You are on page 1of 626

1) 此文档是我在业余时间所写,由于基础知识和能力有限,文档肯定存在错误

之处,还请仅供参考之用。如有朋友希望反馈错误之处或相互交流学习,还
请通过微信号联系我:a346322498

2) 真诚得感谢原著作者 Robert W.Erickson,让我有机会能读到逻辑严谨、语言


优美的精彩书籍!

-1-
第2章 稳态变换器的原理

Problem 2.1

Buck-Boost 变换器的分析与设计:图 2-1 (a)展示了一个 Buck-Boost 变换器,


(b)展示了一个实际应用中使用晶体管和二极管实现的 Buck-Boost 变换器。
1 2
(a)

i (t )
Vg v
C R
L

iT Q1 D1 iD
(b)

i (t )
Vg v
C R
L

图 2-1 Buck-Boost 变换器; (a)理想变换器电路,(b)使用 MOS 和二极管实现

(a) 推导稳态下的输出电压 V、电感电流 I 关于占空比 D、输入电压 Vg、负载


电阻 R 的关系式,你可以假设电感电流纹波和电容电压纹波相当小。
(b) 在 0  D  1的整个范围上,画出上一小问(a)的图形。
(c) DC 设计:对于以下指标:
Vg  3 0 V V  2 0 V

R  4 f s  40kHz

i. 求解 D 和 i (t ) 。

ii. 计算电感量 L 的值,使得电感电流纹波峰值 i 等于电感电流平均值


1
I的 。
10
iii. 计算电容量 C 的值,使得输出电压纹波的 v 峰值为 0.1V。
(d) 对于(c)部分的设计,画出 MOSFET 的漏极电流 iT 。如果考虑到电感电流

纹波的影响,那么漏极电流 iT 的峰值是多少?当电感 L 的值减小至使 i 等于 I

-2-
的一半时,此时漏极电流 iT 会发生什么情况?

(e) 对于(d)部分的两种情况,画出二极管电流 id 的波形。

Solution:
(a) 当开关位于位置 1 时,即 MOS 导通时,变换器的电路简化至图 2-2(a),
1 2
(a)
iC
i (t )
Vg C
R v
L vL

1 2
(b)
iC
i (t )
Vg C
R v
L vL

图 2-2 各开关状态示意图
电感电压 vL、电容电流 iC 满足以下方程:
vL  Vg
v (2-1)
iC  
R

应用小纹波近似,即认为 i (t ) 、v 的纹波相对于它们的直流分量 I、V 很小,

得到以下方程:
vL  Vg
V (2-2)
iC  
R
当开关位于位置 2 时,即二极管导通时,变换器的电路简化至图 2-2(b)。电
感电压 vL、电容电流 iC 满足以下方程:
vL  v
v (2-3)
iC  i 
R
应用小纹波近似,有:
vL  V
V (2-4)
iC   I 
R

-3-
根据电感的伏秒平衡、电容的电荷平衡有:
vL  DVg  DV  0
V V (2-5)
iC  D( )  D( I  )  0
R R
求解式(2-5),有:
D
V Vg
D 1 (2-6)
1 V
I
D 1 R
V I V
(b) 、 的波形如图 2-3 所示,其中 I Load = 。
Vg I Load R

0
0

 3.125

 6.25
V/Vg = D/(D-1)

 9.375

D
 12.5
D1

 15.625

 18.75

 21.875

 21.672
 25
0 0.125 0.25 0.375 0.5 0.625 0.75 0.875 1
0 D 1

0
1
 3.125

 6.25
I/I_load = 1/(D-1)

 9.375
1
 12.5
D1
 15.625

 18.75

 21.875
 22.672
 25
0 0.125 0.25 0.375 0.5 0.625 0.75 0.875 1
0 D 1

图 2-3 V / Vg、I / ILoad 波形图


(c) 对于 C 部分的指标:
i. 根据式(2-6)可以推出所需的 D 和 I:

-4-
V
D  0.4
Vg  V
(2-7)
V 25
I=    8.33A
(1  D) R 3
1
ii. 当 i  I 时, L 的值可根据式(2-8)算得:
10
DTSVg
L  0.18mH (2-8)
2i
iii. 当 v  0.1V 时,根据以下两式:
Q  2C v
1 T (2-9)
Q  i S
2 2
求得:
i  TS
C  26μF (2-10)
8v
(d) 对于(c)中的参数,晶体管集电极电流 iT (t ) 如图 2-4 所示:

iT (t)

iT (max)

0 DTS TS t

图 2-4 晶体管集电极电流波形

iT (t ) 的最大值为:

iT (max)  I  i  8.33  0.833  9.163A (2-11)

1
当 L 的值减小至 i  I 时, iT (t ) 如图 2-5 所示:
2

-5-
iT (t)
iT (max)

I  8.33 A

0 DTS TS t

图 2-5 集电极电流纹波为直流分量的一半时

此时, iT (t ) 的最大值为:

iT (max)  I  i  8.33  0.5  8.33  12.495A (2-12)

注:此时为 CCM 和 DCM 的临界点。


1
(e) i  I 时的 iD (t ) 波形如图 2-6 所示:
10
iD (t)

iD (max)

0 DTS TS

图 2-6 iD (t ) 波形

1
当 L 减小至使 i  I 时的 iD (t ) 波形如图 2-7 所示:
2
iD (t)
iD (max)

I  8.33 A

0 DTS TS t

图 2-7 集电极电流纹波为直流分量的一半时的 iD (t ) 波形

-6-
Problem 2.2

在某一特定的应用场合下,未经整定的直流输入电压可在 18V 至 36V 变化。


对于输出,要求提供 28V 以满足 2A 的负载要求。因此,就需要一个既能升压也
能降压的变换器。由于要求输入电压与输出电压极性相同,故那些反转输出电压
极性的变换器(如 Buck - Boost 变换器)便不符合此场合要求。
对于以上需求,图 2-8 所示的非隔离 SEPIC 电路满足要求。该电路的转换
比率 M ( D) 在不反转输出电压极性的前提下,既能升压也能降压。当变换器正常

运行时,第一个子区间 (0  t  DTS ) 晶体管导通,第二个子区间 ( DTS  t  TS ) 二

极管导通。你可以假设所有元器件均为理想器件。
i1 L1 C1 D1

iD

Vg Q1 L2 C2 R v

图 2-8 SEPIC 变换器


(a) 推导各个电容电压、电感电流直流分量关于占空比 D、输入电压 Vg、负
载电阻 R 的表达式。
(b) 当存在一个控制电路可以自动的调整以输出恒定电压 28V,即输入电压在
(18V  Vg  36) 缓慢变化时,负载电流保持 2A 不变。那么此情况下的占空比

D 的变化范围是多少?输入电感电流的直流分量 I1 的变化范围是多少?
Solution:
(a) 当 MOS 导通时,即处于第一个子区间时,变换器的电路简化至图 2-9(a)
所示。当 MOS 关断、二极管导通时,即处于第二个子区间时,变换器的电路简
化至图 2-9(b)所示:
L1 vC1
i1
(a) vL1 iC1 i2 iC 2
C1

Vg L2 vL 2 C2 R v

L1 vC1
i1
(b) vL1 iC1 C1 i2 iC 2

Vg L2 vL 2 C2 R v

图 2-9 各开关状态示意图

-7-
对于第一个子区间,即 MOS 导通时,对于电感电压 vL1、vL2、电容电流 iC1、
iC2 有:
vL1  Vg
vL 2  vC1
iC1  i2 (2-13)
v
iC 2  
R
应用小纹波近似,即认为电感电流 i、电容电压 v 的纹波相对于它们的直流
分量 I、V 很小,得到以下方程:
vL1  Vg
vL 2  VC1
iC1   I 2 (2-14)
V
iC 2  
R
对于第 2 个子区间,即二极管导通时,对于电感电压 vL1、vL2、电容电流 iC1、
iC2 有:
vL1  Vg  v  vC1
vL 2  v
iC1  i1 (2-15)
v
iC 2  i1  i2 
R
应用小纹波近似,有:
vL1  Vg  V  VC1
vL 2  V
iC1   I1 (2-16)
V
iC 2  I1  I 2 
R
根据电感的伏秒平衡和电容的电荷平衡有:
vL1  D(Vg )  D(Vg  V  VC1 )  0
vL 2  D(VC1 )  D(V )  0
iC1  D( I 2 )  D( I1 )  0 (2-17)
V V
iC 2  D( )  D( I1  I 2  )  0
R R
求解式(2-17),得出:

-8-
VC1  Vg
DVg
Vc 2  
D 1
D 2 Vg (2-18)
I1 
(1  D ) 2 R
D Vg
I2 
(1  D ) R
(b) 根据式(2-18),可推出:
VC 2
D
Vg  VC 2
(2-19)
D 2Vg
I1 
(1  D) 2 R

当 (18V  Vg  36) 时,D 的范围为 (0.6087  D  0.4375) ,如图 2-10 所示,

输入电感电流直流分量 I1 的范围为 (3.111A  I1  1.556A),如图 2-11 所示:

0.62

0.595

0.57

0.545
Vc2
0.52
Vg Vc2

0.495

0.47

0.445

0.42
18 20.25 22.5 24.75 27 29.25 31.5 33.75 36
Vg

图 2-10 D 的变化范围
3.2

2.975

2.75
2
 28 V   Vg
2.525
 28 V V 
 g
2.3
2
1 28 V  R
  28 V V  2.075
  g 

1.85

1.625

1.4
18 20.25 22.5 24.75 27 29.25 31.5 33.75 36
Vg

图 2-11 电感电流直流分量变化范围

-9-
Problem 2.3

对于 Problem 2.2 的 SEPIC 电路:


(a) 推导各个电感电流纹波、电容电压纹波的表达式。将这些纹波表示为开关
周期 Ts、电感值 L1、L2,电容值 C1、C2,占空比 D、输入电压 Vg、负载电阻 R
的函数。
(b) 画出 MOS 的漏源极电压 vDS (t ) 、漏极电流 iD (t ) 的波形,并推导它们的峰

值表达式。
Solution:
(a) 对于电感 L1、L2,它们的电流纹波可根据下式求出:
2i1
Vg  L1
DTS
(2-20)
2i2
V  L2
(1  D)TS

故 i1 、 i2 为:

DTsVg
i1 
2 L1
(2-21)
DTsVg
i2 
2 L2
对于电容电压纹波,要先求出流过电容的电流,并画出电流波形,确定
电荷的增量,再根据电荷的增量来确定电容电压纹波。
先画电感电压波形和电感电流波形,如图 2-12 所示,电感电流纹波值为式
(2-21)确定。
vL1 vL 2

Vg
V
0 DTs Ts t 0 DTs Ts t
V
Vg

i1 i2
I1  i1 I 2  i2
I1 I2

0 DTs Ts t 0 DTs Ts t

图 2-12 电感电压波形与电感电流波形

- 10 -
对于 iC1(t)根据式(2-17)可知,当 0  t  DTs 时, iC1  i2 ,当 DTS  t  TS 时,

iC1  i1 。故 iC1(t)的波形和 vC1(t)的波形如图 2-13 所示:

iC1
I 2  i2
I2
vC1
0 DTs Ts
0 DTs Ts t V  v t
c1 c1
 I1 Vc1  Vg
 I1  i1

图 2-13 电容 C1 电流
故,电容 C1 的电压纹波可根据下式求出:
Q I DT
C1   2 s
2vc1 2vc1
(2-22)
I DT D 2VgTs
vc1  2 s 
2C1 2(1  D) RC1
注:在求电容电压纹波的时候,本质上是根据下式(2-23)求出的,对于积分
部分,可以使用更简便的方法计算,即用电流的平均值(等于直流分量),这样
就不必像刚才那样既要画出电容电流波形,又要计算此电流的积分这么麻烦了
(算出结果后才反应过来自己折腾了一圈…)。
du
iC
dt
(2-23)
1 1
U DTs  U 0  2v   i  dt  IDTs
C C
对于电容 C2 的电压纹波(这回用简单的方法,不折腾了…),根据下式有:
1
C
u i  dt

1  V
2vc 2     DTs (2-24)
C2  R 
VDTs
vc 2 
2C2 R
vc2 的波形如图 2-14 所示:

- 11 -
v
V  v
V

0 DTs Ts t

图 2-14 电容 C2 电压波形(负载电压)

(b) 根据不同开关状态下的电路图,如图 2-9 所示,可知:


 0 (0  t  DTs )
vDS (t )  
v  vc1 ( DT  t  Ts )
(2-25)
 0 (0  t  DTs )
iD (t )  
i1  i2 ( DT  t  Ts )

vDS (t ) 的波形可由图 2-14 的波形减去图 2-13 的右侧波形得到,相减后如图

2-15 所示:
vDS

V  v  Vg  vc1

0 DTs Ts t

图 2-15 MOS 管电压波形

iD (t ) 的波形如图 2-16 所示,最大值已在图中标出(图中红色波形什么条件

下出现还没想明白…)。
iD

i1  i2  i1  i2

DTs Ts t

图 2-16 二极管电流波形

Problem 2.4

图 2-17 对于所示的变换器,其开关的动作是同步的,即 0  t  DTs 时,开

- 12 -
V
关处于位置 1,当 DTs  t  Ts 时,开关处于位置 2。推导电压转换比 M ( D)  的
Vg

表达式,并画出其波形 M(D) vs D。

1 ic 2
iL L
Vg C
vL R
2 v 1

图 2-17 H 桥变换器
Solution:
当开关处于位置 1 时,对于电感电压 vL、电容电流 ic 有:
vL  Vg  v
v (2-26)
ic  iL 
R
应用小纹波近似,有:
vL  Vg  V
V (2-27)
ic  I L 
R
当开关处于位置 2 时,有:
vL  Vg  v
v (2-28)
ic  iL 
R
应用小纹波近似,有:
vL  Vg  V
V (2-29)
ic  I L 
R
根据电感的伏秒平衡和电容的电荷平衡,有:
vL  D (Vg  V )  D( Vg  V )  0
V V (2-30)
ic  D( I L  )  D( I L  )  0
R R
由式(2-30)可推得:
V  (2 D  1)Vg
V (2-31)
IL 
R

- 13 -
故电压转换比 M(D)为:
M ( D)  2 D  1 (2-32)

M(D) vs D 的图形如图 2-18 所示:


1

0.75

0.5

0.25
M(D)

2D 1 0

 0.25

 0.5

 0.75

1
0 0.125 0.25 0.375 0.5 0.625 0.75 0.875 1
D

图 2-18 M(D) vs D 图

Problem 2.5

图 2-19 对于所示的变换器,其开关的动作是同步的,即 0  t  DTs 时,开

V
关处于位置 1,当 DTs  t  Ts 时,开关处于位置 2。推导电压转换比 M (D)  的
Vg

表达式,并画出其波形 M(D) vs D。
iL L
vL
1 ic 2

Vg C
R
2 v 1

图 2-19 电流型 H 桥变换器


Solution:
当开关处于位置 1 时,对于电感电压 vL、电容电流 ic 有:
vL  Vg  v
v (2-33)
ic  iL 
R
应用小纹波近似,有:

- 14 -
vL  Vg  V
V (2-34)
ic  I L 
R
当开关处于位置 2 时,有:
vL  Vg  v
v (2-35)
ic  iL 
R
应用小纹波近似,有:
vL  Vg  V
V (2-36)
ic   I L 
R
根据电感的伏秒平衡和电容的电荷平衡,有:
vL  D(Vg  V )  D(Vg  V )  0
V V (2-37)
ic  D( I L  )  D( I L  )  0
R R
由式(2-37)推得:
Vg
V
2D 1
(2-38)
V
IL 
(2 D  1) R

故电压转换比 M(D)为:
1
M ( D)  (2-39)
2D  1
M(D) vs D 的图形如图 2-20 所示:

- 15 -
50

37.5

25

12.5
M(D) 1
0
2D1

 12.5

 25

 37.5

 50
0 0.125 0.25 0.375 0.5 0.625 0.75 0.875 1
D

图 2-20 电流型 H 桥变换器 M(D) vs D 图

Problem 2.6

对于图 2-17 所示的变换器,推导电感电流纹波 iL 和电容电压纹波 vc 的表

达式。
Solution:
由于 Vg – V 和 -Vg – V 的符号既有可能是正号,也可能是负号,但这两项的
符号一定是不同的(伏秒平衡)。故假设 Vg – V 的符号为正,则电感电压 vL 的波
形如下所示:
vL

Vg  V

0 DTs Ts t

Vg  V

图 2-21 电感电压 vL 波形
电感电流 iL 的波形如图 2-22 所示:

- 16 -
iL

IL

0 DTs Ts t

图 2-22 电感电流波形

则电感电流纹波 iL 为:

(Vg  V ) DTs
iL  (2-40)
2L
对于设计良好的变换器,电感电流的直流分量 IL 全部流过电阻负载,而电
感电流的交流分量 iL 则几乎全部流过电容(通过设计,将此回路在开关频率处

的阻抗设为很小的值)。
故假设 iL 全部流过电容,则电容电流波形如图 2-23 所示:

ic
Ts
2iL 0
DTs t

图 2-23 电容电流波形
故,电容电压纹波则可推出:

iLTs (Vg  V ) DTs


2

vc   (2-41)
8C 16 LC

Problem 2.7

对于图 2-19 所示的变换器,推导电感电流直流分量关于占空比 D、输入电


压 Vg、负载电阻 R 的表达式,并画出图形。
Solution:
根据式(2-38),有:
Vg
IL  (2-42)
(2 D  1) 2 R
如图 2-24 所示:

- 17 -
图 2-24 电感电流直流分量关于 D 的波形

Problem 2.8

对于图 2-19 所示的变换器,推导电感电流纹波 iL 与电容电压纹波 vc 。

Solution:
电感电流纹波 iL 可由下式推出:

diL 2iL
vL  L L (2-43)
dt DTs

故对于 (0  t  DTs ) ,可推出电感电流纹波为:

(Vg  V ) DTs
iL  (2-44)
2L
对于设计良好的变换器,电感电流的交流分量可近似认为全部流过电容,故
根据式(2-41)的算法,可得:

iLTs (Vg  V ) DTs


2

vc   (2-45)
8C 16 LC

Problem 2.9

对于 Buck 变换器,为了降低开关动作所引起的输入电流谐波,可以在输入
处添加 LC 滤波电路,如图 2-25 所示。为了限制传导性的电磁干扰,像这样的
输入滤波电路被广泛得使用以满足各种标准。对于此题,你可以假设所有的电感
和电容值都足够大,因此所有的纹波都足够小。

- 18 -
i1 L1 iT Q1 i2 L2

Vg C1 vc1 D1 C2 R v

图 2-25 带有 LC 输入滤波的 Buck 变换器

(a) 画出 MOS 管电流 iT (t ) 。

(b) 推导各电容电压和各电感电流的直流分量。
(c) 推导输入滤波电感电流和电容电压纹波的幅值。
(d) 给定以下量值:
输入电压 Vg  48V

输出电压 V 36 V

开关频率 fs  1 0 0 k H z

负载电阻 R  6

选定 L1 、C1 的值使得 (i) 电容 C1 的纹波幅值 vc1 是其直流分量 Vc1 的 2% ,(ii )

输入电流的纹波幅值 i1 为 20mA 。

Solution:
(a) iT (t ) 的近似波形图 2-26 如图 2-26 所示:

iT (t)

0 DTs Ts t

图 2-26 iT (t ) 波形

(b) 当 0  t  DTs 时,电路简化至图 2-27(a)所示:

- 19 -
i1 L1 i2 L2
(a)
vL1 vL 2 ic 2
ic1
Vg C1 vc1 C2 R v

i1 L1 i2 L2
(b) vL1 vL 2 ic 2
ic1
Vg C1 vc1 D1 C2 R v

图 2-27 各子区间简化示意图
对于电感电压、电容电流,有:
vL1  Vg  v c1
vL 2  v c1 v
ic1  i1  i2 (2-46)
v
ic 2  i2 
R
应用小纹波近似,可得:
vL1  Vg  Vc1
vL 2  Vc1  V
ic1  I1  I 2 (2-47)
V
ic 2  I 2 
R

当 DTS  t  TS ,电路简化至图 2-27(b)所示,对于电感电压、电容电流有:

vL1  Vg  vc1
vL 2   v
ic1  i1 (2-48)
v
ic 2  i2 
R
应用小纹波近似,可得:
vL1  Vg  Vc1
vL 2  V
ic1  I1 (2-49)
V
ic 2  I 2 
R

- 20 -
应用电感伏秒平衡与电容电荷平衡,有:
vL1  D(Vg  Vc1 )  D(Vg  Vc1 )  0
vL 2  D(Vc1  V )  D(V )  0
ic1  D( I1  I 2 )  D( I1 )  0 (2-50)
V V
ic 2  D( I 2  )  D( I 2  )  0
R R
求解上式,可得:
V
I1  D
R
V
I2  (2-51)
R
Vc1  Vg
Vc 2  DVg

(c) 根据式(2-47)和式(2-49)可画出电感 L2 的电流波形,又由于当电容 C2 的值

足够大时,可认为电感 L2 电流的交流分量全部流过电容 C2 ,而电感 L2 电流的直

流分量全部流过负载,故可画出电感 L2 和电容 C2 的电流波形,如图 2-28 所示,

根据图 2-28 有:
2iL 2
VC1  V  L2
DTS
dvC 2 (t )
iC 2 (t )  C2  (2-52)
dt
D
TS  TS
2
1 T

DTS
iC 2 (t )dt  C2  2vC 2   S  iL 2  2C2 vC 2
2 2
2

iL 2 (t )

0 DTS TS t
iC 2 (t )

t
0 DTS TS

图 2-28 电感 L2 和电容 C2 的电流波形

由式(2-52)可求出电感 L2 的电流纹波幅值和电容 C2 的电压纹波幅值为:

- 21 -
DDVgTS
iL 2 
2 L2
(2-53)
DDVgTS 2
vC 2 
16C2 L2

由于电感 L1 的值很大,可先暂时认为电感电流 iL1 (t )  I1 ,而当 0  t  DTS 时,

有:
iC1  iL1  iL 2  iC1  I1  iL 2 (2-54)

故根据式(2-54)和图 2-28 可画出电容 C1 的电流和电压波形,如图 2-29 所示:

iL 2 (t )

V
R

0 DTS TS t
iL1 (t )
V
iL1 (t )  I1  DI 2  D
R
0 DTS TS t
iC1 (t )

V 0 DTS TS t
I1  (  iL 2 )
R

vC1 (t )
Vg

0 DTS TS t

图 2-29 电容 C1 的电流和电压波形

由图 2-29 可求出电容 C1 的电压纹波幅值为:

dvC1 (t )
TS
DD 2VgTS
iC1 (t )  C1   iC1 (t )dt  2C1vC1  vC1  (2-55)
dt DTS
2 RC1

由于电感 L2 的电流纹波幅值 iL 2 很小,为了便于计算电感 L1 的电流纹波幅

值,故认为电容 C1 的电流 iC1 (t ) 在 0  t  DTS 这个区间内的斜率变化不大,近似为

一水平线,进而对于 0  t  DTS 区间,电容 C1 的电流电压波形可近似为如图 2-30

所示:

- 22 -
iC1 (t )

V V 0 DTS TS t
I1  (  iL 2 )  I1 
R R

vC1 (t )
Vg

0 DTS TS t

图 2-30 电容 C1 的电流电压近似波形

由图 2-30 可得电感 L1 的电压波形如图 2-31 所示:

vL1 (t )  Vg  vC1 (t )

TS
0 DTS t

图 2-31 电感 L1 的电压波形

由图 2-31 可推导出电感 L1 的纹波幅值为:


DTs
TS 
2
diL1 (t )
vL1 (t )  L1
dt

DTS
 vL1 (t )dt  2 L1iL1 
2 (2-56)
vC1TS D 2 DVgTS 2
iL1   iL1 
8L1 16C1 L1 R
(d) 根据式(2-55)和式(2-56),将给定条件代入可得:
D 2 DVgTS
C1   5.859μF
2 RvC1
(2-57)
D 2 DVgTS 2
L1   60μH
16C1 RiL1

- 23 -
第3章 稳态下的等效电路建模、损耗与效率

Problem 3.1

对于图 3-1 所示的 Buck-Boost 变换器,电感的绕组具有电阻 RL,其他的所


有损耗可以忽略。
(a) 推导非理想电压转换比率 V / Vg 的表达式。

(b) 对于 0  D  1, RL / R  0, 0.01 和 0.05 时,画出(a)中结果的图形。

(c) 推导效率的表达式。

Vg L v

图 3-1 非理想 Buck – Boost 变换器,Problem3.1 和 3.2


Solution:
(a) 当开关导通时,即 0  t  DTS 时,电路如图 3-2 所示:

Vg L VL R v

iC
RL

图 3-2 开关导通时的 Buck-Boost 变换器简化电路


对于电感电压和电容电流有:
vL (t )  Vg  i (t ) RL  Vg  IRL
v(t ) V (3-1)
iC (t )   
R R

当开关关断,即 DTS  t  TS 时,电路如图 3-3 所示:

Vg L VL R v
i
iC
RL

图 3-3 开关关断时的 Buck-Boost 变换器简化电路

- 24 -
对于电感电压和电容电流有:
vL (t )  v(t )  i (t ) RL  V  IRL
v(t ) V (3-2)
iC (t )    i (t )    I
R R
应用电感的伏秒平衡和电容的电荷平衡,可得:
DV  DVg  IRL  0
V (3-3)
DI  0
R
可推出电压转换比为:
V DR( D  1)
 (3-4)
Vg R( D  1)2  RL

(b) 当 RL / R  0 时,电压转换比为:

V D
 (3-5)
Vg D  1

其结果如图 3-4 所示:


0

 3.125

 6.25

 9.375
D
 12.5
D 1
 15.625

 18.75

 21.875

 25
0 0.125 0.25 0.375 0.5 0.625 0.75 0.875 1
D

图 3-4 当 RL/R = 0 时的电压转换比

当 RL / R  0.01时,电压转换比为:

V D( D  1)
 2 (3-6)
Vg D  2 D  1.01

其结果如图 3-5 所示:

- 25 -
0
 0.625
 1.25
 1.875
D ( D1)
2  2.5
D 2.0 D 1.01
 3.125
 3.75
 4.375
5
0 0.125 0.25 0.375 0.5 0.625 0.75 0.875 1
D

图 3-5 当 RL/R = 0.01 时的电压转换比

当 RL / R  0.05 时,电压转换比为:

V D( D  1)
 2 (3-7)
Vg D  2 D  1.05

其结果如图 3-6 所示:

 0.225

 0.45

 0.675
D ( D1)
2  0.9
D 2.0 D 1.05
 1.125

 1.35

 1.575

 1.8
0 0.125 0.25 0.375 0.5 0.625 0.75 0.875 1
D

图 3-6 当 RL/R = 0.05 时的电压转换比

(c) 效率 由下式推出:

V2
Po R( D  1) 2
  R  (3-8)
Pin Vg  I  D R ( D  1) 2  RL

- 26 -
Problem 3.2

图 3-1 所示的 Buck-Boost 变换器中的电感具有绕组电阻 RL。其他的所有损


耗可以忽略不计,推导这个变换器的等效电路,推导的等效电路应该明显的展示
变换器的输入端口,此外还应包括两个 DC 变压器。
Solution:
根据式(3-3),可推出图 3-7 所示的电路:

Ig IL L RL

Vg DI L DVg DV DI L V

图 3-7 Buck-Boost 合成电路


故,等效电路如图 3-8 所示:

Ig 1: D IL L RL D :1

Vg V

图 3-8 Buck-Boost 变换器等效电路


验证:
将电路等效至中间部分,如图 3-9 所示:

IL L RL

DVg D2 R DV

图 3-9 归一化至中间部分的 Buck-Boost 等效电路


由上图可推出电压转换比为:
DV DVg V D( D  1) R
    (3-9)
D R RL  D R
2 2
Vg (1  D)2 R  RL

效率为:
 DVI L D V D D( D  1) R (1  D) 2 R
      ( )  (3-10)
DVg I L D Vg D (1  D)2 R  RL (1  D) 2 R  RL

- 27 -
Problem 3.3

图 3-10 所示变换器中的电感具有绕组电阻 RL,其他的所有损耗可以忽略不


计。此变换器的开关同步得动作:当 0  t  DTS 时,开关位于位置 1;当 DTS  t  TS

时,开关位于位置 2。
L
C
1 2
Vg
2 R 1
v
图 3-10 非理想电流型桥式变换器,Problem3.3 和 3.4

(a) 推导非理想的电压转换比 V / Vg 。

(b) 对于 0  D  1, RL / R  0, 0.01 和 0.05 时,画出(a)中结果的图形。

(c) 推导效率的表达式。
Solution:
(a) 当开关位于位置 1 时,即 0  t  DTS 时,电路如图 3-11 所示:

L i RL
VL C iC
1
Vg
R 1
v
图 3-11 开关位于位置 1 时的桥式电流型变换器
对于电感电压和电容电流有:
vL (t )  Vg  i (t ) RL  v(t )  Vg  IRL  V
v(t ) V (3-11)
iC (t )  i (t )  I
R R

当开关位于位置 2 时,即 DTS  t  TS 时,电路如图 3-12 所示:

L i RL
VL C iC
2
Vg
2 R
v
图 3-12 开关位于位置 2 时的桥式电流型变换器
对于电感电压和电容电流有:

- 28 -
vL (t )  Vg  i (t ) RL  v(t )  Vg  IRL  V
v(t ) V (3-12)
iC (t )  i (t )   I 
R R
应用电感的伏秒平衡和电容的电荷平衡,可得:
Vg  (1  2 D)V  IRL  0
V (3-13)
(2 D  1) I  0
R
可推出电压转换比为:
V (2 D  1) R
 (3-14)
Vg (2 D  1)2 R  RL

(b) 当 RL / R  0 时,电压转换比为:

V 1
 (3-15)
Vg 2 D  1

其结果如图 3-13 所示:


50

37.5

25

12.5
1
0
2 D 1
 12.5

 25

 37.5

 50
0 0.125 0.25 0.375 0.5 0.625 0.75 0.875 1
D

图 3-13 当 RL / R = 0 时的电压转换比

当 RL / R  0.01时,电压转换比为:

V 2D  1
 (3-16)
Vg 4 D  4 D  1.01
2

其结果如图 3-14 所示:

- 29 -
6

4.5

1.5
2 D 1
2 0
4 D  4 D 1.01
 1.5

3

 4.5

6
0 0.125 0.25 0.375 0.5 0.625 0.75 0.875 1
D

图 3-14 当 RL / R = 0.01 时的电压转换比

当 RL / R  0.05 时,电压转换比为:

V 4D  2
 (3-17)
Vg 8D  8D  2.1
2

其结果如图 3-15 所示:

2.5
1.875
1.25
0.625
4 D 2
2 0
8 D  8 D 2.1
 0.625
 1.25
 1.875
 2.5
0 0.125 0.25 0.375 0.5 0.625 0.75 0.875 1
D

图 3-15 当 RL / R = 0.05 时的电压转换比


(c) 效率可由下式推出:

V2
P R(2 D  1) 2
 o  R  (3-18)
Pin Vg  I R(2 D  1) 2  RL

Problem 3.4

图 3-10 所示的电流型桥式变换器中的电感绕组具有电阻 RL,其他的所有损


耗可忽略不计,推导该变换器的等效电路。
Solution:
根据式(3-13),可推出图 3-16 所示的电路:

- 30 -
IL L RL

Vg (2 D  1)V (2 D  1) I L R V

图 3-16 电流型桥式变换器的合成电路
故等效电路如图 3-17 所示:
IL L RL 2 D  1:1

Vg R V

图 3-17 电流型桥式变换器的等效电路
验证:
将 DC 变压器副边的电路等效至原边,可得图 3-18 所示的等效电路:
IL L RL

Vg (2 D  1) 2 R (2 D  1)V

图 3-18 副边等效至原边后的等效电路
由上图可推出电压转换比为:
(2 D  1)V Vg V (2 D  1) R
   (3-19)
(2 D  1) R RL  (2 D  1) R
2 2
Vg RL  (2 D  1)2 R

效率为:
(2 D  1)VI L V (2 D  1)2 R
  (2 D  1)  (3-20)
Vg I L Vg RL  (2 D  1) 2 R

Problem 3.5

图 3-19 所示的 Buck 变换器,MOSFET 具有导通电阻 Ron,二极管的正向压


降可以建模成恒压源 VD,其他的所有损耗可以忽略不计。
(a) 推导此变换的等效电路模型。
(b) 根据所建立的模型推导输出电压 V。
(c) 推导效率的表达式。

- 31 -
Q1 L

Vg D1 C R V

图 3-19 非理想 Buck 变换器


Solution:
(a) 当 MOSFET 导通时,即 0  t  DTS 时,电路如图 3-20 所示:

i Ron L
VL iC
Vg C R V

图 3-20 MOSFET 导通时的 Buck 电路


对于电感电压有:
vL (t )  Vg  i(t ) Ron  v(t )  Vg  IRon  V (3-21)

当 MOSFET 关断时,即 DTS  t  TS 时,电路如图 3-21 所示:

i L
VL iC
Vg VD C R V

图 3-21 MOSFET 关断时的 Buck 电路


对于电感电压有:
vL (t )  VD  v(t )  VD  V (3-22)

根据电感的伏秒平衡有:
V  DVD  DVg  DRon I  0 (3-23)

故等效电路模型如图 3-22 所示:


Ig 1: D I DRon DVD

Vg R V

图 3-22 非理想 Buck 电路的等效模型


(b) 根据推导出得等效电路模型,可算得输出电压为:
DRVg  DRVD
V (3-24)
DRon  R

- 32 -
(c) 效率为:
DRVg  DRVD
 (3-25)
DRVg  Ron D 2Vg

Problem 3.6

为了降低 Buck 变换器输入电流的开关谐波,添加了输入滤波器,如图 3-23


所示。电感 L1 和 L2 各自具有绕组电阻 RL1 和 RL2,MOSFET 具有导通电阻 Ron,
此外,二极管正向压降可以使用恒压源 VD 和导通电阻 RD 进行建模,其他的所有
损耗可以忽略不计。
i1 L1 RL1 Q1 i2 L2 RL 2

Vg C1 vC1 D1 C2 R v

图 3-23 带有输入滤波的 Buck 变换器


(a) 推导此电路的等效电路模型。
(b) 求解输出电压 V 的表达式。
(c) 推导效率的表达式。
Solution:
(a) 当 MOSFET 导通时,即 0  t  DTS 时,电路如图 3-24 所示:

i1 L1 RL1 Ron i2 L2 RL 2
vL 2
Vg C1 vC1 C2 R v

图 3-24 MOSFET 导通时的带输入滤波的 Buck 变换器


对于电感 L2 有:
vL 2 (t )  Vg  i1 (t ) RL1  i2 (t ) Ron  i2 (t ) RL 2  v(t ) (3-26)

应用小纹波近似,即:
i1 (t )  i2 (t )  I
(3-27)
v(t )  V

故:
vL 2 (t )  Vg  IRL1  IRon  IRL 2  V (3-28)

当 MOSFET 关断时,即 DTS  t  TS 时,电路如图 3-25 所示:

- 33 -
i1 L1 RL1 i2 L2 RL 2
vL 2
Vg C1 vC1 VD C2 v
R
RD

图 3-25 MOSFET 关断时的带输入滤波的 Buck 变换器


对于电感 L2 有:
vL 2 (t )  VD  i2 (t ) RD  i2 (t ) RL 2  v(t ) (3-29)

应用小纹波近似,可得:
vL 2 (t )  VD  IRD  IRL 2  V (3-30)

由伏秒平衡可得:
V  DVD  DVg  DIRD  IRL 2  DIRL1  DIRon  0 (3-31)

根据(3-31)可画出等效电路,如图 3-26 所示:

Ig DVD DRD DRL1 DRon RL 2


I

Vg R V

1: D
图 3-26 带输入滤波的 Buck 变换器的等效电路
(b) 根据图 3-26 所示的等效电路,可求出输出电压:
DRVg  DRVD
V (3-32)
R  DRon  DRL1  DRD  RL 2

(c) 效率为:
DRVg  DRVD
 (3-33)
DVg ( R  DRD  DRL1  DRon  RL 2 )

Problem 3.7

用一个 1.5V 的锂电池,使用 Buck-Boost 变换器为 5V,1A 的负载供电。变


换器电路中已选择了合适的导通电阻为 35mΩ 的晶体管及正向压降为 0.5V 的肖
特基二极管,此外肖特基二极管的导通电阻可忽略不计,整个变换器的原理图如
图 3-27 所示。

- 34 -
Q1 1A

Vg
100  H 5V Load
1.5V
DTS TS
f S  40kHz

图 3-27 非理想 Buck-Boost 变换器,Problem3.7


(a) 推导变换器直流特性的等效电路,要包含晶体管、二极管的导通损耗和电
感的铜耗,其他的损耗可以忽略不计,推导的模型应该正确描述变换器的直流输
入端口。
(b) 正常情况下变换器要有至少 70%的效率(例如,输入电压是 1.5V,输出
是 5V,1A),在此条件下,电感绕组的阻值最大为多少?占空比的范围是多少?
注:有两种方式可以解决此问题,其中一种很简便,另一种则比较繁琐。
(c) 对于(b)部分的设计,求解各个元器件的功率损耗。
(d) 画出变换器输出电压和效率相对于占空比 0  D  1的图形,关于电感绕组
的阻值,请使用(b)部分设计的值。
(e) 讨论(d)的图形,分析是否符合预期?
Solution:
(a) 当晶体管导通时,即 0  t  DTS 时,电路如图 3-28 所示:

Ron iC
RL
Vg vL v R
L
i
f S  40kHz

图 3-28 晶体管导通时的 Buck-Boost 变换器简化电路


对于电感电压有:
vL (t )  Vg  i(t ) Ron  i(t ) RL  Vg  IRon  IRL (3-34)

当晶体管关断时,即 DTS  t  TS 时,电路如图 3-29 所示:

- 35 -
VD RD

RL iC
Vg vL v R
L
i
f S  40kHz

图 3-29 晶体管关断时的 Buck-Boost 变换器简化电路


对于电感电压有:
vL (t )  v(t )  i(t ) RL  VD  i(t ) RD  V  IRL  VD  IRD (3-35)

由电感的伏秒平衡可得:
DVg  DV  IRL  DIRon  DVD  DIRD  0 (3-36)

根据式(3-36)可推出等效电路如图 3-30 所示:


DRon RL DVD DRD
Ig 1: D I D :1

Vg R V

图 3-30 非理想 Buck-Boost 变换器等效电路


(b) 当不考虑二极管的导通电阻 RD 时,将图 3-30 所示的等效电路全部等效至
中间部分,即如图 3-31 所示:
I DR R DVD L on L

DVg D2 R DV

图 3-31 进一步简化的 Buck-Boost 变换器等效电路


为了使效率达到 70%以上,故有:
DVI L (1  D)V
   70% (3-37)
DVg I L DVg

由式(3-37)可推出:
D  0.8264 (3-38)
为了使效率达到 70%以上,有:
I L ( DRon  RL )  DVD  DVg  30%
DV V (3-39)
IL  
D R DR
2

- 36 -
将 Vg,VD,Ron,V 的值带入上式,可得:
RL  0.95D2  1.415D  0.5 (3-40)

上式在 D = 0.7447 时取得最大值,故有:


RL  0.027 (3-41)

(c) 由式(3-39)可算出各器件的平均损耗为:
D
PQ1  I L 2 DRon  0.035
(1  D) 2
RL
PL  I L 2 RL 
D2 (3-42)
PD  I L * DVD  0.5W
V2
PLoad   5W
R
对于 PL 和 PQ1,当给定 RL 时,根据输出电压 V = 5V 等条件会确定固定的占
空比 D,然后根据 D 的值可求出 PQ1 和 PL。
(d) 对于(b)部分的条件,效率和输出电压的表达式为:
DR( DVg  DVD )
VO 
RD2  RL  DRon
(3-43)
DD2 RVg  D3 RVD

DD2 RVg  D 2Vg Ron  DRLVg
当取 RL = 0.02 时,输出电压如图 3-32 所示:
7

4
2
 2500.0 D 2000.0 D  500.0
 3
2
 1993.0 D 1000.0 D  1004.0
2

1
0 0.125 0.25 0.375 0.5 0.625 0.75 0.875 1
D

图 3-32 输出电压随占空比变化的图形
当取 RL = 0.02 时,效率所示:

- 37 -
1

1

2

 ( D) 3

4

5

6

7
0 0.125 0.25 0.375 0.5 0.625 0.75 0.875 1
D

图 3-33 效率随占空比变化的图形
(e) 对于(d)中的图形,输出电压和效率能出现负数,是因为有 VD = 0.5V 的
前提条件。也即,当 D > 0.25 时,二极管才能导通,输出才会有电压。

Problem 3.8

为了使用 500V 直流源给 400V ,10A 的负载供电,有两种变换器电路可以利


用,一种是 Buck 变换器,另一种是 Buck-Boost 变换器,如图 3-34 所示。MOSFET
具有 0.5  的导通电阻,其余的全部损耗可以忽略不计。
(a) 推导两种变换器电路的等效电路。
(b) 当这两种变换器电路均工作在给定条件下时,求出此时的占空比。
(c) 比较这两种变换器电路的效率和 MOSFET 损耗,总结哪种变换器电路更
适合此题的条件。
10A
(a)

500V 400V

10A
(b)
500V 400V

图 3-34 实现 500V 电压源到 400V 负载。(a):Buck;(b):Buck-Boost


Solution (a) Buck:
(a) 当 MOSFET 导通时,即 0  t  DTS 时,电路如图 3-35 所示:

Ron vL (t ) i

Vg v

图 3-35 MOSFET 导通时的 Buck 电路


对于电感电压,有:

- 38 -
vL (t )  Vg  i (t ) Ron  v(t )  Vg  IRon  V
V v(t ) (3-44)
i (t )  I  
R R

当 MOSFET 关断时,即 DTS  t  TS 时,电路如图 3-36 所示:

i vL (t)

Vg v

图 3-36 MOSFET 关断时的 Buck 电路


对于电感电压,有:
vL (t )  v(t )  V (3-45)

应用电感的伏秒平衡,有:
DVg  V  DIRon  0 (3-46)

故有图 3-37 所示的等效电路:


1: D I DRon

Vg V

图 3-37 考虑 MOSFET 导通电阻的 Buck 电路等效电路


(b) 对于给定的条件,根据式(3-46)可求出占空比为:
V
D  0.808 (3-47)
Vg  IRon

(c) MOSFET 的导通损耗为:


PMOSFET  I 2 DRon  40.4W (3-48)

效率为:
VI
  0.989 (3-49)
DVg I

Solution (b) Buck-Boost:


(a) 当 MOSFET 导通时,即 0  t  DTS 时,电路如图 3-38 所示:

- 39 -
Ron

i
Vg v
vL (t)

图 3-38 MOSFET 导通时的 Buck-Boost 电路


对于电感电压和电容电流,有:
vL (t )  Vg  i (t ) Ron  Vg  IRon
v(t ) V (3-50)
iC (t )   
R R

当 MOSFET 关断时,即 DTS  t  TS 时,电路如图 3-39 所示:

i
Vg v
vL (t)

图 3-39 MOSFET 关断时的 Buck-Boost 电路


对于电感电压和电容电流,有:
vL (t )  v(t )  V
v(t ) V (3-51)
iC (t )  i (t )  I
R R
应用电感的伏秒平衡和电容的电荷平衡,有:
DVg  DIRon  DV  0
V (3-52)
I
DR
故有所示的等效电路:
1: D I DRon D :1

Vg V

图 3-40 考虑 MOSFET 导通电阻的 Buck-Boost 电路等效电路


(b) 对于给定的条件,根据式(3-52)可求出占空比为:
V
D  0.44897 (3-53)
V  Vg  IRon

(c) MOSFET 的导通损耗为:


PMOSFET  I 2 DRon  73.9W (3-54)

- 40 -
效率为:
DVI
  0.982 (3-55)
DVg I

故,比较两种变换器电路的 MOSFET 的损耗可以看出 Buck 更适合此种情况


下的应用。

Problem 3.9

对于 300V 的电池,可以使用 Boost 或 Buck-Boost 变换器为 400V,10A 的


负载供电,如图 3-41 所示。求解两种变换器电路下的效率和功率损耗,哪种变
换器电路更适合此题情况下应用。
10A
(a)

300V 400V

10A
(b)
300V 400V

图 3-41 使用 300V 电池为 400V 负载供电,(a): Boost; (b): Buck-Boost


Solution (a) Boost:
当 MOSFET 导通时,即 0  t  DTS 时,电路如图 3-42 所示:

i vL (t ) iC (t )

Vg Ron v

图 3-42 MOSFET 导通时的 Boost 电路


对于电感电压和电容电流,有:
vL (t )  Vg  i (t ) Ron  Vg  IRon
v(t ) V (3-56)
iC (t )   
R R

当 MOSFET 关断时,即 DTS  t  TS 时,电路如图 3-43 所示:

- 41 -
vL (t )
i
iC (t )
Vg v

图 3-43 MOSFET 关断时的 Boost 电路


对于电感电压和电容电流,有:
vL (t )  Vg  v(t )  Vg  V
v(t ) V (3-57)
iC (t )  i (t )  I
R R
根据电感的伏秒平衡,和电容的电荷平衡有:
Vg  DV  DIRon  0
V (3-58)
DI  0
R
根据式(3-58)可画出如图 3-44 所示的等效电路:
I DRon D :1

Vg V

图 3-44 Boost 变换器的等效电路


故 MOSFET 的功率损耗为:
PMOSFET  I 2 DRon  22.86W (3-59)

效率为:
DVI
  0.9943 (3-60)
Vg I

Solution (b) Buck-Boost:


根据图 3-40 所示的等效电路和式(3-52)可求出:
D  0.581347
I  23.88613A
PMOSFET  I 2 DRon  165.84W (3-61)
DVI
  0.9602
DVg I
故,针对此题的情况,Boost 变换器更适合。

- 42 -
Problem 3.10

利用 230V 交流电源整流后的电为 Buck 变换器供电,即变换器的直流输入


为 Vg  325V  20% ,控制电路可以自动调节占空比 D,以维持恒定的直流输出

电压 V  240V ,直流负载电流 I 的变化范围为 1A  I  10A ,MOSFET 的导通电


阻为 0.8 ,二极管的导通损耗可以使用 0.7V 的恒压源串联 0.2 的电阻进行建
模,其余的所有损耗可以忽略。
(a) 推导等效电路模型
(b) 对于给定的 Vg 和 I 的变化范围,求出占空比 D 的变化范围
(c) 变换器处于哪个运行点时(如 Vg 和 I 为多少时)变换器的损耗最大?此
运行点时的效率是多少?
Solution:
(a) 当 MOSFET 导通时,即 0  t  DTS 时,电路如图 3-45 所示:

Ron vL (t ) i (t )

Vg R v(t )

图 3-45 MOSFET 导通时的 Buck 电路


对于电感电压,有:
vL (t )  Vg  i(t ) Ron  v(t )  Vg  IRon  V (3-62)

当 MOSFET 关断时,即 DTS  t  TS 时,电路如图 3-46 所示:

vL (t ) i (t )

VD
Vg R v(t )
RD

图 3-46 MOSFET 关断时的 Buck 电路


对于电感电压,有:
vL (t )  VD  i(t ) RD  v(t )  VD  IRD  V (3-63)

根据电感的伏秒平衡,有:
DVg  DVD  DIRD  V  DIRon  0
V (3-64)
I
R

- 43 -
根据式(3-64),可画出等效电路如图 3-47 所示:
DVD DR DR
1: D I D on

Vg R V

图 3-47 Buck 变换器的等效电路


(b) 根据式(3-64)可求出占空比为:
VD  V  IRD
D (3-65)
VD  Vg  IRD  IRon

带入数值后:
VD  V  IRD 0.7  240  0.2 I 240.7  0.2 I
D   (3-66)
VD  Vg  IRD  IRon 0.7  Vg  0.2I  0.8I 0.7  Vg  0.6I

故,当负载电流最大,且输入电压最小时,占空比 D 最大,即:
240.7  0.2 10
Dmax   0.9529 (3-67)
0.7  325  80%  0.6 10
当负载电流最小,且输入电压最大时,占空比 D 最小,即:
240.7  0.2 1
Dmin   0.6175 (3-68)
0.7  325 120%  0.6 1
故占空比 D 的变化范围为:
0.6175  D  0.9529 (3-69)
(c) 效率的表达式为:
VI V
  (3-70)
DVg I DVg

由于输出电压 V 是恒定不变的,故当变换器的损耗最大时,效率最低,即
DVg 此时应为最大值。
而根据式(3-69)可知:
( DVg )max  0.9529  (325 120%)  371.631 (3-71)

故:
V 240
min    0.6458 (3-72)
( DVg )max 371.631

Problem 3.11

图 3-48 所示的 Cuk 变换器中,MOSFET 的导通电阻为 Ron,二极管的正向


压降为 VD,其他的所有损耗可以忽略。

- 44 -
(a) 推导变换器的等效电路模型。
(b) 推导变换器输出电压和效率的表达式。
(c) 忽略二极管的正向压降,即 VD  0 ,当 Ron / R  0.01,0.05 时,画出 V / Vg 随

着占空比 D 变化的图形。
(d) 忽略二极管的正向压降,即 VD  0 ,当 Ron / R  0.01,0.05 时,画出变换器

效率随着占空比 D 变化的图形。

L1 C1 L2

Vg Q1 D1 C2 R v

图 3-48 Cuk 变换器,Problem3.11


Solution:
(a) 当 MOSFET 导通时,即 0  t  DTS 时,电路如图 3-49 所示:

iL1 (t ) L1 C1 iC1 (t ) iL 2 (t ) L2
vL1 (t ) vL 2 (t )
vC1 (t )
Vg Ron C2 R v
iC 2 (t )

图 3-49 MOSFET 导通时的 Cuk 电路


对于电感电压和电容电流有:
vL1 (t )  Vg  iL1 (t ) Ron  iL 2 (t ) Ron  Vg  I L1Ron  I L 2 Ron
vL 2 (t )  v(t )  iL1 (t ) Ron  iL 2 (t ) Ron  vC1 (t )  V  I L1Ron  I L 2 Ron  VC1
iC1 (t )  iL 2 (t )   I L 2 (3-73)
v(t ) V
iC 2 (t )  iL 2 (t )   I L2 
R R

当 MOSFET 关断时,即 DTS  t  TS 时,电路如图 3-50 所示:

iL1 (t ) L1 C1 iC1 (t ) iL 2 (t ) L2
vL1 (t ) vL 2 (t )
vC1 (t )
Vg VD C2 R v
iC 2 (t )

图 3-50 MOSFET 关断时的 Cuk 电路


对于电感电压和电容电流有:

- 45 -
vL1 (t )  Vg  VD  vC1 (t )  Vg  VD  VC1
vL 2 (t )  v(t )  VD  V  VD
iC1 (t )  iL1 (t )  I L1 (3-74)
v(t ) V
iC 2 (t )  iL 2 (t )   I L2 
R R
根据电感的伏秒平衡和电容的电荷平衡,有:
Vg  DVD  DVC1  D( I L1  I L 2 ) Ron  0
V  DVC1  DVD  D( I L1  I L 2 ) Ron  0
DI L 2  DI L1 (3-75)
V
I L2  
R
将式(3-75)的第一个子式消去 IL2,将第二个子式消去 IL1,有:
Vg  DVD  DVC1  I L1 Ron  0
(3-76)
D2VD  DV  DI L 2 Ron  DDVC1  0
再将式(3-76)的第二个子式除以 D  ,有:
Vg  DVD  DVC1  I L1 Ron  0
D (3-77)
DVD  V  I L 2 Ron  DVC1  0
D
通过式(3-75)的第三子式建立直流变压器,故有如图 3-51 所示的等效电路:
D
I L1 Ron DVD I L2 Ron DVD
D : D D

Vg R V

图 3-51 Cuk 变换器的等效电路


(b) 将图 3-51 中 DC 变压器的副边等效至原边,如图 3-52 所示:

D2
D VD
I L1 Ron DVD Ron D
D

D2 D
Vg R V
D2 D

图 3-52 简化后的 Cuk 变换器等效电路


根据图 3-52 所示的等效电路,可求出输出电压的表达式为:
DDRVg  D2 RVD
V  (3-78)
D2 R  DRon

- 46 -
效率为:
DD2 RVg  D3 RVD
 (3-79)
DD2 RVg  D 2 RonVg

(c) 不考虑二极管的正向压降时,即 VD  0 ,当 Ron / R  0.01 时,V / Vg 的表达

式为:
V 100 DD
 (3-80)
Vg 100 D 2  199 D  100

当 Ron / R  0.01 时, V / Vg 如图 3-53 所示:

0
 0.625
 1.25
 1.875
1.0 D ( 100.0 D 100.0)
2  2.5
100.0 D  199.0 D 100.0
 3.125
 3.75
 4.375
5
0 0.125 0.25 0.375 0.5 0.625 0.75 0.875 1
D

图 3-53 Ron / R = 0.01 时的 V / Vg

当 Ron / R  0.05 时, V / Vg 的表达式为:

V 20 DD
 (3-81)
Vg 20 D 2  39 D  20

当 Ron / R  0.05 时, V / Vg 如图 3-54 所示:

- 47 -
0

 0.313

 0.625

 0.938
1.0 D ( 20.0 D 20.0)
2  1.25
20.0 D  39.0 D 20.0
 1.563

 1.875

 2.188

 2.5
0 0.125 0.25 0.375 0.5 0.625 0.75 0.875 1
D

图 3-54 Ron / R = 0.05 时的 V / Vg

(d) 不考虑二极管的正向压降时,即 VD  0 ,当 Ron / R  0.01 时,效率的表达

式为:

100 D2  200 D  100


 (3-82)
100 D2  199 D  100

当 Ron / R  0.01 时,效率如图 3-35 所示:

0.875

0.75

0.625
2
100.0 D  200.0 D 100.0
0.5
2
100.0 D  199.0 D 100.0
0.375

0.25

0.125

0
0 0.125 0.25 0.375 0.5 0.625 0.75 0.875 1
D

图 3-55 Ron / R = 0.01 时的效率

当 Ron / R  0.05 时,效率的表达式为:

20D2  40D  20
 (3-83)
20D2  39 D  20

当 Ron / R  0.05 时,效率如图 3-56 所示:

- 48 -
1

0.875

0.75

0.625
2
20.0 D  40.0 D 20.0
0.5
2
20.0 D  39.0 D 20.0
0.375

0.25

0.125

0
0 0.125 0.25 0.375 0.5 0.625 0.75 0.875 1
D

图 3-56 Ron / R = 0.05 时的效率

- 49 -
第4章 开关的实现方式

Problem 4.1

对于 Problem4.1 ~ 4.6,输入电压 Vg 为直流,极性如各图所示。请指定如何


用数量最少的二极管和晶体管实现半导体开关,使得变换器可以在占空比
0  D  1的范围内运行,开关的状态如图 4-1 所示那样变化。你可以认为电感电
流纹波和电容电压纹波都很小,对于每一个 Problem,请回答以下几个小问题:
开关
位置
2

1
0 DTS TS t

图 4-1 开关控制方法,Problem4.1~ 4.6


(a) 使用理想的 SPST 开关去实现,并定义各个开关的电压和电流参考方向。
(b) 以变换器的电感电流、电容电压、输入电压这三个量描述各个开关的导通
电流和阻断电压。
(c) 使用 Chapter2 中讲解的稳态分析方法求解电感电流和电容电压。
(d) 说明各个开关导通电流和阻断电压的极性,这些极性是否会随着占空比的
变化而变化?
(e) 说明各个开关如何用晶体管和二极管去实现?这些实现的方式用到了哪
种形式的开关?如单象限、电流双向两象限、电压双向两象限、四象限开关。
Solution:
电路如图 4-2 所示:
2 1

Vg

1 2
图 4-2 Problem4.1 电路
(a) 使用理想 SPST 开关实现,各个开关电压、电流的参考方向如图 4-3 所示:
i2 2 i1 1

v2 v1

Vg
i1 3 4 i2
v3 v4

图 4-3 Problem4.1 电路,理想 SPST 实现

- 50 -
(b) 定义电感电流和电容电压的参考方向,如图 4-4 所示:
i2 2 i1 1

v2 v1

Vg iL vC
i1 3 4 i2
v3 v4

图 4-4 Problem4.1,定义电感电流和电容电压方向
开关 1 的导通电流和阻断电压为:
ion1  iL
(4-1)
voff 1  Vg  vC

开关 2 的导通电流和阻断电压为:
ion2 = iL
(4-2)
voff2 = Vg + vC

开关 3 的导通电流和阻断电压为:
ion3 = iL
(4-3)
voff3 = Vg

开关 4 的导通电流和阻断电压为:
ion4 = iL
(4-4)
voff4 = Vg

(c) 当开关 1 和开关 3 导通时,电路如图 4-5 所示:

vL
Vg iL vC
i1
iC

图 4-5 Problem4.1,开关 1 和开关 3 导通时的简化电路


故,有:
vL  vC  Vg
vC (4-5)
iC  iL 
R
当开关 2 和开关 4 导通时,电路如图 4-6 所示:

- 51 -
i2

vL
Vg iL vC
iC

图 4-6 Problem4.1,开关 2 导通时的简化电路


故,有:
vL  Vg
vC (4-6)
iC  
R
根据电感的伏秒平衡和电容的电荷平衡有:
D(vC  Vg )  D(Vg )  0
vC v (4-7)
D(iL  )  D( C )  0
R R
由上式可推出:
vC
iL 
DR
(4-8)
(1  2 D)Vg
vC 
D
(d) 根据式(4-8)和式(4-1),开关 1 的导通电流和阻断电压为:
v
ion1 = iL = C
DR
(4-9)
DVg
voff1 = Vg  vC = 
D
故根据上式可知,开关 1 的导通电流方向始终为正,电流的方向不随占空比
D 的变化而改变。开关 1 的阻断电压极性始终为负,且阻断电压的极性不随占空
比 D 的变化而改变。
根据式(4-8)和式(4-2)可知,开关 2 的导通电流和阻断电压为:
v
ion2 = iL = C
DR
(4-10)
DVg
voff2 = Vg + vC =
D
故根据上式可知,开关 2 的导通电流方向始终为正,电流方向不随占空比 D
的变化而改变。开关 2 的阻断电压极性始终为正,且阻断电压的极性不随占空比
D 的变化而改变。
根据式(4-8)和式(4-3)可知,开关 3 的导通电流和阻断电压为:
vC
ion3 = iL =
DR (4-11)
voff3 = Vg

- 52 -
故根据上式可知,开关 3 的导通电流方向始终为正,电流方向不随占空比 D
的变化而改变。开关 3 的阻断电压极性始终为负,且阻断电压的极性不随占空比
D 的变化而改变。
根据式(4-8)和式(4-4)可知,开关 4 的导通电流和阻断电压为:
vC
ion4 = iL =
DR (4-12)
voff4 = Vg

故根据上式可知,开关 4 的导通电流方向始终为正,电流方向不随占空比 D
的变化而改变。开关 4 的阻断电压极性始终为正,且阻断电压的极性不随占空比
D 的变化而改变。
(e) 当用晶体管和二极管去实现电路时,电路如图 4-7 所示,所使用的开关形
式为单象限开关。
v2 v1
i2 i1
Vg iL vC
v3 v4 i2
i1

图 4-7 使用晶体管和二极管实现 Problem4.1 电路

Problem 4.2

电路如图 4-8 所示,所需回答的问题如 Problem 4.1 所述。

1 2
Vg
2 1

图 4-8 Problem4.2
Solution:
(a) 使用理想 SPST 开关实现,各个开关电压、电流的参考方向如图 4-9 所示:
iL

1 v1 v2 2
i1 i2
Vg i2 i1
v4 v3
4 3

图 4-9 Problem4.2 电路,理想 SPST 实现


(b) 定义电感电流和电容电压的参考方向,如图 4-10 所示:

- 53 -
iL
vL
1 v1 v2 2
iC
i1 i2
Vg i2 i1
v4 vC v3
4 3

图 4-10 Problem4.2,定义电感电流和电容电压方向
开关 1 的导通电流和阻断电压为:
ion1 = iL
(4-13)
voff1 = vC

开关 2 的导通电流和阻断电压为:
ion2 = iL
(4-14)
voff2 = vC

开关 3 的导通电流和阻断电压为:
ion3 = iL
(4-15)
voff3 = vC

开关 4 的导通电流和阻断电压为:
ion4 = iL
(4-16)
voff4 = vC

(c) 当开关 1 和开关 3 导通时,电路如图 4-11 所示:


iL
vL
iC
i1
Vg
vC

图 4-11 Problem4.2,开关 1 和开关 3 导通时的简化电路


故有:
vL  Vg  vC
vC (4-17)
iC  iL 
R
当开关 2 和开关 4 导通时,电路如图 4-12 所示:

- 54 -
iL
vL
1
iC
i2
Vg i2
vC

图 4-12 Problem4.2,开关 2 和开关 4 导通时的简化电路


故有:
vL  Vg  vC
vC (4-18)
iC  iL 
R
根据电感的伏秒平衡和电容的电荷平衡有:
D(Vg  vC )  D(Vg  vC )  0
vC v (4-19)
D(iL  )  D(iL  C )  0
R R
由上式可推出:
Vg
vC 
2D 1
(4-20)
Vg
iL 
(2 D  1) 2 R

(d) 根据式(4-20)和式(4-13),可知开关 1 的导通电流和阻断电压为:


Vg
ion1  iL 
(2 D  1) 2 R
(4-21)
Vg
voff 1  vC  
2D  1
故根据上式可知,开关 1 的导通电流方向始终为正,电流的方向不随占空比
D 的变化而改变。开关 1 的阻断电压极性可正可负,阻断电压的极性随着占空比
D 的变化而改变。
根据式(4-20)和式(4-14),可知开关 2 的导通电流和阻断电压为:
Vg
ion 2  iL 
(2 D  1) 2 R
(4-22)
Vg
voff 2  vC 
2D 1
故根据上式可知,开关 2 的导通电流方向始终为正,电流的方向不随占空比
D 的变化而改变。开关 2 的阻断电压极性可正可负,阻断电压的极性随着占空比
D 的变化而改变。

- 55 -
根据式(4-20)和式(4-15),可知开关 3 的导通电流和阻断电压为:
Vg
ion 3  iL 
(2 D  1) 2 R
(4-23)
Vg
voff 3  vC  
2D 1
故根据上式可知,开关 3 的导通电流方向始终为正,电流的方向不随占空比
D 的变化而改变。开关 3 的阻断电压极性可正可负,阻断电压的极性随着占空比
D 的变化而改变。
根据式(4-20)和式(4-16),可知开关 4 的导通电流和阻断电压为:
Vg
ion 4  iL 
(2 D  1) 2 R
(4-24)
Vg
voff 4  vC 
2D 1
故根据上式可知,开关 4 的导通电流方向始终为正,电流的方向不随占空比
D 的变化而改变。开关 4 的阻断电压极性可正可负,阻断电压的极性随着占空比
D 的变化而改变。
(e) 当用晶体管和二极管去实现电路时,电路如图 4-13 所示,所使用的开关
形式为电流单向、电压双向的二象限开关。
iL
vL

1 2
iC
i2
Vg
vC

4 3

图 4-13 使用晶体管和二极管实现 Problem4.2 电路

Problem 4.3

电路如图 4-14 所示,所需回答的问题如 Problem 4.1 所述。


1

2
Vg

图 4-14 Problem4.3

- 56 -
Solution:
(a) 使用理想 SPST 开关实现,各个开关电压、电流的参考方向如图 4-15 所
示:
i1 1
v1 i2
Vg v2 2

图 4-15 Problem4.3 电路,理想 SPST 实现


(b) 定义电感电流和电容电压的参考方向,如图 4-16 所示:
vC1 vL 2
i1 1 iC1
v1 i2 iL 2
Vg vL1 v2 2 vC 2
iC 2
iL1

图 4-16 Problem4.3,定义电感电流和电容电压方向
开关 1 的导通电流和阻断电压为:
ion1  iL1  iL 2
(4-25)
voff 1  Vg  vC1

开关 2 的导通电流和阻断电压为:
ion 2  iL 2  iL1
(4-26)
voff 2  Vg  vC1

(c) 当开关 1 导通时,电路如图 4-17 所示:


vC1 vL 2
i1 iC1
iL 2
Vg vL1 vC 2
iC 2
iL1

图 4-17 Problem4.3,开关 1 导通时的简化电路


故有:
vL1  Vg
vL 2  vC 2  Vg  vC1 (4-27)
iC1  iL 2
当开关 2 导通时,电路如图 4-18 所示:

- 57 -
vC1 vL 2
iC1
iL 2
Vg vL1 vC 2
iC 2
iL1

图 4-18 Problem4.3,开关 2 导通时的简化电路

故有:
vL1  vC1
vL 2  vC 2 (4-28)
iC1  iL1

根据电感的伏秒平衡和电容的电荷平衡有:
D(Vg )  D(vC1 )  0
D(vC 2  Vg  vC1 )  D(vC 2 )  0 (4-29)
D(iL 2 )  D(iL1 )  0
由上式可推出:
DVg
vC1 
D 1
DVg
vC 2 
D 1 (4-30)
DiL 2   DiL1
vC 2
iL 2 
R
(d) 根据式(4-30)和式(4-25)可推出,开关 1 的导通电流和阻断电压为:
DVg
ion1  iL1  iL 2 
(1  D) 2 R
(4-31)
V
voff 1  Vg  vC1  g
D
故根据上式可知,开关 1 的导通电流方向始终为正,电流的方向不随占空比
D 的变化而改变。开关 1 的阻断电压极性始终为正,阻断电压的极性不随着占空
比 D 的变化而改变。
根据式(4-30)和式(4-26)可推出,开关 2 的导通电流和阻断电压为:
DVg
ion 2  iL 2  iL1  
(1  D) 2 R
(4-32)
V
voff 2  Vg  vC1  g
D

- 58 -
故根据上式可知,开关 2 的导通电流方向始终为负,电流的方向不随占空比
D 的变化而改变。开关 1 的阻断电压极性始终为正,阻断电压的极性不随着占空
比 D 的变化而改变。
(e) 当用晶体管和二极管去实现电路时,电路如图 4-19 所示,所使用的开关
形式为单象限开关。

Vg

图 4-19 使用晶体管和二极管实现 Problem4.3 电路

Problem 4.4

电路如图 4-20 所示,所需回答的问题如 Problem 4.1 所述。

2 1

Vg 1 2

图 4-20 Problem4.4 电路
Solution:
(a) 使用理想 SPST 开关实现,各个开关电压、电流的参考方向如图 4-21 所
示:
i1
2 i2 1
v2 v1

i3 i4
Vg 3 4
v3 v4

图 4-21 Problem4.4 电路,理想 SPST 实现


(b) 定义电感电流和电容电压的参考方向,如图 4-22 所示:

i1
2 i2 1
v2 v1

iC1 vC1 iL 2
iL1
vL1 i3 i4 vL 2
Vg 3 4 vC 2
v3 v4

图 4-22 Problem4.4,定义电感电流和电容电压方向

- 59 -
开关 1 的导通电流和阻断电压为:
ion1  iL 2
(4-33)
voff 1  vC1

开关 2 的导通电流和阻断电压为:
ion 2  iL 2
(4-34)
voff 2  vC1

开关 3 的导通电流和阻断电压为:
ion 3  iL1  iL 2
(4-35)
voff 3  vC1

开关 4 的导通电流和阻断电压为:
ion 4  iL 2  iL1
(4-36)
voff 4  vC1

(c) 当开关 1 和开关 3 导通时,电路如图 4-23 所示:

iC1 vC1 iL 2
iL1
vL1 vL 2
Vg vC 2

图 4-23 Problem4.4,开关 1 和开关 3 导通时的简化电路


故有:
vL1  Vg
vL 2  vC1  vC 2
iC1  iL 2 (4-37)
vC 2
iL 2 
R
当开关 2 和开关 4 导通时,电路如图 4-24 所示:

iL1 iL 2

vL1 iC1 vL 2
Vg vC1 vC 2

图 4-24 Problem4.4,开关 2 和开关 4 导通时的简化电路


故有:

- 60 -
vL1  Vg  vC1
vL 2  vC1  vC 2
iC1  iL1  iL 2 (4-38)
vC 2
iL 2 
R
根据电感的伏秒平衡和电容的电荷平衡有:
D(Vg )  D(Vg  vC1 )  0
D(vC1  vC 2 )  D(vC1  vC 2 )  0 (4-39)
D(iL 2 )  D(iL1  iL 2 )  0
由上式可推出:
Vg
vC1 
D
(2 D  1)Vg
vC 2  
D
(4-40)
(1  2 D) 2 Vg
iL1 
D 2 R
(1  2 D)Vg
iL 2 
DR
(d) 根据式(4-40)和式(4-33)可知,开关 1 的导通电流和阻断电压为:
(1  2 D)Vg
ion1  iL 2  
DR
(4-41)
V
voff 1  vC1  g
D
故根据上式可知,开关 1 的导通电流方向可正可负,电流的方向随着占空比
D 的变化而改变。开关 1 的阻断电压极性始终为正,阻断电压的极性不随着占空
比 D 的变化而改变。
根据式(4-40)和式(4-34)可知,开关 2 的导通电流和阻断电压为:
(1  2 D)Vg
ion 2  iL 2 
DR
(4-42)
V
voff 2  vC1  g
D
故根据上式可知,开关 2 的导通电流方向可正可负,电流的方向随着占空比
D 的变化而改变。开关 2 的阻断电压极性始终为正,阻断电压的极性不随着占空
比 D 的变化而改变。
根据式(4-40)和式(4-35)可知,开关 3 的导通电流和阻断电压为:

- 61 -
(2 D  1) DVg
ion 3  iL1  iL 2 
D2 R
(4-43)
V
voff 3  vC1  g
D
故根据上式可知,开关 3 的导通电流方向可正可负,电流的方向随着占空比
D 的变化而改变。开关 3 的阻断电压极性始终为正,阻断电压的极性不随着占空
比 D 的变化而改变。
根据式(4-40)和式(4-36)可知,开关 4 的导通电流和阻断电压为:
(1  2 D) DVg
ion 4  iL 2  iL1 
D2 R
(4-44)
V
voff 4  vC1  g
D
故根据上式可知,开关 4 的导通电流方向可正可负,电流的方向随着占空比
D 的变化而改变。开关 4 的阻断电压极性始终为正,阻断电压的极性不随着占空
比 D 的变化而改变。
(e) 当用晶体管和二极管去实现电路时,电路如图 4-25 所示,所使用的开关
形式为电流双向、电压单向双象限开关。

iL1 iL 2

vL1 vL 2
Vg vC 2

图 4-25 使用晶体管和二极管实现 Problem4.4 电路

Problem 4.5

电路如图 4-26 所示,所需回答的问题如 Problem 4.1 所述。

1 2
Vg

2 1

图 4-26 Problem4.5 电路

- 62 -
Solution:
(a) 使用理想 SPST 开关实现,各个开关电压、电流的参考方向如图 4-27 所
示:

i1 i2
1 2
v1 v2
Vg

v4 v3
4 i4 i3 3

图 4-27 Problem4.5 电路,理想 SPST 实现


(b) 定义电感电流方向和电容电压参考方向,如图 4-28 所示:
iC1 vC1
iL1
vL1 i1 i2 iL 3
1 2
v1 v2 vL 3
Vg

v4 v3 vC 3
4 i4 i3 3
iL 2
vL 2 iC 2 vC 2
图 4-28 Problem4.5,定义电感电流和电容电压方向
开关 1 的导通电流和阻断电压为:
ion1  iL1  iL 3
(4-45)
voff 1  vC1

开关 2 的导通电流和阻断电压为:
ion 2  iL 3  iL1
(4-46)
voff 2  vC1

开关 3 的导通电流和阻断电压为:
ion 3  iL 2  iL 3
(4-47)
voff 3  vC 2

开关 4 的导通电流和阻断电压为:
ion 4  iL 2  iL 3
(4-48)
voff 4  vC 2

- 63 -
(c) 当开关 1 和开关 3 导通时,电路如图 4-29 所示:
iC1 vC1
iL1
vL1 iL 3
vL 3
Vg

vC 3

iL 2
vL 2 iC 2 vC 2
图 4-29 Problem4.5,开关 1 和开关 3 导通时的简化电路
故有:
vL1  Vg
vL 2  Vg  vC 2
vL 3  vC1  vC 3 (4-49)
iC1  iL 3
iC 2  iL 2
当开关 2 和开关 4 导通时,电路如图 4-30 所示:
iC1 vC1
iL1
vL1 iL 3
vL 3
Vg

vC 3

iL 2
vL 2 iC 2 vC 2
图 4-30 Problem4.5,开关 2 和开关 4 导通时的简化电路
故有:
vL1  Vg  vC1
vL 2  Vg
vL 3  vC 2  vC 3 (4-50)
iC1  iL1
iC 2  iL 3
根据电感的伏秒平衡和电容的电荷平衡有:

- 64 -
D(Vg )  D(Vg  vC1 )  0
D(Vg  vC 2 )  D(Vg )  0
D(vC1  vC 3 )  D(vC 2  vC 3 )  0 (4-51)
D(iL 3 )  D(iL1 )  0
D(iL 2 )  D(iL 3 )  0
由上式可推出:
Vg
vC1 
D
Vg
vC 2 
D
(1  2 D)Vg
vC 3 
DD (4-52)
(1  2 D)Vg
iL1  
D2 R
(1  2 D)Vg
iL 2 
D2 R
(1  2 D)Vg
iL 3 
DDR
(d) 根据式(4-52)和式(4-45)可知,开关 1 的导通电流和阻断电压为:
(2 D  1)Vg
ion1  iL1  iL 3 
DRD2
(4-53)
V
voff 1  vC1  g
D
故根据上式可知,开关 1 的导通电流方向可正可负,电流的方向随着占空比
D 的变化而改变。开关 1 的阻断电压极性始终为正,阻断电压的极性不随着占空
比 D 的变化而改变。
根据式(4-52)和式(4-46)可知,开关 2 的导通电流和阻断电压为:
(2 D  1)Vg
ion 2  iL 3  iL1  
DRD2
(4-54)
V
voff 2  vC1  g
D
故根据上式可知,开关 2 的导通电流方向可正可负,电流的方向随着占空比
D 的变化而改变。开关 2 的阻断电压极性始终为正,阻断电压的极性不随着占空
比 D 的变化而改变。
根据式(4-52)和式(4-47)可知,开关 3 的导通电流和阻断电压为:

- 65 -
(2 D  1)Vg
ion 3  iL 2  iL 3 
D 2 DR
(4-55)
Vg
voff 3  vC 2 
D
故根据上式可知,开关 3 的导通电流方向可正可负,电流的方向随着占空比
D 的变化而改变。开关 3 的阻断电压极性始终为正,阻断电压的极性不随着占空
比 D 的变化而改变。
根据式(4-52)和式(4-48)可知,开关 4 的导通电流和阻断电压为:
(2 D  1)Vg
ion 4  iL 2  iL 3  
D 2 DR
(4-56)
Vg
voff 4  vC 2 
D
故根据上式可知,开关 4 的导通电流方向可正可负,电流的方向随着占空比
D 的变化而改变。开关 4 的阻断电压极性始终为正,阻断电压的极性不随着占空
比 D 的变化而改变。
(e) 当用晶体管和二极管去实现电路时,电路如图 4-31 所示,所使用的开关
形式为电流双向、电压单向双象限开关。
iC1 vC1
iL1
vL1 iL 3
vL 3
Vg

vC 3

iL 2
vL 2 iC 2 vC 2
图 4-31 使用晶体管和二极管实现 Problem4.5 电路

Problem 4.6

电路如图 4-32 所示,所需回答的问题如 Problem 4.1 所述。


1
2 2
Vg
1

图 4-32 Problem4.6 电路
Solution:

- 66 -
(a) 使用理想 SPST 开关实现,各个开关电压、电流的参考方向如图 4-33 所
示:

v3
3 i3
2 4
v2
i2 v4
Vg
1 i4
v1
i1

图 4-33 Problem4.6 电路,理想 SPST 实现


(b) 定义电感电流和电容电压参考方向,如图 4-34 所示:

iL1 vL1 v3 iL 2 vL 2
3 i3
2 4
v2 vC1
i2 iC1 v4
Vg vC 2
1 i4
v1
i1

图 4-34 Problem4.6,定义电感电流和电容电压方向
开关 1 的导通电流和阻断电压为:
ion1  iL1  iL 2
(4-57)
voff 1  Vg

开关 2 的导通电流和阻断电压为:
ion2 = iL1
(4-58)
voff2 = Vg

开关 3 的导通电流和阻断电压为:
ion 3  iL 2
(4-59)
voff 3  vC1  Vg

开关 4 的导通电流和阻断电压为:
ion 4  iL 2
(4-60)
voff 4  vC1

(c) 当开关 1 和开关 3 导通时,电路如图 4-35 所示:

- 67 -
iL1 vL1 iL 2 vL 2

Vg vC1 vC 2
iC1

图 4-35 Problem4.6,开关 1 和开关 3 导通时的简化电路


故有:
vL1  Vg  vC1
vL 2  vC1  vC 2
iC1  iL1  iL 2 (4-61)
vC 2
iL 2 
R
当开关 2 和开关 4 导通时,电路如图 4-36 所示:

iL1 vL1 iL 2 vL 2

vC1
iC1
Vg vC 2

图 4-36 Problem4.6,开关 2 和开关 4 导通时的简化电路


故有:
vL1  vC1
vL 2  vC 2
iC1  iL1 (4-62)
vC 2
iL 2 
R
根据电感的伏秒平衡和电容的电荷平衡有:
D(Vg  vC1 )  D(vC1 )  0
D(vC1  vC 2 )  D(vC 2 )  0 (4-63)
D(iL1  iL 2 )  D(iL1 )  0
由上式可推得:

- 68 -
vC1  DVg
vC 2  D 2Vg
D 3Vg (4-64)
iL1 
R
D 2Vg
iL 2 
R
(d) 根据式(4-64)和式(4-57)可知,开关 1 的导通电流和阻断电压为:
D 2 DVg
ion1  iL1  iL 2  
R (4-65)
voff 1  Vg

故根据上式可知,开关 1 的导通电流方向为负,电流的方向不随占空比 D
的变化而改变。开关 1 的阻断电压极性始终为正,阻断电压的极性不随占空比 D
的变化而改变。
根据式(4-64)和式(4-58)可知,开关 2 的导通电流和阻断电压为:
D3Vg
ion 2  iL1  
R (4-66)
voff 2  Vg

故根据上式可知,开关 2 的导通电流方向为负,电流的方向不随占空比 D
的变化而改变。开关 2 的阻断电压极性始终为正,阻断电压的极性不随占空比 D
的变化而改变。
根据式(4-64)和式(4-59)可知,开关 3 的导通电流和阻断电压为:
D 2Vg
ion3  iL 2 
R (4-67)
voff 3  vC1  Vg  ( D  1)Vg

故根据上式可知,开关 3 的导通电流方向为正,电流的方向不随占空比 D
的变化而改变。开关 3 的阻断电压极性始终为正,阻断电压的极性不随占空比 D
的变化而改变。
根据式(4-64)和式(4-60)可知,开关 4 的导通电流和阻断电压为:
D 2Vg
ion 4  iL 2  
R (4-68)
voff 4  vC1  DVg

故根据上式可知,开关 4 的导通电流方向为负,电流的方向不随占空比 D
的变化而改变。开关 4 的阻断电压极性始终为正,阻断电压的极性不随占空比 D
的变化而改变。
(e) 当用晶体管和二极管去实现电路时,电路如图 4-37 所示,所使用的开关
形式为单象限开关。

- 69 -
Vg

图 4-37 使用晶体管和二极管实现 Problem4.4 电路

Problem 4.7

对于 Buck 变换器中的 IGBT 和硅二极管,其 IGBT 的波形如图 4-38 所示。


变换器的工作条件为:输入电压 Vg = 400V,输出电压 V = 200V,负载电流 I = 10A。

vCE iC
vCE (t ) vCE (t )
400V 40A
300V 30A
200V 20A
100V 10A iC (t )
iC (t )
0V 0A
0 1 2 t , μs

图 4-38 IGBT 的电压电流波形,Problem4.7


(a) 估算开关转换过程中的总能量损耗。
(b) IGBT 的正向压降为 2.5V,二极管的正向压降为 1.5V,导致传导损耗产生
的其他原因可以忽略不计,估算半导体的传导损耗。
(c) 画出变换器的效率关于开关频率 1kHz  f S  100kHz 的曲线。

Solution:
(a) 对于 IGBT 由导通转换至关断的过程中,当 vCE (t ) 上升到 Vg 之前,二极

管的电流几乎为 0,故此阶段的二极管的损耗几乎为 0;当 vCE (t ) 达到 Vg ,并且

IGBT 的集电极电流 iC (t ) 下降至 0 之前,此阶段二极管已正偏,故二极管的电

压几乎为 0,故此阶段的二极管的损耗几乎为 0。综上,对于 IGBT 的关断过程,


二极管的损耗可忽略不计。
对于 IGBT 由关断转换至导通的过程中,由图 4-38 可看出,二极管为“snappy”
二极管,故对于 IGBT 的导通过程,二极管的损耗也可忽略不计(可忽略的原因
详见书中 4.3.2 节中的 Fig. 4.51)。
综上,开关转换的过程中的开关管损耗只计算 IGBT 的开关损耗即可,二极

- 70 -
管的开关损耗可忽略不计。
IGBT 导通时的开关损耗为:
WD   vCE (t )  iC (t )dt
1 1 1 1 1 1 
 400   106  35  10     106  (100  400)   10 6 100 
2 4 2 2 2 2 
 3.25 103 J
(4-69)
IGBT 关断时的开关损耗为:
Woff   vCE (t )  iC (t )dt
1 1 1 1 1 3 
 10    10-6  400  400     10-6 (10  5)  10-6  5
2 2 2 2 2 2 
 4 10-3 J
(4-70)
故,开关转换过程中的开关管总损耗为:
Wsw  WD  Woff  7.25 10-3 J (4-71)

(b) IGBT 的传导损耗功率为:


Pon- IGBT  VF - IGBT  iC (t )  2.5 10  25W (4-72)

二极管的传导损耗功率为:
Pon- DIODE  VF - DIODE  iC (t )  1.5 10  15W (4-73)

(c) 当 Buck 变换器的输入为 400V,输出为 200V 时,故其占空比 D = 0.5,即


对于任意一个开关周期 TS 来说,IGBT 和二极管的导通时间各占一半 TS。
故变换器的效率表达式为:
Pout 200 100
  (4-74)
Pout  Plost 200 100  Wsw  f s  0.5  Pon  IGBT  0.5  Pon  DIODE

效率关于开关频率的曲线如图 4-39 所示:

- 71 -
1

0.995

0.99

0.985

200 100
0.98
200 100 Wsw fs 0.5 Pon_IGBT  0.5 Pon_DIODE

0.975

0.97

0.965

0.96
3 4 4 4 4 4 4 4 5
110 1.33710 2.57510 3.81310 5.0510 6.28710 7.52510 8.76210 110
fs

图 4-39 效率关于开关频率曲线

Problem 4.8

为了给电池进行充放电,使用两个 MOSFET 作为电流双向两象限开关应用


在 DCDC Buck 变换器中。此变换器用来在 16V 电池和 28V 主功率 BUS 之间进
行能量交互,电池的最大电流为 40A,MOSFET 具有 35mΩ 的导通电阻,MOSFET
的体二极管的正向压降为 1.0V,恢复电荷 Qr 为 25 μC ,并且在此电路中,二极

管的反向恢复时间 t r 为 200ns ,你可以认为此问题中的所有二极管都具有“snappy”

的反向恢复特性,并且二极管的存储电荷是开关损耗的主要原因。除了半导体的
传导损耗和由二极管存储电荷引起的开关损耗之外,你可以忽略其他所有损耗。
利用了 MOSFET 体二极管的电流双向两象限开关如图 4-40 所示。

图 4-40 利用 MOSFET 体二极管作为电流双向两象限开关


(a) 当电池以最大电流进行充电,并且开关频率为 100kHz 时,估算开关损耗、
传导损耗和变换器效率。
当使用如图 4-41 所示的额外二极管时,这些额外二极管的正向压降为 1.0V,
恢复电荷为 5 μC ,反向恢复时间 t r 为 40ns 。

- 72 -
图 4-41 使用外部二极管作为电流双向两象限开关
(b) 重复 Part(a)的分析。
(c) 开关频率处于哪个范围时,额外的二极管提升了变换器的效率?
Solution:
(a) 当电池以最大电流进行充电时,电路如图 4-42 所示:

40A

Vg 28V 16V

图 4-42 以最大电流为电池充电时
由于二极管具有“snappy”反向恢复特性,故可以忽略二极管的开关损耗。
并且二极管存储电荷只影响的是 MOSFET 导通时的开关损耗,由于只考虑二极
管存储电荷导致的开关损耗,故根据题意,可忽略 MOSFET 关断时的开关损耗。
对于 MOSFET 由关断转换至导通的过程中,二极管存储电荷导致的开关损
耗为:
WD   vCE (t )  iC (t )dt   Vg  [iL  iB (t )]dt  Vg iLtr  Vg Qr
 9.24 104 J (4-75)

根据题中所述的条件,MOSFET 的占空比 D 为:
Vout 16
D   0.5714 (4-76)
Vg 28

当 MOSFET 导通时,其传导损耗为:
PonMOS  iL 2 Ron  402  35 103  56W (4-77)

当 MOSFET 关断时,另一个 MOSFET 的体二极管的传导损耗为:


Pon DIODE  VF iL  1.0  40  40W (4-78)

变换器的效率为:

- 73 -
Pout

Pout  Plost
16  40
 4
(4-79)
16  40  9.24 10 100 103  0.5714  56  (1  0.5714)  40
 0.819
(b) 使用外部二极管作为电流双向两象限开关时,电路如图 4-43 所示:

40A

Vg 28V 16V

图 4-43 使用外部二极管时的电路
对于 MOSFET 由关断转换至导通的过程中,二极管存储电荷导致的开关损
耗为:
WD   vCE (t )  iC (t )dt   Vg  [iL  iB (t )]dt  Vg iLtr  Vg Qr
(4-80)
 1.848 104 J
当 MOSFET 导通时,各半导体器件的传导损耗为:
PonMOS  iL 2 Ron  VF iL  402  35 103  1.0  40  96W (4-81)

当 MOSFET 关断时,各半导体器件的传导损耗为:
Pon DIODE  VF iL  1.0  40  40W (4-82)

变换器的效率为:
Pout 16  40
  4
Pout  Plost 16  40  1.848 10 100 103  0.5714  96  (1  0.5714)  40
 0.876
(4-83)
(c) 当额外的二极管提升了变换器的效率时,即:
16  40
4

16  40  9.24 10  f S  0.5714  56  (1  0.5714)  40
(4-84)
16  40
4
16  40  1.848 10  f S  0.5714  96  (1  0.5714)  40
可推出:
f S  30.919kHz (4-85)

- 74 -
Problem 4.9

一个以开关频率为 100kHz 运行的开关变换器,变换器的波形在开关周期结


束之前,展现出了由晶体管关断发起的衰减缓慢但最终趋于 0 的正弦阻尼振铃现
象。这种振铃现象出现在由电路中的寄生电感、寄生电容形成的串联谐振电路。
振铃现象的频率为 5MHz。在正弦振铃的第一个周期中,交流电感电流达到了
0.5A 的峰值,交流电容电压达到了 200V 的峰值,求解以下变量:
(a) 总的寄生电感值和总的寄生电容值。
(b) 每个开关周期中,由振铃现象导致的能量损失。
(c) 这种振铃现象导致的开关损耗(功率)。
(d) 推导由振铃导致的开关损耗的通用公式,这种公式应为开关频率,振铃频
率,第一个振铃周期中电压和电流峰值的函数。
Solution:
(a) 当发生振铃现象时,电路模型如图 4-44 所示:
i R L

Vg C

图 4-44 发生振铃时的电路模型
振铃发生时,寄生电感和寄生电容发生谐振,即:
1
L  (4-86)
C
根据电感电流和电容电压的峰值有:
du (t )
iL (t )  iC (t )  C C (4-87)
dt
假设:
uC (t )  200sin t (4-88)

根据电感电流的峰值,并将式(4-88)带入(4-87)可得:
0.5cos t  200C cos t (4-89)
由上式可推得:
0.5 0.5
C   79.6pF (4-90)
200 200  2    5 106
将式(4-90)带入到式(4-86)可得:
1
L  2  12.7μH (4-91)
C
(b) 对于一个开关周期,由振铃现象导致的能量损失为:
1 1 1
Wlost  LI m 2  CU m 2  12.7 106  0.52  1.59 106 J (4-92)
2 2 2
(c) 由振铃现象导致的开关损耗为:

- 75 -
Plost  Wlost  f S  1.59 106 100 103  0.159W (4-93)

(d) 记开关频率为 f S 、振铃频率为 f r 、第一个振铃周期中电容交流电压峰值

为 U m 、第一个振铃周期中电感交流电流峰值为 I m 。

根据振铃的谐振频率有:
1 1 1
L   LC  2  (4-94)
C   2    fr 
2

根据寄生电感存储的能量峰值等于寄生电容存储的能量峰值,有:
2
1 1 L U 
LI m 2  CU m 2    m  (4-95)
2 2 C  Im 
可推出寄生电感的值为:
Um
L (4-96)
2 I m f r

故对于一个开关周期来说,由振铃现象导致的能量损耗为:
1 U I
Wlost  LI m 2  m m (4-97)
2 4 f r

故由振铃现象导致的开关损耗为:
Um Im fS
Plost  Wlost f S  (4-98)
4 f r

- 76 -
第5章 断续模式(DCM)

Problem 5.1

图 5-1 所示的 Buck-Boost 变换器中的元器件均为理想器件,即所有的损耗


均可忽略,对于(a)和(b)小问,你得到的答案应与书中的 Table 5.2 一致。

Q1 D1
i (t )
Vg L C R V

图 5-1 Buck-Boost 变换器

(a) 证明当 K  K crit 时,变换器运行于断续模式,并推导 K 和 K crit 的表达式。

(b) 推导 Buck-Boost 变换器在断续模式下的直流电压转换比率 V / Vg 。

(c) 当 K  0.1时,画出 V / Vg 关于占空比 0  D  1的曲线。

(d) 当 K  0.1、 D  0.3 时,画出电感电压和电感电流的波形,并标注其静态


特性。
(e) 当变换器处于空载时,即 ( R  ) 时,输出电压 V 如何变化,试解释。

Solution:
(a) 当变换器运行于 CCM 模式时,并且 0  t  DTS 时,电路如图 5-2 所示:

i (t ) iC
Vg vL L C R V

图 5-2 CCM 模式,开关管导通时


故有:
vL  Vg
V (5-1)
iC  
R

当 DTS  t  TS 时,电路如图 5-3 所示:

- 77 -
D1
i (t ) iC
Vg vL L C R V

图 5-3 CCM 模式,开关管关断时


故有:
vL  V
V (5-2)
iC   I 
R
由式(5-1)和式(5-2)可推出电感电流的直流分量为:
DVg
I (5-3)
D2 R
电感电压和电流波形如图 5-4 所示:
vL (t )

Vg

0 DTS TS t

iL (t ) V
2iL

0 DTS TS t
图 5-4 CCM 模式下,电感电压电流波形
由上图可推出电感电流纹波大小为:
DTSVg
iL  (5-4)
2L
故当电感电流的纹波大小大于电感电流的直流分量时,DCM 模式便会发生,
即有:
I  iL for DCM
DVg DTSVg 2L
   D 2
D R
2
2L RTS
(5-5)
2L
K
RTS
K crit  D2

- 78 -
(b) 当 0  t  D1TS 时,电路如图 5-5 所示:

i (t ) iC
Vg vL L C R V

图 5-5 DCM 模式, 0  t  D1TS 时

对于电容电压,可以使用小纹波近似,而对于 DCM 下电感电流,则不可以


使用小纹波近似。
故有:
vL  Vg
V (5-6)
iC  
R

当 D1TS  t  ( D1  D2 )TS 时,电路如图 5-6 所示:

D1
i (t ) iC
Vg vL L C R V

图 5-6 DCM 模式, D1TS  t  ( D1  D2 )TS 时

故有:
vL  V
V (5-7)
iC  i (t ) 
R

当 ( D1  D2 )TS  t  TS 时,电路如图 5-7 所示:

i (t ) iC
Vg vL L C R V

图 5-7 DCM 模式, ( D1  D2 )TS  t  TS 时

故有:

- 79 -
vL  0, i (t )  0
V (5-8)
iC  
R
根据电感的伏秒平衡有:

1 g  D2V  D3  0  0
DV (5-9)

由上式可推出:
V D
 1 (5-10)
Vg D2

由于 D2 未知,故需推导 D2,根据电容的电荷平衡,可得电容电流的直流分
量为:
iC  0 (5-11)

故可知二极管的直流分量全部流过负载如图 5-8 所示,即:


V
iD   (5-12)
R
V
D1 iD   R

iC  0
C R V

图 5-8 直流分量示意图
由于电感电流的波形如图 5-9 所示:
iL (t )
i pk

iL  I

0 D1TS D2TS D3TS t

图 5-9 DCM 下的电感电流波形


故由上图可知二极管的电流波形如图 5-10 所示:
iD (t )
i pk

iD

0 D1TS D2TS D3TS t

图 5-10 DCM 下的二极管电流波形

- 80 -
根据图 5-9 可求得
Vg D1TS
iL ( D1TS )  i pk  (5-13)
L
而二极管电流的直流分量又等于其平均值,即:
1 Ts 1 1 1
iD 
TS  0
iD (t )dt   i pk D2TS  i pk D2
TS 2 2
(5-14)

由式(5-12)、式(5-13)和式(5-14)可推得 D2 为:
2L
D2   K (5-15)
RTS

由式(5-10)和式(5-15)可推出 DCM 模式下的电压转换比为:


V D D D
 1  1  (5-16)
Vg D2 K K
(c) 当 K  0.1时,由于 DCM 模式发生的条件为:
K  K crit (5-17)

根据式(5-5)可推出,DCM 发生的条件为:

0.1  D2  0.3162  D  D  0.6838 (5-18)


当 D  0.6838 时,变换器运行于 CCM 模式,根据式(5-1)和式(5-2)可推出:
V D
 (5-19)
Vg 1 D

故当 K  0.1时,电压转换比如图 5-11 所示:

 3.125

 6.25

 9.375


if  D 0.6838 
D

D 
1 D
  12.5
 0.1 

 15.625

 18.75

 21.875

 25
0 0.125 0.25 0.375 0.5 0.625 0.75 0.875 1
D

图 5-11 K = 0.1 时的电压转换比

(d) 当 K  0.1, D  0.3 时,根据式(5-18)可知,此时变换器运行于 DCM 模式。

- 81 -
电感电压和电感电流的波形如图 5-12 所示:
vL (t )

Vg

0 DT D2TS D3TS t
1 S

V
iL (t ) 6Vg
i pk 
R

iL  I

0 DT D2TS D3TS t
1 S

图 5-12 电感电压和电感电流波形
(e) 当变换器运行于空载时,K  0 ,故变换器运行于 DCM 模式,根据式(5-16),
输出电压会变得极大,因为电容的能量无处泄放。

Problem 5.2

对于书中 4.1.5 节中的同步 Buck 整流器,如图 5-13 所示:


(a) 轻载时,变换器是否会运行在 DCM 模式?
(b) 当处于空载时 ( R  ) ,并且占空比 D  0.5 时,画出电感电流波形。

Q1 L

Vg Q2 C R

图 5-13 同步 Buck 整流器


Solution:
(a) 轻载时,变换器不会运行在 DCM 模式,因为当电感电流降为 0 时,由于
Q2 可双向导通,故电容为电感和负载电阻提供能量,电感电流反向流动。
(b) 当 Q1 导通时,电路如图 5-14 所示:

Q1 L
vL
Vg C R

图 5-14 Q1 导通时的同步 Buck 整流器

- 82 -
故有:
vL  Vg  V (5-20)

当 Q2 导通时,电路如图 5-15 所示:


L
vL
C R

图 5-15 Q2 导通时同步 Buck 整流器


故有:
vL  V (5-21)

应用小纹波近似,即忽略输出电压的纹波,当 D  0.5 时,有:


V  0.5Vg (5-22)

当变化运行在空载时,可知电感电流的直流分量为 0,故电感电流波形如图
5-16 所示:

0 0.5TS TS t

图 5-16 空载时的电感电流波形

Problem 5.3

直流输入电压 Vg 的变化范围为 35V  Vg  70V ,现在使用 Buck 变换器来获

得 28V 的输出电压,通过反馈环路来随时改变占空比以保证在不同的输入电压
下,输出电压固定在 28V,负载功率的变化范围为 10W  PLoad  1000W ,元器件

的值为 L  22μH, C  470μF, f S  75kHz 。忽略所有损耗。

(a) 输入电压 Vg 和负载电流在什么范围内时变换器运行于 CCM 模式?

(b) 求稳态时占空比的最大最小值。
Solution:
(a) 当变换器运行于 CCM 模式时,并且开关管导通时,电路如图 5-17 所示:

- 83 -
L iL
vL iC
Vg C R V

图 5-17 开关管导通时的 Buck 电路


故有:
vL  Vg  V (5-23)

当开关管关断时,电路如图 5-18 所示:


L iL
vL iC
C R V

图 5-18 开关管关断时的 Buck 电路


故有:
vL  V (5-24)

由于电感电流的直流分量全部流过负载电阻,有:
V
IL  (5-25)
R
当变换器运行于 CCM 模式时,电感电流的波形为如图 5-19 所示:
iL (t )

2iL

0 DTS TS t

图 5-19 CCM 模式下的电感电流波形


故 CCM 模式下电感电流的纹波大小为:
2iL (V  V ) DTS
Vg  V  L  iL  g (5-26)
DTS 2L

故当变换器运行于 CCM 模式时,由式(5-25)和式(5-26)有:


2L
I L  iL  Vg  IL (5-27)
DDTS

上式可变形为:
 2L 
Vg 1  IL   V (5-28)
 VTs 

- 84 -
将各值带入后,有:
Vg 1  0.118 I L   28 (5-29)

上式中的 IL 为电感电流的直流分量,也即负载电流。故当输入电压和负载
电流满足式(5-29)时,变换器运行于 CCM 模式。
(b) 当变换器运行于 CCM 模式时,占空比为:
V
D (5-30)
Vg

由于输入电压的变化范围为 35V  Vg  70V ,可知占空比 D 的变化范围为:

28 28
0.4 
D  0.8 (5-31)
70 35
再根据式(5-29)可知,CCM 模式下的负载电流范围应为:
Vg 1  0.118I L   28  1  0.118 I L  D (5-32)

由上式可推得:
1 D
IL  (5-33)
0.118
当输入电压 Vg  35V ,根据上式可知:

I L  1.7A  PLoad  47.6W (5-34)

当输入电压 Vg  70V ,根据上式可知:

I L  5.1A  PLoad  142W (5-35)

当变换器运行于 DCM 模式时,并且开关管导通时,电路如图 5-20 所示:


L iL
vL iC
Vg C R V

图 5-20 DCM 模式,开关管导通


故有:
vL  Vg  V (5-36)

当变换器运行于 DCM 模式时,并且开关管关断,电流没有断续时,电路如


图 5-21 所示,故有:
vL  V (5-37)

- 85 -
L iL
vL iC
C R V

图 5-21 DCM 模式,开关管关断,断续之前


当变换器运行于 DCM 模式时,并且开关管关断,电流已经断续时,此时有:
vL  0 (5-38)

故对于电感电压,有:
D1 (Vg  V )  D2 (V )  D3 (0)  0 (5-39)

由上式可推出:
D1 V
 (5-40)
D1  D2 Vg

对于上式,D2 未知,故需要推导 D2,由于电感电流的直流分量全部流过负


载,故有:
V
iL  (5-41)
R
而电感电流的直流分量为:
TS
1 1 1 VD T
iL 
TS  i (t )dt  T
0
L
S
  ( D1  D2 )TS  2 S
2 L
(5-42)

根据式(5-41)和式(5-42),可求出 D2 为:

D12  4 K  D1
D2 
2 (5-43)
2L
K
RTS
将式(5-43)带入到式(5-40),可推出:
V 2 D1
 (5-44)
Vg D12  4 K  D1

根据上式可求出:
K
D1  V (5-45)
Vg  VVg
2

而 DCM 模式下,根据式(5-27)可推出:

- 86 -
2L 2 L V VK
Vg  I L  Vg   
DDTS DDTS R DD
(5-46)
V
K  D  K  (1  )
Vg

故,当输入电压为 35V 时,根据式(5-46)可推出:


K  0.2 (5-47)
由于负载电阻的范围为:
V2 282 282
R  0.784  R  78.4 (5-48)
PLoad 1000 10

故 K 的范围为:
0.042  K  4.2 (5-49)
将式(5-49)和式(5-47)代入到式(5-45)后,可推出:
0.367  D1  0.8 (5-50)

当输入电压为 70V 时,根据式(5-46),可推出:


K  0.6 (5-51)
将式(5-49)和式(5-51)代入到式(5-45)后,可推出:
0.106  D1  0.4 (5-52)

故稳态下的占空比的范围为:
0.106  D  0.8 (5-53)

Problem 5.4

图 5-22 所示变换器中的晶体管由相同的门极驱动信号驱动,因此这两个晶
体管同步得开通与关断。
2L
(a) 当稳态下的占空比 D 和无量纲参数 K  在什么范围内时,变换器运行
RTS

于 DCM 模式?
(b) 对于(a)小问,当占空比 D  0.5 时会发什么现象?
(c) 推导直流电压转换比率,并画出当 K  0.1,10 时的电压转换比率曲线。

D1 Q1

Vg Q2 D2 C R v

图 5-22 Watkins-Johnson 变换器

- 87 -
Solution:
(a) 当开关管导通时,电路如图 5-23 所示:
Q1

iC (t )
vL
Vg Q2 i (t ) C R v

图 5-23 开关管导通时的 Watkins-Johnson 变换器


故有:
vL  Vg  V
V (5-54)
iC  i (t ) 
R
当开关管关断时,电路如图 5-24 所示:

i (t ) D1

iC (t )
vL
Vg Q2 D2 C R v

图 5-24 开关管关断时的 Watkins-Johnson 变换器


故有:
vL  Vg
V (5-55)
iC  
R
根据电感的伏秒平衡可推出:
V 2D  1
 (5-56)
Vg D

当变换器运行发生断续时,可知电感电流在输入电压的作用下减少至 0,然
后二极管反向偏置。故可知变换器运行于 DCM 模式下的条件为电感电流的直流
分量小于电感电流的纹波大小,即:
I L  iL (5-57)

当开关管导通时,根据图 5-23 可知,电感电流的纹波为:


2iL (V  V ) DTS
vL  Vg  V  L  iL  g (5-58)
DTS 2L

而当开关管导通时,此阶段的电感电流直流分量等于负载电流,即:
DTS
V 1

R TS  i (t )dt
0
L (5-59)

- 88 -
当变换器运行于 CCM 时,电感电流波形如图 5-25 所示:

iL (t )

0 DTS TS t

图 5-25 CCM 下的电感电流波形


故:
TS
1
iL (t )  I L 
TS  i (t )dt
0
L (5-60)

根据图 5-25 中的波形可知:


DTS
V 1

R TS  i (t )dt  DI
0
L L (5-61)

根据式(5-58)和式(5-61)可知,变换器运行于 CCM 模式时有:

V (V  V ) DTS
I L  iL   g (5-62)
DR 2L
根据式(5-56)和式(5-62),可推出变换器运行于 CCM 的条件为:

D 2 (1  D)
K
2D 1 (5-63)
2D 1  0
故变换器运行于 DCM 的条件为:

D 2 (1  D)
K
2D  1 (5-64)
2D 1  0

D2 (1  D)
(b) 当 K  ,并且 D  0.5 时,输出电压很小很小,几乎为 0,故负
2D  1
载的功率也很小很小,一个开关周期内电源提供的能量很小,但这些能量会在开
关管关断时被负载所消耗,最终达到一个输出电压很小,负载功率也很小的稳态。
(c) 当 K  0.1时,由式(5-64)可知当 0.5  D  0.9 时,变换器处于 DCM 模式。
当变换器处于 DCM 模式时,并且开关管导通时,电路如图 5-26 所示:

- 89 -
Q1

iC (t )
vL
Vg Q2 i (t ) C R v

图 5-26 DCM 模式,开关管导通时


故有:
vL  Vg  V
V (5-65)
iC  i (t ) 
R
当变换器运行于 DCM 模式时,并且开关管关断,电感电流没有降为 0 时,
电路如图 5-27 所示:

i (t ) D1

iC (t )
vL
Vg Q2 D2 C R v

图 5-27 DCM 模式,开关管关断,电感电流降为 0 之前


故有:
vL  Vg
V (5-66)
iC  
R
当变换器运行于 DCM 模式,并且开关管关断,电感电流为 0 时,电路如图
5-28 所示:

i (t ) D1

iC (t )
vL
Vg Q2 D2 C R v

图 5-28 DCM 模式,开关管关断,电感电流为 0 时


故有:
vL  0
V (5-67)
iC  
R
根据电感的伏秒平衡可求得:
V D1  D2
 (5-68)
Vg D1

- 90 -
根据电感电流的波形及开关管 Q1 电流的直流分量等于负载电流,可知:
V
iQ1 
R
(5-69)
1 1 V DT
iQ1    D1TS  g 2 S
TS 2 L

通过式(5-69)求出占空比 D2,然后代入到式(5-68),可得 DCM 模式下的电压


转换比率:
V D2
 21 (5-70)
Vg D1  K

CCM 模式下的电压转换比率为式(5-56)。
故当 K  0.1时,电压转换比率为:
V D12
 for 0.5  D  0.9
Vg D12  K
V 2D  1
 for 0.9  D  1 (5-71)
Vg D
K  0.1
K  0.1时的电压转换比率曲线如图 5-29 所示:

0.963

0.925

0.888
 2
2 D 1 
if  D 0.9 
D
 0.85
 2 D 
 D  0.1 
0.813

0.775

0.738

0.7
0.5 0.563 0.625 0.688 0.75 0.813 0.875 0.938 1
D

图 5-29 K = 0.1 时的电压转换比率


当 K  10 时,无论占空比 0.5  D  1为何值,式(5-63)总是成立,故变换器
总是以 CCM 模式运行,故电压转换比率图 5-30 如图 5-30 所示:

- 91 -
1

0.875

0.75

0.625
2D 1
0.5
D
0.375

0.25

0.125

0
0.5 0.563 0.625 0.688 0.75 0.813 0.875 0.938 1
D

图 5-30 k = 10 时的电压转换比率

Problem 5.5

对于图 5-31 所示的 Cuk 变换器进行 DCM 模式边界分析,可以认为电容的


纹波极小。

L1 vC1 L2
i1 i2
C1 iD
Vg Q1 D1 C2 R V

图 5-31 Cuk 变换器


(a) 画出 CCM 模式下二极管电流波形,并将二极管电流的峰值用两个电感电
流的纹波及直流分量表示出来。
(b) 推导 Cuk 变换器运行于 DCM 模式下的条件,用 K  K crit 的形式表达,并

给出 K 和 K crit 的表达式。

Solution:
(a) 当 0  t  DTS 时,即开关管导通时,电路如图 5-32 所示:

L1 vC1 L2
i1 i2
vL1 iC1 C1 vL 2 iC 2
Vg Q1 C2 R V

图 5-32 开关管导通时的 Cuk 电路


故有:

- 92 -
vL1  Vg
vL 2  V  VC1 (5-72)
iC1   I 2
当开关管关断时,电路如图 5-33 所示:

L1 vC1 vL 2
i1 i2
vL1 iC1 C1 iD1 L2 iC 2
Vg D1 C2 R V

图 5-33 开关管关断时的 Cuk 电路


故有:
vL1  Vg  VC1
vL 2  V (5-73)
iC1  I1
应用电感的伏秒平衡和电容的电荷平衡可得:
Vg
VC1 
D
DVg
V 
D (5-74)
DV
I1  
D R
V
I2  
R
根据式(5-72)和式(5-73),可画出一个开关周期内,两电感电流的波形如图
5-34 所示:
i1 (t )
I1  iL1
I1
I1  iL1

0 DTS TS t
i2 (t )
I 2  iL 2
I2
I 2  iL 2

0 DTS TS t

图 5-34 CCM 模式下的电感电流波形


由于二极管的电流为:

- 93 -
iD1  i1  i2 for DTS  t  TS (5-75)

故二极管电流的峰值及直流分量为:
I D  D( I1  I 2 )
(5-76)
iD _ pk  I1  I 2  iL1  iL 2

(b) Cuk 变换器运行于 CCM 模式的条件即为:


I1  I 2  iL1  iL 2 (5-77)

对于两个电感的纹波大小有:
2iL1
Vg  L1
DTS
(5-78)
2iL 2
V  VC1  L2
DTS
将式(5-74)和式(5-78)代入到式(5-77),可推得变换器运行于 CCM 模式下的
条件为:
1 RT ( L  L )
 S 1 2 (5-79)
D 2
2 L1 L2

上式可变形为:
2 L1 L2
K  D2  K crit (5-80)
( L1  L2 ) RTS

故变换器运行于 DCM 模式的条件为:


2 L1 L2
 D2
( L1  L2 ) RTS (5-81)
K  K crit

Problem 5.6

对图 5-35 所示的 Cuk 变换器进行 DCM 模式下的电压转换比率分析。


(a) 当元器件参数为所示时,变换器运行在 CCM 模式和 DCM 模式的边界处,
画出二极管电流 iD (t ) 、电感电流 i1 (t ) 和 i2 (t ) ,并标出纹波和直流分量大小。

D  0.4 f s  100kHz
Vg  120V R  10Ω
(5-82)
L1  54μH L2  27μH
C1  47μF C2  100μF
(b) 假设变换器运行于 DCM 模式,并且有不一样的元器件值,推导电压转换

- 94 -
比率 M ( D, K ) 。

(c) 画出 DCM 模式下的二极管电流 iD (t ) ,和电感电流 i1 (t ) 和 i2 (t ) 的波形。

L1 vC1 L2
i1 i2
C1 iD
Vg Q1 D1 C2 R V

图 5-35 Cuk 变换器


Solution:
(a) 根据 Problem 5.5 节中的式(5-74),可求出电感电流的直流分量为:
Vg
VC1    200V
D 1
V   DVC1  80V
V (5-83)
I2    8A
R
DI 2
I1    5.33A
D 1
根据式(5-78)可求出电感电流的纹波为:
Vg DTS
i1   4.44A
2 L1
(5-84)
(V  VC1 ) DTS
i2   8.89A
2 L2
根据式(5-74)、式(5-78)、式(5-75)可画出电感电流和二极管电流波形如图
5-36 所示:

- 95 -
i1 (t ) 9.78A

5.33A

0.89A
0 0.4TS TS t
i2 (t ) 16.89A

8A

0.89A t
iD (t )
26.67A

13.33A
0A
0 0.4TS TS t

图 5-36 CCM 与 DCM 边界处的电感电流和二极管电流波形

(b) 当变换器运行于 DCM 模式,并且开关管导通时,即 0  t  D1TS 时,电路

如图 5-37 所示:

L1 vC1 L2
i1 i2
vL1 iC1 C1 vL 2 iC 2
Vg Q1 C2 R V

图 5-37 DCM 模式,开关管导通时的 Cuk 电路


故有:
vL1  Vg
vL 2  V  VC1 (5-85)
iC1  i2
当变换器运行于 DCM 模式,开关管关断并且二极管电流断续之前,即
D1TS  t  ( D1  D2 )TS 时,电路如图 5-38 所示:

L1 vC1 vL 2
i1 i2
vL1 iC1 C1 iD1 L2 iC 2
Vg D1 C2 R V

图 5-38 DCM 模式,开关管关断且电流断续之前时的 Cuk 电路


故有:

- 96 -
vL1  Vg  VC1
vL 2  V (5-86)
iC1  i1
当变换器运行于 DCM 模式,开关管关断并且二极管电流断续之后时,即
( D1  D2 )TS  t  TS 时,电路如图 5-39 所示:

L1 vC1 vL 2
i1 i2
vL1 iC1 C1 L2 iC 2
Vg C2 R V

图 5-39 DCM 模式,开关管关断且电流断续之后时的 Cuk 电路


故有:
vL1  0
vL 2  0 (5-87)
iC1  i1

根据电感的伏秒平衡和电容的电荷平衡有:
D1 (Vg )  D2 (Vg  VC1 )  D3 (0)  0
(5-88)
D1 (V  VC1 )  D2 (V )  D3 (0)  0

由上式可推出:
V D
 1 (5-89)
Vg D2

由于电容 C2 的直流分量为 0,电感 L2 的直流分量全部流过负载电阻,故有:


V
iL 2 (t )   (5-90)
R
而 DCM 模式下的电感电流 iL 2 (t ) 的波形如图 5-40 所示:

iL 2 (t ) I 2  iL 2

I2
V
iL 2  
R 0 t
H1TS H 2TS
I 2  iL 2
D1TS D2TS D3TS

图 5-40 DCM 模式下的电感电流 iL 2 (t ) 波形

- 97 -
图 5-40 中的各量为:
H1 I 2  iL 2 D ( I  iL 2 )
  H1  1 2
D1 2iL 2 2iL 2
(5-91)
H 2 I 2  iL 2 D ( I  iL 2 )
  H2  2 2
D2 2iL 2 2iL 2
根据上式可求得:
iL 2  I 2  iL 2  D1iL 2  D2 iL 2
2iL 2 VD T (5-92)
V  L2  iL 2   2 S
D2TS 2 L2
而式(5-92)中的 I2 未知,接下来根据电容 C1 的电荷平衡来求 I2。电容 C1 的
电流波形根据式(5-85)、式(5-86)和式(5-87)可画出,如图 5-41 所示:
iC1 (t )
 I 2  iL 2  2iL1

 I 2  iL 2  I 2  iL 2
0 t

 I 2  iL 2

D1TS D2TS D3TS

图 5-41 DCM 模式下电容电流 iC1 (t )

根据电容的电荷平衡有:
iC1 (t )  0
(5-93)
iC1 (t )   I 2  iL 2  D1iL 2  D2 iL1
由上式可求出:
I 2  iL 2  D1iL 2  D2 iL1 (5-94)

将式(5-94)代入到式(5-92)后,有:
iL 2  D2 (iL1  iL 2 ) (5-95)

联立式(5-95)和式(5-90),有:
V
D2 (iL1  iL 2 )   (5-96)
R
由于两个电感的纹波电流大小为:

- 98 -
2iL1 V DT
Vg  L1  iL1  g 1 s
D1Ts 2 L1
(5-97)
2iL 2 VD T
V  L2  iL 2  2 s
D2Ts 2 L2
将式(5-97)代入到式(5-96),并根据式(5-89),可求得 D2 为:
L1 L2
2
L1  L2
D2  (5-98)
RTs

将式(5-98)代入到式(5-89),可求得 DCM 模式下的电压转换比率为:


V D
 1
Vg K
L1 L2 (5-99)
2
L1  L2
K
RTs
(c) 电感电流和二极管电流波形如图 5-42 所示:
i1 (t )

0 t
i2 (t )

t
iD (t )

0 t
D1TS D2TS D3TS

图 5-42 DCM 模式下的电感电流波形和二极管电流波形

Problem 5.7

对于图 5-43 所示的 Sepic 变换器进行 DCM 模式边界分析,可以认为电容的

- 99 -
纹波极小。

i1 L1 D1 iD
C1 i2
Vg Q1 L2 C2 R V

图 5-43 Sepic 电路
(a) 画出 CCM 模式下二极管电流波形,并将二极管电流的峰值用两个电感电
流的纹波及直流分量表示出来。
(b) 推导 Cuk 变换器运行于 DCM 模式下的条件,用 K  K crit 的形式表达,并

给出 K 和 K crit 的表达式。

Solution:
(a) 当开关管导通,即 0  t  DTS 时,电路如图 5-44 所示:

i1 vL1 vC1
L1 iC1 C1 i2 iC 2
Vg Q1 L2 vL 2 C2 R V

图 5-44 CCM 模式下开关管导通时的 Sepic 电路


故有:
vL1  Vg
vL 2  VC1
iC1   I 2 (5-100)
V
iC 2  
R

当开关管关断,即 DTS  t  TS 时,电路如图 5-45 所示:

vL1 vC1 D1 iD
i1
L1 iC1 C1 i2 iC 2
Vg L2 vL 2 C2 R V

图 5-45 CCM 模式下开关管关断时的 Sepic 电路


故有:

- 100 -
vL1  Vg  V  VC1
vL 2  V
iC1  I1 (5-101)
V
iC 2  I1  I 2 
R
根据电感的伏秒平衡和电容的电荷平衡,可推得:

DVg
V
D
DV
VC1 
D (5-102)
DV
I1 
DR
DI1
I2 
D
故一个开关周期内,两电感电流波形和二极管电流波形如图 5-46 所示:
i1 (t )
I1  iL1
I1
I1  iL1

0 DTS TS t
i2 (t )
I 2  iL 2
I2
I 2  iL 2

0 DTS TS t
iD (t )
I1  I 2  iL1  iL 2
I1  I 2
I1  I 2  iL1  iL 2

0 DTS TS t

图 5-46 CCM 模式下,两电感电流和二极管电流波形


故二极管的电流为:
iD1  i1  i2 for DTS  t  TS (5-103)

二极管电流的直流分量及峰值为:
I D  D( I1  I 2 )
(5-104)
iD _ pk  I1  I 2  iL1  iL 2

- 101 -
(b) Sepic 电路运行于 CCM 模式的前提条件为:
I1  I 2  iL1  iL 2 (5-105)

对于两个电感电流的纹波大小,有:
2iL1 V DT
Vg  L1  iL1  g S
DTS 2 L1
(5-106)
2iL 2 VDTS
V  L2  iL 2 
DTS 2 L2
将式(5-102)和式(5-106)代入到式(5-105),可 Sepic 电路运行于 CCM 模式的
前提条件为:
1 RT ( L  L )
 S 1 2 (5-107)
D 2
2 L1 L2

将上式进行变形,有:
2 L1 L2
K  D2  K crit (5-108)
( L1  L2 ) RTS

故 Sepic 电路运行于 DCM 模式的前提条件为:


2 L1 L2
 D2
( L1  L2 ) RTS (5-109)
K  K crit

Problem 5.8

对图 5-47 所示的 Sepic 电路进行 DCM 模式分析。

i1 L1 D1 iD
C1 i2
Vg Q1 L2 C2 R V

图 5-47 DCM 模式下运行的 Sepic 电路


(a) 当元器件参数为所示时,变换器运行在 CCM 模式和 DCM 模式的边界处,
画出二极管电流 iD (t ) 、电感电流 i1 (t ) 和 i2 (t ) ,并标出纹波和直流分量大小。

D  0.225 f s  100kHz
Vg  120V R  10Ω
(5-110)
L1  50μH L2  75μH
C1  47μF C2  200μF

- 102 -
(b) 假设变换器运行于 DCM 模式,并且有不一样的元器件值,推导电压转换
比率 M ( D, K ) 。

(c) 画出 DCM 模式下的二极管电流 iD (t ) ,和电感电流 i1 (t ) 和 i2 (t ) 的波形。

Solution:
(a) 根据 Problem 5.7 中的式(5-106),可求出两电感电流的纹波大小为:
Vg DTS
iL1   2.7A
2 L1
(5-111)
VDTS
iL 2   1.8A
2 L2
根据式(5-102),可求出两电感电流的直流分量为:
DV
I1   1.011A
DR (5-112)
DI1
I2   3.484A
D
故两电感电流和二极管电流波形如图 5-48 所示:

i1 (t )
3.711A
1.011A
0 t
1.689A
i2 (t )
5.284A
3.484A
1.684A
t
iD (t )
8.995A
I1  I 2  4.495A

I D  D( I1  I 2 )
 0A
 3.484A t
0 0.225TS TS

图 5-48 Sepic 电路电感电流和二极管电流波形

(b) 当变换器运行于 DCM 模式,且开关管导通时,即 0  t  D1TS 时,电路如

图 5-49 所示:

- 103 -
i1 vL1 vC1
L1 iC1 C1 i2 iC 2
Vg Q1 L2 vL 2 C2 R V

图 5-49 DCM 模式,开关管导通时


故有:
vL1  Vg
vL 2  VC1
iC1  i2 (5-113)
V
iC 2  
R
当 变 换 器 运 行 DCM 模 式 , 开 关 管 关 断 且 二 极 管 电 流 断 续 之 前 , 即
D1TS  t  ( D1  D 2)TS 时,电路如图 5-50 所示:

vL1 vC1 D1 iD
i1
L1 iC1 C1 i2 iC 2
Vg L2 vL 2 C2 R V

图 5-50 DCM 模式,开关管关断,二极管电流断续之前


故有:
vL1  Vg  V  VC1
vL 2  V
iC1  i1 (5-114)
V
iC 2  i1  i2 
R
当开关管关断并且二极管电流已经断续时,电路如图 5-51 所示:

vL1 vC1
i1
L1 iC1 C1 i2 iC 2
Vg L2 vL 2 C2 R V

图 5-51 DCM 模式,开关管关断,二极管电流断续之后


故有:

- 104 -
vL1  0
vL 2  0
iC1  i1 (5-115)
V
iC 2  
R
根据电感的伏秒平衡有:
V D
 1 (5-116)
Vg D2

根据电容 C2 的电荷平衡,可知二极管电流的直流分量等于负载电流,即:
V
iD (t)  (5-117)
R
DCM 模式下,两电感电流和二极管电流波形如图 5-52 所示,故有:
V
iD (t )  I D  D2 ( I1  I 2 )  (5-118)
R

i1 (t )
I1  iL1
I1
t
I1  iL1
i2 (t )
I 2  iL 2
I2
I 2  iL 2
t
iD (t )

I1  I 2

I D  D2 ( I1  I 2 )
t
D1TS D2TS D3TS
图 5-52 DCM 模式下,电感电流和二极管电流波形
而式(5-118)中 I1 和 I2 未知,注意由于 D3 区间的存在,I1 和 I2 便不再两电感
电流的直流分量。
根据式(5-113)、式(5-114)和式(5-115),可画出电容电流 iC1 (t ) 的波形,如图

5-53 所示:

- 105 -
iC1 (t )
I 3  2iL1

I3 t
I 3  I1  iL1 I 3  2iL 2
I 3  iL 2  I 2
D1TS D2TS D3TS

图 5-53 DCM 模式下,电容电流 iC1 (t ) 波形

由电容的电荷平衡,有:
iC1 (t )  0
1  ( I 3  I 3  2iL 2 ) D1TS ( I 3  2iL1  I 3 ) D2TS  (5-119)
   I 3 D3TS   0
TS  2 2 
由式(5-119)和式(5-116)可推出:
D12 L1TSVg  D1D2 L2TSVg
I3  (5-120)
2 L1 L2

又由于:
I 3  I1  iL1
(5-121)
I 3  iL 2  I 2

由式(5-121)、式(5-120)、式(5-118)、式(5-116),可推出:
L1 L2
2
L1  L2
D2  (5-122)
RTs

将式(5-122)代入到式(5-116),可推出 DCM 模式下的电压转换比率:


V D
 1
Vg K
L1 L2 (5-123)
2
L1  L2
K
RTs
(c) DCM 模式下的二极管电流、两电感电流波形如图 5-52 所示。

Problem 5.9

如图 5-54 所示,LC 输入滤波器被添加到了 Buck 电路中,电感 L1 、电感 L2

- 106 -
和电容 C2 的值都很大,以至于可认为它们的开关纹波极小,所有的损耗可以忽

略。

i1 L1 Q1 i2 L2

Vg C1 vC1 D1 C2 R v

图 5-54 带输入滤波的 Buck 电路

(a) 画出电容 C1 的电压波形 v1 (t ) ,并推导其直流分量 V1 和纹波幅值 v1 。

(b) 当负载电流增加(负载电阻 R 减小)以至于 v1 大于 V1 时:

i. 画出电容电压波形 v1 (t ) 。

ii. 对于各个子区间,判断哪个半导体器件处于导通状态。
iii. 推导 DCM 模式发生的条件,用 K  K crit ( D) 的形式表达,并给出 K 和

Kcrit ( D) 的表达式。

Solution:
(a) 当变换器运行于 CCM 模式时,并且开关管导通时,即 0  t  DTS 时,电

路如图 5-55 所示:

i1 L1 i2 L2

vL1 iC1 vL 2
Vg C1 vC1 C2 R v

图 5-55 开关管导通时的电路
故有:
vL1  Vg  VC1
vL 2  VC1  V (5-124)
I C1  I1  I 2

当开关管关断时,即 DTS  t  TS 时,电路如图 5-56 所示:

- 107 -
i1 L1 i2 L2

vL1 iC1 vL 2
Vg C1 vC1 C2 R v

图 5-56 开关管关断时的电路
故有:
vL1  Vg  VC1
vL 2  V (5-125)
I C1  I1
根据式(5-124)和式(5-125)可推得:
VC1  Vg
V  DVC1  DVg
I1  DI 2 (5-126)
V
I2 
R

由式(5-126)可知电容 C1 的电压波形的直流分量 V1 为:

V1  VC1  Vg (5-127)

由式(5-124)、式(5-125)和式(5-126)可画出电容 C1 的电流电压波形,如图 5-57

所示:
DV
iC1 (t )
R
DV 0 DTS TS t
I1  I 2  
R

vC1 (t )
Vg

0 DTS TS t

图 5-57 电容 C1 的电流电压波形

由图 5-57 可推出电容 C1 的电压纹波幅值 v1 为:

- 108 -
dvC1 (t )
TS
DD 2VgTS
iC1 (t )  C1   iC1 (t )dt  2C1vC1  vC1   v1 (5-128)
dt DTS
2 RC1

(b) DCM 模式发生的条件根据式(5-128)可有:


v1  Vg (5-129)

也即:
2 RC1
K  D 2 D  K crit (5-130)
TS

DCM 模式下电容 C1 的电流电压波形如图 5-58 所示,各个子区间中,处于

导通状态的元器件也已在图中标出。
iC1 (t )

0 t

vC1 (t )

0 t
D1TS D2TS D3TS

Q1 Q1 D1
D1

图 5-58 DCM 模式下,电容 C1 的电流电压波形

Problem 5.10

推导 Problem 5.9 节中的变换器运行于 DCM 模式下的电压转换比率。注:D


为晶体管的导通占空比。
Solution:
当晶体管导通,且输入滤波电容的电压降为 0 之前,即输入滤波电容电流断
续之前,电路如图 5-59 所示:

i1 L1 i2 L2

vL1 iC1 vL 2
Vg C1 vC1 C2 R v

图 5-59 开关管导通,且输入滤波电容电流断续之前

- 109 -
故有:
vL1  Vg  vC1
vL 2  vC1  V (5-131)
iC1  I1  I 2
当晶体管导通,且输入滤波电容的电流断续之后,电路如图 5-60 所示:

i1 L1 i2 L2

vL1 iD vL 2
Vg D1 C2 R v

图 5-60 开关管导通,且输入滤波电容电流断续之后
故有:
vL1  Vg
vL 2  V
(5-132)
I C1  0
I D  I 2  I1
当晶体管关断时,电路如图 5-61 所示:

i1 L1 i2 L2

vL1 iC1 vL 2
Vg C1 vC1 C2 R v

图 5-61 晶体管关断之后
故有:
vL1  Vg  vC1
vL 2  V (5-133)
iC1  I1

对电容 C1 、 C2 使用电荷平衡,故有:

D1 ( I1  I 2 )  D2 (0)  D3 ( I1 )  0 (5-134)

由式(5-134)可推出:
D1
I1  I2
1  D2
(5-135)
V
I2 
R

- 110 -
由于变换器运行于 DCM 模式,故 vC1 纹波很大。电容 C1 的电流电压波形如

图 5-62 所示。
D1 V
iC1 (t ) 1  D2 R

D3 V 0 t
I1  I 2  
1  D2 R

vC1 (t )

0 t
D1TS D2TS D3TS

Q1 Q1 D1
D1

图 5-62 DCM 模式下,电容 C1 的电流电压波形

由图 5-62 可推导出 DCM 模式下,电容 C1 的纹波幅值为:

D1 V 2vC1 D1 D3VTS
 C1  vC1  (5-136)
1  D2 R D3TS ( D1  D3 )2C1R

根据图 5-62 中电容 C1 的电压波形,可画出电感 L1 和 L2 的电压波形,如图

5-63 所示:
vL1 (t ) vL 2 (t )
Vg 2vC1  V

0 t

0
t
Vg  2vC1 V
D1TS D2TS D3TS D1TS D2TS D3TS

图 5-63 DCM 模式下,电感 L1 和 L2 的电压波形

根据图 5-63,对电感 L1 和 L2 应用伏秒平衡,有:

- 111 -
1  (Vg  Vg  2vC1 ) D1TS (V  Vg  2vC1 ) D3TS 
  Vg D2TS  g 0
TS  2 2  (5-137)
1  (2vC1  V  V ) D1TS 
  V ( D2  D3 )TS   0
TS  2 
将式(5-136)代入到式(5-137)化简后的公式里,可推出:
2C1 RVg
V
D1 D3TS
(5-138)
C1 R  C1 R(2 D32TS  C1R )
D1 
D3TS
由式(5-138)可求出 DCM 模式下的电压转换比率为:
V 2
 (5-139)
Vg 2
2 D3 TS
1 1
C1 R

由于 D 为晶体管的导通占空比,故有:
D  D3  1 (5-140)

将式(5-140)代入到式(5-139)后,有:
V 2 2
  (5-141)
Vg 2(1  D)2 TS 4(1  D)2
1 1 1 1
C1 R K

Problem 5.11

图 5-64 所示 Cuk 变换中的电感 L1 、 L2 和电容 C2 的值都很大,以至于可认

为他们的开关纹波极小,此外所有的损耗可以忽略。

L1 vC1 L2
i1 i2
C1 iD
Vg Q1 D1 C2 R V

图 5-64 Cuk 变换器

(a) 假设变换器运行于 CCM 模式,画出电容 C1 的电压波形 vC1 (t ) ,推导其直

流分量 V1 和纹波幅值 vC1 。

(b) 当负载电流增加(负载电阻 R 减小)以至于 v1 大于 V1 时:

- 112 -
i. 画出电容电压波形 vC1 (t ) 。

ii. 对于各个子区间,判断哪个半导体器件处于导通状态。
iii. 推导 DCM 模式发生的条件,用 K  K crit ( D) 的形式表达,并给出 K 和

Kcrit ( D) 的表达式。

Solution:
(a) 当变换器运行于 CCM 模式,且开关管导通时,即 0  t  DTS 时,电路如

图 5-65 所示:

L1 vC1 L2
i1 i2
vL1 iC1 C1 vL 2 iC 2
Vg Q1 C2 R V

图 5-65 CCM 模式,开关管导通时的 Cuk 变换器


故有:
vL1  Vg
vL 2  V  VC1 (5-142)
iC1   I 2

当开关管关断时,即 DTS  t  TS 时,电路如图 5-66 所示,故有:

L1 vC1 vL 2
i1 i2
vL1 iC1 C1 iD1 L2 iC 2
Vg D1 C2 R V

图 5-66 CCM 模式,开关管关断时的 Cuk 变换器


vL1  Vg  VC1
vL 2  V (5-143)
iC1  I1
根据电感伏秒平衡和电容的电荷平衡,有:

- 113 -
Vg
VC1 
D
DVg
V 
D (5-144)
DV
I1  
D R
V
I2  
R

故由式(5-144)可知电容 C1 的电压直流分量 V1 为:

Vg
V1  VC1  (5-145)
D

由式(5-142)、式(5-143)和式(5-144)可画出电容 C1 的电流电压波形,如图 5-67

所示:
iC1 (t )
DV
I1  
D R

0
DTS TS t
V
I2 
R
vC1 (t )

2vC1 V1  VC1

0
DTS TS t

图 5-67 CCM 模式下,电容 C1 的电流电压波形

由图 5-67 可推出电容 C1 的电压纹波幅值 vC1 为:

2vC1 D 2VgTS
 I 2  C1  vC1  (5-146)
DTS (1  D)2C1R

(b) DCM 模式发生的条件为:


vC1  V1 (5-147)

由式(5-147)、式(5-146)和式(5-145)可推出:

- 114 -
2C1 R
K  D 2  K crit (5-148)
TS

DCM 模式下电容 C1 的电流电压波形如图 5-68 所示,各个子区间内处于导

通状态的半导体器件名称也已在图中标出。
iC1 (t )

0
t

vC1 (t )

2vC1
0 TS t
D1TS D2TS D3TS
Q1 Q1 D1
D1

图 5-68 DCM 模式下,电容 C1 的电流电压波形

Problem 5.12

推导 Problem 5.11 中 Cuk 变换器运行于 DCM 模式下的电压转换比率


M ( D, K ) ,注: D 代表的是晶体管导通时的占空比。

Solution:
当晶体管导通,且电容 C1 的电压下降至 0 之前,即 0  t  D1TS 时,电路如图

5-69 所示:

L1 vC1 L2
i1 i2
vL1 iC1 C1 vL 2 iC 2
Vg Q1 C2 R V

图 5-69 当 0  t  D1TS 时的 Cuk 变换器

- 115 -
故有:
vL1  Vg
vL 2  V  vC1 (5-149)
iC1   I 2

当 D1TS  t  ( D1  D2 )TS 时,电路如图 5-70 所示:

i1 L1 L2 i2
vL1 iC1 vL 2 iC 2
Vg Q1 D1 C2 R V

图 5-70 当 D1TS  t  ( D1  D2 )TS 时的 Cuk 变换器

故有:
vL1  Vg
vL 2  V
iC1   I 2 (5-150)
vC1  0
iC1  0

当 ( D1  D2 )TS  t  TS 时,电路如图 5-71 所示:

L1 vC1 vL 2
i1 i2
vL1 iC1 C1 iD1 L2 iC 2
Vg D1 C2 R V

图 5-71 当 ( D1  D2 )TS  t  TS 时的 Cuk 变换器

故有:
vL1  Vg  vC1
vL 2  V (5-151)
iC1  I1

对电容 C1 和 C2 使用电荷平衡,故有:

- 116 -
DV
D1 ( I 2 )  D2 (0)  D3 ( I1 )  0  I1   1
D3 R
(5-152)
V
I2  
R
根据 Problem 5.11 中的图 5-68 和式(5-149)、式(5-150)、式(5-151),可画出
电感 L1 、 L2 的电压波形,如图 5-72 所示:

vL1 (t ) vL 2 (t )
V  2vC1

Vg
0 t

0
t
Vg  2vC1 V
D1TS D2TS D3TS D1TS D2TS D3TS

图 5-72 DCM 模式下 Cuk 变换器的电感电压波形


根据图 5-72,对两电感使用伏秒平衡,可得:
1  (Vg  Vg  2vC1 ) D3TS 
Vg ( D1  D2 )TS  0
TS  2  (5-153)
1  (V  2vC1  V ) D1TS 
  V ( D2  D3 )TS   0
TS  2 
将式(5-153)化简后有:
Vg  D3 vC1  0
(5-154)
V  D1vC1  0

根据式(5-149)、式(5-150)、式(5-151)可画出电容 C1 的电流电压波形,如图

5-73 所示:
根据图 5-73 可求出电容 C1 的纹波幅值为:

V 2vC1 DVT
 C1  vC1   1 S (5-155)
R D1TS 2C1 R

- 117 -
iC1 (t )
D1V
I1  
D3 R

0
t
V
I2 
R
vC1 (t )

2vC1
0 TS t
D1TS D2TS D3TS
Q1 Q1 D1
D1

图 5-73 DCM 模式 Cuk 变换器电容 C1 的电流电压波形

将式(5-155)代入到式(5-154)后,可求出:
V 1 2C1 R K
  (5-156)
Vg D3 TS D3

又由于 D 代表的是晶体管导通时的占空比,故有:
D  D1  D2  1  D3 (5-157)

将式(5-157)代入到式(5-156),可求出 DCM 模式下的电压转换比率为:


V K K
  (5-158)
Vg D3 D 1

注:DCM 模式发生的原因,一种原因是由于电感值不够大,导致电感电流
或二极管电流纹波过大,进而引发电感电流或二极管电流的断续,如 Problem 5.5
的 Cuk 电路。另一种原因是由于电容值不够大,导致电容电压纹波过大,使得
电容电压在某一区间内降为 0,进而使得电容电流断续,如 Problem 5.12 的 Cuk
电路。

Problem 5.13

需要设计一个运行在以下条件的 DCM 模式 Buck-Boost 变换器,如图 5-74


所示:
136V  Vg  204V
5W  PLoad  100W
V  150V
f s  100kHz

- 118 -
Q1 D1
i (t )
Vg L C R V

图 5-74 Buck-Boost 变换器


你可以假设存在一个反馈环,使得占空比一直变化以保证 150V 的恒定输出
电压。
设计此变换器,需要满足以下条件:
 变换器一直运行于 DCM 模式。
 在上述的运行前提下,选择器件的值使得电感峰值电流最小。
 输出电压纹波小于 1V 。
需要求出:
(a) 电感值 L 。
(b) 输出电容值 C 。
(c) 最坏情况下的电感峰值电流 i pk 。

(d) 晶体管占空比 D 的最大值和最小值。


Solution:
如 Problem 5.1 的推导,变换器运行于 DCM 模式的条件为:
2L
K  (1  D1 ) 2  K crit (5-159)
RTS

而在 DCM 模式下,电压转换比率为:
V D
 1 (5-160)
Vg K
将式(5-160)代入到式(5-159),可推导出 K 范围为:
Vg
K  K  0.476  K  0.2266 (5-161)
Vg  V

根据给定条件,可推出负载电阻的变化范围为:

V2 1502 1502
R  225  R  4500 (5-162)
P 100 5

根据式(5-159),为了保证变换器一直运行在 DCM 模式,可求出电感值 L 的


范围为:
0.2266RTS
L  L  0.2549mH (5-163)
2
当晶体管导通时,即 0  t  D1TS 时,电路如图 5-75 所示:

- 119 -
i (t ) iC
Vg vL L C R V

图 5-75 晶体管导通时的 Buck-Boost 变换器


故有:
vL  Vg
V (5-164)
iC  
R

当晶体管关断,且电感电流下降至 0 之前,即 D1TS  t  ( D1  D2 )TS 时,电路

如图 5-76 所示:
故有:
vL  V
V (5-165)
iC   i
R
D1
i (t ) iC
Vg vL L C R V

图 5-76 晶体管关断,且电感电流断续之前的 Buck-Boost 变换器

当电感电流断续之后,即 ( D1  D2 )TS  t  TS 时,电路如图 5-77 所示:

i (t ) iC
Vg vL L C R V

图 5-77 电感电流断续之后的 Buck-Boost 变换器


故有:
vL  0
V (5-166)
iC  
R
根据式(5-164)、式(5-165)和式(5-166)可画出电感电流波形,如图 5-78 所示:

- 120 -
iL (t )
i pk

0 D1TS D2TS D3TS t

图 5-78 DCM 模式下 Buck-Boost 变换器的电感电流波形


由图 5-78 可求出电感电流峰值为:
Vg D1TS
i pk  (5-167)
L
将式(5-160)、式(5-159)代入到式(5-167)可将电感峰值电流的表达式变形为:

2TS
i pk   V (5-168)
RL
由式(5-168)可知,当电感值 L 取最大值时,对于任意的负载电阻,能保证电
感峰值电流最小,故根据式(5-163)可知电感值应取:
L  0.2549mH (5-169)
根据式(5-168)可求出最恶劣情况下的电感峰值电流为:

2TS
i pk_max   V  2.801A for R  225 (5-170)
RL
为了保证变换器一直运行在 DCM 模式下,且任意负载电阻下的电感峰值电
流最小,已确定出电感值 L ,电感值 L 的确定可推导出给定条件下的系数 K 的范
围为:
0.0113  K  0.2266 (5-171)
根据式(5-160)可知,占空比 D 的范围为:
KV
D  D1  
Vg
Dmax  0.525 for K  0.2266 Vg  136V (5-172)
Dmin  0.078 for K  0.0113 Vg  204V
根据图 5-78 及式(5-164)、式(5-165)和式(5-166)可画出输出电容的电流电压
波形,如图 5-79 所示。
根据图 5-79 和式(5-167)可求出时间间隔 D4TS 为:

V  V
  i pk  i pk  
D4 R R D
  D4    2 (5-173)
D2 i pk  i pk 
 

- 121 -
iC (t ) vC (t )  v
D4TS
V D4TS

R
0 TS t 0 TS t

2vC
V

V D1TS D2TS D3TS


  i pk
R D1TS D2TS D3TS

图 5-79 DCM 模式 Buck-Boost 变换器输出电容的电流电压波形


故输出电容的纹波为:
( D1  D4 )TS
dvC (t ) ( D  D4 )TS
iC  C
dt
 
D1TS
iC (t )dt  Cvc (t ) D T1
1 S

( D  D4 )TS
Cvc (t ) D T1  2C vC (5-174)
1 S

( D1  D4 )TS
1  V 

D1TS
iC (t )dt 
2
D4TS    i pk 
 R 
由式(5-174)、式(5-167)、式(5-173)可求出输出电容纹波为:
D2 ( LV  D1 RTSVg ) 2
vC  (5-175)
4CD1 LR 2Vg

将 Problem 5.1 中的式(5-15)和式(5-160)代入到式(5-175),可求出:


K
V ( L  K RTS ) 2 V (  K ) 2 TS 2
vC    2 (5-176)
4CLR 2 4CL
由于输出电压纹波要小于 1V ,故上式可变形为:
K K
V (  K ) 2 TS 2 (V )(  K ) 2 TS 2
vC   2  1V  2 C (5-177)
4CL 4L
对于式(5-177),可知:
K
(  K )2  0.1316 for K  0.226 (5-178)
2 max

根据式(5-178)和式(5-177),可求出电容值 C 的取值范围应为:
K
(V )(  K ) 2 TS 2
C 2  C  1.936μF (5-179)
4L

- 122 -
Problem 5.14

需要设计一个 DCM 模式下的 Boost 变换器,如图 5-80 所示,运行在以下


给定条件:
18V  Vg  36V
5W  PLoad  100W
V  48V
f s  150kHz
你可以假设存在一个反馈环路可以随时改变占空比的值,使得输出为恒定的
48V 电压。

i (t ) L D1 iD (t )
vL (t ) iC (t )
Vg Q1 C R v(t )

图 5-80 Boost 变换器


设计这个变换器,以满足下列要求:
 变换器一直运行在 DCM 模式,为了保证足够的设计裕度,电感值 L 的
选择要保证在所有的运行点上, K 的值不超过 K crit 的 75% 。

 在上述的要求下,选择器件值使得电感峰值电流最小。
 输出电压纹波小于 1V 。
需要求出:
(a) 电感值 L 。
(b) 输出电容值 C 。
(c) 最恶劣情况下的峰值电感电流 i pk 。

(d) 晶体管导通占空比 D 的最小值与最大值。


(e) 以下运行点处的 D 、 K 和 K crit 值:

i. Vg  18V , PLoad  5W

ii. Vg  36V , PLoad  5W

iii. Vg  18V , PLoad  100W

iv. Vg  36V , PLoad  100W

Solution:
对于 Boost 变换器,其运行于 DCM 模式的条件为:

- 123 -
2L
 K  K crit  D(1  D) 2 (5-180)
RTS

又由于 DCM 模式下,电压转换比率为:

4D2
1 1
V K
 (5-181)
Vg 2

为了保证一定的设计裕度,即 K  0.75Kcrit ,利用式(5-181)求出 D 的表达式

代入到式(5-180),有:
0.75 KV (V  Vg )(Vg  KV (V  Vg )) 2
K (5-182)
Vg 3

令 K  Z ,则式(5-182)等效为:

ZVg 3  0.75 V (V  Vg )(Vg  Z V (V  Vg )) 2 (5-183)

将式(5-183)进行合并同类项,可得:
3 3
0.75V (V  Vg ) Z 2   Vg 3  1.5VVg (V  Vg )  Z
2 2
(5-184)
 0.75Vg 2
V (V  Vg )  0

式(5-184)的形状为开口向上的抛物线,其两个根为:

 Vg 2 3V 2  3VVg  Vg 2 
1.333 0.75V Vg  0.75VVg 
2 2
 0.5Vg 3 
 2 
Z1   
3 3
V 2 (V  Vg ) 2
(5-185)
 Vg 2 3V 2  3VVg  Vg 2 
1.333 0.75V 2Vg  0.75VVg 2   0.5Vg 3 
 2 
Z2   
3 3
V 2 (V  Vg ) 2

根 Z1 关 于 输 入 电 压 Vg 的 曲 线 如 图 5-81 所 示 , 其 中 Z1 _ m a  ,
x 7.187

Z1_ min  0.815 ,根 Z 2 关于输入电压 Vg 的曲线如图 5-82 所示,其中 Z2_ max  0.333 ,

Z2_min  0.276 ,故 Z 的范围为 Z  7.187或Z  0.276 ,故系数 K 的范围为:

K  51.65或K  0.076 (5-186)

- 124 -
8

 2 2 2 5
 2 2
Vg  3.0 Vo  3.0 Vo Vg Vg
3
1.3333333333333333333  0.75 Vo  Vg 0.75 Vo Vg   0.5 Vg 
 2 4
3 3
2

Vo  Vo 1.0 Vg  2
3

0
18 20.25 22.5 24.75 27 29.25 31.5 33.75 36
Vg

图 5-81 根 Z1 关于输入电压 Vg 曲线

0.34

0.331

0.323

 2 2 2  0.314
 2 2
Vg  3.0 Vo  3.0 Vo Vg Vg
3
1.3333333333333333333  0.75 Vo  Vg 0.75 Vo Vg   0.5 Vg 
 2  0.305
3 3
2

Vo  Vo 1.0 Vg 2 0.296

0.288

0.279

0.27
18 20.25 22.5 24.75 27 29.25 31.5 33.75 36
Vg

图 5-82 根 Z 2 关于输入电压 Vg 曲线

当 K  51.65 时,式(5-181)求出的占空比 D 会大于 1,故此范围的 K 值要舍


去,综上,系数 K 的范围为:
K  0.076 (5-187)
根据给定条件,可推出负载电阻的变化范围为:

V2 482 482
R  23.04  R  460.8 (5-188)
P 100 5
对于 DCM 模式下的 Boost 变换器,电感电流的峰值为:
Vg
i pk  DTS (5-189)
L
将式(5-181)代入到式(5-189),有:
KV (V  Vg )TS 2V (V  Vg )TS
i pk   (5-190)
L LR
由式(5-190)可看出,为了使电感峰值电流最小,电感值 L 应取最大值,故根
据式(5-188)和式(5-180),可推导出电感值的最大值为:
0.076RTS
K  0.076  L   L  5.837μH  L  5.837μH (5-191)
2
根据式(5-191)和式(5-190),可求出最恶劣情况下的电感峰值电流为:

- 125 -
2V (V  Vg )TS 2  48  (48  18)TS
i pk _ max    11.95A (5-192)
LR 5.837 106  23.04
根据已确定的电感值 L ,可求出系数 K 的真实范围为:

3.8 103  K  0.076 (5-193)


由式(5-193)和式(5-181),可知实际中占空比 D 的范围为:
KV (V  Vg )
D
Vg
Dmax  0.5812 for K  0.076 Vg  18V (5-194)
Dmin  0.0411 for K  3.8 103 Vg  36V
DCM 模式下,输出电容的电流电压波形如图 5-83 所示:
iC (t )
V
 +i pk vC (t )  v
R D4TS

V
2vC

V TS t

R
D4TS
0 D1TS D2TS D3TS TS t
D1TS D2TS D3TS

图 5-83 DCM 模式 Boost 变换器输出电容的电流电压波形

根据图 5-83 和式(5-189),可求出时间间隔 D4TS 为:

V  V
 i pk  i pk  
D4
 R  D4   R D
 2 (5-195)
D2 i pk  i pk 
 
故输出电容的纹波为:
( D1  D4 )TS
dv (t ) ( D  D4 )TS
iC  C C 
dt 
D1TS
iC (t )dt  Cvc (t ) D T1
1 S

( D  D4 )TS
Cvc (t ) D T1  2C vC (5-196)
1 S

( D1  D4 )TS
1  V 

D1TS
iC (t )dt 
2
D4TS    i pk 
 R 
由式(5-196)及输出电压纹波小于 1V 的给定条件,可求出:

- 126 -
TS 2 ( KV  2 KV (V  Vg ))2
C (5-197)
16 L(V  Vg )
由于开关频率和电感值已确定,故需要求出式(5-197)右边项的最大值,即需
求出下式的最大值:
( KV  2 KV (V  Vg )) 2
f ( K , Vg ) 
(V  Vg )
3.8 103  K  0.076 (5-198)
18  Vg  36

函数 f ( K ,Vg ) 关于 K 的偏导数为:

2( KV  2 KV (V  Vg ))  V (V  Vg ) 
f K  ( K ,Vg )  V   (5-199)
(V  Vg )  K 
 

令 f K  ( K ,Vg )  0 ,可求出:

 V  Vg
 V
K  (5-200)
 4V  4Vg
 V

函数 f ( K ,Vg ) 关于 Vg 的偏导数为:

( KV  2 KV (V  Vg )) 2
fVg  ( K , Vg )  
(V  Vg ) 2
(5-201)
2 KV ( KV  2 KV (V  Vg ))
3
(V  Vg ) 2

令 fVg  ( K ,Vg )  0 ,可求出:

 0

K   4V  4Vg (5-202)

 V

4V  4Vg
故函数 f ( K ,Vg ) 如果有极值的话,一定在 ( ,Vg ) 这一系列的点处取得。而
V

4V  4Vg
在式(5-198)中的给定范围内,会发现 K  ,故在式(5-198)的范围内,函数
V

- 127 -
f ( K ,Vg ) 无极值,所以函数 f ( K ,Vg ) 的最大值一定边界处取得。故函数 f ( K ,Vg ) 的

最大值为:
f ( K ,18) max  9.947 for K  0.076 Vg  18
f ( K ,36) max  7.656 for K  0.076 Vg  36
(5-203)
f (3.8 103 ,Vg )  0.674 for K  3.8 103 Vg  18
max

f (0.076,Vg )  9.947 for K  0.076 Vg  18


max

故当 K  0.076 , Vg  18 时函数 f ( K ,Vg ) 取得最大值,故式(5-197)右边项的最

大值为:
TS 2 ( KV  2 KV (V  Vg )) 2 TS 2
  9.947  4.734μF (5-204)
16 L(V  Vg ) 16 L
max

故为了保证输出电压纹波小于 1V ,电容值要满足下式:
C  4.734μF (5-205)

故电容值 C  4.734μF 。

各运行点处的 D 、 K 和 K crit 值可根据式(5-180)和式(5-194)求出:

K  3.8 103 , D  0.13, K crit  0.098 for Vg  18V , PLoad  5W


3
K  3.8 10 , D  0.041, K crit  0.038 for Vg  36V , PLoad  5W
K  0.076, D  0.581, K crit  0.102 for Vg  18V , PLoad  100W
K  0.076, D  0.184, K crit  0.122 for Vg  36V , PLoad  100W

Problem 5.15

对于使用电池供电的便携式设备中的 DCDC 变换器,当负载处于睡眠模式


时,即低功率模式时,要求 DCDC 变换器既能输出所需的电压又能保持高效。
然而开关损耗和晶体管门极驱动电路所产生的损耗主要取决于开关频率,而不是
取决于负载电流。因此为了在低负载功率时保持高效率,就需要可变频率的控制
策略,即开关频率随着负载电流的下降而降低。
考虑图 5-84 中的(a)所示的 Boost 变换器,电池组由两个镍铬电池单体构
成,能够产生 Vg  2.4V  0.4V 的电压,变换将此输入电压调整至 5V 。如图 5-84

中的(b)所示,当变换器运行在 DCM 模式,其晶体管的导通时间 ton 是恒定的,

晶体管的关断时间 toff 是受控制器控制以维持恒定的输出电压。

- 128 -
(a) 推导出 CCM 和 DCM 模式的边界方程和电压转换比率 M  V / Vg ,以 ton ,

toff , L 和有效负载电阻 R 为参数。

对于(b)和(c)小问,负载电流的变化范围为 100μA 1A ,晶体管的导通

时间是固定的,即 ton  10μs 。

(b) 选择电感值 L 和电容值 C ,以满足下列条件:


 输出电压纹波小于 50mV 。
 变换器始终运行在 DCM 模式。
 电感峰值电流尽可能小。
(c) 对于(b)中的设计,求出占开关频率的最大值和最小值
(a) i (t ) I load

Vg v

(b) i (t )
i pk

0 t
ton toff

图 5-84 便携式设备中的 Boost 变换器


Solution:
(a) 当使用占空比 D  ton / (ton  toff ) 这个参数时,CCM 和 DCM 的边界条件及

电压转换比为:
2L V 1
CCM : I  iL  K  K crit  D(1  D) 2 
RTS Vg (1  D)
4D2 (5-206)
1 1
2L V K
DCM : I  iL  K  K crit  D(1  D) 2 
RTS Vg 2

为了使用 ton , toff , L 和有效负载电阻 R 为参数表达上式,将式代入到式

(5-206),可得出:

- 129 -
ton
D
ton  toff (5-207)
TS  ton  toff

V ton  toff
CCM : K  K crit 
Vg toff
2 Rton 2  L(ton  toff )
1
V L(ton  toff )
DCM : K  K crit 
Vg 2 (5-208)
2L
K
R(ton  toff )
tontoff 2
K crit 
(ton  toff )3
(b) 由于变换器需要始终运行在 DCM 模式,故有:
2L
 K  K crit  D(1  D) 2 (5-209)
RTS

又由于 DCM 模式下的电压转换比为:

4D2
1 1
V K
 (5-210)
Vg 2

利用式(5-210)求出 D 的表达式后,代入到式(5-209)后,有:
KV (V  Vg )(Vg  KV (V  Vg ))2
K (5-211)
Vg 3

令 Z  K ,代入到式(5-211),并化简后有:

ZVg 3  V (V  Vg )(Vg  Z V (V  Vg )) 2 (5-212)

进一步化简后有:
 52 3
 2
  Vg V  Vg  Z   2VVg  2V Vg  Vg  Z
2 2 3

2
V V Vg V
  (5-213)
 Vg 2
V (V  Vg )  0

由于 Boost 变换器只能升压,可判断 Z 2 的系数大于 0,因此式(5-213)为开口


向上的抛物线,然后令式(5-213)等于 0,求出其两个根为:

- 130 -
Vg 2 (Vg  2V ) Vg 3
V Vg  VVg  2
 2

Z1  2 2
5 3
V 2 V  Vg  V 2Vg V  Vg
(5-214)
V 2 (V  2V ) Vg 3
V Vg  VVg  g g2
 2

Z2  2 2
5 3
V 2 V  Vg  V 2Vg V  Vg

由于输出电压 V 为恒定的 5V ,输入电压的范围为 2V  Vg  2.8V ,故可画出

两个根 Z1 、 Z 2 关于输入电压 Vg 的曲线, Z1 关于输入电压 Vg 的曲线如图 5-85 所

示,其中 Z1_ max  0.371, Z1_ min  0.31 。Z 2 关于输入电压 Vg 的曲线如图 5-86 所示,

其中 Z2_ max  1.919 , Z2_ min  0.861 。

0.38

0.37

0.36

2 2
2
 
Vg  Vg 2 Vo Vg 0.35
3

Vo  Vg Vo Vg  
2 2
0.34
5 3
2 2 0.33
Vo  Vo Vg Vo  Vg Vo Vg
0.32

0.31

0.3
2 2.1 2.2 2.3 2.4 2.5 2.6 2.7 2.8
Vg

图 5-85 Z1 关于输入电压 Vg 曲线

1.85

1.7

2 2
2

Vg  Vg 2 Vo  3
Vg 1.55
Vo Vg Vo  Vg 
2 2
 1.4
5 3
2 2
Vo  Vo VgVo  Vg Vo Vg 1.25

1.1

0.95

0.8
2 2.1 2.2 2.3 2.4 2.5 2.6 2.7 2.8
Vg

图 5-86 Z 2 关于输入电压 Vg 曲线

- 131 -
故为了使变换器一直运行于 DCM 模式,有:
Z  1.919 or Z  0.31
(5-215)
Z K
即:
K  3.683 or K  0.096
(5-216)
当 K  3.683 时,根据式(5-210)可推出 D  1 ,故需舍去。综上,为了使变换
器始终运行于 DCM 模式,要保证 K  0.096 。
由于电感峰值电流的表达式为:
Vg Vg V  Vg Vg ton
i pk  DTS  ton  D2TS  D2TS  (5-217)
L L L V  Vg

根据式(5-217)可得出电感电流从峰值下降至 0 的时间最小为:
Vg ton
D2TS   D2T  6.667μs for Vg  2V (5-218)
V  Vg

根据式(5-218)可知,开关周期的最小值为:
TS  D1TS  D2TS  D3TS  D1TS  D2TS  ton  D2TS  16.667μs (5-219)

由于负载电流的变化范围为 100μA 1A ,故可知负载电阻的变化范围为:

5V 5V
5 104   R  5 (5-220)
100μA 1A

由于晶体管导通时间 ton 为定值,为了使电感峰值电流最小,故需使电感值 L

尽可能大。而根据式(5-216)可推导出:
2L
K  0.096   0.096  L  0.048RTS
RTS (5-221)
 L  0.048  5 16.667 106  4μH
故为了使变换器始终运行在 DCM 模式,并且尽可能得使电感峰值电流最小,
电感值 L  4μH 。

根据式(5-210)可推出:
RVg 2ton 2
TS 
2 LV (V  Vg )
(5-222)
Vg 2ton 2
3.333 106   8.909 10 6
2 LV (V  Vg )
根据式(5-222)和式(5-220)可推出开关周期的范围为:
16.667 106  3.333106  5  TS  8.909 106  5104  0.445 (5-223)

- 132 -
根据式(5-223)和式(5-209),可推出 K 的真实范围为:
2  4 106 2L
3.596 1010  K  0.909 (5-224)
5 10  0.445
4
RTS

根据 Problem 5.14 中的式(5-196),有:


ton
TS 2 ( KV  2 Vg ) 2
TS ( KV  2 KV (V  Vg ))
2 2
TS
vC   (5-225)
16CL(V  Vg ) 16CL(V  Vg )

ton
根据式(5-210)及 D  ,可求出 K 的表达式为:
TS

Vg 2ton 2
K (5-226)
TS 2V (V  Vg )

将式(5-226)代入到式(5-225)可推出:
Vg 2ton 2 (2TSVg  2TSV  Vg ton ) 2
vC  (5-227)
16CLTS 2 (V  Vg )3

由于要保证输出电压纹波小于 50mV ,故式(5-227)可变形为:


Vg 2ton 2 (2TSVg  2TSV  Vg ton ) 2
 50 103CL (5-228)
16TS 2 (V  Vg )3

式(5-228)的左半部分可看成 f (Vg , TS ) 二元函数,令 f (Vg , TS ) 关于 Vg 和 TS 的偏

导数等于 0,可推出:

 f  (V , T )  0
 Vg g S Vg ton
  TS  (5-229)
 fTS  (Vg , TS )  0 2V  2Vg

将式(5-229)代入到式(5-226)后,会发现式(5-226)中的 K 的范围不在式(5-224)
中,故可知式(5-228)左半部分的最大值在边界处取得。即有:
f (Vg ,16.667 106 )  3.4 1011 for Vg  2.8V
max

f (Vg , 0.445)  8.911011 for Vg  2.8V


max
(5-230)
f (2, TS ) max  3.33 1011 for TS  0.445
f (2.8, TS ) max  8.911011 for TS  0.445
由式(5-230)可知式(5-228)左半部分的最大值为:
f (Vg , TS )  8.911011 for Vg  2.8V TS  0.445 (5-231)
max

由式(5-231)可将式(5-228)重写为:

- 133 -
8.911011  50 103 CL  445.5μF  C (5-232)

根据式(5-232),电容值选取为:
C  445μF (5-233)

(c) 开关频率 f S 的最大最小值根据式(5-223)可求得:

1 1
2.247Hz   fS   60kHz (5-234)
0.445 16.667 106

- 134 -
第6章 变换器电路

Problem 6.1

使用带有中间抽头电感的 Boost 变换器,Boost 变换器有时被修改成如图 6-1


所示的拓扑以获得更大的转换比率。电感的总匝数为 n1  n2 ,电感的左半部分匝

数为 n1 ,与晶体管相连接。此中间抽头的电感可看成匝数为 n1 : n2 的两绕组变压

器,只不过在此变压器中,两绕组是串联的。整个匝数 n1  n2 的电感值为 L 。

n1 n2
D1

Vg Q1 C R V

图 6-1 带中间抽头的 Boost 变换器,Problem6.1


(a) 画出此变换器的等效电路,此等效电路应包括激磁电感和理想变压器,并
在等效电路中标出激磁电感值与匝数比。
(b) 假设变换器运行在 CCM 模式,且晶体管、二极管、电感与电容均无损耗,
推导电压转换比率 M  V / Vg 。

(c) 对于 n1  n2 ,画出 M ( D) 关于 D 的图形,并和 n2  0 时的情况相比较。

Solution:
(a) 等效电路如图 6-2 所示:
n1 : n2 D1

Lm
Vg C R V
Q1

图 6-2 带中间抽头的 Boost 变换器等效电路

关于激磁电感 Lm 的推导,由于整个匝数 n1  n2 的电感值为 L ,对于图 6-3

所示的电路,有:

- 135 -
i i1 n1 : n2 i2
im
Lm vLm v2
vg

图 6-3 激磁电感值的推导
i  i1  im
i1 n2

i2 n1
i2  i
vLm n1 (6-1)

v2 n2
vg  vLm  v2
dim
vLm  Lm
dt
由式(6-1)可推出:
2
 n  n  di
vg  Lm  1 2 
 n1  dt
2
(6-2)
n n 
L  Lm  1 2 
 n1 

由式(6-2)可推出激磁电感值为:
2
 n 
Lm   1  L (6-3)
 n1  n2 
(b) 当晶体管导通时,电路如图 6-4 所示:

i i1 n1 : n2 i2 D1
im
vLm v2 iC
Lm
Vg C R V
Q1

图 6-4 晶体管导通时的等效电路
故有:

- 136 -
vLm  Vg
V
iC   (6-4)
R
iI
当晶体管关断时,电路如图 6-5 所示:

i i1 n1 : n2 i2 D1
im
vLm v2 iC
Lm
Vg C R V

图 6-5 晶体管关断时的等效电路
故有:
vLm n1

v2 n2
V
i2  
R (6-5)
i1 n2

i2 n1
vLm  Vg  V  v2
对于激磁电感使用伏秒平衡有:
Vg  v2  Dv2
D(Vg )  (1  D)(Vg  V  v2 )  0  V  (6-6)
1 D

利用式(6-5)中的第 1 个子式和第 4 个子式可求出 v2 的表达式,有:

n2 (V  Vg )
v2   (6-7)
n1  n2

将式(6-7)代入到式(6-6)后,可得:
V 1  n2 
M ( D)   1  D  (6-8)
Vg 1  D  n1 

(c) 当 n1  n2 时,电压转换比率的图形如图 6-6 所示:

- 137 -
45

39.375

33.75

28.125

1 D
22.5
1D

16.875

11.25

5.625

0
0 0.125 0.25 0.375 0.5 0.625 0.75 0.875 1
D

图 6-6 n1  n2 时的电压转换比率

当 n2  0 时,电压转换比率如图 6-7 所示:

25

21.875

18.75

15.625

1
12.5
1D

9.375

6.25

3.125

0
0 0.125 0.25 0.375 0.5 0.625 0.75 0.875 1
D

图 6-7 n2  0 时的电压转换比率

Problem 6.2

对 DCM 模式下的反激变换器进行分析,变换器电路如图 6-8 所示:

- 138 -
1: n D1

Lm C V
Vg
Q1

图 6-8 DCM 模式下的反激变换器


(a) 使用激磁电感与理想变压器的并联对此 DCM 模式下的反激变换器进行建
模,画出 3 个子区间的变换器电路。
(b) 推导变换器运行于 DCM 模式的边界条件。
(c) 推导稳态输出电压 V 与二极管的导通占空比 D2 的表达式。

Solution:
(a) 等效电路如图 6-9 所示:
ig i1 1: n i2 D1
i iC

vT Lm C V
Vg
Q1

图 6-9 使用激磁电感与理想变压器对 DCM 反激变换器建模

当晶体管导通,即 0  t  D1TS 时,变换器的等效电路如图 6-10 所示:

ig i1 1: n
i iC

vT Lm C V
Vg
Q1

图 6-10 0  t  D1TS 时的等效电路

当晶体管关断、二极管导通且电流断续之前时,即 D1  t  ( D1  D2 )TS 时,

- 139 -
变换器的等效电路如图 6-11 所示:
ig i1 1: n i2 D1
i iC

vT Lm C V
Vg

图 6-11 D1  t  ( D1  D2 )TS 时的等效电路

当电流断续之后,即 ( D1  D2 )TS  t  TS 时,变换器的等效电路如图 6-12 所

示:
ig i1 1: n
i iC

vT Lm C V
Vg

图 6-12 ( D1  D2 )TS  t  TS 时的等效电路

(b) 当反激变换器处于 CCM 模式时,且晶体管导通时,即 0  t  DTS 时,等

效电路如图 6-13 所示:


ig i1 1: n
i iC

vT Lm C V
Vg
Q1

图 6-13 CCM 模式下,晶体管导通时等效电路

当反激变换器处于 CCM 模式时,且晶体管关断时,即 DTS  t  TS 时,等效

电路如图 6-14 所示:

- 140 -
ig i1 1: n i2 D1
i iC

vT Lm C V
Vg

图 6-14 CCM 模式下,晶体管关断时等效电路


CCM 模式下,激磁电感的电压电流波形和电容电流波形如图 6-15 所示:
vT

Vg

0 DTS TS t
V

n
i V
Vg 
n
I

0 t
DTS TS
iC
I V

n R

0
DTS TS t
V

R
图 6-15 CCM 模式下的激磁电感电压电流波形和电容电流波形
对激磁电感使用伏秒平衡,对电容使用电荷平衡,有:
V
D(Vg )  D( )  0
n
(6-9)
V I V
D( )  D(  )  0
R n R
由式(6-9)可推出激磁电感中电流的直流分量为:
Dn 2Vg
I (6-10)
(1  D ) 2 R
根据激磁电感中电流的波形,可推出激磁电感的电流纹波为:

- 141 -
2i DTSVg
Vg  Lm  i  (6-11)
DTS 2 Lm

当激磁电感的电流纹波大小大于其直流分量时,DCM 模式便会发生,即:
i  I for DCM (6-12)

将式(6-10)和式(6-11)代入到式(6-12)中,有:
2 Lm 2
K n  (1  D) 2  K crit for DCM (6-13)
RTS

(c) 当变换器运行于 DCM 模式时,根据图 6-10、图 6-11 和图 6-12 可画出


激磁电感的电压波形和二极管的电流波形,如图 6-16 所示:

vT

Vg

0 TS t
V

n
i2
i2 pk

I
0 t

D1TS D2TS D3TS

图 6-16 DCM 模式下的激磁电感电压波形与二极管电流波形


对激磁电感使用伏秒平衡后,有:
V DV n
1 g  D2 ( 
DV )  D3  0  0  V  1 g (6-14)
n D2

二极管电流的峰值 i2 pk 与激磁电感电流峰值 i pk 的关系为:

i pk
i2 pk  (6-15)
n
而激磁电感电流峰值为:
i pk D1TSVg
Vg  Lm  i pk  (6-16)
D1TS Lm

对输出电容使用电荷平衡可知二极管电流的直流分量等于负载电流,即:

- 142 -
1 1  V
 D2TS i2 pk   (6-17)
TS 2  R
由式(6-15)、式(6-16)和式(6-17)可求出二极管导通的占空比为:
2 LmVn
D2  (6-18)
RTS DV1 g

将式(6-14)代入到式(6-18)后,可得:
2 Lm 2
D2  n  K (6-19)
RTS

将式(6-19)代入到式(6-14),可推出电压转换比率为:
V nD1
 (6-20)
Vg K

Problem 6.3

对如图 6-17 所示的隔离型反转 SEPIC 电路进行分析,你可以假设变换器运


行于 CCM 模式,并且电感电流纹波和电容电压纹波都很小。
C1 L2
1: n

D1 C2 R v
Vg
Q1

图 6-17 隔离型反转 SEPIC 电路


(a) 推导磁化电流、电感电流和电容电压的直流分量。
(b) 推导原副边电流的有效值,注意原副边电流有效值的关系并不仅仅是匝数
比的关系。
Solution:
(a) 当晶体管导通时,即 0  t  DTS 时,电路如图 6-18 所示:

- 143 -
C1 L2
ig i1 1: n i2 iC1 iL 2
iM vC1 vL 2 iC 2

Lm vT v2 C2 R v
Vg
Q1

图 6-18 晶体管导通时的等效电路
故有:
vT  Vg
vL 2  vC1  v2  v
vT 1

v2 n
iC1  i2 (6-21)
i1
n
i2
V
I L2 
R

当晶体管关断时,即 DTS  t  TS 时,等效电路如图 6-19 所示:

C1 L2
ig i1 1: n i2 iC1 iL 2
iM vC1 vL 2 iC 2

Lm vT v2 D1 C2 R v
Vg

图 6-19 晶体管关断时的等效电路
故有:
v2
vT 
n
v2  vC1
vL 2   v
iC1  i2 (6-22)
V
I L2 
R
i1
i2 
n
i1  iM

- 144 -
使用小纹波近似(注意两个子区间段的 V2 不一样,使用小纹波近似时需要使

用各自区间段的量,一定不能混用),有:
iI
vC1  VC1
iL 2  I L 2 (6-23)
v V
v2  V2
将式(6-23)代入到式(6-21)可得:
vT  Vg
vL 2  VC1  V  nVg (6-24)
V
iC1  
R
将式(6-23)代入到式(6-22)可得:
VC1
vT  
n
vL 2  V (6-25)
IM
iC1 
n

对激磁电感 Lm 和电感 L2 使用伏秒平衡,有:

VC1
D(Vg )  (1  D)(  )0
n (6-26)
D(VC1  V  nVg )  (1  D)( V )  0

由式(6-26)可得:
nDVg
VC1 
1 D
(6-27)
nDVg
V
1 D

对电容 C1 、 C2 使用电荷平衡,有:

V I nDV
D( )  (1  D)( M )  0  I M 
R n (1  D) R
(6-28)
V
I L2 
R
(b) 当晶体管导通时,原副边绕组的电流为:

- 145 -
V
i1  ni2  n
R
V
i2  iL 2  I L 2  (6-29)
R
nV
ig  i1  iM 
(1  D) R

当晶体管关断时,原副边绕组的电流为:
i1  iM   I M
i1  I M
i2   (6-30)
n n
ig  0

而实际中流经原副边绕组的电流为 ig 和 i2 ,其波形如图 6-20 所示:

ig

nV
(1  D) R

0 DTS TS t
i2

V
R

0 DTS TS t
DV

(1  D) R
图 6-20 原副边绕组电流波形
由图 6-20 可求出原副边绕组电流的有效值为:
n DV
I g _ rms 
(1  D) R
(6-31)
DV
I 2_ rms 
(1  D) R
原副边绕组电流的比值为:
I g _ rms n
 (6-32)
I 2_ rms (1  D)

- 146 -
Problem 6.4

如图 6-21 所示的双管反激变换器有时使用在直流输入电压非常高的场合。
晶体管 Q1 、 Q2 使用同一驱动信号驱动,故 Q1 和 Q2 同时开通或关断,并产生相同

的占空比 D 。二极管 D1 、 D2 用来保证晶体管的阻断电压不超过 Vg 。变换器运行

在 DCM 模式。原边绕组侧的激磁电感值为 LM 。

1: n

D3
D1 Q1

Vg C R v

D2 Q2

图 6-21 双管反激变换器
(a) 推导稳态时的输出电压表达式。
(b) 占空比在什么范围时变压器会正确得复位?
Solution:
(a) 变压器的等效电路如图 6-22 所示:
1: n

D1 Q1
D3

Vg Lm C R v

D2 Q2

图 6-22 双管反激变换器的等效电路

当 Q1 、 Q2 导通时,即 0  t  D1TS 时,等效电路可简化为如图 6-23 所示:

- 147 -
1: n
D3

im iC
Vg Lm vT v2 C R v

图 6-23 Q1 、 Q2 导通时的等效电路

故有:
vT  Vg
V (6-33)
iC  
R

当 晶 体 管 Q1 、 Q2 关 断 时 , 且 激 磁 电 感 的 电 流 在 断 续 之 前 时 , 即

D1TS  t  ( D1  D2 )TS 时,等效电路可简化为如图 6-24 所示:

D3 i2
1: n
im iC
Lm vT C R v

图 6-24 Q1 、 Q2 关断且激磁电感电流断续之前的等效电路

故有:
V
vT  
n
(6-34)
V
iC  i2 
R
当 晶 体 管 Q1 、 Q2 关 断 时 , 且 激 磁 电 感 的 电 流 在 断 续 之 后 时 , 即

( D1  D2 )TS  t  TS 时,等效电路可简化为如图 6-25 所示:

1: n
im iC
Lm vT v2 C R v

图 6-25 Q1 、 Q2 关断且激磁电感电流断续之后的等效电路

- 148 -
故有:
vT  0
V (6-35)
iC  
R
对激磁电感使用伏秒平衡,有:
V
1 g  D2 ( 
DV )  D3  0  0
n (6-36)
D3  1  D1  D2

由式(6-36)可推出:
nDV
V 1 g
(6-37)
D2

由于电容电流的直流分量为 0,故二极管电流的直流分量就等于负载电流的
直流分量,即:
V
I2  (6-38)
R
根据图 6-23、图 6-24 和图 6-25 可画出二极管电流的波形,如图 6-26 所示:
i2 (t )
i2 pk

I2
0 t

D1TS D2TS D3TS D1TS


图 6-26 二极管电流波形
由图 6-26 可求出二极管电流的直流分量为:
1 1 
I2   D2TS i2 pk  (6-39)
TS  2 
激磁电感电流的波形如图 6-27 所示:
im (t )
impk

0 t

D1TS D2TS D3TS D1TS


图 6-27 激磁电感电流波形

- 149 -
故可求的:
impk D1TSVg
Vg  Lm  impk  (6-40)
D1TS Lm

而激磁电感电流的峰值与二极管电流峰值的关系为:
impk
n (6-41)
i2 pk

由式(6-38)、式(6-39)、式(6-40)和式(6-41)可求出:
2nLmV
D2  (6-42)
D1RTSVg

将式(6-42)代入到式(6-37)中,可求得输出电压的表达式为:
DV RTS DV
V 1 g
 1 g

2 Lm K
(6-43)
2L
K m
RTS
(b) 为了保证变压器正确得复位,故有:
D3  1  D1  D2  0 (6-44)

将式(6-43)代入式(6-42)求出可求出:

D2  n K (6-45)

再将式(6-45)代入到式(6-44),可求出:

D1  1  n K (6-46)

此外为了保证能量能传送至负载还需要求:
V
 Vg (6-47)
n
将式(6-37)代入到式(6-47)可求出:

D1  D2  n K (6-48)

综上所述,占空比的范围应为:

D1  1  n K 且 D1  n K (6-49)

Problem 6.5

如图 6-28 所示的非理想反激变换器运行在 CCM 模式,MOSFET 具有导通


电阻 Ron ,二极管具有恒定的导通压降 VD ,反激变换器原边绕组的电阻为 R p ,

- 150 -
副边绕组的电阻为 Rs 。

(a) 推导 CCM 模式下稳态情况的等效电路,该等效电路可以正确得描述损耗。


(b) 推导变换器效率的表达式。
1: n D1

Lm C V
Vg
Q1

图 6-28 反激变换器
Solution:
(a) 当晶体管导通时,即 0  t  DTS 时,电路如图 6-29 所示:

1: n

Rp
Rs iC
im
Lm vT C V
Vg
Q1
Ron

图 6-29 晶体管导通时的等效电路
故有:
vT  Vg  ( R p  Ron )im
V (6-50)
iC  
R

当晶体管关断时,即 DTS  t  TS 时,电路如图 6-30 所示:

- 151 -
1: n
VD i
D
Rp
Rs iC
im
Lm vT v2 C V
Vg

图 6-30 晶体管关断时的等效电路
故有:

v2  V  VD  Rs iD
im
n
iD
vT 1 (6-51)

v2 n
V
iC  iD 
R
对激磁电感和输出电容使用伏秒平衡和电荷平衡,有:
nV  nVD  Rs I m
D Vg  ( R p  Ron )im   (1  D)( )0
n2
(6-52)
V I R  nV
D( )  (1  D)( m )0
R nR
将上式化简,可得:
nDVg   DV  DVD  DRS I D   (n 2 DR p  n 2 DRon ) I D  0
V (6-53)
DI D 
R
根据式(6-53)可画出等效电路,如图 6-31 所示:
DVD DRs  n 2 D ( R p  Ron )
ID DI D

nDVg DV R V

D :1
图 6-31 非理想反激变换器等效电路
而图 6-31 反应了占空比和输出端口的特性,并没有反应输入端口和变压器
匝数比特性,由于:

- 152 -
Ig  DI m
Ts
(6-54)
I m  nI D
故利用式(6-54),图 6-31 可将输入端口特性分离出来,如图 6-32 所示:

Ig DVD DRs  n 2 D ( R p  Ron )


TS ID DI D

Vg nDVg DV R V

1: nD D :1

图 6-32 非理想反激变换器的完整等效电路
(b) 将图 6-32 所示的完整等效电路进一步化简,可得到简化的等效电路,如
图 6-33 所示:

DVD DRs  n 2 D ( R p  Ron )


ID

nDVg D2 R DV

图 6-33 非理想反激变换器简化等效电路
由图 6-33 可知:
nDVg I D D Vg
 n (6-55)
DVI D D V

而:
nDVg  DVD   DRS  n 2 D  Rp  Ron   I D  DV
DV (6-56)
ID  2
D R
由式(6-55)和式(6-56),可求出效率的表达式为:
nDVg ( DRS  D2 R  n 2 DR p  n 2 DRon )
 (6-57)
nDD2 RVg  D3 RVD

Problem 6.6

可以使用同步整流器作为计算机的低压电源。目前数字集成电路的趋势是使
用越来越低的供电电压,而在低压输入下的达到高效率却是难度很大,这是由于
副边二极管的导通损耗很大。因此,此题的一个任务是评估正激变换器的效率随
着输入电压的降低会发生如何变化,此外再对同步整流器进行评估。
对于如图 6-34 所示的正激变换器,其输入电压 Vg  325V ,提供的负载电流

- 153 -
为 20A ,考虑以下三种情况: (i) V  5V, (ii) V  3.3V, (iii) V  1.5V 。对于所述的

每一种情况,选择匝数比 n3 / n1 使得变换器在可以提供所需输出电压的基础上保

持占空比 D  0.4 。MOSFET 的导通电阻为 Ron  5 ,副边肖特基二极管的正向

压降 VF  0.5V ,其他的所有元素均可视为理想元器件。

(a) 推导正激变换器的等效电路,该等效电路可以反映出损耗情况。
(b) 求解所述的三种情况下的匝数比 n3 / n1 和效率。

(c) 当副边的肖特基二极管被 MOSFET 取代,并且变换器以同步整流方式运


行,此 MOSFET 的导通电阻为 Ron  4mΩ ,再次求解所述的三种情况下匝数比和

效率。

n1 : n2 : n3 D2 L

D3 C R V
Vg

Q1 D1

图 6-34 正激变换器
Solution(假设电感 L 的值很大,即其电流工作在连续模式):
(a) 除了原边侧 MOSFET 具有导通电阻,副边侧肖特基二极管具有正向压降
之外,其余元器件根据题意视为理想元器件,故变压器的激磁电感相当于无穷大,
变压器为理想变压器。
在此基础上,当晶体管导通时,即 0  t  DTS 时,等效电路如图 6-35 所示:

I1 n1 : n2 : n3 VF L IL
vL iC
v1 v2 C R V
Vg

Ron

图 6-35 晶体管导通时的等效电路
故有:

- 154 -
v1  Vg  I1 Ron
I1 n3

I L n1
V
iC  I L  (6-58)
R
vL  v2  V  VF
v1 n1

v2 n3

当晶体管关断时,即 DTS  t  TS 时,等效电路如图 6-36 所示:

n1 : n2 : n3 L IL
vL iC
v2 VF C R V
Vg

图 6-36 晶体管关断,激磁电感电流断续之前时的等效电路
故有:
vL  V  VF
V (6-59)
iC  I L 
R
对滤波电感和输出电容使用伏秒平衡和电荷平衡,有:
2
n n 
D 3 Vg  V  VF  D  3  Ron I L
n1  n1  (6-60)
V
IL 
R
而输入电流的平均值为:
n3
I g  DI1  D IL (6-61)
n1

根据式(6-60)和式(6-61)可画出等效电路如图 6-37 所示:

- 155 -
n3 n3
Ig  D IL 1: D VF IL
n1 n1

n3
Vg Vg D Vg R V
n1

2
n 
D  3  Ron
 n1 
图 6-37 正激变换器等效电路
图 6-37 所示的等效电路可进一步简化,如图 6-38 所示:
VF IL

n3
D Vg R V
n1

2
n 
D  3  Ron
 n1 
图 6-38 正激变换器简化等效电路
(b) 根据图 6-38,有:
2
n n 
D 3 Vg  VF  V  D  3  Ron I L
n1  n1 
VI L V (6-62)
 
n n
D 3 Vg I L D 3 Vg
n1 n1
根据题中给定的参数和式(6-62),可求出三种情况下的匝数比和效率为:
n3
 0.04287   0.8971 for V  5V
n1
n3
 0.02950   0.8605 for V  3.3V (6-63)
n1
n3
 0.01546   0.7464 for V  1.5V
n1
(c) 当副边的肖特基二极管被 MOSFETs 取代后,简化的等效电路如图 6-39
所示:

- 156 -
RM IL

n3 RM  4mΩ
D Vg R V
n1

2
n 
D  3  Ron Ron  5
 n1 

图 6-39 肖特基二极管被 MOSFETs 取代后的简化等效电路


故有:
2
n n 
D 3 Vg  RM I L  V  D  3  Ron I L
n1  n1 
VI L V (6-64)
 
n n
D 3 Vg I L D 3 Vg
n1 n1
使用 MOSFETs 取代肖特基二极管后的三种情况下的匝数比和效率根据式
(6-64)可求出:
n3
 0.03956   0.9723 for V  5V
n1
n3
 0.02621   0.9685 for V  3.3V (6-65)
n1
n3
 0.01220   0.9458 for V  1.5V
n1

Problem 6.7

开关单元的旋转。如图 6-40 所示,一个包含开关和无功元器件的网络具有


3 个端点 a、b、c,端口电压满足 Vbc / Vac   ( D) 关系式。

(a) vbc
A a   ( D) b B
vac
vac c vbc
Vg v
C

(b) (c )
a 1 b a 1: n b

2
vac vbc
c vac vbc
1 2

图 6-40 三端开关单元的旋转

- 157 -
(a) 对于以下三种情况,推导电压转换比率的表达式 V / Vg  M ( D) ,其中

M ( D) 使用  ( D) 来表示。
i. a A bB c C
ii. aB b C c A
iii. a C b A cB

(b) 考虑图 6-40 中(b)的情况,推导  ( D) 表达式,并检查所得到的 Buck、

Boost 和 Buck-Boost 变换器的结果。


(c) 考虑图 6-40 中的(c)的情况,推导  ( D) 表达式,看看产生的变换器是

哪种变换器?
Solution:
(a) 对于 i. 所述的情况,即 a  A b  B c  C ,有:
Vg  vac
V  vbc (6-66)
V vbc
M ( D)     ( D)
Vg vac
对于 ii. 所述的情况,即 a  B b C c  A ,有:
Vg  vcb
V  vab (6-67)
V vab vac  vbc v 1
M ( D)     1  ac  1 
Vg vcb vbc vbc  ( D)
对于 iii. 所述的情况,即 a  C b A c  B ,有:
Vg  vba
V  vca (6-68)
V vca vac 1 1
M ( D)     
Vg vba vbc  vac vbc  vac 1   ( D)
vac
(b) 对于图 6-40 中的(b),认为开关位于位置 1 的占空比为 D ,当开关位于
位置 1,即 0  t  DTS 时,有:

vL  vab  vac  vbc (6-69)

当开关位于位置 2 时,即 DTS  t  TS 时,有:

vL  vcb (6-70)

- 158 -
对电感 L 使用伏秒平衡,有:
D(vac  vbc )  (1  D)(vcb )  0
vbc vcb (6-71)
 ( D)   D
vac vac

当此开关单元使用 a  A b  B c  C 这一连接方式时,所得到的是 Buck


变换器,根据式(6-66)和式(6-71)可求得:
M ( D)   ( D )  D (6-72)

当 此 开关 单 元使 用 a  B b C c  A 这 一 连接 方 式时 ,所 得 到的 是
Buck-Boost 变换器,根据式(6-67)和式(6-71)可求得:
1 1
M ( D)  1   1 (6-73)
 ( D) D
当此开关单元使用 a  C b  A c  B 这一连接方式时,所得到的是 Boost
变换器,根据式(6-68)和式(6-71)可求得:
1 1 1
M ( D)    (6-74)
1   ( D) 1  D D
(c) 对于图 6-40 中的(c),其等效电路如图 6-41 所示:
a 1: n b

vT Lm

vac vbc
1 2

c
图 6-41 图 6-40 中(c)的等效电路

当 0  t  DTS 时,即开关位于位置 1 时,激磁电感上的电压为:

vT  vac (6-75)

当 DTS  t  TS 时,即开关位于位置 2 时,激磁电感上的电压为:

vbc
vT   (6-76)
n
对激磁电感使用伏秒平衡,有:
vbc
Dvac  (1  D)( )0
n
(6-77)
v nD
 ( D)  bc 
vac 1  D

- 159 -
当此开关单元使用 a  A b  B c  C 这一连接方式时,所得到的是反激变
换器,根据式(6-66)和式(6-77),可求得:
nD
M ( D)   ( D)  (6-78)
1 D
当此开关单元使用 a  B b  C c  A 这一连接方式时,根据式(6-67)和式
(6-77),可求得:
1 1 (n  1) D  1 D
M ( D)  1   1   1 (6-79)
 ( D) nD nD nD
1 D
当此开关单元使用 a  C b  A c  B 这一连接方式时,根据式(6-68)和式
(6-78),可求得:
1 1 1 D
M ( D)    (6-80)
1   ( D) 1  nD 1  (n  1) D
1 D

Problem 6.8

使用变压器隔离的电流采样电路。实际中经常需要对晶体管的导通电流进行
采样,一个与变压器副边相串联的无感电阻用来产生与晶体管漏极电流 iD (t ) 成

正比的电压 v(t ) 。通过变压器来获得功率晶体管与控制电路的隔离。变压器的匝

数比可以降低采样电阻上的电流与损耗,同时也可以增大采样电阻上的电压 v(t ) 。

此题将集中于如图 6-42 所示的具有变压器隔离的电流采样电路的设计。

1: n D1

DZ VZ v(t )
R
iD (t )
D2
Q1

图 6-42 具有变压器隔离的电流采样电路
变压器具有匝数为 1 的原边,和匝数为 n 的副边,晶体管运行时的占空比为
D ,其开关频率为 f S ,当晶体管导通时,其电流为固定值 I ,二极管 D1 、 D2 为

普通的具有正向压降 VD 的硅二极管。二极管 DZ 为齐纳二极管,可以用极性如图

6-42 所示的电压源 VZ 进行建模,为了获得一个合适的设计,变压器的磁化电流

和二极管 D2 工作在 DCM 模式。在已优化好的设计当中,磁化电流远小于晶体

- 160 -
管的导通电流。该电路对于每一个开关周期来说,具有 3 个子区间; Q1 和 D1 导

通时,为第一子区间; D2 和 DZ 导通时为第二子区间; Q1 、 D1 和 D2 均关断时为

第三子区间。
(a) 使用变压器和齐纳二极管的等效电路重新画出电流采样电路。
(b) 画出晶体管导通电流 iD (t ) 、变压器磁化电流 iM (t ) 、原边绕组电压和采样

电阻上的电压 v(t ) 的波形,标出它们的静态特性。

(c) 为了保证变压器磁化电流在开关周期结束之前成功复位,齐纳二极管的电
压 VZ 应满足什么条件?

(d) 给定以下指标:
开关频率 fS  1 0 0 k H z

晶体管占空比 D0.75

晶体管最大电流 m a xiD t ( ) 25 A

当晶体管电流达到 25A 时,输出电压 v(t ) 应为 5V ,为了避免变压器铁芯的

饱和,当晶体管导通时施加在 1 匝的原边绕组上的伏秒数不能超过 2volt- sec ,

硅二极管的正向压降为 VD  0.7V 。

设计电路:选择 R 、 n 和 VZ 的值。

Solution:
(a) 使用变压器和齐纳二极管的等效电路重画电流采样电路,如图 6-43 所示:
1: n D1
i2
iM
VZ
v1 LM vT R v(t )
iD (t ) D2
Q1

图 6-43 电流采样电路的等效电路
(b) 各电量的波形如图 6-44 所示,并已在图中标出其静态特性:

- 161 -
iD (t ) v1 (t )
V  VD
I
n
0
t VZ  VD t
iM (t ) 
n
I v(t )

0 t V
D1TS D2TS D3TS t
0
D1TS D2TS D3TS

图 6-44 各个电量波形
(c) 激磁电感的电压波形如图 6-45 所示:

vT (t )

VD  V
0
t
VD  VZ

D1TS D2TS D3TS

图 6-45 激磁电感的电压波形
对激磁电感使用伏秒平衡,可得:
D1 (VD  V )  D2 (VD  VZ )  0 (6-81)

当晶体管导通时,有:
I
n
I2
iM I2 (6-82)
I2 R  V

根据式(6-81)和式(6-82),可得:
D1 ( IR  nVD )
D2  (6-83)
n(VD  VZ )

为了使变压器正确得复位,故有:
D1  D2  1 (6-84)

将式(6-83)代入到式(6-84),可得:
D1 IR  nVD (2 D1  1)
VZ  (6-85)
n(1  D1 )

- 162 -
(d) 当晶体管电流达到 25A 时,输出电压 v(t ) 应为 5V ,故根据式(6-82)可得到

第一个条件:
V n I 25
I n    5 (6-86)
R R V 5
又由于晶体管导通时施加在 1 匝的原边绕组上的伏秒数不能超过 2volt- sec ,

可得到第二个条件为:
V  VD 5  0.7 1
 D1TS  2 106   0.75   2 106 (6-87)
n n 100 103
将式(6-87)化简可得到第二个条件为:
n  21.375 (6-88)
为了保证变压器正确得复位,必须要满足式(6-85),将式(6-86)代入到式(6-85),
可得到第三个条件为:
D1 I  3.5  (2 D1  1)
VZ  for 0  D1  0.75 0  I  25 (6-89)
5(1  D1 )

式(6-89)表达的是,对于一定范围内的占空比和电流,电压 VZ 要大于式(6-89)

右半部分的最大值。
将式(6-89)的右半部分看成一个二元函数 f ( D1 , I ) ,则有:

f ( D1 , I ) D1

I 5  5D1
(6-90)
f ( D1 , I ) 5(7 D1  D1I  3.5) 7  I
 
D1 (5  5D1 ) 2 5  5D1

如果二元函数 f ( D1 , I ) 在某一点处取得极值,则式(6-90)中的两个子式要同

时等于 0。令式(6-90)中的两个子式分别等于 0,则可求得:


D1  0
(6-91)
I  3.5
根据式(6-91)可知,二元函数 f ( D1 , I ) 不会在 0  D1  0.75 0  I  25 范围内

取得极值,故二元函数 f ( D1 , I ) 的最大值一定在此范围的边界处取得,则有:

f (0, I ) max  0.7 for 0  I  25


f (0.75, I ) max  16.4 for I  25
(6-92)
f ( D1 , 0) max  1.4 for D1  0.75
f ( D1 , 25) max  16.4 for D1  0.75

- 163 -
根据式(6-92),可得到第三个条件为:
VZ  16.4V (6-93)

故 R 、 n 和 VZ 的值应同时满足式(6-86)、式(6-88)和式(6-93)。 R 、 n 和 VZ 的

值可选择为:
n  22
R  4.4 (6-94)
VZ  16.4V

Problem 6.9

正激变换器复位优化。如图 6-46 所示,可以不利用输入电压 Vg ,使用另一

个电压源来复位正激变换器。相关文献中有很多类似的策略。通过选择复位电压
Vr 的值,晶体管 Q1 和二极管 D2 的电压应力可降低。占空比的最大值也可以增加,

以降低变压器的匝数比和晶体管的导通电流。特别是当直流输入电压的变化范围
较宽时,通过这种方法可以在变压器成本及效率方便得到很大改善。
(a) 如果将 Vr 看成是输入电压 Vg 、晶体管占空比 D 和变压器匝数比的函数,

那么能使变压器磁化电流在开关周期结束之前正确复位的 Vr 最小值为多少?

(b) 对于(a)中选择的 Vr 值,晶体管 Q1 所受的电压应力是多少?

根据(a)小问选定的 Vr 值,并且此变换器运行在以下参数。

输入电压 Vg 的变化范围为 127 至 380 V ,通过占空比的调节使输出电压稳定

在 12V ,负载功率是 480W 。


(c) 选择匝数比 n3 / n1 使得总开关应力最小,对于此时的匝数比 n3 / n1 ,占空比

的变化范围是多少?晶体管的峰值电流是多少?
(d) 通用的选择策略是让 n1  n2 , Vr  Vg ,对比此通用策略和(c)中所得到

的策略。此外比较两种策略中的最恶劣情况下的晶体管峰值电压和峰值电流。
(e) 提供一种实现电压源 Vr 的方法。给出实现方式的原理图,如果还需另外的

控制电路的话,还请用简短的语言描述一下。

- 164 -
n1 : n3 D2

D3 R v
Vg

Q1
D1
Vr

: n2

图 6-46 具有额外复位绕组的正激变换器
Solution:(c、d、e 这 3 小问,目前还不能给出确定的答案)
(a) 图 6-46 的等效电路如图 6-47 所示:
n1 : n3 D2
im vL

Lm vT v3 D3 R v
Vg

Q1
D1
v2 Vr

: n2

图 6-47 具有额外复位绕组的正激变换器等效电路

当晶体管导通时,即 0  t  D1TS 时,有:

vT  Vg (6-95)

当晶体管关断,且激磁电感中的电流未复位之前,即 D1TS  t  ( D1  D2 )TS 时,

有:
n1
vT   Vr (6-96)
n2

当晶体管关断,且激磁电感中的电流复位之后,即 ( D1  D2 )TS  t  TS 时,有:

vT  0 (6-97)

根据式(6-95)、式(6-96)和式(6-97),对激磁电感使用伏秒平衡,有:
 n  n DV
D1Vg  D2   1 Vr   0  D2  2 1 g (6-98)
 n2  n1Vr

- 165 -
为了使变压器正确的复位,有:
D1  D2  1 (6-99)

将式(6-98)代入到式(6-99),可得:
n2 DV
Vr  1 g
(6-100)
n1 (1  D1 )

则 Vr 的最小值根据式(6-100),有:

n2 DV
Vr  1 g
(6-101)
min
n1 (1  D1 )

(b) 当 D1TS  t  ( D1  D2 )TS 时,晶体管会受到电压应力,此电压应力为:

n1
Vstress  Vg  Vr (6-102)
n2

将式(6-101)代入到式(6-102),可得:
n1 V
Vstress  Vg  Vr  g (6-103)
n2 1  D1

(c) 晶体管 Q1 的电压电流波形如图 6-48 所示:

VQ1

Vg
1  D1

0
I Q1
n3 V
n1 R

0
D1TS D2TS

图 6-48 三个半导体器件的电压电流应力
根据图 6-48 可求得总的有效开关应力为:
 V   n V
S  VQ1 I Q1_ rms   g    D1  3  (6-104)
 1  D1   n1 R 

对滤波电感使用伏秒平衡有:

- 166 -
n3 n V
D1 ( Vg  V )  (1  D1 )(V )  0  D1  1 (6-105)
n1 n3 Vg

将式(6-105)代入到式(6-104)中,有:
Vn1 V
VVg 2 n32 VVg 2 k 2
Vg n3 Vg k n3
S  k (6-106)
(n3Vg  n1V ) Rn1 (Vg k  V ) R n1

令 Vg k  T ,则有:

V
VT 2
S T (6-107)
(T  V ) R

dS
令  0 ,则有:
dT
T  3V
(6-108)
并且当 T  3V 时, S 取极小值,故为了使总的有效开关应力最小,有:
n3 3V 36
  (6-109)
n1 Vg Vg

由于输入电压在一定范围内变动,所以使总的有效开关应力最小的匝数比并
不是固定值,并且得到式(6-109)的前提条件为式(6-103)。
(d) 当 n1  n2 ,且 Vr  Vg 时,晶体管的峰值电压根据式(6-102),有:

Vpeak  2Vg (6-110)

故当输入电压达到最大值 380V 时,晶体管的峰值电压为 760V 。


晶体管的峰值电流为:
V n3
I peak  (6-111)
R n1

(e) 根据(c)中的回答,难道 Vr 的由反激变换器得到?

Problem 6.10

设计多输出 DCDC 反激变换器。对于此问题,你可以忽略所有的损耗和变


压器漏感。所需的反激变换器如图 6-49 所示,此变换器具有三个输出,并且以
f S  100kHz 的开关频率运行在 DCM 模式。额定的运行条件在图 6-49 中已给出,

你可以认为输入电压或者负载电流是恒定的。设定占空比 D3  0.1 (此占空比时

- 167 -
间内 ,所有的半导体都是关断状态)。此题的任务是尽可能得使半导体的峰值阻
断电压和峰值电流小。

ip n p : n1
i1
15V
1A
Vg
: n2
165V dc
i2
15V
0.5A

: n3
i3
5V
4A

图 6-49 三输出反激变换器
(a) 寻找一组设计,稳态时使晶体管阻断的峰值电压小于 300V ,所有的二极
管的阻断电压均小于 35V ,并给出此设计的晶体管占空比 D 、折算于原边侧的
激磁电感 Lm 和匝数比 n1 / np 、 n3 / n p 。

(b) 对于(a)中的设计,求解四个绕组内的电流有效值,注意它们的关系并
不仅仅是匝数比的关系。
Solution:
(a) 由于三组输出是同步的,即副边绕组的电流同一时刻上升,同一时刻下降。
故设计时只考虑其中一组即可。故其简化的等效电路如图 6-50 所示:

ipk n p : nk
imk ik
Vk
vk
Lmk vT Ik

Vg
165V dc

图 6-50 简化的三输出反激变换器等效电路

当晶体管导通时,即 0  t  D1TS ,有:

vT  Vg (6-112)

- 168 -
当晶体管关断,且激磁电感电流断续之前,即 D1TS  t  ( D1  D2 )TS 时,有:

np
vT   Vk (6-113)
nk

对激磁电感(对于每组输出,都有各自对应的激磁电感)使用伏秒平衡有:
np DV
1 g nk
1 g  D2 ( 
DV Vk )  0  D2  (6-114)
nk Vk n p

晶体管的阻断电压为:
np
VQ  Vk  Vg  300V (6-115)
nk

二极管的阻断电压为:
nk
VD  Vk  Vg  35V (6-116)
np

由于设定占空比 D3 的值,故有:

D1  D2  0.9 (6-117)

由式(6-117)和式(6-114),有:
np DV
 1 g
(6-118)
nk (0.9  D1 )Vk

将式(6-118)代入到式(6-115)和式(6-116),有:
D1  0.9  0.003Vg  0.405
0.9 (6-119)
D1  Vk
35

由于 Vk  15V 或 Vk  5V ,故由式(6-119)可知 D1 的范围应为:

0.9
15  0.386  D1  0.405 (6-120)
35
根据式(6-120),选定 D1  0.4 ,可保证晶体管阻断电压和二极管阻断电压在

给定范围内。
选定占空比 D1  0.4 之后,可根据式(6-118)确定匝数比,即有:

- 169 -
np D1Vg

nk (0.9  D1 )Vk
np
 8.8 for V1  15V (6-121)
n1
np
 26.4 for V3  5V
n3
故根据式(6-121)可确定匝数比为:
n1 15

n p 132
(6-122)
n3 5

n p 132
对于 3 组输出中的任意一组,二极管电流波形如图 6-51 所示:
iDk (t )

iDk _ peak

I Dk

0 t

0.4TS 0.5TS 0.1TS


图 6-51 二极管电流波形
由于二极管电流的直流分量等于负载电流的直流分量,故有:
1 1 2I
I k  I Dk  D2TS iDk _ peak  iDk _ peak  k  4 I k (6-123)
TS 2 D2

而激磁电流峰值与二极管电流峰值的又要满足:
imk _ peak nk
 (6-124)
iDk _ peak np

又由于:
im _ peak  i
k 1,2,3
mk _ peak (6-125)

故根据式(6-123)、式(6-124)和式(6-125),可求出激磁电感电流的峰值为:
nk  15 15 5 
im _ peak  
k 1,2,3
4
np
Ik  4  
 132
1 
132
 0.5   4   1.288A
132 
(6-126)

而当晶体管导通时,有:
im _ peak
Vg  Lm  Lm  0.512mH (6-127)
D1TS

- 170 -
要注意的是,当激磁电感的电流达到峰值时,变压器不能饱和,因为饱和后,
激磁电感的值会变得极小。
(b) 对于图 6-51 所示的波形,其有效值为:
( D1  D2 )TS

 (t )dt  I Dk
2 2
iDk _ rmsTS (6-128)
D1TS

对于(6-128)的左半部分有:
( D1  D2 )TS ( D1  D2 )TS 2
 I Dk _ peak 
 i (t )dt    (t  0.9TS )  dt 
2
Dk
D1TS D1TS  0.5TS  (6-129)
2
I Dk _ peak
2
 0.243TS 3
0.25TS
将式(6-129)和式(6-123)代入到式(6-128),可求出:

1 1
I Dk _ rms  I Dk _ peak   4 I Dk (6-130)
6 6
由式(6-130)可求出副边绕组电流的有效值为:
I D1_rms  1.63A
I D 2 _rms  0.82A (6-131)
I D 3_rms  6.53A
原边激磁绕组中的电流波形如图 6-52 所示:
im (t )

im _ peak

0 t

0.4TS 0.5TS 0.1TS


图 6-52 原边激磁绕组电流波形
则激磁绕组电流的有效值可由下式求出:
( D1  D2 )TS

 i (t )  dt  I m2 _ rmsTS
2
m (6-132)
0

对于式(6-132)的左半部分,有:
( D1  D2 )TS 2
 im _ peak 
D1TS

 i (t )  dt   t  dt 
2
m 
0 0  D1TS 
( D1  D2 )TS 2
 im _ peak 

D1TS

 D2TS
(t  D1TS  D2TS )  dt

(6-133)

- 171 -
将式(6-133)简化,可得:
( D1  D2 )TS
( D1  D2 )TS im2 _ peak
 im (t ) dt 
2
(6-134)
0
3

将式(6-134)和式(6-126)代入到式(6-132),可求得:

( D1  D2 )
I m _ rms  im _ peak  0.705A (6-135)
3

Problem 6.11

SpreadSheet 设计(一种类似 Excel 的软件):


(a) 推导书中正激变换器设计范例 Table6.2 中“Results”和“Worst-case stress”
的表达式。
(b) 将(a)中得到的表达式键入到 SpreadSheet 软件中,并检验结果。
(c) 如果需要将正激变换器晶体管峰值电压设计成不能超过 650V ,修改设计
变量以达成此项需求,并讨论对其他应力的影响。
(d) 对于 CCM 正激变换器,晶体管最大利用率是多少?如何选择设计变量以
实现最大晶体管利用率设计?
Solution:
(a) 变换器的等效电路如图 6-53 所示:
n1 : n2 : n3 D2 L iL
im i1 i3 vL iC
vT v3 D3 C R V
Vg

Q1 D1

图 6-53 正激变换器等效电路
当变换器运行于 CCM 模式(指滤波电感 L 的电流不断续,而非指激磁电感
中的电流)时,且晶体管 Q1 导通时,即 0  t  DTS 时,等效电路如图 6-54 所示:

n1 : n2 : n3 D2 L iL
im i1 i3 vL iC
vT v3 C R V
Vg

Q1

图 6-54 CCM 模式,晶体管导通时的等效电路

- 172 -
故有:
vT  Vg
vT n1

v3 n3
vL  v3  V
V (6-136)
IL 
R
i3  iL  I L
i1 n3

i3 n1

当变换器运行于 CCM 模式时,且晶体管关断时,即 DTS  t  TS 时,等效电

路如图 6-55 所示:

n1 : n2 : n3 L iL
im i1 i3 vL iC
vT v3 D3 C R V
Vg

D1

图 6-55 CCM 模式,晶体管关断时的等效电路


故有:
n1
vT   Vg
n2
vL  V (6-137)
V
IL 
R
对滤波电感 L 使用伏秒平衡,有:
n1V  n3Vg n1V
D(  )  (1  D)(V )  0  D  (6-138)
n1 n3Vg

故对于 CCM 模式占空比的最大值和最小值为:


n1V
Dmax 
n3Vg _ min
(6-139)
n1V
Dmin 
n3Vg _ max
滤波电感 L 的电压电流波形如图 6-56 所示:

- 173 -
vL iL

n3 2iL
Vg  V
n1
DTS TS DTS TS
0 t 0 t
V

图 6-56 CCM 模式,滤波电感的电压电流波形


故滤波电感的电流纹波大小为:
2iL T V (1  D)
V  L  iL  S (6-140)
(1  D)TS 2L

故对于 CCM 模式,滤波电感的电流纹波最大值为:


T V (1  Dmin )
iL _ max  S (6-141)
2L
为了保证滤波电感的电流纹波最大值小于设定值,故有:
TSV (1  Dmin ) T V (1  Dmin )
iL _ max   iL _set  L  S (6-142)
2L 2iL _set

假设滤波电感电流的直流分量全部流过负载,则滤波电容的电流波形如图
6-57 所示:

iL

2iL

DTS TS
0 t
iC
DTS
TS
2iL
t

图 6-57 滤波电容的电流波形
故电容电压的纹波为:

dv 1 1 1 TS T 2V (1  D)
iC  C  2vC   iC dt  iL  vC  S (6-143)
dt C C2 2 16CL
故根据式(6-143),电容电压纹波的最大值为:

TS 2V (1  Dmin )
vC _ max  (6-144)
16CL
为了保证电容电压纹波的最大值小于设定值,故有:

- 174 -
TS 2V (1  Dmin ) T 2V (1  Dmin )
vC _ max   vC _ set  C  S (6-145)
16CL 16 LvC _ set

以上分析是建立在滤波电感电流连续的条件下,即:
V
IL   iL  K  (1  D)
R
(6-146)
2L
K
RTS

如果允许的滤波电感电流纹波较大,并且大于轻载时的负载电流,则变换器
会在轻载时处于 DCM 模式。
当变换器运行于 DCM 模式,且晶体管导通时,即 0  t  D1TS 时,变换器的

等效电路如图 6-58 所示:


n1 : n2 : n3 D2 L iL
im i1 i3 vL iC
vT v3 C R V
Vg

Q1

图 6-58 DCM 模式,晶体管导通时的等效电路


故有:
vT  Vg
vT n1
 (6-147)
v3 n3
vL  v3  V
当变换器运行于 DCM 模式,晶体管断开且滤波电感电流断续之前时,即
D1TS  t  ( D1  D2 )TS 时,变换器的等效电路如图 6-59 所示:

n1 : n2 : n3 L iL
im i1 i3 vL iC
vT v3 D3 C R V
Vg

D1

图 6-59 DCM 模式,滤波电感电流断续之前时的等效电路


故有:

- 175 -
vL  V (6-148)

当 变 换 器 运 行 于 DCM 模 式 , 且 滤 波 电 感 电 流 断 续 之 后 时 , 即
( D1  D2 )TS  t  TS ,变换器的等效电路如图 6-60 所示:

n1 : n2 : n3 L iL
im i1 i3 vL iC
vT v3 C R V
Vg

D1

图 6-60 DCM 模式,滤波电感电流断续之后时的等效电路


故有:
vL  0 (6-149)

对滤波电感使用伏秒平衡有:
n3Vg  n1V n1 D2V
D1 ( )  D2 (V )  0  D1  (6-150)
n1 n3Vg  n1V

滤波电感电流的波形如图 6-61 所示:

iL
iL _ peak

IL
0 TS t
D1TS D2TS

图 6-61 DCM 模式下的滤波电感电流波形


由于滤波电感的直流分量等于负载电流的直流分量,即:
V
IL  (6-151)
R
而滤波电感的直流分量为:
TS
1 1 1 
IL 
TS  i dt  T
0
L
S
 2 ( D1  D2 )TS iL _ peak  (6-152)

而 DCM 模式下的滤波电感电流峰值为:
iL _ peak D2TSV
V  L  iL _ peak  (6-153)
D2TS L

- 176 -
利用式(6-153)、式(6-152)和式(6-151),可求出占空比 D2 为:

D12  4 K  D1
D2  (6-154)
2

将式(6-154)代入到式(6-150),可求出占空比 D1 为:

K
D  D1  V 2
(6-155)
 n3  n3
 Vg   VgV
 n1  n1
故 DCM 模式下的最小占空比为:
K
Dmin  D1_ min  V 2
(6-156)
 n3  n3
 Vg _ max   Vg _ maxV
 n1  n1
故,占空比的最小值取式(6-139)和式(6-156)的较小者。
对于最恶劣情况下的应力,应考虑满载时的情况,即 CCM 模式下的满载情
况:
PLoad _ max
I L _ max  (6-157)
V

当 DTS  t  TS 时,晶体管 Q1 承受阻断电压,有:

n1
vQ1  vT  Vg  (  1)Vg (6-158)
n2

故晶体管 Q1 的峰值阻断电压为:

n1
vQ1_ max  (  1)Vg _ max (6-159)
n2

为了计算晶体管 Q1 电流有效值的最大值,假设激磁电感中的电流 im 远小于原

边绕组中的电流 i1 ,则晶体管 Q1 电流波形如图 6-62 所示:

- 177 -
iQ1

2iQ1 I Q1

0 DTS TS t

图 6-62 最恶劣情况下的晶体管 Q1 电流波形

最恶劣情况下,晶体管 Q1 电流的有效值达到最大,此时发生在满载且占空比

最大时,此时晶体管 Q1 电流的纹波 iQ1 和 IQ1 ,有:

iQ1 I Q1 n3
 
iL IL n1
PLoad _ max
IL 
V (6-160)
T V (1  Dmax )
iL  S
2L
n1V
Dmax 
n3Vg _ min
则根据书中附录 Appendix A 中的式(A.6)及式(6-160),最恶劣情况下的晶
体管电流有效值的最大值为:
2
1  i 
I Q1_ rms  I Q1 Dmax 1   Q1  (6-161)
3  I Q1 
将式(6-160)代入到式(6-161),有:

n iL2
I Q1_ rms  3 Dmax I 
2
L (6-162)
n1 3
根据式(6-159)和式(6-162),可求出开关利用率为:
PLoad _ max PLoad _ max
U  (6-163)
S vQ1_ max  I Q1_ rms

当 DTS  t  TS 时,二极管 D2 所承受的阻断电压为:

n3
vD 2  Vg (6-164)
n2

故二极管 D2 所承受的峰值阻断电压为:

- 178 -
n3
vD 2 _ max  Vg _ max (6-165)
n2

最恶劣情况下,二极管 D2 的电流波形如图 6-63 所示:

iD 2

2iL IL

DTS TS
0 t

图 6-63 最恶劣情况下,二极管 D2 的电流波形

故最恶劣情况下二极管 D2 电流的有效值达到最大,此时发生在满载且占空

比最大时,根据书中附录 Appendix A 中的式(A.6)可求出:


2
1  i 
I D 2 _ rms  I L Dmax 1  L 
3  IL 
PLoad _ max
IL 
V (6-166)
T V (1  Dmax )
iL  S
2L
n1V
Dmax 
n3Vg _ min

当 0  t  DTS 时,二极管 D3 所承受的阻断电压为:

n3
vD 3  Vg (6-167)
n1

故二极管 D3 所承受的峰值阻断电压为:

n3
vD 3_ max  Vg _ max (6-168)
n1

最恶劣情况下,二极管 D3 的电流波形如图 6-64 所示:

- 179 -
iD 3

2iL IL

0 DTS TS t

图 6-64 最恶劣情况下,二极管 D3 的电流波形

最恶劣情况下,二极管 D3 电流有效值达到最大,此时发生在满载且占空比

最小时,根据书中附录 Appendix A 中的式(A.6),可求出最恶劣情况下二极管 D3

电流有效值为:
2
1  i 
I D 3_ rms  I L (1  Dmin ) 1   L 
3  IL 
PLoad _ max
IL 
V (6-169)
T V (1  Dmin )
iL  S
2L
n1V
Dmin 
n3Vg _ max
最恶劣情况下,输出滤波电容的电流波形如图 6-65 所示:
iC
DTS
TS
2iL
t

图 6-65 最恶劣情况下,输出滤波电容的电流波形
根据书中附录 Appendix A 中的式(A.9),可求出输出滤波电容电流的有效
值为:
iL
IC _ rms  (6-170)
3

根据式(6-170)可知,输出滤波电容电流有效值的最大值发生在 iL 取最大值

时,而 iL 达到最大值时根据式(6-142)可知发生在占空比最小时,故输出滤波电

容电流有效值的最大值为:

- 180 -
iL _ max
I C _ rms 
3
T V (1  Dmin )
iL _ max  S (6-171)
2L
n1V
Dmin 
n3Vg _ max

(b) 由于没有使用过 SpreadSheet 软件,这里改为用 Matlab 进行计算,使用


Matlab 计算后将结果存入 Excel 文件中,计算结果如图 6-66 所示,此结果与书
中的结果一致。

图 6-66 Matlab 程序计算结果


Matlab 计算所使用的程序文件为:

(c) 为了使晶体管 Q1 的峰值阻断电压小于 650V ,根据式(6-159)可求出:

n2
 1.5 (6-172)
n1

n2
令  1.5 ,重新使用 Matlab 计算,计算结果如图 6-67 所示:
n1

- 181 -
图 6-67 晶体管 Q1 峰值阻断电压不超过 650V 时的计算结果

(d) 为了简化分析,在忽略电感纹波的情况下,式(6-162)可简化为:
n3
I Q1_ rms  Dmax I L (6-173)
n1

将式(6-159)、式(6-173)代入到式(6-163)中,可得:
PLoad max
U (6-174)
1 n
(1  ) 3 Vg max I L Dmax
n2 n1
n1
对于最恶劣情况有:
PLoad max
IL 
Vo
(6-175)
n1V
Dmax 
n3Vg _ min

将式(6-175)代入到式(6-174)可得:
Vg minV
U (6-176)
1 n
(1  )Vg max 3
n2 n1
n1

1 1
为了使开关利用率达到最大,式(6-176)中的 (1  ) 应越小越好,而 (1  ) 的
n2 x
n1

n2 n
曲线如图 6-68 所示,故 的值取图 6-68 中的拐点较为合适,即 2  9 :
n1 n1

- 182 -
1.5

1.438

1.375

1.313

1
1 1.25
x

1.188

1.125

1.063

1
0 6.25 12.5 18.75 25 31.25 37.5 43.75 50
x

1
图 6-68 1  曲线图
x
n3
由式(6-176)可知,匝数比 越小可获得越大的开关利用率,而 x 的曲线如
n1

n3
图 6-69 所示,故 取 0.1 较为合适:
n1

0.5

0.438

0.375

0.313

x 0.25

0.188

0.125

0.063

0
0 0.031 0.063 0.094 0.125 0.156 0.188 0.219 0.25
x

图 6-69 x 曲线图
对于电感电流纹波的大小,现要求其为:
iL  1A (6-177)

使用 MATLAB 程序设计的结果如图 6-70 所示:

- 183 -
图 6-70 获取较大开关利用率时的设计

Problem 6.12

使用 SpreadSheet 设计如图 6-71 所示的隔离型 Cuk 变换器,要求设计出的


变换器满足书中 Table6.2 的指标。此外,此变换器需要在满载时以 CCM 模式运
行。
L1 C1a C1b L2

Vg Q1 D1 C2 R v

1: n
图 6-71 隔离型 Cuk 变换器
(a) 推导下列变量的表达式:
 CCM 模式下的最大最小占空比
 两个半导体器件的峰值阻断电压和电流有效值
 电容电压纹波和电感电流纹波
 电容电流有效值
 晶体管的开关利用率 U
(b) 将(a)小问推导出的表达式键入 SpreadSheet 中,考虑下应选择哪些量作
为设计变量?
(c) 对于书中 Table6.2 中的指标,选择你认为的最佳的设计变量的值以满足指
标要求。并将你设计的变换器的性能与 Table6.2 中的反激和正激变换器性能作比
较。
Solution:
(a) 变换器的等效电路如图 6-72 所示:

- 184 -
iL1 L1 iC1 C1a i1 i2 C1b iL 2 L2
vL1 vC1 im vC 2 vL 2 iC 2
Vg Q1 v1 v2 D C2 R v
1

1: n
图 6-72 隔离型 Cuk 变换器等效电路

当变换器运行于 CCM 模式时,且开关管 Q1 导通,即 0  t  DTS 时,等效电

路简化至如图 6-73 所示:


iL1 L1 iC1 C1a i1 i2 C1b iL 2 L2
vL1 vC1 im vC 2 vL 2 iC 2
Vg Q1 v1 v2 C2 R v

1: n
图 6-73 CCM 模式,开关管导通时的隔离型 Cuk 等效电路
故有:
VL1  Vg
I C1  I1
I1
 n
I2
I2  I L2 (6-178)
VL 2  VC 2  V2  V
V1  VC1
V1 1

V2 n

当变换器运行于 CCM 模式,且开关管 Q1 关断,即 DTS  t  TS 时,等效电路

简化至如图 6-74 所示:


iL1 L1 iC1 C1a i1 i2 C1b iL 2 L2
vL1 vC1 im vC 2 vL 2 iC 2
Vg v1 v2 D C2 R v
1

1: n
图 6-74 CCM 模式,开关管关断时的隔离型 Cuk 等效电路
故有:

- 185 -
VL1  Vg  V1  VC1
V1 1

V2 n
VL 2  V
V2  VC 2 (6-179)
I C1  I L1
I1
 n
I2
I C1  I1

对两电感和激磁电感使用伏秒平衡(对激磁电感使用伏秒平衡时,要假设励
磁电流远小于原边绕组上的电流)、对电容使用电荷平衡有:
VC 2  nVg  nVC1
D(Vg )  (1  D)( )0
n
D(nVC1  VC 2  V )  (1  D)(V )  0
VC 2
D(VC1 )  (1  D)( )0 (6-180)
n
D(nI L 2 )  (1  D)( I L1 )  0
V
I L2 
R
由式(6-180)可求出:
VC 2  V
(1  D)V
VC1 
nD
V
D
V  nVg
nDV (6-181)
I L1 
(1  D) R
V
I L2 
R
V nD
M ( D)  
Vg (1  D)
故 CCM 模式下,占空比的最大最小值为:
V
Dmax 
V  nVg min
(6-182)
V
Dmin 
V  nVg max

滤波电感 L1 的电压电流波形为如图 6-75 所示:

- 186 -
vL1 iL1

2iL1
Vg
DTS TS DTS TS
t 0 t
nDVg  V
nD

图 6-75 滤波电感 L1 的电压电流波形

故滤波电感 L1 的电流纹波大小为:

2iL1 DTSVg
Vg  L1  iL1  (6-183)
DTS 2 L1

将式(6-181)中的占空比 D 的表达式代入到式(6-183)中,可求出:
VTS
iL1  (6-184)
V
2 L1 (  n)
Vg

故当输入电压达到最大值时,电感 L1 的电流纹波最大,即:

VTS
iL1max  (6-185)
V
2 L1 (  n)
Vg max

为了保证电感电流纹波小于设计值,即:
VTS
iL1max   iL1_ set (6-186)
V
2 L1 (  n)
Vg max

故有:
VTS
L1  (6-187)
V
2iL1_ set (  n)
Vg max

滤波电感 L2 的电压电流波形如图 6-76 所示:

- 187 -
vL 2 iL 2

(1  D)V 2iL 2
D
DTS TS DTS TS
t 0 t
V

图 6-76 滤波电感 L2 的电压电流波形

故滤波电感 L2 的电流纹波为:

(1  D)V 2iL 2 (1  D)VTS


 L2  iL 2  (6-188)
D DTS 2 L2

将式(6-181)中的占空比 D 的表达式代入到式(6-188),可求出滤波电感 L2 的

电流纹波最大值为:
nVTS
iL 2max  (6-189)
V
2 L2 (  n)
Vg max

为了使滤波电感 L2 的电流纹波小于设计值 iL 2 _ set ,即:

nVTS
iL 2max   iL 2 _ set (6-190)
V
2 L2 (  n)
Vg max

由式(6-190)可推出:
nVTS
L2  (6-191)
V
2iL 2 _ set (  n)
Vg max

假设励磁电流 im 远小于原边绕组电流 i1 ,且忽略两滤波电感的电流纹波,则

电容 C1a 、 C1b 电流波形如图 6-77 所示:

- 188 -
iC1a iC1b

nDV V
(1  D) R R
DTS TS DTS TS
t t
V DV
n 
R (1  D) R

图 6-77 C1a 、 C1b 电容电流波形

由图 6-77 可求出两电容的电压纹波为:
nDV 2vC1a nDVTS
 C1a  vC1a 
(1  D) R (1  D)TS 2C1a R
(6-192)
V 2vC1b DVTS
 C1b  vC1b 
R DTS 2C1b R
将式(6-181)中的占空比 D 的表达式代入到式(6-192),可得:
nV 2TS
vC1a 
2C1a R(V  nVg )
(6-193)
V 2TS
vC1b 
2C1b R(V  nVg )

注意,式(6-193)应用的前提是电感 L1 、 L2 的电流连续,即处于 CCM 模式。

而通过式(6-193)可知,当负载电阻 R 最小,且输入电压取最小值时,两电容电压
纹波最大,而当负载电阻 R 变大时,可能导致电感 L1 、 L2 的电流不连续,即可

能会运行在 DCM 模式,这里的设计仅保证 CCM 模式 C1a 、 C1b 两电容电压纹波

的最大值小于设计值 vC1a 、 vC1b 。故有:

nV 2TS
vC1a   vC1a _ set
2C1a Rmin (V  nVg min )
(6-194)
V 2TS
vC1b   vC1b _ set
2C1b Rmin (V  nVg min )

为了保证 CCM 模式下, C1a 、C1b 两电容电压纹波的最大值小于设计值,由

式(6-194)可推出:

- 189 -
nV 2TS
C1a 
2vC1a _ set Rmin (V  nVg min )
(6-195)
V 2TS
C1b 
2vC1b _ set Rmin (V  nVg min )

对于输出滤波电容 C2 ,假设滤波电感 L2 电流的直流分量全部流过负载,而

滤波电感 L2 电流的交流分量全部流过输出滤波电容,故电容 C2 的电流波形如图

6-78 所示:
iC 2
DTS
TS
2iL 2
t

图 6-78 电容 C2 的电流波形

故电容 C2 的电压纹波为:

dvC 2 1 1 1 TS iL2 TS
C2  2
iC 2  C2  2vC 2  iC dt  iL  vC 2  (6-196)
dt C2 2 2 2
8C2

将式(6-188)代入到式(6-196)中,可得:
nVTs2
vC 2  (6-197)
V
16C2 L2 (  n)
Vg

为了保证 CCM 模式下输出滤波电容 C2 的电压纹波最大值小于设计值 vC 2 ,

即:
nVTs2
vC 2max   vC 2 _ set (6-198)
V
16C2 L2 (  n)
Vg max

故有:
nVTs2
C2  (6-199)
V
16vC 2 _ set L2 (  n)
Vg max

对于 C1a 、 C1b 两电容,它们的电流有效值可根据图 6-77 求出,即:

- 190 -
2
 nDV 
2
 V
I 2
T   n  DTS    (1  D)TS
 (1  D) R 
C1a _ rms S
 R
2
(6-200)
 DV 
2
V 
I C21b _ rmsTS    DTS     (1  D)TS
R  (1  D) R 

由式(6-200)可求出两电容 C1a 、 C1b 的电流有效值为:

D V
I C1a _ rms  n
1 D R
(6-201)
D V
I C1b _ rms 
1 D R
将式(6-181)中的占空比 D 的表达式代入到式(6-201)中,可求出 CCM 模式下
两电容 C1a 、 C1b 电流有效值的最大值,即:

nV V
I C1a _ rms max 
Rmin nVg min
(6-202)
V V
I C1b _ rms max 
Rmin nVg min

对于输出电容 C2 的电流有效值,根据图 6-78 和书中附录 Appendix A 中的

式(A.9),可求出输出滤波电容电流的有效值为:
iL 2
I C 2 rms  (6-203)
3
将式(6-189)代入到式(6-203),可求出输出滤波电容电流有效值的最大值为:
nVTS
I C 2 rms max  (6-204)
V
2 3L2 (  n)
Vg max

当 DTS  t  TS 时,晶体管 Q1 承受阻断电压,有:

vQ1  VC1  V1 (6-205)

将式(6-179)中 V1 的表达式和式(6-181)代入到式(6-205),可求出晶体管 Q1 承受

的阻断电压为:
V
vQ1  (6-206)
nD
将式(6-181)中的占空比 D 的表达式代入到式(6-206)中,可得:

- 191 -
V  nVg
vQ1  (6-207)
n

故晶体管 Q1 承受阻断电压的最大值为:

V  nVg max V
vQ1max    Vg max (6-208)
n n

如果为了保证晶体管 Q1 承受的阻断电压最大值小于设计值 vQ1_ set ,通过式

(6-208)可求出:
V V
vQ1max   Vg max  vQ1_ set  n  (6-209)
n vQ1_ set  Vg max

在两电感电流纹波远小于它们的直流分量的前提下,晶体管 Q1 的电流波形如

图 6-79 所示:
iQ1
nV
I L1  I C1 
(1  D) R

0 DTS TS

图 6-79 晶体管 Q1 的电流波形

根据书中附录 Appendix A 中的式(A.5),可求出晶体管 Q1 电流的有效值为:

nV
I Q1rms  D (6-210)
(1  D) R

故当占空比 D 最大时,且当负载电阻取最小值时, I Q1rms 达到最大值,即:

nV Dmax
I Q1rms max  (6-211)
(1  Dmax ) Rmin

晶体管 Q1 的开关利用率为:

PLoad _ max PLoad _ max


U  (6-212)
S vQ1_ max  I Q1_ rms max

为了分析晶体管 Q1 开关利用率何种条件下能够达到最大,先为了简化分析,

- 192 -
认为 iL1 相对于 I L1 很小,故由式(6-208)、式(6-211)和式(6-212),可推出晶体管 Q1

开关利用率的表达式为:
PLoad max Rmin 1  Dmax 
U (6-213)
Dmax V (V  nVg max )

由式(6-213)可知,当匝数比取最小值时,可使晶体管 Q1 的开关利用率取最大

值。
对于二极管 D1 ,其承受的阻断电压为:

vD1  VC 2  V2 (6-214)

将式(6-178)中的 V2 子式和式(6-181)代入到式(6-214)中,可求出二极管 D1 的

阻断电压为:
V
vD1  (6-215)
D
将式(6-181)中的占空比 D 的表达式代入到式(6-215)可得二极管 D1 的阻断电

压最大值为:
vD1max  V  nVg max (6-216)

二极管 D1 的电流波形如图 6-80 所示:

iD1
V
I L2  I2 
(1  D) R

0 DTS TS t

图 6-80 二极管 D1 的电流波形

根据图 6-80 可求出二极管 D1 的电流有效值为:

2
 V  V
I 2
T   (1  D)TS  I D1rms  (6-217)
 (1  D) R  R 1 D
D1rms S

将式(6-181)中占空比 D 的子表达式代入到式(6-217)中,可求出二极管 D1 电

流有效值的最大值为:

- 193 -
V
I D1rms max  (6-218)
n
Rmin
V
n
Vg min

(b) 由于没有使用过 SpreadSheet 软件,这里改用 Matlab 进行计算,计算文件


为:

所使用的设计变量为:
 匝数比 n
 电感 L1 的电流纹波 iL1

 电感 L2 的电流纹波 iL 2

 电容 C1a 的电压纹波 vC1a

 电容 C1b 的电压纹波 vC1b

(c) 设计结果如图 6-81 所示:

图 6-81 隔离型 Cuk 变换器的设计结果

- 194 -
第7章 交流等效电路建模

Problem 7.1

一个理想的 Boost 变换器运行于 CCM 模式下。


(a) 推导该变换器的非线性周期平均后的方程。
(b) 构建小信号模型,使:
vg (t )  Vg  vˆg (t )
Ts

d (t )  D  dˆ (t )
 i (t )  I  iˆ(t )
Ts

 v(t ) Ts
 V  vˆ(t )

Vg 、 D 、 I 、 V 为稳态直流分量, vˆg (t ) 、 dˆ (t ) 是功率和控制的交流小信号

输入。iˆ(t ) 、vˆ(t ) 是 vˆg (t ) 、dˆ (t ) 在电感电流和输出电压中引起的交流小信号扰动,

推导以下模型:
大信号直流分量:
0   DV  Vg
V
0  DI 
R
小信号交流分量:

diˆ(t )
L   Dvˆ(t )  Vdˆ (t )  vˆg (t )
dt
dvˆ(t ) vˆ(t )
C  Diˆ(t )  Idˆ (t ) 
dt R
Solution:
(a) Boost 变换器电路图如图 7-1 所示:
L

Vg Q1 C R v

图 7-1 Boost 变换器电路图


当开关管导通时,电路如图 7-2 所示:

- 195 -
iL L
vL iC
Vg Q1 C R v

图 7-2 开关管导通时的 Boost 电路


故有:
vL (t )  vg (t )
v(t ) (7-1)
iC (t )  
R
当开关管关断时,电路如图 7-3 所示:

iL L
vL iC
Vg C R v

图 7-3 开关管关断时的 Boost 电路


故有:
vL (t )  vg (t )  v(t )
v(t ) (7-2)
iC (t )  iL (t ) 
R
使用周期平均进行近似后,可得到 Boost 变换器的周期平均后的非线性方程:

 
d iL (t ) T
L S
 d (t ) vg (t )  d (t ) vg (t )  v(t ) T
dt TS TS S

(7-3)
d v(t ) T  v(t ) T   v(t ) T 
C S
 d (t )   S
  d (t )  iL (t ) T  S

dt  R   S
R 
   
(b) 将周期平均近似的量表示成静态量与小信号交流变量的和,即:
vg (t )  Vg  vˆg (t )
TS

d (t )  D  dˆ (t )
d (t )  1  D  dˆ (t ) (7-4)
iL (t ) TS
 I L  iˆL (t )
v(t ) TS
 V  vˆ(t )
将式(7-4)代入到式(7-3)中,可得:

- 196 -

d I L  iˆL (t ) 
L
dt
V g 
 DV    Dvˆ(t )  vˆg (t )  Vdˆ (t ) 

 dˆ (t )vˆ(t )  (7-5)
d V  vˆ(t )   V  vˆ(t ) 
C   DI L     DiˆL (t )  I L dˆ (t )  
dt  R  R 

  dˆ (t )iˆ (t )L 
由于静态量的导数等于 0,故大信号方程为:
dI
0  L L  Vg  DV
dt
(7-6)
dV V
0C  DI L 
dt R
小信号交流方程为:

diˆL (t )
L   Dvˆ(t )  vˆg (t )  Vdˆ (t )
dt (7-7)
dvˆ(t ) vˆ(t )
C  DiˆL (t )  I L dˆ (t ) 
dt R

Problem 7.2

构建 Problem 7.1 推导的 Boost 变换器小信号交流方程的等效电路。


Solution:
根据式(7-7)中的电感环路方程,可画出等效电路如图 7-4 所示:

iˆL (t ) L Vdˆ (t )

vˆL (t )
vˆg (t ) Dvˆ(t )

图 7-4 电感环路的交流小信号等效电路
根据式(7-7)中的电容节点方程,可画出等效电路如图 7-5 所示:

DiˆL (t ) C I L dˆ (t ) R vˆ(t )

图 7-5 电容节点的交流小信号等效电路
将图 7-4 和图 7-5 进行合并,并将受控源用理想变压器替代,可得到最终
的等效电路,如图 7-6 所示:

- 197 -
iˆL (t ) L Vdˆ (t )

vˆL (t )
vˆg (t ) C I L dˆ (t ) R vˆ(t )

D :1
图 7-6 Boost 变换器小信号等效电路

Problem 7.3

将 Problem 7.2 中推导的等效电路变换成标准形式,所得到的标准形式应与


书中 Table7.1 中的结果一致。
Solution:
根据图 7-6 可求出稳态时的电感电流 I L 为:

 V2
Vg I L  R V
  IL  (7-8)
 Vg  D DR
 V
先将图 7-6 中的独立电流源移至原边侧,再继续将独立电流源移动至靠近电
源侧,具体步骤如图 7-7 所示:
iˆL (t ) L Vdˆ (t )

vˆL (t )
vˆg (t ) C I L dˆ (t ) R vˆ(t )

D :1
将电流源移至原边侧

iˆL (t ) L Vdˆ (t )

vˆL (t )
I L dˆ (t )
vˆg (t ) C R vˆ(t )
D

D :1
将电流源移至电源旁边
iˆL (t ) L Vdˆ (t )

I L dˆ (t )
vˆg (t ) I L dˆ (t ) C R vˆ(t )
D
D
D :1
图 7-7 Boost 等效电路转换成标准形式,步骤 1
再将独立电压源、串联电感和并联的独立电流源使用戴维宁定理进行简化,

- 198 -
然后再将电感移动至副边侧,获得标准形式的交流小信号等效电路模型,如图
7-8 所示:

iˆL (t ) L Vdˆ (t )

I L dˆ (t )
vˆg (t ) I L dˆ (t ) C R vˆ(t )
D
D
D :1
使用戴维宁定义获得简化
的等效电路
 ˆ sLI L dˆ 
Vd (t ) 
 D 

iˆL (t ) L 

I L dˆ (t )
vˆg (t ) C R vˆ(t )
D

D :1

 ˆ sLI L dˆ  L 将电感移至副边侧
 Vd (t )   D 2
iˆL (t )  D 

vˆg (t ) I L dˆ (t )
C R vˆ(t )
D

D :1

图 7-8 Boost 等效电路转换成标准形式,步骤 2


注意,到此为止得到的是交流小信号的标准形式的等效电路,但是此等效电
路也同样适用于稳态直流量,也即获得了最终的适用于直流和交流小信号的标准
形式等效电路。将(7-8)代入到图 7-8 可发现结果与书中 Table7.1 中一致。

Problem 7.4

一个理想的运行于 CCM 模式的电流型桥式变换器如图 7-9 所示:


(a) 推导此变换器的非线性平均化方程。
(b) 对非线性平均化方程进行扰动和线性化,推导此变换器的小信号交流方程。
(c) 构建此变换器的小信号交流等效电路。
iL L
vL
1 ic 2

vg C
R
2 v 1

图 7-9 电流型桥式变换器
Solution:

- 199 -
(a) 当开关处于位置 1 时,此时占空比为 d (t ) ,电路如图 7-10 所示:

iL L
vL
1 ic

vg C
R
v 1

图 7-10 开关处于位置 1 时的电路


故有:
vL (t )  vg (t )  v(t )
v(t ) (7-9)
iC (t )  iL (t ) 
R
当开关处于位置 2 时,电路如图 7-11 所示:
iL L
vL
ic 2

vg C
R
2 v

图 7-11 开关处于位置 2 时的电路


故有:
vL (t )  vg (t )  v(t )
v(t ) (7-10)
iC (t )  iL (t ) 
R
使用周期平均进行近似后,可得到周期平均后的非线性方程为:

   
d iL (t ) T
L S
 d (t ) vg (t )  v(t ) T  d (t ) vg (t )  v(t ) T
dt TS S TS S

(7-11)
d v(t ) T  v(t ) T   v(t ) T 
C S
 d (t )  iL (t ) T  S
  d (t )   iL (t ) T  S

dt  S
R   S
R 
   
(b) 将周期平均近似的量表示成静态量与小信号交流变量的和,即:

- 200 -
vg (t )  Vg  vˆg (t )
TS

d (t )  D  dˆ (t )
d (t )  1  D  dˆ (t ) (7-12)
iL (t ) TS
 I L  iˆL (t )
v(t ) TS
 V  vˆ(t )
将式(7-12)代入到式(7-11),可得:

L

d I L  iˆL (t )    D  D V  V    D  D  vˆ(t )  2Vdˆ (t )  vˆ (t ) 
dt  g   g 


 2dˆ (t )vˆ(t )  (7-13)
d V  vˆ(t )   V  vˆ(t ) 
C   D  D  I L     D  D  iˆL (t )  2 I L dˆ (t ) 
dt  R  R 

 2dˆ (t )iˆ (t ) L 
故小信号交流方程为:

diˆL (t )
L   D  D  vˆ(t )  2Vdˆ (t )  vˆg (t )
dt (7-14)
dvˆ(t ) vˆ(t )
C   D  D  iˆL (t )  2 I L dˆ (t ) 
dt R
(c) 根据式(7-14)中的电感电压环路方程,可构建小信号等效电路如图 7-12
所示:
2Vdˆ (t)
iˆL (t ) L
vˆL (t )
vˆg (t )  D  D vˆ(t )

图 7-12 电感电压环路小信号交流模型
根据式(7-14)中的电容电流节点方程,可构建小信号等效电路如图 7-13 所示:

 D  D  iˆL (t ) 2 I L dˆ (t ) C R vˆ(t )

图 7-13 电容电流节点小信号交流模型
将图 7-12 和图 7-13 进行合并,并将受控源用理想变压器替代,可得到最
终的等效电路,如图 7-14 所示:

- 201 -
2Vdˆ (t)
iˆL (t ) L
vˆL (t )
vˆg (t ) 2 I L dˆ (t ) C R vˆ(t )

D  D :1
图 7-14 电流型桥式变换器的小信号交流等效电路

Problem 7.5

对于如图 7-15 所示的运行于 CCM 模式的反激变换器,构建其完整的小信


号交流模型。变压器具有折算于原边侧的激磁电感 L ,此外,变压器具有不可忽
略的铁耗,该铁耗可以使用并联在原边绕组的电阻 RC 进行建模。其余所有的元

器件均为理想器件,你可以使用任何方法解决此问题。你的模型应当可以正确的
反映出 ig (t ) 的变化。

ig (t ) 1: n D1

L C R v(t )
vg (t )

Q1

图 7-15 反激变换器
Solution:
等效电路如图 7-16 所示:
ig (t ) 1: n D1
iL iC

RC vT L C R v(t )
vg (t )

Q1

图 7-16 含有变压器铁耗的反激变换器等效电路
当开关管导通时,等效电路简化至如图 7-17 所示:

- 202 -
ig (t ) 1: n
iL iC

RC vT L C R v(t )
vg (t )

Q1

图 7-17 开关管导通时的反激变换器等效电路
故有:
vT  vg (t )
v(t )
iC   (7-15)
R
vg (t )
ig  iL (t ) 
RC
当开关管关断时,等效电路可简化至如图 7-18 所示:
故有:
v(t )
vT  
n
v(t )
 iL (t )
nRC v(t )
iC    (7-16)
n R
ig  0

ig (t ) 1: n D1
iL iC

RC vT L C R v(t )
vg (t )

图 7-18 开关管关断时的反激变换器等效电路
使用周期平均进行近似后,可得到反激变换器周期平均后的非线性方程:

- 203 -
d iL (t )  v(t ) T 
 d (t )  
 d (t ) vg (t ) 
TS
L S

dt TS  n 
 
 v(t ) T 
d v(t ) T  v(t ) T   S
 iL (t ) v(t ) 
  (7-17)
TS
nR
 d (t )     d (t )  
C TS
C S S

dt  R   n R 
   
 
 
 vg (t ) 

ig (t )  d (t ) iL (t ) T 
TS
  d (t )  0
TS  S
RC 
 
将周期平均近似的量表示成静态量与小信号交流变量的和,即:
vg (t )  Vg  vˆg (t )
TS

d (t )  D  dˆ (t )
d (t )  1  D  dˆ (t ) (7-18)
iL (t ) TS
 I L  iˆL (t )
v(t ) TS
 V  vˆ(t )
将式(7-18)代入到式(7-17),可得反激变换器周期平均后的非线性方程:

L

d I L  iˆL (t )  
  DVg 
DV   Dvˆ(t )
 
 V 
 Dvˆg (t )   Vg   dˆ (t ) 
 n    n
dt n 
 dˆ (t )vˆ(t ) 
  dˆ (t )vˆg (t )  
 n 
d V  vˆ(t )   DI L V DV 
C    2 
dt  n R n RC 
 vˆ(t ) DiˆL (t ) Dvˆ(t ) I L dˆ (t ) Vdˆ (t ) 
     2   2 
 R n n RC n n RC 
 dˆ (t )iˆL (t ) dˆ (t )vˆ(t ) 
    2 
 n n RC 

 DVg   ˆ ˆ 
I g  iˆg (t )   DI L    ˆ  ˆ (t )  Dvg (t )  Vg d (t ) 
 DiL (t ) I L d
 RC   RC RC 
 dˆ (t )vˆg (t ) 
  dˆ (t )iˆL (t )   (7-19)
 RC 
根据式(7-19)可得到小信号方程为:

- 204 -
diˆL (t ) Dvˆ(t )  V
L   Dvˆg (t )   Vg   dˆ (t )
dt n  n
dvˆ(t ) vˆ(t ) DiˆL (t ) Dvˆ(t ) I L dˆ (t ) Vdˆ (t )
C    2   2 (7-20)
dt R n n RC n n RC
Dvˆ (t ) V dˆ (t )
iˆg (t )  DiˆL (t )  I L dˆ (t )  g  g
RC RC
根据式(7-20)中的电感电压环路小信号方程可画出等效电路如图 7-19 所示:

 Vˆ
 Vg   d (t )
iˆL (t ) L  n
vˆL (t )
D
Dvˆg (t ) vˆ(t )
n

图 7-19 电感电压环路小信号交流模型
根据式(7-20)中的电容电流节点小信号方程可画出等效电路如图 7-20 所示:

D ˆ Vdˆ (t ) I L dˆ (t ) n2
iL (t ) RC C R vˆ(t )
n n 2 RC n D

图 7-20 电容电流节点小信号交流模型
根据式(7-20)中的输入电流小信号方程可画出等效电路如图 7-21 所示:
iˆg (t )

Vg dˆ (t ) 1
vˆg (t ) I L dˆ (t ) RC DiˆL (t )
RC D

图 7-21 输入电流小信号方程
将图 7-19、图 7-20 和图 7-21 合并,并将受控源用理想变压器替代,可得
到最终的小信号等效电路,如图 7-22 所示:
 Vˆ
 Vg   d (t )
iˆg (t ) iˆL (t ) L  n

vˆL (t )
Vg dˆ (t ) 1 Vdˆ (t ) I L dˆ (t ) n2
vˆg (t ) I L dˆ (t ) RC RC R vˆ(t )
RC D n 2 RC n D C

D
1: D :1
n

图 7-22 反激变换器小信号模型(考虑了铁耗)

- 205 -
Problem 7.6

对 Cuk 变换器进行建模,你可以使用任何方法解决此问题。
(a) 推导可以描述理想 Cuk 变换器的小信号动态方程。
(b) 构建完整的 Cuk 变换器小信号等效电路。
Solution:
(a) Cuk 变换器电路如图 7-23 所示:

iL1 L1 iC1 C1 L2 iL 2
vL1 vC1 vL 2
vg Q1 D1 C2 R v2

图 7-23 Cuk 变换器电路


当开关管导通时,电路如图 7-24 所示:

iL1 L1 iC1 C1 L2 iL 2
vL1 vC1 vL 2
vg Q1 C2 R v2

图 7-24 开关管导通时的 Cuk 电路


故有:
vL1  vg (t )
vL 2  vC1 (t )  v2 (t ) (7-21)
iC1  iL 2 (t )
当开关管关断时,电路如图 7-25 所示:

iL1 L1 iC1 C1 L2 iL 2
vL1 vC1 vL 2
vg D1 C2 R v2

图 7-25 开关管关断时的 Cuk 电路


故有:
vL1  vg (t )  vC1 (t )
vL 2  v2 (t ) (7-22)
iC1  iL1 (t )
使用周期平均进行近似后,可得到 Cuk 变换器周期平均后的非线性方程:

- 206 -
 
d iL1 (t )
 d (t ) vg (t )  d (t ) vg (t )  vC1 (t )
TS
L1 TS
dt TS TS

   d (t )   v (t ) 
d iL 2 (t )
 d (t )  vC1 (t )  v2 (t )
TS
L2 TS TS 2 TS
dt
(7-23)
   d (t )  i 
d v1 (t )
 d (t ) iL 2 (t )
TS
C1 TS L1 (t ) TS
dt
v2 (t )
 iL 2 (t ) 
TS
iC 2 (t ) TS TS
R
(b) 将周期平均近似的量表示成静态量与小信号交流变量的和,即:
vg (t )  Vg  vˆg (t )
TS

d (t )  D  dˆ (t )
d (t )  1  D  dˆ (t )
iL1 (t ) TS
 I L1  iˆL1 (t ) (7-24)
iL 2 (t ) TS
 I L 2  iˆL 2 (t )
vC1 (t ) TS
 VC1  vˆC1 (t )
v2 (t ) TS
 V2  vˆ2 (t )
将式(7-24)代入到式(7-23),可得 Cuk 变换器的周期平均后的非线性方程:

L1

d I L1  iˆL1 (t )   V  DVC1 
dt  g

 vˆg (t )  VC1dˆ (t )  DvˆC1 (t ) 

  dˆ (t )vˆC1 (t ) 

L2

d I L 2  iˆL 2 (t )    V  DVC1 
2
dt
  vˆ2 (t )  DvˆC1 (t )  VC1dˆ (t ) 

  dˆ (t )vˆC1 (t ) 


d VC1  vˆC1 (t ) 
C1   DI L1  DI L 2 
dt
   I L1dˆ (t )  DiˆL 2 (t )  I L 2 dˆ (t )  DiˆL1 (t ) 

  dˆ (t )iˆL1 (t )  dˆ (t )iˆL 2 (t ) 


d VC 2  vˆC 2 (t )   V   vˆ (t )  (7-25)
C2   I L 2  2   iˆL 2 (t )  2 
dt  R  R 
根据式(7-25)可推导出 Cuk 变换器的小信号方程为:

- 207 -
diˆL1 (t )
L1  vˆg (t )  VC1dˆ (t )  DvˆC1 (t )
dt
diˆ (t )
L2 L 2  vˆ2 (t )  DvˆC1 (t )  VC1dˆ (t )
dt (7-26)
dvˆC1 (t )
C1   I L1dˆ (t )  DiˆL1 (t )  DiˆL 2 (t )  dˆ (t ) I L 2
dt
dvˆC 2 (t ) ˆ vˆ (t )
C2  iL 2 (t )  2
dt R

(c) 根据式(7-26)中电感 L1 的电压环路小信号方程,可画出等效电路如图 7-26

所示:

VC1dˆ (t )
iˆL1 (t ) L1

vˆg (t ) DvˆC1 (t )

图 7-26 电感 L1 的电压环路小信号等效模型

根据式(7-26)中电感 L2 的电压环路小信号方程,可画出等效电路如图 7-27

所示:

VC1dˆ (t )
iˆL 2 (t ) L2

DvˆC1 (t ) vˆ2 (t )

图 7-27 电感 L2 的电压环路小信号等效模型

根据式(7-26)中的电容 C1 的电流节点小信号方程,可画出等效电路如图 7-28

所示:

DiˆL1 (t ) I L1dˆ (t ) C1 dˆ (t ) I L 2 DiˆL 2 (t )

图 7-28 电容 C1 的电流节点小信号等效模型

根据式中的电容 C2 的电流节点小信号方程,可画出等效电路如图 7-29 所示:

- 208 -
iˆL 2 (t ) C2 R vˆ2 (t )

图 7-29 电容 C2 的电流节点小信号等效模型

将图 7-26、图 7-27、图 7-28 和图 7-29 合并,并将受控源用理想变压器替


代,可得到最终的小信号等效电路,如图 7-30 所示:
VC1dˆ (t ) VC1dˆ (t )
iˆL1 (t ) L1 iˆL 2 (t ) L2

vˆg (t ) I L1dˆ (t ) C1 dˆ (t ) I L 2 C2 R vˆ2 (t )

D :1 1: D

图 7-30 Cuk 变换器小信号等效电路模型

Problem 7.7

对如图 7-31 所示的反转式 SEPIC 电路进行建模,你可以使用任何方法解决


此问题。
(a) 推导描述此电路的小信号动态方程。
(b) 构建完整的小信号等效电路

ig (t ) C1 L2

vg L1 C2 R v(t )

图 7-31 反转式 SEPIC 电路


Solution:
(a) 当开关管导通时,电路如图 7-32 所示:

ig (t ) iC1 (t ) C1 L2 iL 2 (t )
vL 2 (t )
vC1 (t )
vg vL1 (t ) L1 C2 R v(t )
iL1 (t ) iC 2 (t )

图 7-32 开关管导通时的反转式 SEPIC


故有:

- 209 -
vL1 (t )  vg (t )
vL 2 (t )  vC1 (t )  vg (t )  v(t )
iC1 (t )  iL 2 (t ) (7-27)
v(t )
iC 2 (t )  iL 2 (t ) 
R
当开关管关断时,电路如图 7-33 所示:

ig (t ) iC1 (t ) C1 L2 iL 2 (t )
vL 2 (t )
vC1 (t )
vg vL1 (t ) L1 C2 R v(t )
iL1 (t ) iC 2 (t )

图 7-33 开关管关断时的反转式 SEPIC


故有:
vL1 (t )  vC1 (t )
vL 2 (t )  v(t )
iC1 (t )  iL1 (t ) (7-28)
v(t )
iC 2 (t )  iL 2 (t ) 
R
使用周期平均进行近似后,可得到反转式 SEPIC 周期平均后的非线性方程:

 
d iL1 (t )
 d (t ) vg (t )  d (t ) vC1 (t )
TS
L1 TS
dt TS

 
d iL 2 (t )
 d (t )  vC1 (t )  vg (t )  v (t )
TS
L2 TS TS
dt TS


 d (t )  v(t ) TS  (7-29)

   d (t )   i 
d v1 (t )
 d (t ) iL 2 (t )
TS
C1 TS L1 (t ) TS
dt
d v2 (t ) v(t )
 iL 2 (t ) 
TS TS
C2 TS
dt R
将周期平均近似的量表示成静态量与小信号交流变量的和,即:

- 210 -
vg (t )  Vg  vˆg (t )
TS

d (t )  D  dˆ (t )
d (t )  1  D  dˆ (t )
iL1 (t ) TS
 I L1  iˆL1 (t ) (7-30)
iL 2 (t ) TS
 I L 2  iˆL 2 (t )
vC1 (t ) TS
 VC1  vˆC1 (t )
v(t ) TS
 V  vˆ(t )
将式(7-30)代入到式(7-29)中,可得:

L1

d I L1  iˆL1 (t ) 
  DVg  DVC1 
dt
  Dvˆg (t )  Vg dˆ (t )  VC1dˆ (t )  DvˆC1 (t ) 

  dˆ (t )vˆg (t )  dˆ (t )vˆC1 (t ) 

L2

d I L 2  iˆL 2 (t )    DV  V  DVC1 
dt  g

  vˆ(t )  Dvˆg (t )  Vg dˆ (t )  Dvˆc1 (t )  VC1dˆ (t ) 

  dˆ (t )vˆg (t )  dˆ (t )vˆC1 (t ) 


d VC1  vˆC1 (t ) 
C1   DI L 2  DI L1    I L1dˆ (t )  DiˆL 2 (t )  I L 2 dˆ (t )  DiˆL1 (t ) 
dt
  dˆ (t )iˆL1 (t )  dˆ (t )iˆL 2 (t ) 
d VC 2  vˆC 2 (t )   V   vˆ (t ) 
C2   I L 2  2   iˆL 2 (t )  2 
dt  R  R 
(7-31)
故反转式 SEPIC 的小信号方程为:

diˆL1 (t )
L1  Dvˆg (t )  Vg dˆ (t )  VC1dˆ (t )  DvˆC1 (t )
dt
diˆ (t )
L2 L 2  vˆ(t )  Dvˆg (t )  Vg dˆ (t )  Dvˆc1 (t )  VC1dˆ (t )
dt (7-32)
ˆ
dv (t )
C1 C1  I L1dˆ (t )  DiˆL 2 (t )  I L 2 dˆ (t )  DiˆL1 (t )
dt
dvˆ (t ) vˆ(t )
C2 C 2  iˆL 2 (t ) 
dt R

(b) 根据式(7-32)中的电感 L1 的环路电压小信号方程,可画出等效小信号模型

电路如图 7-34 所示:

- 211 -
iˆL1 (t ) L1 V
g  VC1  dˆ (t )

Dvˆg (t ) DvˆC1 (t )

图 7-34 电感 L1 的电压环路小信号等效模型

根据式(7-32)中的电感 L2 的环路电压小信号方程,可画出等效小信号模型电

路如图 7-35 所示:

iˆL 2 (t ) L2 Vg  VC1  dˆ (t )

Dvˆg (t ) vˆ(t )  Dvˆc1 (t )

图 7-35 电感 L2 的电压环路小信号等效模型

根据式(7-32)中的电容 C1 的电流节点小信号方程,可画出等效小信号模型电

路如图 7-36 所示:

DiˆL1 (t )  I L1  I L 2  dˆ (t ) C1 DiˆL 2 (t )

图 7-36 电容 C1 的电流节点小信号等效模型

根据式(7-32)中的电容 C2 的电流节点小信号方程,可画出等效小信号模型电

路如图 7-37 所示:

iˆL 2 (t ) C2 R vˆ(t )

图 7-37 电容 C2 的电流节点小信号等效模型

而对于输入电流,其周期平均后的非线性方程为:

- 212 -
ig (t )  d (t )  iL1 (t )  iL 2 (t ) 
TS  TS TS 
I g  iˆg (t )   DI L1  DI L 2 
(7-33)
  DiˆL1 (t )  I L1dˆ (t )  DiˆL 2 (t )  I L 2 dˆ (t ) 

  dˆ (t )iˆL1 (t )  dˆ (t )iˆL 2 (t ) 

故输入电流的小信号方程为:
iˆg (t )  DiˆL1 (t )  I L1dˆ (t )  DiˆL 2 (t )  I L 2dˆ (t ) (7-34)

将图 7-34、图 7-35、图 7-36 和图 7-37 合并,可得到合并后的小信号等效


电路,如图 7-38 所示:

L1  g
V  VC1  dˆ (t )
1: D iˆL1 (t )

D :1
 I L1  I L 2  dˆ (t )

C1

D(iˆL1 (t )  iˆL 2 (t )) iˆL 2 (t )


1: D

L2

vˆg (t )
V g  VC1  dˆ (t )
C2 R vˆ(t )

1: D
图 7-38 反转式 SEPIC 交流小信号等效电路

Problem 7.8

对于如图 7-39 所示的非理想 Buck 变换器,输入电压源 vg (t ) 具有内阻 Rg ,

其他器件的非理想性可忽略。
(a) 使用状态空间平均法推导小信号交流方程,此方程可以正确描述 i 、v 和 ig

受占空比 d 和输入电压 vg 变化而引起的变化。

(b) 根据(a)推导出的小信号方程,构建小信号交流等效电路。

- 213 -
(c) 根据小信号模型,求解控制输入到输出的传递函数。

ig (t ) Rg L i (t )

vg (t ) C R v(t )

图 7-39 非理想 Buck 变换器


Solution:
(a) 状态向量定义为:
 i (t ) 
x (t )    (7-35)
v(t ) 
输入向量定义为:
u (t )  vg (t )  (7-36)

由于输出电压 v(t ) 已包含再状态向量中,故输出向量就可以仅包含 ig (t ) ,故

定义输出向量为:
y (t )  ig (t )  (7-37)

当开关管导通时,电路如图 7-40 所示:

ig (t ) Rg L i (t )

vg (t ) C R v(t )

图 7-40 开关管导通时的非理想 Buck 变换器


故有:
di (t )
L   Rg i (t )  vg (t )  v(t )
dt
dv(t ) v(t )
C  i (t )  (7-38)
dt R
ig (t )  i(t )

将式(7-38)写成状态空间形式,故有:

- 214 -
 di (t ) 
 R 1 
 L 0   dt   g  i (t )  1 
 0 C   dv(t )    1       [vg (t )]
   1  v(t )  0 
 R (7-39)
 dt 
 i (t ) 
ig (t )   1 0     0[vg (t )]
v(t ) 
当开关管关断时,电路如图 7-41 所示:

ig (t ) Rg L i (t )

vg (t ) C R v(t )

图 7-41 开关管关断时的非理想 Buck 变换器


故有:
di (t )
L  v(t )
dt
dv(t ) v(t )
C  i (t )  (7-40)
dt R
ig (t )  0

将式(7-40)写成状态空间形式,故有:
 di (t ) 
0 1 
 L 0   dt    i (t )  0
 0 C   dv(t )    1       [vg (t )]
   1  v(t )  0 
 dt   R (7-41)

 i (t ) 
ig (t )    0 0     0[vg (t )]
v(t ) 
根据式(7-39)和式(7-41),可得:

- 215 -
  Rg 1  0 1    DRg 1 
A  DA1  DA2  D  1  D 
 1 1
 1   1    1  
 R  R  R
1  0  D 
B  DB1  DB2  D    D     
0 0  0 
C  DC1  DC2  D 1 0  D  0 0   D 0
E  DE1  DE2  [0]
(7-42)
  Rg 1  0 1 
A1  A2        Rg 0
 1 1 1  0 
  1    0
 R  R
1   0  1 
B1  B2         
0  0 0
C1  C2  1 0   0 0  1 0
E1  E2   0
小信号方程为:
dx (t )
K  Axˆ (t )  Buˆ (t )   A1  A2  X   B1  B2 U  dˆ (t )
dt (7-43)
yˆ (t )  Cxˆ (t )  Euˆ (t )   C1  C2  X   E1  E2 U  dˆ (t )

将式(7-42)代入到式(7-43)中,可得到非理想 Buck 变换器的小信号交流方程:

 diˆ(t ) 
 L 0   dt 
0 C  ˆ  
   dv(t ) 
 dt 
  DRg 1  ˆ
   i (t )    D  vˆ (t )     Rg 0  I  1  ˆ
   Vg  d (t ) (7-44)
 1 1     g    
  vˆ(t )   0   0 0  V  0 
 R
iˆg (t ) 
 iˆ(t )   I  ˆ
 D 0     0 vˆg (t )  1 0     0Vg  d (t )
vˆ(t )   V  
(b) 根据式(7-44)可画出小信号等效电路如图 7-42 所示:
iˆg (t ) iˆ(t ) L DRg V g  Rg I  dˆ (t )

vˆg (t ) Idˆ (t) C R vˆ(t )

1: D
图 7-42 非理想 Buck 变换器的小信号等效电路

- 216 -
(c) 控制输入到输出的传递函数为:
vˆ( s )
Gvd ( s )  (7-45)
dˆ ( s ) vˆg ( s ) 0

当 vˆg (t )  0 时,图 7-42 所示的小信号等效电路可简化至如图 7-43 所示:

iˆ(t ) L DRg V
g  Rg I  dˆ (t )

C R vˆ(t )

图 7-43 当 vˆg (t )  0 时的非理想 Buck 变换器小信号等效电路

故有:
ˆ
V g  R g I  ˆ (t )  vˆ(t )  L di (t )  DR iˆ(t )
d
dt
g
(7-46)
dvˆ(t ) vˆ(t )
iˆ(t )  C 
dt R
将式(7-46)写成拉普拉斯形式:
Vg  Rg I  dˆ ( s)  vˆ( s)   sL  DRg  iˆ(s)
1  (7-47)
iˆ( s)    sC  vˆ( s)
R 
根据式(7-47)可求出控制输入到输出的传递函数为:

Gvd ( s) 
vˆ( s)

Vg  Rg I  R (7-48)
dˆ ( s) vˆg ( s )0 CLRs   CDRRg  L  s  R  DRg
2

Problem 7.9

对于如图 7-44 所示的带有输入滤波的 Buck 变换器,使用电路平均方法推


导其直流和小信号交流等效电路。所有的元器件均可以认为理想器件。
L1 L2

Q1

Vg C1 D1 C2 R v

图 7-44 带有输入滤波的 Buck 变换器

- 217 -
Solution:
首先定义开关网络的端口电压和电流,如图 7-45 所示:
L1 i1 (t ) i2 (t ) L2

Q1

Vg C1 v1 (t ) D1 v2 (t ) C2 R v

图 7-45 带有输入滤波的 Buck 变换器的开关网络端口

选择 v1 (t ) 和 i2 (t ) 作为独立输入,则当晶体管导通时,即 0  t  dTS 时,有:

v2 (t )  v1 (t )
(7-49)
i1 (t )  i2 (t )

当晶体管关断时,即 dTS  t  TS ,有:

v2 (t )  0
(7-50)
i1 (t )  0

对受控源的波形进行平均,有:
v2 (t ) TS
 d (t ) v1 (t ) TS
(7-51)
i1 (t ) TS
 d (t ) i2 (t ) TS

对平均后的物理量进行扰动,有:
v1 (t ) T  V1  vˆ1 (t )
S

i2 (t ) TS
 I 2  iˆ2 (t )
v2 (t ) TS
 V2  vˆ2 (t ) (7-52)
i1 (t ) TS
 I1  iˆ1 (t )

d (t )  D  dˆ (t )
将式(7-52)代入到(7-51)中,可得:
V2  vˆ2 (t )  DV1  Dvˆ1 (t )  V1dˆ (t )  dˆ (t )vˆ1 (t )
(7-53)
I  iˆ (t )  DI  Diˆ (t )  I dˆ (t )  dˆ (t )iˆ (t )
1 1 2 2 2 2

对于式(7-53),忽略其幅值更小的二次非线性分量,可得:
V  vˆ (t )  D V  vˆ (t )  V dˆ (t )
2 2 1 1 1
(7-54)
I1  iˆ1 (t )  D  I 2  iˆ2 (t )   I 2 dˆ (t )
根据式(7-54),可画出开关网络的直流和小信号交流的等效电路,如图 7-46

- 218 -
所示:

I1  iˆ1 (t ) I 2  iˆ2 (t )

I 2 dˆ (t ) V1dˆ (t )
V1  vˆ1 (t ) V2  vˆ2 (t )

1: D

图 7-46 平均开关网络模型
使用图 7-46 所示的平均开关网络模型替代图 7-45 中开关网络,即可得到
最终的直流和交流小信号等效电路,如图 7-47 所示:
L1 I1  iˆ1 (t ) I 2  iˆ2 (t ) L2

V1dˆ (t )
Vg  vˆg (t ) V1  vˆ1 (t ) V2  vˆ2 (t ) C2 R
I 2 dˆ (t )
1: D
图 7-47 带输入滤波的 Buck 变换器直流和交流小信号等效电路

Problem 7.10

一个运行在 CCM 模式下的反激变换器,MOSFET 具有导通电阻 Ron ,副边

二极管具有恒定的正向压降 VD ,反激变换器原边绕组的电阻为 RP ,副边绕组的

电阻为 RS 。

(a) 推导此变换器的交流小信号方程。
(b) 推导此变换器的交流小信号模型,该模型适用于 CCM 模式,并且可以正
确的反映以上所述的所有损耗,此外还可以描述变换器的输入输出端口特性。
Solution:
(a) 当 MOSFET 导通时,电路如图 7-48 所示:

- 219 -
1: n
RP

LM vT C R v
vg
i iC
Q1

Ron

图 7-48 MOSFET 导通时的反激变换器


故有:
di
LM  vg  i( RP  Ron )
dt
dv v
C  (7-55)
dt R
ig  i

当开关管关断时,电路如图 7-49 所示:


1: n i2 VD

LM vT C R v
vg i
RS iC

Ron

图 7-49 MOSFET 关断时的反激变换器


故有:

 i 
  VD  v  RS 
 
di n 
LM
dt n
dv i v
C   (7-56)
dt n R
ig  0

使用周期平均近似后,有:

- 220 -
d i

TS
LM
dt
  i 
   VD  v T  TS RS  
  n 
d (t )  vg  i T ( RP  Ron )   d (t )   
S

  
TS S
 n  (7-57)
 
 
d vT  vT   iT vT 
C S
 d (t )   S
  d (t )  S  S

dt  R   n R 
ig  d (t )  i T   d (t )  0
TS  S
将周期平均近似的量表示成静态量与小信号交流变量的和,即:
vg (t )  Vg  vˆg (t )
TS

d (t )  D  dˆ (t )
d (t )  1  D  dˆ (t ) (7-58)
i (t ) TS
 I  iˆ(t )
v(t ) TS
 V  vˆ(t )
将式(7-58)代入到式(7-57)中,可得:
d  I  iˆ(t )   V V IR 
LM   DVg  D  D D  DI  RP  Ron   D 2S  
dt  n n n 
 dˆ (t ) vˆ(t ) dˆ (t ) 
 Dvˆg (t )  Vg dˆ (t )  V  D  VD 
 n n n 
 DRP iˆ(t )  IRP dˆ (t )  DRoniˆ(t )  IRon dˆ (t )  
 
 IRS dˆ (t ) RS iˆ(t ) 
 2
 D 2 
 n n 
vˆg (t )dˆ (t )  dˆ (t )iˆ(t )  RP  Ron   
 
 dˆ (t )vˆ(t ) R dˆ (t )iˆ(t ) 
  S 2 
 n n 
d V  vˆ(t )  DI DV DV 
C     (7-59)
dt  n R R 
 Dvˆ(t ) Dvˆ(t ) Idˆ (t ) Diˆ(t ) 
     
 R R n n 
 dˆ (t )iˆ(t ) 
  
 n 
I g  iˆg (t )   DI    Diˆ(t )  Idˆ (t )    dˆ (t )iˆ(t ) 

- 221 -
故此反激变换器的交流小信号方程为:

 dˆ (t ) vˆ(t ) dˆ (t ) 
 Dvˆg (t )  Vg dˆ (t )  V  D  VD 
 n n n 
diˆ(t )  
LM  DRPiˆ(t )  IRP dˆ (t )  DRoniˆ(t )  IRon dˆ (t ) 
dt  
 IRS dˆ (t ) RS iˆ(t ) 
 2
 D 2 
 n n 
dvˆ(t )  Dvˆ(t ) Dvˆ(t ) Idˆ (t ) Diˆ(t ) 
C       (7-60)
dt  R R n n 
iˆg (t )   Diˆ(t )  Idˆ (t ) 

(b) 根据式(7-60)的第一子式可画出等效电路如图 7-50 所示:

 V  VD IRS  ˆ
V    R  R  I  d (t )
n 2 
g P on
iˆ(t ) LM  n

Dvˆg (t ) vˆ(t )
D
n
RS
DRP  DRon  D
n2
图 7-50 激磁电感电压回路的小信号等效模型
根据式(7-60)的第二子式可画出等效电路如图 7-51 所示:

Diˆ(t ) Idˆ (t )
C R vˆ(t )
n n

图 7-51 电容电流节点的小信号等效模型
根据式(7-60)的第三子式可画出等效电路如图 7-52 所示:

iˆg (t )

vˆg (t ) Idˆ (t ) Diˆ(t )

图 7-52 输入端口的小信号模型
将图 7-50、图 7-51 和图 7-52 合并,可得到最终的小信号等效电路如图 7-53
所示:

- 222 -
 V  VD IRS  ˆ
V    R  R  I  d (t )
n 2 
g P on
iˆg (t ) iˆ(t ) LM  n

Idˆ (t )
vˆg (t ) Idˆ (t ) R C R vˆ(t )
DRP  DRon  D 2S n
n
1: D D
:1
n
图 7-53 反激变换器交流小信号等效电路

Problem 7.11

对如图 7-54 所示的桥式逆变电路使用电路平均。


(a) 证明图 7-54 中的(a)和(b)是等效的。
(b) 使用电路平均法推导此变换器的大信号模型。
(c) 扰动并线性化上一小问中的方程,来获取此变换器的直流和交流小信号等
效电路。
(a)
C
1 2
i (t ) L
vg (t )
R
2 1
v(t )

(b) L

vg (t ) i1 (t ) v1 (t ) C R v(t )

图 7-54 桥式逆变器
Solution:
(a) 由于图 7-54 中(a)和(b)中的开关网络端口电压电流完全相同,故(a)
和(b)是等效的。
(b) 如图 7-55 所示定义开关网络的端口电压电流:
i1

C
1 i2 2
L
vg (t ) v1
R
2 1
v2

图 7-55 定义开关网络的端口电压和电流

- 223 -
选择 v1 和 i2 作为独立输入源,则当开关处于位置 1 时,即 0  t  dTS 时,有:

v2  v1
(7-61)
i1  i2

当开关处于位置 2 时,即 dTS  t  TS 时,有:

v2  v1
(7-62)
i1  i2

进行周期平均后,有:
v2 TS
  d (t )  d (t )  v1 TS
(7-63)
i1 TS
  d (t )  d (t ) i2 TS

故此开关网络可以使用如图 7-56 所示的周期平均后的等效电路进行替代:


i1 TS
  d (t )  d (t )  i2 TS
i2 TS

v1 TS
v2 TS
  d (t )  d (t )  v1 TS

图 7-56 使用受控源进行等效的开关网络
图 7-56 可进一步使用理想变压器进行再一步等效,故桥式逆变器的大信号
等效模型如图 7-57 所示:
L

vg (t ) C R v(t )

1:  d (t )  d (t ) 

图 7-57 桥式逆变器大信号模型
(c) 将式(7-63)进行扰动并线性化,即将式(7-64)代入到式(7-63)中:
v1 TS
 V1  vˆ1 (t )
i2 TS
 I 2  iˆ2 (t )
(7-64)
d (t )  D  dˆ (t )
d (t )  D  dˆ (t )
忽略幅值很小的二次非线性分量,即有:

- 224 -
V2  vˆ2 (t )   D  D V1   D  D  vˆ1 (t )  2V1dˆ (t )
(7-65)
I1  iˆ1 (t )   D  D  I 2   D  D  iˆ2 (t )  2 I 2 dˆ (t )
根据式(7-65)可画出开关网络的直流和交流小信号等效模型,如图 7-58 所示:
2V1dˆ (t ) I 2  iˆ2 (t )
I1  iˆ1 (t )

V1  vˆ1 (t ) 2 I 2 dˆ (t ) V2  vˆ2 (t )

1:  D  D 
图 7-58 开关网络的直流和交流小信号等效模型
故桥式逆变器的直流和交流小信号等效电路如图 7-59 所示:
2V1dˆ (t ) L

Vg  vˆg (t ) 2 I 2 dˆ (t ) C R V  vˆ(t )

1:  D  D 

图 7-59 桥式逆变器的直流和交流小信号等效模型

Problem 7.12

使用电路平均法推导 Buck-Boost 变换器的直流和交流小信号等效电路,你


可以认为此变换器运行于 CCM 模式,且所有元器件均为理想器件。
(a) 画出时不变的等效电路,此等效电路的开关器件被等效电压源和等效电流
源取代,并定义端口波形。
(b) 推导此变换器的大信号平均模型。
(c) 扰动并线性化(b)中推导的大信号模型,以此来获得直流和交流小信号
等效电路模型。
Solution:
(a) 电路如图 7-60 所示:
Q1 D1

Vg L C R v

图 7-60 Buck-Boost 变换器


抽取其开关网络,并定义其端口电压电流,如图 7-61 所示:

- 225 -
i1 i2

v1 v2

Vg L C R v

图 7-61 抽取开关网络

选取 v1 、 i2 作为独立输入源,故其时不变的等效电路如图 7-62 所示:

i1 i2

v1 v2

Vg L C R v

图 7-62 Buck-Boost 变换器的时不变等效电路

(b) 当开关管导通时,即 0  t  dTS 有:

v2  v  vg
i1  iL
(7-66)
v1 0
i2  0

当开关管关断时,即 dTS  t  TS 时,有:

v2  0
i1  0
(7-67)
v1  vg  v
i2  iL
进行周期平均后,有:

- 226 -
v2 (t )  d (t )  v(t ) T  vg (t ) 
TS  S TS 
i1 (t ) TS
 d (t ) iL (t ) T
S
(7-68)
v1 (t )  d (t )  vg (t )  v(t ) T 
TS  TS S 

i2 (t ) TS
 d (t ) iL (t ) TS

由式(7-68)可得:
d (t )
v2 (t )  v1 (t ) T
TS
d (t ) S

(7-69)
d (t )
i1 (t )  i2 (t ) T
TS
d (t ) S

故 Buck-Boost 变换器的大信号平均模型如图 7-63 所示:


d (t )
i1 (t )  i2 (t )
TS
d (t ) TS i2 (t ) TS

d (t )
v1 (t ) v2 (t ) v2 (t )  v1 (t )
TS TS TS
d (t ) TS

Vg L C R v

图 7-63 Buck-Boost 变换器大信号平均模型


(c) 对式(7-69)进行扰动并线性化,即将式图 7-71 代入到式(7-69)中:
v1 TS
 V1  vˆ1 (t )
v2 TS
 V2  vˆ2 (t )
i1 TS
 I1  iˆ1 (t )
(7-70)
i2 TS
 I 2  iˆ2 (t )

d (t )  D  dˆ (t )
d (t )  D  dˆ (t )
可得:
D V2  v2   V2 dˆ (t )   D V1  v1   V1dˆ (t )
(7-71)
D  I1  i1   I1dˆ (t )   D  I 2  i2   I 2 dˆ (t )
根据式(7-71)可画出此开关网络的直流和交流小信号等效电路,如图 7-64
所示:

- 227 -
V2  V1  dˆ (t )
I1  iˆ1 (t ) D I 2  iˆ2 (t )

V1  v1
 I1  I 2  dˆ (t ) V2  v2
D

 D : D
图 7-64 开关网络的直流和小信号交流等效电路
故 Buck-Boost 变换器的直流和小信号交流等效电路如图 7-65 所示:
I1  iˆ1 (t )  D : D I 2  iˆ2 (t )

 I1  I 2  dˆ (t ) V2  V1  dˆ (t )
D D

Vg  vˆg (t ) C R V  vˆ(t )

图 7-65 Buck-Boost 变换器直流和交流小信号等效电路

Problem 7.13

如图 7-66 所示,两输出的反激变换器运行于 CCM 模式,可以认为此变换


器是无损耗的。
(a) 推导小信号交流等效电路。
(b) 将其小信号交流等效电路写成如图 7-67 所示的标准形式,并给出 e( s ) 和

j (s) 的解析式。

ip n p : n1

C1 R1 v1 (t )
L

vg (t )

C2 R2 v2 (t )

图 7-66 两输出反激变换器

- 228 -
iˆg ( s ) e( s )dˆ ( s ) Dn p : n1
1: D iˆ( s ) L

vˆg ( s ) j ( s )dˆ ( s ) C1 R1 vˆ1 (t )

C2 R2 vˆ2 (t )

: n2

图 7-67 反激变换器的小信号交流等效电路
Solution:
(a) 当开关管导通时,等效电路如图 7-68 所示:
ip n p : n1

C1 R1 v1 (t )
vT L
i
vg (t )

C2 R2 v2 (t )

图 7-68 开关管导通时的反激变换器等效电路
故有:
di
L  vg
dt
dv v
C1 1   1
dt R1 (7-72)
dv2 v
C2  2
dt R2
ip  i
当开关管关断时,等效电路如图 7-79 所示:
ip n p : n1 i
1

C1 R1 v1 (t )
vT L
i i2
vg (t )

C2 R2 v2 (t )

图 7-69 开关管关断时的反激变换器等效电路

- 229 -
故有:

di n n
L  v1 p  v2 p
dt n1 n2
dv1 v
C1  i1  1
dt R1
dv2 v
C2  i2  2 (7-73)
dt R2
n1 n
i i1  2 i2
np np
ip  0

使用周期平均近似后,有:
d i  n n 
 d (t ) vg  d (t )   v1 T p  v2 T p 
TS
L
dt  n1 n2 
TS S S

d v1 T  v1 T   v1 T 
C1 S
 d (t )   S
  d (t )  i1 T  S

dt  R   S
R 
 1   1 
(7-74)
d v2 T  v2 T   v2 T 
C2 S
 d (t )   S
  d (t )  i2 T  S

dt  R   S
R 
 2   2 
n n 
i p  d (t )  1 i1 T  2 i2 T 
TS n np S 
 p 
S

将周期平均近似的量表示成静态量与小信号交流变量的和,即:
vg (t )  Vg  vˆg (t )
TS

d (t )  D  dˆ (t )
d (t )  1  D  dˆ (t )
i TS
 I  iˆ(t )
i1 TS
 I1  iˆ1 (t ) (7-75)
i2 TS
 I 2  iˆ2 (t )
v1 (t ) TS
 V1  vˆ1 (t )
v2 (t ) TS
 V2  vˆ2 (t )
ip  I p  iˆp (t )
TS

将式(7-75)代入到式(7-74)中,可得:

- 230 -
 dI diˆ(t )   DV1n p DV2 n p 
L 1     DVg   
 dt dt   n1 n2 
 V dˆ (t )n p V2 dˆ (t )n p Dn p vˆ1 (t ) Dn p vˆ2 (t ) 
  Dvˆg (t )  Vg dˆ (t )  1    
 n1 n2 n1 n2 
 dˆ (t )vˆ1 (t )n p dˆ (t )vˆ2 (t )n p 
  dˆ (t )vˆg (t )   
 n1 n2 
 dV dvˆ (t )   V   vˆ (t ) 
C1  1  1     1  DI1     1  dˆ (t ) I1  Diˆ1 (t ) 
 dt dt   R1   R1 
  dˆ (t )iˆ1 (t ) 

 dV dvˆ (t )   V   vˆ (t ) 
C2  2  2     2  DI 2     2  dˆ (t ) I 2  Diˆ2 (t ) 
 dt dt   R2   R2 
  dˆ (t )iˆ2 (t ) 
 DI n DI n 
I p  iˆp (t )   1 1  2 2 
 n p n p 
 Diˆ (t )n1 I1dˆ (t )n1 Diˆ2 (t )n2 I 2 dˆ (t )n2 
  1     (7-76)
 n p np np n p 
 dˆ (t )iˆ1 (t )n1 dˆ (t )iˆ2 (t )n2 
   
 np np 
根据式(7-76)可得交流小信号方程为:

diˆ(t ) V dˆ (t )n p V2 dˆ (t )n p Dn p vˆ1 (t ) Dn p vˆ2 (t )


L  Dvˆg (t )  Vg dˆ (t )  1   
dt n1 n2 n1 n2
dvˆ (t ) vˆ (t )
C1 1   1  dˆ (t ) I1  Diˆ1 (t )
dt R1
(7-77)
dvˆ2 (t ) vˆ2 (t ) ˆ
C2   d (t ) I 2  Diˆ2 (t )
dt R2
Diˆ (t )n1 I1dˆ (t )n1 Diˆ2 (t )n2 I 2 dˆ (t )n2
iˆp (t )  1   
np np np np

假设激磁电感 L 无限大,即激励电流恒定,将式(7-73)中的倒数第 2 个子式


代入到式(7-77)中的最后一个子式,可得:
Diˆ (t )n1 I1dˆ (t )n1 Diˆ2 (t )n2 I 2 dˆ (t )n2
iˆp (t )  1     Diˆ(t )  Idˆ (t ) (7-78)
np np np np

根据式(7-77)中电感电压方程,可画出电感电压回路的等效电路如图 7-70
所示:

- 231 -
V2 dˆ (t )n p
Vg dˆ (t )
iˆ(t ) L n2
Dn p vˆ1 (t )
V1dˆ (t )n p n1
Dvˆg (t )
n1 Dn p vˆ2 (t )
n2
图 7-70 电感电压回路的交流小信号等效电路

根据式(7-77)中的电容 C1 的电流节点交流小信号方程,可画出等效电路如图

7-71 所示:

Diˆ1 (t ) I1dˆ (t ) C1 R1 vˆ1 (t )

图 7-71 电容 C1 电流节点的交流小信号等效电路

根据式(7-77)中的电容 C2 的电流节点交流小信号方程,可画出等效电路如图

7-72 所示:

Diˆ2 (t ) I 2 dˆ (t ) C2 R2 vˆ2 (t )

图 7-72 电容 C2 电流节点的交流小信号等效电路

根据式(7-78)可画出输入电流的交流小信号等效电路,如图 7-73 所示:


iˆp (t )

vˆg (t ) Idˆ (t ) Diˆ(t )

图 7-73 输入电流的交流小信号等效电路
将图 7-70、图 7-71、图 7-72 和图 7-73 合并,可得到最终的交流小信号等
效电路如图 7-74 所示:

- 232 -
Dn p : n1
V2 dˆ (t )n p
Vg dˆ (t )
iˆp (t ) L
1: D n2 I1dˆ (t ) C1 R1 vˆ1 (t )
iˆ(t )
V1dˆ (t )n p
vˆg (t ) Idˆ (t )
n1
I 2 dˆ (t ) C2 R2 vˆ2 (t )

Dn p : n2

图 7-74 两输出反激变换器的交流小信号等效电路
(b) 图 7-74 可变换为如图 7-75 所示的形式:
V V 
Vg dˆ (t )   1  2  dˆ (t )n p Dn p : n1
 n1 n2 
iˆp (t ) D
L iˆ(t ) C1 R1 vˆ1 (t )

dˆ (t )
vˆg (t ) Idˆ (t )  n1I1  n2 I 2 
Dn p

1: D C2 R2 vˆ2 (t )

Dn p : n2
图 7-75 反激变换器交流小信号等效电路中间形式 1
图 7-75 可进一步变换成如图 7-76 所示的形式:
V V 
Vg dˆ (t )   1  2  dˆ (t )n p Dn p : n1
 n1 n2  ˆ
sLd (t )
 n1I1  n2 I 2 
iˆp (t ) D
Dn p L iˆ(t )
C1 R1 vˆ1 (t )

dˆ (t )
vˆg (t ) Idˆ (t )  n1I1  n2 I 2 
Dn p

1: D C2 R2 vˆ2 (t )

Dn p : n2
图 7-76 反激变换器交流小信号等效电路中间形式 2
图 7-76 可变换成最终的标准形式,如图 7-77 所示:

- 233 -
Dn p : n1
ˆ ( s)
Vg  sIL  dDD 
iˆp ( s ) L iˆ( s )
C1 R1 vˆ1 ( s )
dˆ ( s )
vˆg ( s ) I
D

1: D C2 R2 vˆ2 ( s )

Dn p : n2
图 7-77 反激变换器交流小信号等效电路的标准形式
故有:
I
j ( s) 
D
(7-79)
V  sIL
e( s )  g
DD

Problem 7.14

如图 7-78 所示,在一个脉宽调制电路中,锯齿波发生器被三角波发生器所
取代。
(a) 推导变换器的开关频率,单位 Hz。
(b) 推导电路的增益,即  (t ) / vc (t ) 。

(c) 为了使(b)中推导的增益有效, vc 的范围应该是多少?

vtri (t )
三角波 vtri (t )
发生器 比较器
2
 (t )
0 t
PWM波形
vc (t ) 2
模拟输入 50  s 100  s
图 7-78 脉宽调制电路
Solution:
(a) 变换器的开关频率为:
1
f   20000Hz (7-80)
50μs

(b) 对于不同大小的模拟输入,比较器的输出,即占空比为:

- 234 -
1vc  2

 (t )  0.25  vc  2    vc  2 (7-81)
 0v  2
 c

故增益为:
d  (t )
 0.25 (7-82)
dvc (t )

(c) 为了使(b)中推导的增益有效,显然比较器正向输入端的信号大小不能
大于三角波的最大值,同时也不能小于三角波的最小值,即:
2  vc (t )  2 (7-83)

Problem 7.15

使用电路平均方法推导如图 7-79 所示的 Buck-Boost 变换器的交流等效电路


模型。

ig (t ) Q1 D1
i (t )
vg (t ) L C R v(t )

图 7-79 Buck-Boost 变换器


(a) 将图 7-79 所示的开关网络用如图 7-80 所示的平均开关模型替代。
(b) 比较替代后的结果和书中 Fig7.16(b)中的结果。这两种模型可以推导出同
样的从交流输入到输出的交流小信号传递函数 Gvg (s)  vˆ / vˆg 。

V1 ˆ
d
DD D : D
i1 (t ) i2 (t ) I1  iˆ1 I 2  iˆ2

I2 ˆ
v1 (t ) v2 (t ) V1  vˆ1 d (t ) V2  vˆ2
DD

图 7-80 开关网络模型
Solution:
(a) 将图 7-79 所示的 Buck-Boost 变换器进行变形,如图 7-81 所示:

- 235 -
i1 (t )

v1 (t ) v2 (t )
i2 (t )

vg (t ) L C R v(t )

图 7-81 变换后的 Buck-Boost 电路


用图 7-80 所示的开关模型取代图 7-81 中的开关网络,如图 7-82 所示:
D : D

V1 ˆ
d
DD

I2 ˆ
vg (t ) d (t ) R v(t )
L DD C

图 7-82 替代开关网络后的 Buck-Boost 等效电路


(b) 交流输入到输出的增益为:
vˆ( s )
Gvg ( s )  (7-84)
vˆg ( s )
dˆ  0

令 dˆ  0 ,图 7-82 可简化至如图 7-83 所示:

iˆ1 D : D

v̂1 v̂2

iˆ2
vˆg L C R v(t )
iˆ3

图 7-83 dˆ  0 时的 Buck-Boost 等效电路

即有:

- 236 -
v1 D

v2 D
i1 D

i2 D
vg  v  v1  v2 (7-85)
vg  v1  sLi3
i1  i2  i3
v 
i2     sCv 
R 
通过式(7-85),可求得:
vˆ( s) DDR
Gvg ( s)   (7-86)
vˆg ( s) CLRs  Ls  D2 R
2
dˆ  0

此结果与根据书中 Fig7.16(b)的等效电路求得的增益结果相同。

Problem 7.16

对书中 Fig7.50 所示的直流与小信号交流平均开关模型进行修改,使其可以


对 MOSFET 导通电阻 Ron 和二极管正向压降 VD 进行建模。

Solution:
(a) 对于书中 Fig7.50 中的(a)开关网络,以 Buck 变换器作为分析例子,当
开关管导通时,等效电路如图 7-84 所示:
i1 Ron i2 L

vg v1 v2 C R v

图 7-84 开关管导通时的 Buck 等效电路


即有:
v1  iL Ron  v2
(7-87)
i1  i2  iL

当开关管关断时,等效电路如图 7-85 所示:


i1 i2 L

vg v1 vD v2 C R

图 7-85 开关管关断时的 Buck 等效电路

- 237 -
即有:
v1  vg
v2  VD
(7-88)
i1  0
i2  iL
则 Buck 类型的基本开关网络的平均端口波形为:
i1 TS
 d iL TS

i2 TS
 d iL TS
 d  iL TS
 iL TS
(7-89)

v2 TS 
 d v1 TS
 iL TS 
Ron  d   VD 

即:
i1 TS
 d i2 TS

 
(7-90)
v2 TS
 d v1 TS
 i2 TS
Ron  d   VD 
根据式(7-90)可画出大信号平均开关网络等效电路如图 7-86 所示:

i1 dRon d VD i2
TS 1: d TS

v1 TS
v2 TS

图 7-86 Buck 类型开关网络的平均开关模型


(b) 对于书中 Fig7.50 中的(b)开关网络,以 Boost 变换器作为分析例子,当
开关管导通时,等效电路如图 7-87 所示:
iL L i1 i2

vg v1 v2 C R

图 7-87 开关管导通时的 Boost 等效电路


故有:
v1  iL Ron
(7-91)
i1  iL

当开关管关断时,等效电路如图 7-88 所示:

- 238 -
iL L i1 i2

vg v1 v2 C R

图 7-88 开关管关断时的 Boost 等效电路


故有:
v1  v2  VD
(7-92)
i1  i2  iL

则 Boost 类型的基本开关网络的平均端口波形为:
i2 T
i1 T  S

S
d
(7-93)
 i2 T 
v1 T  d  S
Ron   d   v2  VD 
S  d 
 
根据式(7-93)可画出大信号平均开关网络等效电路如图 7-89 所示:

i1 VD
TS dRon d  :1 i2 TS

v1 TS
v2 TS

图 7-89 Boost 类型开关网络的平均开关模型


(c) 对于书中 Fig7.50 中的(c)开关网络,以 SEPIC 变换器作为分析例子,
当开关管导通时,等效电路如图 7-90 所示:

iL1 L1 C1

vC1
vg L2 C2 R vC 2
iL 2

i1

v1 v2
i2

图 7-90 开关管导通时的 SEPIC 等效电路


故有:

- 239 -
i1  iL1  iL 2
i2  0
(7-94)
v1   iL1  iL 2  Ron
v2  vC 2  vC1   iL1  iL 2  Ron
当开关管关断时,等效电路如图 7-91 所示:

iL1 L1 C1

vC1
vg L2 C2 R vC 2
iL 2

i1

v1 v2
i2

图 7-91 开关管关断时的 SEPIC 等效电路


故有:
i1  0
i2  iL1  iL 2
(7-95)
v1  vC1  VD  vC 2
v2  VD
则 SEPIC 类型的基本开关网络的平均端口波形为:
i1 TS 
 d iL1 TS
 iL 2 TS 
i2 TS
 d i L1 TS  iL 2 TS  (7-96)
v1 TS
d i L1 TS  iL 2 TS R on 
 d  vC1 TS
 vC 2 TS
 VD 
v2 TS
d v C1 TS  vC 2 TS 
  iL1  iL 2  Ron  d   VD 

将式(7-96)进行变形,可得:
d
i2 T  i1 T
S
d S
(7-97)
R d
v1 T  on i1 T   v2  VD 
S
d S
d
根据式(7-97)可画出平均开关模型如图 7-92 所示:

- 240 -
Ron
i1 VD
TS d d : d i2 TS

v1 TS
v2 TS

图 7-92 SEPIC 类型开关网络的平均开关模型

Problem 7.17

使用平均开关模型法对如图 7-93 所示的反激变换器建立直流和交流等效模


型。你可以忽略所有损耗和变压器的漏感。
ig (t ) 1: n D1

vg (t ) L C R v(t )

Q1

图 7-93 反激变换器

(a) 像书中 Fig7.39(a)那样定义晶体管 Q1 和二极管 D1 的开关网络。推导大信

号平均开关模型。此模型应考虑到变压器的匝数比。
(b) 扰动和线性化你推导的(a)小问的结果,来获取直流和交流小信号平均
开关模型。为了检验获得的模型,当 n  1 时,你的模型应该和书中 Fig7.39(d)
一致。
(c) 使用(b)小问的推导的模型来获取完整的反激变换器的直流和交流小信
号模型。根据此模型来求出稳态的电压转换比 M ( D)  V / Vg 。

(d) (a)小问推导的模型和(b)小问推导的模型,哪一个可以在其他的含有
隔离变压器的变换器中使用?
Solution:
(a) 定义开关网络如图 7-94 所示:
i1 i2

v1 v2

图 7-94 定义的开关网络
反激变换器的等效电路如图 7-95 所示:

- 241 -
ig (t ) 1: n D1
iT
vg (t )
Lm C R v(t )

Q1

图 7-95 反激变换器的等效电路
当开关管导通时,有:
v1  0
v2  v  nvg
i1  iT (7-98)
i2  0
ig  iT
当开关管关断时,有:
v
v1  vg 
n
v2  0
i1  0 (7-99)
iT
i2 
n
ig  0

故平均的端口波形为:
 v 
 d   vg  
TS
v1 TS  TS n 

v2 TS
d v  TS
 n vg
TS 
i1 TS
 d iT TS
(7-100)
iT
 d
TS
i2 TS
n
ig  d iT TS
TS

根据式(7-100)可推出:
d
v2 T v1 TS

nd S
(7-101)
d
i2 T  i1 T
S
nd S

根据式(7-101)可画出大信号平均开关模型如图 7-96 所示:

- 242 -
i1 TS
i2 TS

d d
v1 TS
v2 TS i1 v2 TS
nd nd TS

图 7-96 大信号开关网络平均模型
(b) 对式(7-101)进行扰动和线性化,即将式(7-102)代入到式(7-101)中。
v1 TS
 V1  vˆ1 (t )
v2 TS
 V2  vˆ2 (t )
i1 TS
 I1  iˆ1 (t )
(7-102)
i2 TS
 I 2  iˆ2 (t )

d (t )  D  dˆ (t )
d (t )  D  dˆ (t )
可得:
D V2  vˆ2  dˆ (nV1  V2 )
V1  vˆ1  
nD nD (7-103)
D  I  nI 2 
I 2  iˆ2 
nD

I1  iˆ1  dˆ 1 nD
根据式(7-103)可画出直流和小信号交流模型如图 7-97 所示:
dˆ (nV1  V2 )
I1  iˆ1 nD
D : nD I 2  iˆ2

 I  nI 2 
dˆ 1
V1  vˆ1 V2  vˆ2
nD

图 7-97 直流和交流小信号开关网络等效电路
当 n  1 时,图 7-97 和书中 Fig7.39(d)是一致的。
(c) 反激变换器完整的直流和交流小信号模型如图 7-98 所示:

- 243 -
ig (t ) 1: n

vg (t ) L C R v(t )

D : nD
I1  iˆ1 I 2  iˆ2
dˆ (nV1  V2 )  I  nI 2 
dˆ 1
nD nD

图 7-98 完整的反激变换器的直流和交流小信号模型
当不考虑扰动,只考虑稳态时,图 7-98 可以简化为如图 7-99 所示:
ig (t ) 1: n

vg (t ) L C R v(t )

D : nD
I1 I2

图 7-99 只考虑稳态时的反激变换器等效电路
注意稳态的信号对于 1: n 的变压器是无效的,该变压器只对交流信号起作用。
而 D : nD 的这个变压器既可以对直流信号起作用,也可以对交流信号起作用。
故当只考虑稳态的直流信号时,可将 1: n 的变压器忽略掉,图 7-99 可进一步简
化为如图 7-100 所示:
Ig D : nD

Vg C R V

图 7-100 只考虑稳态信号的简化的等效电路
故稳态的电压转换比率为:
V D
M ( D)  n (7-104)
Vg D

- 244 -
(d) (a)和(b)中推导的模型或许只能用于类似反激变换器工作原理的变换
器中(此结论不一定正确,只是我自己所想的)。

Problem 7.18

在如图 7-101 所示的反激变换器中,晶体管的导通电阻为 Ron ,二极管的正

向压降为 VD ,其余损耗和变压器的漏感可以忽略。推导开关网络的直流和小信

号交流平均开关模型。此模型应该可以反映晶体管的导通电阻 Ron 和二极管的正

向压降 VD ,及变压器的匝数比 n 。

ig (t ) 1: n D1

vg (t ) L C R v(t )

Q1

图 7-101 反激变换器
Solution:
(a) 当开关管导通时,其等效电路如图 7-102 所示:
ig (t ) 1: n i2
v2
iT
vg (t ) Lm C R v(t )
i1
Ron v1

图 7-102 开关管导通的反激变换器等效电路
故有:
v1  iT Ron
v2  v  n  vg  iT Ron 
(7-105)
i1  iT  ig
i2  0
当开关管关断时,其等效电路如图 7-103 所示:

- 245 -
VD
ig (t ) 1: n i2
iT v2
vg (t ) Lm C R v(t )
i1
v1

图 7-103 开关管关断的反激变换器等效电路
故有:

v1 
 v  VD   v
g
n
v2  VD
(7-106)
i1  ig  0
iT
i2 
n
故开关网络的平均端口波形为:

v1  d  i1 
Ron  d 
 v V
 TS D 
 vg 
 
TS  TS   n TS 
 

v2 TS
d v
 TS 
 n vg  iT T Ron   d   VD 
TS S   (7-107)

nd
i1  i2
TS
d TS

根据式(7-107)可进一步得出:
d d VD
v1 TS
 v2 T  iT TS
Ron 
nd S
nd
d
i2 TS
 i1 T (7-108)
nd S

i1 TS
 d iT T
S

根据式(7-108)可得:

d i1 T d VD
v1 TS
 v2 T  S
Ron 
nd S
d nd (7-109)
d
i2 TS
 i1 T
nd S

对式(7-109)进行扰动和线性化,即将式(7-110)代入到式(7-109)中。

- 246 -
v1 TS
 V1  vˆ1 (t )
v2 TS
 V2  vˆ2 (t )
i1 TS
 I1  iˆ1 (t )
(7-110)
i2 TS
 I 2  iˆ2 (t )

d (t )  D  dˆ (t )
d (t )  D  dˆ (t )
可得:

V1  vˆ1 
D D dˆ
V2  vˆ2   VD  V2  VD  nV1  
I1  iˆ1 Ron  
nD nD nD D (7-111)
D dˆ
I 2  iˆ2 
nD
 
I1  iˆ1 
nD
( I1  nI 2 )

根 据式 (7-111)可画出 开关网络的直流和小信号交流平均开关模型如 图
7-104 所示:
dˆ (nV1  V2  VD )
I1  iˆ1 nD
D : nD
VD I 2  iˆ2

Ron
 I  nI 2 
dˆ 1
V1  vˆ1 D V2  vˆ2
nD

图 7-104 含有导通电阻和正向压降的平均开关模型

Problem 7.19

在如图 7-105 所示的 Boost 变换器中, v1 (t ) 和 i2 (t ) 的波形如图 7-106 所示。

当晶体管导通时,二极管的反向电流会将二极管存储电荷移除。如图 7-106 所示,


反向电流的尖峰面积为 Qr ,持续时间为 t r ,电感绕组的电阻为 RL ,除了由反向

电流导致的开关损耗和由电感绕组电阻导致的导通损耗外,可以忽略其余的所有
损耗。
(a) 推导 Boost 变换器的平均开关模型。
(b) 使用(a)中推导的结果画出 Boost 变换器的直流等效电路
(c) 二极管的存储电荷可以表达成电流 I1 的函数:

Qr  kq I1

反向恢复时间 t r 可以认为是固定的常数,当 Vg  100V ,D  0.5 , f s  100kHz ,

- 247 -
kq  100nc/A1/2 , tr  100ns , RL  0.1 ,负载电流 1A  I Load  10A 在此范围变动

时,画出效率随着负载电流变化的曲线。
iL (t ) L i1 (t ) i2 (t )
I Load
Vg v1 (t ) v2 (t ) C v(t )

图 7-105 Boost 变换器


v1 (t )
v2 v2

0 0
t
tr dTS

TS
i2 (t )
i1 i1

0 0
t

Qr

图 7-106 Boost 变换器中反向恢复电流波形


Solution:
(a) 根据图 7-106 可得出:
v1 TS
 d  v2 TS

1 1 
  d TS  tr  i1  Qr 
TS
i2  i1 (t )dt  (7-112)
TS
TS 0 TS  TS 
tr Qr
 d  i1 TS
 i1 TS

TS TS
根据式(7-112)可画出开关网络的平均开关模型,如图 7-107 所示:

i1 TS d  :1 i2 TS

tr Qr
v1 TS
i1 TS
v2 TS
TS TS

图 7-107 大信号平均开关模型
(b) 根据图 7-107 可画出稳态直流信号的平均开关模型,如图 7-108 所示:

- 248 -
I1 D :1 I2

tr Qr
V1 I1 V2
TS TS

图 7-108 稳态直流信号的开关网络的平均开关模型
故 Boost 变换器的直流等效模型如图 7-109 所示:
IL RL I1 D :1 I2
I Load
Vg tr Qr
V1 I1 V2 C V
TS TS

图 7-109 Boost 变换器直流平均开关模型


(c) 图 7-109 可进一步简化为如图 7-110 所示:
IL RL
I Load
tr Qr D
Vg I1 DV
TS D TS D

图 7-110 简化的 Boost 变换器直流平均开关模型


故有:
tr I1 Q I
IL   r  Load
DTS DTS D
Vg  I L RL  DV
(7-113)
Qr  kq I1
VI Load

Vg I L
根据式(7-113)可用 MathCad 求出效率曲线,如图 7-111 所示:

- 249 -
0.968

0.966

0.964

0.963

0.961

0.959

0.957

0.956

0.954
1 2.125 3.25 4.375 5.5 6.625 7.75 8.875 10

图 7-111 效率关于负载电流的曲线

- 250 -
第8章 变换器的传递函数

Problem 8.1

将书中 Fig8.62(a)至(c)所示的增益渐近线表达成零极点形式。你可以
假设所有的零极点均具有负实部。
Solution:
(a) 转折频率 f 0 之前的第一段,其斜率为 20 dB ,故:

G1 ( s)  s (8-1)

转折频率 f 0 处,其斜率由 20dB 变为 0dB ,故:

1
G2 ( s)  (8-2)
s
1
2 f 0

转折频率 f1 处,其斜率由 0dB 变为 20dB ,故:

1
G3 ( s)  (8-3)
s
1
2 f1

故书中 Fig8.62(a)增益表达式的形式为:
G  G0  G1  G2  G3 (8-4)

对于 G0 的取值,当只考虑第一段的增益渐近线时,有:

20 log10  G0  2 f 0   Gm (8-5)

可推出系数 G0 为:
Gm
10 20
G0  (8-6)
2 f 0
故书中 Fig8.62(a)的增益表达式为:
Gm
10 20
s
2 f 0
G  G1  G2  G3  (8-7)
 s  s 
1  1  
 2 f 0   2 f1 

- 251 -
(b) 书中 Fig8.62(b)的增益表达式形式为:
 s 
G0 1  
G  2 f 2  (8-8)
 s  s 
1   1  
 2 f1   2 f 3 

对于 G0 的取值,如果中频段的增益为 Gm ,则转折频率 f1 之前的增益为

 f 
20 log10  2   Gm ,故有:
 f1 

 f 
20 log10 G0  20 log10  2   Gm (8-9)
 f1 
根据式(8-9)可推出:
f 
20log10  2   Gm
 f1 
G0  10 20
(8-10)
则书中 Fig8.62(b)的增益表达式为:
f 
20log10  2   Gm
 s 
 f1 

1  
20
10
G  2 f 2  (8-11)
 s  s 
1   1  
 2 f1  2 f3 
(c) 书中 Fig8.62(c)的增益表达式形式为:
 s 
G0 s  1  
G  2  (8-12)
2
s  s 
1  
Q1  1 

对于 G ,有:

 2
20 log10  G0 1   G (8-13)
 2 
根据式(8-13)可推出:
2 G 

G0   10 20
(8-14)
12
故书中 Fig8.62(c)的增益表达式为:

- 252 -
2 G 
 s 
10 20  s 1  
 2
 2 
G 1 2 (8-15)
s  s 
1  
Q1  1 

注意:这里的 Q 是没有单位的,即不是以 dB 为单位,如果 Q  5 ,那么

QdB  20 log10 5  13.9dB,那么 Q  5 的 Bode 图上的增益超调为 QdB  13.9dB 。

Problem 8.2

将书中 Fig8.63(a)至(c)所示的增益渐近线表达成零极点形式。你可以
假设所有的零极点均具有负实部。
Solution:
(a) 书中 Fig8.63(a)的增益表达式形式为:
G0 s
G (8-16)
 s 
1  
 2 f 0 

对于 G 的取值,有:

20 log10  2 f 0G0   G (8-17)

根据式(8-17)可推得:
G
1
G0  10 20 (8-18)
2 f 0

故书中 Fig8.63(a)的增益表达式为:
G
1
10 20  s
2 f 0
G (8-19)
 s 
1  
 2 f 0 
(b) 这里认为书中 Fig8.63(b)图中的 Q 是没有单位的,即不是以 dB 为单位,
如果 Q  5 ,那么 QdB  20log10 5  13.9dB ,那么 Fig8.63(b)Bode 图上的增益超

调为 QdB  13.9dB ,即图中的向上超调或向下超调为 13.9dB ,而不是 5dB 。

故书中 Fig8.63(b)的增益表达式形式为:

- 253 -
 s  s  
2

G0 1   
 Q2  2 f 2  2 f 2  
G   (8-20)
 s  s  
2

1   
 Q1  2 f1  2 f1  
 

对于 G 的取值,有:

  2 f1  
2

20log10 G0      G (8-21)


  2 f 2  
根据式(8-21)可求出:
2
 2 f 2  G20
G0    10 (8-22)
 2 f1 
故书中 Fig8.63(b)的增益表达式为:
2
 2 f 2  G20 s  s  
2

  10 1   
 2 f 1 
 Q2  2 f 2  2 f 2  
G   (8-23)
 s  s  
2

1   
 Q1  2 f1  2 f1  
 
(c) 书中 Fig8.63(c)的增益表达式形式为:
 s  s  
2

G0 1   
 Q  2 f3  2 f3  
G   (8-24)
 s  s 
1  1  
 2 f1  2 f 2 
对于 G 的取值,有:

 2 f  2 f 
20log10 G0 1
2 
2
 G (8-25)
  3  
2 f
根据式(8-25)可求得:

 2 f3 
2 G
G0  10 20 (8-26)
2 f1  2 f
故书中 Fig8.63(c)的增益表达式为:

- 254 -
 2 f3 
2 G  s  s  
2

10  1   
 Q  2 f3  2 f3  
20
2 f1  2 f 2
G   (8-27)
 s  s 
1  1  
 2 f1  2 f 2 

Problem 8.3

推导书中 Fig8.63(a)至(c)增益波特图的低频增益渐近线表达式。
Solution:
(a) 书中 Fig8.63(a)的低频增益渐进线为:
y  G  20 log10   log10  2 f 0   (8-28)

即:
  
y  20 log10    G (8-29)
 2 f 0 
(b) 由(8-23)可知,书中 Fig8.63(b)的低频增益渐进线为:
 2 f 2 G   f2 
y  20log10  2
 10   40log10    G
20
(8-30)
 2 f1    f1 
(c) 由(8-27)可知,书中 Fig Fig8.63(c)的低频增益渐进线为:
  2 f3 2 G    2 f3 2 
y  20log10  10   20log10 
20
  G (8-31)
 2 f1  2 f 2   2 f1  2 f 2 

Problem 8.4

推导书中 Fig8.16 所示的 3 个增益渐近线的表达式。


Solution:
低频段的增益渐近线表达式为:
y  32dB (8-32)

中频段的增益渐近线表达式为:
 f 
y  20 log10    32  20 log10 f  72 (8-33)
 f1 
高频段的增益渐近线表达式为:
y  40log10 (2 f )  170 (8-34)

- 255 -
Problem 8.5

传递函数的实验性测量。书中 Fig8.64 显示了通过实验测量方式得到的某放


大器的增益和相位数据曲线。此题的目的是推导增益函数 A( s) 的表达式。你可以

凭借实验测量得到的增益相位曲线来推导增益渐近线和转折频率。你的增益渐近
线和相位渐近线必须满足以下规则:增益渐近线的斜率必须是 20dB 每十倍频程
的整数倍,对于实数极点的相位渐近线必须是 45 每十倍频程的整数倍。相位
渐进线和增益渐近线必须是彼此一致的。
你可以先从增益数据来猜测增益函数 A( s) ,进而得到相位曲线,再与实验得

到的相位曲线相对比。如果和实验得到的相位曲线有偏差,那么修改你猜测的增
益函数 A( s) ,再重新得到相位曲线,然后再次对比。你应该可以得到增益函数

A( s) 的数学表达式,此外还应该在你得到的增益曲线图上标出转折频率和斜率。

Solution:
首先画出增益曲线的渐近线,如图 8-1 所示:

图 8-1 Problem8.5 的增益曲线渐近线


从图中可得到近似的转折频率:
f1  20Hz 1  2 f1
f 2  130Hz2  2 f 2 
(8-35)
f3  4.2kHz3  2 f3
f 4  130kHz4  2 f 4
根据式(8-35),及书中 Fig8.64 的增益相位 Bode 图猜测其增益函数为:

- 256 -
 s  s 
G0 1    1  
1   4 
A( s )   (8-36)
 s  s 
1   1  
 2   3 

注意在转折频率 f 4 的位置,增益 Bode 图的斜率变化为 20dB ,而相位 Bode

 s   s   s 
图仍旧是递减的,故其形式为  1   ,而非  1   。如果是  1   的形式,
 4   4   4 

那么其相位 Bode 图中的相位在转折频率 f 4 附近会有减缓之前的相位递减的趋势。

根据第一段的水平增益渐近线的值,可求出 G0 的值为:

20log10 G0  18  G0  8 (8-37)

将式(8-35)和式(8-37)代入到式(8-36)中,变可得到 A( s) 的数学表达式:

7.794 108 s 2  0.06365s  8


A( s)  (8-38)
4.639 108 s 2  0.001262s  1

使用 Matlab 画出 A( s) 的增益相位 Bode 图,如图 8-2 所示:

Bode Diagram
35

30

25
Magnitude (dB)

20

15

10

0
90

45

0
Phase (deg)

-45

-90

-135

-180
1 2 3 4 5 6
10 10 10 10 10 10
Frequency (Hz)

图 8-2 根据得到的增益表达式画出的 Bode 图


注意使用 Matlab 画图时,注意横轴的单位为 Hz ,而非 rad/s 。此外在 Bode
图的属性里,双击得到 Bode 图即可出现属性对话框,在属性对话框的 Option 里,
要把 Unwrap phase 取消,这样相位的区间就转变为 180 180 。如图 8-3 所
示:

- 257 -
图 8-3 Matlab 画图注意事项

Problem 8.6

传递函数的实验性测量。书中 Fig8.65 画出了某一无源网络通过实验测量在


某一测试点处的阻抗 Z (s) 的增益相位 Bode 图。此题的目的是得到 Z (s) 的数学表

达式。通过在增益相位 Bode 图上画出增益渐近线,来推导 Z (s) 的数学表达式。

你应该可以得到 Z (s) 的数学表达式,并根据 Z (s) 的数学表达式画出与书中

Fig8.65 一样的增益相位 Bode 图。


Solution:
首先画出增益渐近线,如图 8-4 所示:

8dB

图 8-4 书中 Fig8.65 的增益渐近线

- 258 -
根据图 8-4 中的渐近线,可得到转折频率为:
f1  130Hz 1  2 f1
(8-39)
f 2  800Hz 2  2 f 2

根据图 8-4 中的渐近线,猜测阻抗 Z (s) 为:

 s  s  
2

G0 1    
 Q2  2  
Z ( s)  (8-40)
 s 
1  
 1 
由图 8-4 可知,增益超调为 8dB ,故可知:
20log10 Q  8dB  Q  2.5 (8-41)

由于图 8-4 中的低频水平渐近线的增益值为 14dB ,故可知:


20log10 G0  14dB  G0  5 (8-42)

将式(8-39)、式(8-41)和式(8-42)代入到式(8-40)中,可得到阻抗 Z (s) 的表达

式为:

1.979 107 s 2  0.0003979s  5


Z ( s)  (8-43)
0.001224s  1

使用 Matlab 画出阻抗 Z (s) 的增益相位 Bode 图,如图 8-5 所示:

Bode Diagram
30

20
Magnitude (dB)

10

-10
90

45
Phase (deg)

-45

-90
1 2 3 4
10 10 10 10
Frequency (Hz)

图 8-5 根据得到的增益表达式画出的 Bode 图

Problem 8.7

在书中的 7.2.9 节中,已推导了非理想的反激变换器的小信号交流模型,如


书中 Fig7.27 所示。构建变换器输出阻抗 Z out ( s ) 的增益相位 Bode 图,并给出

- 259 -
Z out ( s ) 的数学表达式。注意,Z out ( s ) 包含负载电阻 R 。元器件的值为:D  0.4 ,

n  0.2 , R  6 , L  600μH , C  100μF , Ron  5 。

Solution:
这里重画书中 Fig7.27 所示的小信号交流模型,如图 8-6 所示:
 Vˆ
 Vg  IRon   d (t )
iˆg (t ) iˆ(t ) L  n

DRon
Idˆ (t )
vˆg (t ) Idˆ (t ) C R vˆ(t )
n

1: D D : n
图 8-6 反激变换器的小信号模型

阻抗 Z out ( s ) 的计算方式为:

vˆ( s)
Zout ( s)   (8-44)
iˆLoad ( s) dˆ 0
vˆg 0

令图 8-6 中的占空比扰动 dˆ  0 、输入电压 vˆg  0 ,并添加电流源 iˆLoad (s) ,

如图 8-7 所示:

DRon
C R vˆ(t ) iˆLoad ( s )

D : n

图 8-7 输出阻抗的计算
图 8-7 可简化为如图 8-8 所示:

 DRon  sL    
n
C R vˆ(t ) iˆLoad ( s )
D 

图 8-8 进一步简化,以便计算输出阻抗
则输出阻抗为:
1 1 1 1
 2
  (8-45)
1 R
 DRon  sL    
Z out n
D  sC

由(8-45)可推出:

- 260 -
Rn2  DRon  sL 
Z out ( s)  (8-46)
CLRn2 s 2   Ln2  CDRRon n2  s  D2 R  DRon n2

将各元器件的值带入到式(8-46)中,可得:
0.000144s  0.48
Zout (s)  (8-47)
1.44 108 s 2  0.000072s  2.24
输出阻抗的增益相位 Bode 图如图 8-9 所示:
Bode Diagram
10

0
Magnitude (dB)

-5

-10

-15

-20
90

45
Phase (deg)

-45

-90
1 2 3 4
10 10 10 10
Frequency (Hz)

图 8-9 输出阻抗的增益相位 Bode 图

Problem 8.8

对于书中 7.2.9 节中的非理想反激变换器。


(a) 推导从控制到输出的传递函数 Gvd ( s) 、从输入到输出的传递函数 Gvg ( s) ,

并表达成标准形式。
(b) 推导传递函数静态特性的数学表达式。
(c) 使用以下数值构建从控制到输出的传递函数,n  2 ,Vg  48V ,D  0.3 ,

R  5 , L  250μH , C  100μF , Ron  1.2 。并在 Bode 图中标出渐近线、渐

近线的斜率、 Q 值、所有的转折频率。

Solution:
(a) 书中 Fig7.2.9 节所述的非理想反激变换器的交流小信号模型如图 8-10 所
示:

- 261 -
 Vˆ
 Vg  IRon   d (t )
iˆg (t ) iˆ(t ) L  n

DRon
Idˆ (t )
vˆg (t ) Idˆ (t ) C R vˆ(t )
n

1: D D : n
图 8-10 非理想反激变换器交流小信号模型

对于从控制到输出的传递函数的推导,首先令 vˆg  0 ,则图 8-10 可简化为

图 8-11 所示:
 Vˆ
 Vg  IRon   d (t )
L  n

DRon
Idˆ (t )
C R vˆ(t )
n

D : n

图 8-11 vˆg  0 时的非理想反激变换器交流小信号模型

应用叠加定理,当只考虑电压源,忽略掉电流源时,并将副边侧等效至原边
侧时,图 8-11 可简化为图 8-12 所示:

 Vˆ
 Vg  IRon   d (t )
L  n

DRon
 R  D  D
2

   vˆ(t )
 1  sCR  n  n

图 8-12 vˆg  0 时的非理想反激变换器简化后的交流小信号模型

根据图 8-12 可知:

 Vˆ D
 Vg  IRon   d (t ) vˆ(t )
 n
 n (8-48)
 R   D   R   D 
2 2

sL  DRon       
 1  sCR   n   1  sCR   n 
由上式可推出:
vˆ DR V  Vg n  IRon n 
Gvdv   (8-49)
dˆ Idˆ
0 CLRn2 s 2   Ln2  CDRRon n2  s  D2 R  DRon n 2
n

- 262 -
当只考虑电流源,忽略掉电压源时,并将原边侧等效至副边侧时,图 8-11
可简化为图 8-13 所示:

Idˆ (t )
2

 sL  DRon    
n
C R vˆ(t )
D  n

图 8-13 vˆg  0 时的非理想反激变换器简化后的交流小信号模型

根据图 8-13 可得:

Idˆ 1
vˆ   (8-50)
n sC  1  1
2
R  n 
    sL  DRon 
D 
由上式可推出:


Gvdi 
dˆ  Vˆ
 Vg  IRon   d ( t )  0
 n
(8-51)
 IRn  DRon  sL 

CLRn 2 s 2   Ln 2  CDRRon n 2  s  D2 R  DRon n 2

故从控制到输出的传递函数为:
 R  DIRon n  DVg n  DV  DIRon n  ILns 
Gvd  Gvdv  Gvdi  (8-52)
CLRn2 s 2   Ln2  CDRRon n 2  s  D2 R  DRon n 2

对于从输入到输出的传递函数,令 dˆ  0 ,图 8-10 可简化为如图 8-14 所示:

iˆg (t ) iˆ(t ) L

DRon
vˆg (t ) C R vˆ(t )

1: D D : n

图 8-14 dˆ  0 时的交流小信号模型

将图 8-14 中左右部分等效至中间部分,如图 8-15 所示:

- 263 -
iˆ(t ) L DRon

 R   D  D
2

Dvˆg (t )    vˆ(t )
 1  sCR   n  n

图 8-15 dˆ  0 时的简化后的交流小信号模型

根据图 8-15 可得:


D
Dvˆg vˆ
 n (8-53)
 R  D   R  D 
2 2

sL  DRon       
 1  sCR  n   1  sCR  n 
由上式可推出:
vˆ DDRn
Gvg   (8-54)
vˆg
dˆ  0
CLRn s   Ln  CDRRon n 2  s  D2 R  DRon n 2
2 2 2

对于稳态的直流量,图 8-10 可简化为图 8-16 所示:

I DRon

 D  D
2
DVg   R V
 n  n

图 8-16 简化的稳态直流模型
故有:
n V
I
D R
(8-55)
DDRVg n
V 2
D R  DRon n 2

Gvd 的标准形式为:

s
Gd 0 (1  )
z
Gvd  2
(8-56)
s  s 
1  
Q0  0 
将式(8-52)变换成式(8-56)的形式,可得:

- 264 -
D2 R  DRon n 2
0 
CLRn 2
CLR  D2 R  DRon n 2 
Q
n  L  CDRRon  (8-57)
RVg D2 n  RVD2  RonVn 2
Gd 0 
D3 R  DRon Dn 2
RVg D2 n  RVD2  RonVn 2
z 
LVn 2

Gvg 的标准形式为:

Gg 0
Gvg  2
(8-58)
s  s 
1  
Q0  0 

其中 Gg 0 为:

DDRn
Gg 0  (8-59)
D R  DRon n 2
2

(b) 静态特性(自然特性)的表达式为式(8-57)和式(8-59)。
(c) 将元器件的具体数值代入到式(8-57)和式(8-59),可得:
0  2.7893 103 rad/s
Q 0.8108
Gd 0  64.0123 (8-60)
z  6.7267 103 rad/s
Gg 0  0.5398
两个传递函数为:
0.03702s  249
Gvd 
5 107 s 2  0.00172s  3.89
(8-61)
2.1
Gvg  7 2
5 10 s  0.00172s  3.89
Gvd 的 Bode 图如图 8-17 所示:

- 265 -
Bode Diagram
40

30

20

Magnitude (dB)
10

-10

-20
360

315

270
Phase (deg)

225

180

135

90
2 3 4 5 6
10 10 10 10 10
Frequency (rad/s)

图 8-17 Gvd 的 Bode 图

Gvg 的 Bode 图如图 8-18 所示:

Bode Diagram
0

-10

-20
Magnitude (dB)

-30

-40

-50

-60

-70
0

-45
Phase (deg)

-90

-135

-180
2 3 4 5
10 10 10 10
Frequency (rad/s)

图 8-18 Gvg 的 Bode 图

Problem 8.9 (待继续完成)

RLC 滤波电路的增益 Bode 图。如图 8-19 所示,推导戴维宁输出阻抗 Z out 和

传递函数 H ( s)  v2 / v1 ,并画出它们的增益 Bode 图。给出近似的表达式及相应

的近似转折频率,是否全部的元器件均对 Z out 和 H ( s ) 影响重大?

- 266 -
R1 L1
10Ω 10mH
C2
220μF Z out
v1 C1 R3 v2
R2 47nF 1kΩ
100
图 8-19 滤波电路
Solution:
对于输出阻抗的求解,如图 8-20 所示:
R1 L1
10Ω 10mH
C2
220μF Z out
C1 R3 v2
R2 47nF 1kΩ
100
图 8-20 输出阻抗的求解
故:
1
Z out  (8-62)
1 sC2 1
  sC1 
R1  sL1 1  sR2C2 R3

将元器件的数值代入到式(8-62),可得:
109  s  103  11s  500 
Z out  (8-63)
517 s 3  1.215 108 s 2  1.222 1012 s  5.05 1013
输出阻抗的增益 Bode 图如图 8-21 所示:
Bode Diagram
40

30
Magnitude (dB)

20

10

0
90

45
Phase (deg)

-45

-90
1 2 3 4 5 6 7
10 10 10 10 10 10 10
Frequency (rad/s)

图 8-21 输出阻抗的 Bode 图

对于传递函数 H ( s ) ,有:

- 267 -
Z1  R1  sL1
1
Z 2  R2 
sC2
1
Z3 
sC1
Z 4  R3 (8-64)
1
ZZ 
1 1 1
 
Z 2 Z3 Z 4
ZZ
H (s) 
Z1  Z Z
将元器件的数值代入到式(8-64),可得:

1011 11s  500 


H ( s)  (8-65)
517 s 3  1.215 108 s 2  1.222 1012 s  5.05 1013

传递函数 H ( s ) 的 Bode 图如图 8-22 所示:

Bode Diagram
0

-20
Magnitude (dB)

-40

-60

-80

-100
0

-45
Phase (deg)

-90

-135

-180
1 2 3 4 5 6 7
10 10 10 10 10 10 10
Frequency (rad/s)

图 8-22 H ( s ) 的 Bode 图

Problem 8.10

运放滤波电路。如图 8-23 所示,此电路是 PID 控制的实际实现电路,经常


被用来修改环路增益以提高电路性能。画出传递函数 v2 ( s) / v1 ( s) 的幅值 Bode 图,

你可以假设运放的增益是无穷大的。

- 268 -
R4 C2
2kΩ 1μF

R3 20kΩ
R2 C3 800pF
1kΩ

R1 C1
v1 100Ω 24nF v2

图 8-23 运放实现的 PID 控制器电路


Solution:
如下定义阻抗:
1
Z1  R1 
sC1
Z 2  R2
Z1Z 2
Z3 
Z1  Z 2
1
Z 4  R4  (8-66)
sC2
Z 5  R3
1
Z6 
sC3
1 1 1 1
  
Z7 Z 4 Z5 Z6

故有:
v1 v2
 (8-67)
Z3 Z 7

将元器件值带入到式(8-67)中,可得:
v2 ( s) 1250000  s  500  33s  1250000 
 (8-68)
v1 ( s)  3s  1250000   s 2  688000s  31250000
增益和相位的 Bode 图如图 8-24 所示:

- 269 -
Bode Diagram
30

20

Magnitude (dB)
10

-10

-20
270

225
Phase (deg)

180

135

90
0 1 2 3 4 5 6 7 8
10 10 10 10 10 10 10 10 10
Frequency (rad/s)

图 8-24 PID 控制器 Bode 图

Problem 8.11

构建 Problem 8.10 的传递函数 v2 ( s) / v1 ( s) 的相位 Bode 图。

Solution:
相位 Bode 图如图 8-24 所示。

Problem 8.12 (待继续完成)

构建图 8-25 所示的输入滤波网络输出阻抗的增益 Bode 图,各器件的值为:


L1  100μHL2  16mH
C1  1000μFC2  10μF
R1  5ΩR2  50Ω

L1 R2 L2

Z out
v1 C1 R1 C2

图 8-25 输入滤波网络
Solution:
定义阻抗如下:

- 270 -
1 1 1
   sC1
Z1 sL1 R1
Z 2  R2  sL2
Z 3  Z1  Z 2 (8-69)
1
Z4 
sC2
1 1 1
 
Z5 Z 4 Z3
将各器件的值带入到式(8-69),可得:

105  s 3  3.325 103 s 2  1.069 107 s  3.125 1010 


Z5  (8-70)
s 4  3.325 103 s 3  1.694 107 s 2  3.25 1010 s  6.25 1013
输出阻抗的 Bode 图如图 8-26 所示:
Bode Diagram
40

30
Magnitude (dB)

20

10

0
0
Phase (deg)

-45

-90
2 3 4 5
10 10 10 10
Frequency (rad/s)

图 8-26 输出阻抗的 Bode 图

Problem 8.13 (待完成)

如图 8-27 所示的两级滤波电路,其输出阻抗应该满足相应的输入滤波电路
标准,因此需要构建输出阻抗的幅值增益 Bode 图,尽管此滤波电路含有 6 个无
源器件,但其幅值 Bode 图可以用简单的方法画出。元器件的值为:
L1  32mHC1  32μF
L2  400μHC2  6.8μF
L3  800μHR1  10Ω
L4  1μHR2  1Ω

(a) 构建 Z s 的 Bode 图

(b) 改变器件的值,使得 Z s 在 500Hz 时为 5 ,在 1kHz 时为 2.5 ,最好通

过改变 1 个器件的值来达到此目的。

- 271 -
L1 L3

R1 L2 R2 L4

Zs
vg C1 C2

图 8-27 输入滤波电路
Solution:
待做!!!

Problem 8.14 (待完成)

Problem 8.15

包含输入滤波电路的开环 Buck-Boost 变换器。其小信号交流模型如图 8-28


所示,各元器件的值如下所述,构建变换器输出阻抗的 Bode 图。
D  0.6L f  150μH
R  6ΩC f  16μF
C  0.33μFCb  2200μF
L  25μHR f  1Ω

Lf L
V
g  V  dˆ ( s )

Rf Z out ( s )
vˆg (t ) Cb
Cf Idˆ ( s ) Idˆ ( s ) C R

1: D D :1
图 8-28 带输入滤波的 Buck-Boost 变换器小信号模型
Solution:
令 vˆg  0 , dˆ  0 ,输出阻抗的计算如图 8-29 所示:

Lf L
Rf Z out ( s )
Cf C R
Cb

1: D D :1
图 8-29 输出阻抗计算电路图
令:

- 272 -
1 1 1
   sC f
Z1 sL f R  1
f
sCb (8-71)
1 1
 sC 
Z2 R

则图 8-29 可以简化为如图 8-30 所示:

Z out ( s )
D 2 Z1 Z2

D :1

图 8-30 简化的输出阻抗计算电路图
则输出阻抗为:
1 1 1
  2 (8-72)
Z out Z 2 D Z1  sL
D2
将式(8-71)代入到式(8-72)中,可得:

3.03 106 s 4  1.911011 s3  3.99 1015 s 2  1.811018 s


Zout  (8-73)
s5  5.68 105 s 4  5.25 1010 s3  1.88 1015 s 2  8.38 1018 s  3.67 1021
输出阻抗的 Bode 图如图 8-31 所示:
Bode Diagram
20

10
Magnitude (dB)

-10

-20

-30
90

45
Phase (deg)

-45

-90
2 3 4 5 6 7
10 10 10 10 10 10
Frequency (rad/s)

图 8-31 输出阻抗 Bode 图

Problem 8.16

处于 CCM 模式的 Watkins-Johnson 变换器的小信号方程为:

- 273 -
diˆ(t )
L   Dvˆ(t )   2Vg  V  dˆ (t )   D  D  vˆg (t )
dt
dvˆ(t ) vˆ(t )
C  Diˆ(t ) 
dt R
iˆg (t )   D  D  iˆ(t )  2 Idˆ (t )

(a) 推导 Gvg ( s) 和 Gvd ( s) 的解析表达式。

(b) 推导 Gvg ( s) 和 Gvd ( s) 的静态特性,并将其表示成 Vg 、 D 、 R 、 L 、 C 的函

数。
(c) 当 Vg  28V 、D  0.25 、R  28Ω 、C  100μF 、L  400μH 时,画出 Gvd ( s)

的 Bode 图。
Solution:
(a) 根据小信号方程,可构建等效电路如图 8-32 所示:

iˆg (t ) iˆ(t ) L  2V
g  V  dˆ (t )

vˆg (t ) 2 Idˆ (t ) C R vˆ(t )

1:  D  D  D :1

图 8-32 Watkins-Johnson 变换器

对于 Gvd ( s) ,首先令 vˆg  0 ,则图 8-32 可简化为如图 8-33 所示:

L  2V g  V  dˆ (t )

C R vˆ(t )

D :1

图 8-33 Watkins-Johnson 变换器计算 Gvd ( s ) 的等效电路

图 8-33 可继续简化为如图 8-34 所示:

- 274 -
L  2V g  V  dˆ (t )

RD 2
Dvˆ(t )
1  sCR

图 8-34 简化的 Gvd ( s ) 的等效电路

故有:
vˆ( s ) 2 DRVg  DRV
Gvd ( s )   (8-74)
dˆ ( s ) vˆg 0 CLRs  Ls  RD
2 2

对于 Gvg ( s) ,令 dˆ  0 ,则图 8-32 可简化为如图 8-35 所示:

iˆg (t ) iˆ(t ) L

vˆg (t ) C R vˆ(t )

1:  D  D  D :1

图 8-35 计算 Gvg ( s) 的等效电路

图 8-35 可继续简化为如图 8-36 所示:

iˆ(t ) L

RD 2
 D  D  vˆg (t ) Dvˆ(t )
1  sCR

图 8-36 计算 Gvg ( s) 的简化的等效电路

故有:
vˆ( s) DR  D  D 
Gvg ( s)   (8-75)
vˆg ( s) CLRs 2  Ls  RD 2
dˆ  0

(b) Gvd ( s) 的标准形式为:

- 275 -
 s 
Gd 0 1  
Gvd   z 
2 (8-76)
s  s 
1  
Q0  0 
比较式(8-74)和式(8-76)可得:
C
Q  DR
L
D
0 
CL (8-77)
2Vg  V
Gd 0 
D
z  

Gvg (s) 的标准形式为:

Gg 0
Gvg ( s )  2
(8-78)
s  s 
1  
Q0  0 
比较式(8-75)和式(8-78)可得:

C
Q  DR
L
D
0  (8-79)
CL
D  D
Gg 0 
D
根据图 8-32,其稳态直流量的关系为:
 D  DVg  DV
 D  DVg (8-80)
V
D
将式(8-80)代入到式(8-77)可得:
Vg
Gd 0  (8-81)
D2
(c) 将元器件的值代入到式(8-74)可得:
784
Gvd (s)  6 2
(8-82)
1.12 10 s  0.0004s  1.75
其 Bode 图如图 8-37 所示:

- 276 -
Bode Diagram
80

60

Magnitude (dB)
40

20

0
0

-45
Phase (deg)

-90

-135

-180
2 3 4
10 10 10
Frequency (rad/s)

图 8-37 Gvd ( s ) 的 Bode 图

Problem 8.17

如图 8-38 所示的 Buck 变换器,其参数为:


Vg  120VD  0.6
R  10ΩRg  2Ω
L  550μHC  100μF

(a) 推导从输入到输出的传递函数 Gvg 的解析表达式。

(b) 求解 Gvg 的静态特性。

(c) 构建 Gvg 的 Bode 图。

ig (t ) Rg L i (t )

vg (t ) C R v(t )

图 8-38 Buck 变换器


Solution:
(a) 交流小信号模型已在 Problem 7.8 中推导,这里重新画出如图 8-39 所示:
iˆg (t ) iˆ(t ) L DRg V
g  Rg I  dˆ (t )

vˆg (t ) Idˆ (t) C R vˆ(t )

1: D

图 8-39 Buck 变换器的交流小信号模型

- 277 -
对于 Gvg 的推导,首先令 dˆ  0 ,则图 8-39 可继续简化为如图 8-40 所示:

iˆ(t ) L DRg

Dvˆg (t ) C R vˆ(t )

图 8-40 dˆ  0 时的 Buck 变换器交流小信号模型

则有:
vˆ( s) DR
Gvg   (8-83)
vˆg ( s)
dˆ  0
CLRs   L  CDRRg  s   R  DRg 
2

(b) Gvg 的标准形式为:

Gg 0
Gvg  2
(8-84)
s  s 
1  
Q0  0 
比较式(8-84)和式(8-83),可得:

R  DRg
0 
CLR
CLR  R  DRg 
Q (8-85)
L  CDRRg
DR
Gg 0 
R  DRg

将各元器件值代入到式(8-85),可得:
0  4.51rad/s
Q  1.4182 (8-86)
Gg 0  0.5357

(c) Gvg 的 Bode 图如图 8-41 所示:

- 278 -
Bode Diagram
0

-10

-20

Magnitude (dB)
-30

-40

-50

-60
0

-45
Phase (deg)

-90

-135

-180
2 3 4 5
10 10 10 10
Frequency (rad/s)

图 8-41 输入到输出的传递函数 Bode 图

Problem 8.18

电容的损耗机制。介电损耗和接触电阻可以使用等效串联电阻(esr)来建
模。对于那些使用具有较高介电常数的介电材料的电容,如电解电容、钽电容、
多层陶瓷电容,其等效串联电阻都较大。
某 Buck 变换器的电感值为 1.6mH ,其稳态运行时的占空比 D  0.5 ,输出电
容可以使用 16μF 的电容加上 0.2Ω 的等效串联电阻进行建模。负载电阻为 10Ω ,

变换器运行在 CCM 模式,稳态输入电压 Vg  120V 。

(a) 推导 Gvg ( s) 的解析表达式。

(b) 求解 Gvg ( s) 的静态特性。

(c) 构建 Gvg ( s) 的 Bode 图。

Solution:
(a) Buck 变换器的交流小信号模型如图 8-42 所示:

Vg dˆ (t )
iˆg (t ) iˆ(t ) L

Resr
vˆg (t ) Idˆ (t) R vˆ(t )
C
1: D
图 8-42 含有电容等效串联电阻的 Buck 小信号交流模型

对于 Gvg ( s) 的求解,首先令 dˆ  0 ,则图 8-42 可简化为如图 8-43 所示:

- 279 -
iˆ(t ) L

Resr
Dvˆg (t ) R vˆ(t )
C

图 8-43 求解 Gvg ( s) 的简化等效电路

则有:
1 1 1
 
Z1 R R  1
esr
sC (8-87)
Dvˆg vˆ

sL  Z1 Z1

根据式(8-87)可求出:
vˆ DR 1  CResr s 
Gvg   (8-88)
vˆg CL  R  Resr  s 2   L  CRResr  s  R
dˆ  0

(b) Gvg ( s) 的标准形式为:

 s 
Gg 0  1  
Gvg   1 
2 (8-89)
s  s 
1  
Q0  0 
比较式(8-88)和式(8-89),可得:

R
0 
CL  R  Resr 
CLR  R  Resr 
Q
L  CRResr (8-90)
Gg 0  D
1
1 
CResr

(c) 将元器件的值代入到式(8-90),可得:
0  6.19 103 rad/s
Q  0.99
(8-91)
Gg 0  0.5
1  312.5 103 rad/s

- 280 -
Gvg (s) 的 Bode 图如图 8-44 所示:

Bode Diagram
50

0
Magnitude (dB)

-50

-100

-150
0

-45
Phase (deg)

-90

-135

-180
1 2 3 4 5 6 7
10 10 10 10 10 10 10
Frequency (rad/s)

图 8-44 Gvg ( s) 的 Bode 图

Problem 8.19 (待完成)

LCC 谐振逆变电路包含以下传递函数:
sC1R
H ( s)  (8-92)
1  sR  C1  C2   s 2 LC1  s 3 LC1C2 R

(a) 当 C1 足够大时,此传递函数可以简化成一个反转极点和一对二次极点的形

式。推导此形式下的转折频率和 Q 值,画出一般形式下的幅值渐近线,并推导简

化形式的有效性前提条件。
(b) 当 C2 足够大时,此传递函数也可以简化成一个反转极点和一对二次极点

的形式。推导此形式下的转折频率和 Q 值,画出一般形式下的幅值渐近线,并推

导简化形式的有效性前提条件。
(c) 当 C1  C2 时,并且二次极点具有足够大的 Q 值,此时传递函数也可以简化

成一个反转极点和一对二次极点的形式。推导此形式下的转折频率和 Q 值,画出

一般形式下的幅值渐近线,并推导简化形式的有效性前提条件。
Solution:

- 281 -
Problem 8.20 (待完成)

Problem 8.21

如图 8-45 所示的 Boost 变换器运行于 CCM 模式,其静态占空比 D  0.6 ,


在对数坐标系中,构建以下传递函数的 Bode 图。
(a) 从控制到输出的传递函数 Gvd ( s)

(b) 从交流输入到输出的传递函数 Gvg ( s)

(c) 输出阻抗 Z out ( s )

(d) 输入阻抗 Z in ( s )

L
100μH
vg C R
48V v
33μF 12

f s  200kHz

图 8-45 Boost 变换器


Solution:
(a) Boost 变换器的小信号交流模型如图 8-46 所示:

iˆL (t ) L Vdˆ (t )

vˆL (t )
vˆg (t ) C I L dˆ (t ) R vˆ(t )

D :1

图 8-46 Boost 变换器的小信号交流模型

对于 Gvd ( s) 推导,首先令 vˆg  0 ,则图 8-46 可简化为如图 8-47 所示

iˆL (t ) L Vdˆ (t )

vˆL (t )
C I L dˆ (t ) R vˆ(t )

D :1

图 8-47 Gvd ( s ) 的推导

- 282 -
再利用叠加定理,首先忽略掉电流源的作用,然后将副边等效至原边侧,则
图 8-47 可继续简化为如图 8-48 所示:
iˆL (t ) L Vdˆ (t )

vˆL (t )
RD2
Dvˆ(t )
1  RCs

图 8-48 忽略掉电流源时的 Gvd ( s ) 的推导

则有:
DRV
Gvdv  (8-93)
CLRs  Ls  RD2 2

忽略掉电压源时,并将原边侧等效至副边时,图 8-47 可继续简化至如图


8-49 所示:

sL
C I L dˆ (t ) R vˆ(t )
D 2

图 8-49 忽略掉电压源时的 Gvd ( s ) 的推导

则有:
 I L LRs
Gvdi  (8-94)
CLRs 2  Ls  RD2
对于 I L 的求解,当只考虑稳态分量时,有:

Vg
IL 
RD2 (8-95)
1
V  Vg
D
根据式(8-93)、式(8-94)和式(8-95),可得:
D3 RV  LsVg
Gvd  (8-96)
CLRs 2
 Ls  RD2  D2

将器件值代入到式(8-96),可得:
0.0048s  92.16
Gvd  (8-97)
6.336 109 s 2  1.6 105 s  0.3072
Gvd ( s) 的 Bode 图如图 8-50 所示:

- 283 -
Bode Diagram
60
Gvd

40

Magnitude (dB)
20

-20
360
Gvd

270
Phase (deg)

180

90
2 3 4 5 6
10 10 10 10 10
Frequency (rad/s)

图 8-50 Gvd ( s ) 的 Bode 图

(b) 对于 Gvg ( s) 的推导,令 dˆ  0 ,再将副边侧等效至原边,则图 8-46 可简化

为如图 8-51 所示:


iˆL (t ) L
vˆL (t )
RD2
vˆg (t ) Dvˆ(t )
1  RCs

图 8-51 Gvg ( s) 的推导

则有:
DR
Gvg  (8-98)
CLRs  Ls  RD2 2

将器件值代入到式(8-98),有:
4.8
Gvg  8 2
(8-99)
3.96 10 s  0.0001s  1.92
Gvg (s) 的 Bode 图如图 8-52 所示:

Bode Diagram
20
Gvg
10

0
Magnitude (dB)

-10

-20

-30

-40
0
Gvg

-45
Phase (deg)

-90

-135

-180
2 3 4 5
10 10 10 10
Frequency (rad/s)

图 8-52 Gvg ( s) 的 Bode 图

- 284 -
(c) 输出阻抗的计算如图 8-53 所示:
L
vˆL (t )
Z out
C R

D :1
图 8-53 输出阻抗的计算
则有:
LRs
Zout ( s)  (8-100)
CLRs  Ls  RD2 2

将器件值代入到式(8-100),可得:
0.0012s
Zout (s)  8 2
(8-101)
3.96 10 s  0.0001s  1.92
Z out ( s ) 的 Bode 图如图 8-54 所示:

Bode Diagram
30
Zout
20

10
Magnitude (dB)

-10

-20

-30
90
Zout

45
Phase (deg)

-45

-90
2 3 4 5
10 10 10 10
Frequency (rad/s)

图 8-54 Z out ( s ) 的 Bode 图

(d) 输入阻抗的计算如图 8-55 所示:


L
vˆL (t )
Z in
C R

D :1
图 8-55 输入阻抗的计算
故有:

CLRs 2  Ls  RD2
Zin  (8-102)
CRs  1
将器件值代入到式(8-102),有:

- 285 -
3.96 108 s 2  0.0001s  1.92
Zin  (8-103)
0.000396s  1

输入阻抗 Z in ( s ) 的 Bode 图如图 8-56 所示:

Bode Diagram
20
Zin
15

10
Magnitude (dB)

-5

-10

-15
90
Zin

45
Phase (deg)

-45
2 3 4 5
10 10 10 10
Frequency (rad/s)

图 8-56 Z in ( s ) 的 Bode 图

Problem 8.22

如图 8-57 所示的运行于 CCM 模式下的正激变换器。其静态电压值为


Vg  380V 、V  28V 。变压器的匝数比为 n1 / n3  4.5 。构建以下传递函数的 Bode

图:
(a) 从控制到输出的传递函数 Gvd ( s)

(b) 从输入到输出的传递函数 Gvg ( s)

注意:对于变压器的参数,只有其匝数比 n1 / n3 会影响正激变换器的小信号

模型。
ig n1 : n2 : n3 L
500μH
vg (t ) C R
v(t )
10μF 7Ω

Q1
f s  150kHz

图 8-57 正激变换器
Solution:

- 286 -
(a) 当开关管导通时,电路如图 8-58 所示:
ig n1 : n2 : n3 L iL
vL iC
vg (t )
C R v(t )

Q1

图 8-58 开关管导通时的正激变换器等效电路模型
故有:
n3
vL  vg  v
n1
v
iC  iL  (8-104)
R
ig n3

iL n1
当开关管关断时,电路如图 8-59 所示:
ig n1 : n2 : n3 L iL
vL iC
vg (t )
C R v(t )

图 8-59 开关管关断时的正激变换器等效电路模型
故有:
vL   v
v
iC  iL  (8-105)
R
ig  0

使用周期平均进行近似后,可得到非线性方程:
n 
 
d iL (t ) T
L S
 d (t )  3 vg (t )  v(t ) T   d (t )  v(t ) TS
dt  n1 
TS S

(8-106)
n 
ig (t )  d (t )  3 iL (t ) T 
 n1 
TS S

将周期平均近似的量表示成静态量与小信号交流变量的和,即:

- 287 -
vg (t )  Vg  vˆg (t )
TS

d (t )  D  dˆ (t )
d (t )  1  D  dˆ (t )
(8-107)
iL (t ) TS
 I L  iˆL (t )
ig (t )  I g  iˆg (t )
TS

v(t ) TS
 V  vˆ(t )

将式(8-107)代入到式(8-106)中,可得:

d I L  iˆL (t )    DV n3 
V 
L  g
dt  n1 
 n n 
  D 3 vˆg (t )  vˆ(t )  Vg dˆ (t ) 3 
 n1 n1 
(8-108)
 n 
  dˆ (t )vˆg (t ) 3 
 n1 
 n   n n   n 
I g  iˆg (t )   D 3 I L    D 3 iˆL (t )  I L 3 dˆ (t )   dˆ (t ) 3 iˆL (t ) 
 n1   n1 n1   n1 
故正激变换器的小信号方程为:
diˆL (t ) n n
L  D 3 vˆg (t )  vˆ(t )  Vg dˆ (t ) 3
dt n1 n1
(8-109)
n n
iˆg (t )  D 3 iˆL (t )  I L 3 dˆ (t )
n1 n1
根据式(8-109)可构建正激变换器的小信号等效电路,如图 8-60 所示:
n
Vg dˆ (t ) 3
iˆg (t ) iˆL (t ) L n1

n3 ˆ
vˆg (t ) IL d (t ) C R vˆ(t )
n1

n3
1: D
n1

图 8-60 正激变换器的小信号等效电路

对于 Gvd 的推导,令 vˆg  0 ,则图 8-60 可简化为如图 8-61 所示:

- 288 -
n3
Vg dˆ (t )
L n1

C R vˆ(t )

图 8-61 正激变换器 Gvd 的推导

故有:
1 1
  sC
Z1 R
n (8-110)
Vg dˆ (t ) 3
n1 vˆ

sL  Z1 Z1

可求得:
n3
Vg R
vˆ n1
Gvd   (8-111)
dˆ vˆg  0 CLRs 2  Ls  R

将元器件的值代入到式(8-111)可得:
591.1
Gvd  8 2
(8-112)
3.5 10 s  0.0005s  7
其 Bode 图如图 8-62 所示:
Bode Diagram
40
Gvd

20
Magnitude (dB)

-20

-40
0
Gvd

-45
Phase (deg)

-90

-135

-180
2 3 4 5 6
10 10 10 10 10
Frequency (rad/s)

图 8-62 Gvd 的 Bode 图

对于 Gvg 的推导,令 dˆ  0 ,则图 8-60 可继续简化为如图 8-63 所示:

- 289 -
iˆL (t ) L

n3 vˆ(t )
D vˆg (t ) C R
n1

图 8-63 Gvg 的推导

故有:
n3
D R
n1
Gvg  (8-113)
CLRs 2  Ls  R
将元器件的值代入到式(8-113),可得:
0.5158
Gvg  8 2
(8-114)
3.5 10 s  0.0005s  7
Gvg 的 Bode 图如图 8-64 所示:

Bode Diagram
-20
Gvg

-40
Magnitude (dB)

-60

-80

-100
0
Gvg

-45
Phase (deg)

-90

-135

-180
2 3 4 5 6
10 10 10 10 10
Frequency (rad/s)

图 8-64 Gvg 的 Bode 图

Problem 8.23

如图 8-65 所示的 Boost 变换器运行于 CCM 模式,其静态特性为:Vg  120V 、

V  300V ,如果想要控制变换器的输入电流波形,就需要推导小信号传递函数:
iˆg ( s )
Gid ( s ) 
dˆ ( s ) vˆg ( s )  0

(a) 推导 Gid ( s) 的解析表达式,将其表达成标准形式,并推倒转折频率、 Q 、

DC 增益的表达式。

- 290 -
(b) 画出 Gid ( s) 的 Bode 图。

ig L
400μH
vg C R
v
10μF 120Ω

f s  100kHz

图 8-65 Boost 变换器


Solution:
(a) Boost 变换器的小信号模型如图 8-66 所示:

iˆL (t ) L Vdˆ (t )

vˆL (t )
vˆg (t ) C I L dˆ (t ) R vˆ(t )

D :1
图 8-66 Boost 变换器小信号模型等效电路

对于 Gid 的推导,令 vˆg  0 ,则图 8-66 可简化为如图 8-67 所示:

iˆL (t ) L Vdˆ (t )

vˆL (t )
C I L dˆ (t ) R vˆ(t )

D :1

图 8-67 vˆg  0 时的 Boost 变换器小信号等效电路

根据叠加定理,首先忽略掉电压源,并将原边侧等效至副边侧,则图 8-67
可继续简化至如图 8-68 所示:
DiˆL (t )
sL
C I L dˆ (t ) R vˆ(t )
D 2

图 8-68 忽略掉电压源时的 Boost 变换器小信号等效电路


故有:

- 291 -
1 1 1
  sC 
Z sL R
D 2
(8-115)
ˆ  Diˆ sL
I L dZ
D 2
L

根据式(8-115)可求得:
DI L R
Gidi (8-116)
CLRs  Ls  RD2 2

当忽略掉电流源时,并将副边侧等效至原边侧,时图 8-67 可继续简化至如


图 8-69 所示:

iˆL (t ) L Vdˆ (t )

vˆL (t )
D2 R
1  RCs

图 8-69 忽略掉电流源时的 Boost 变换器小信号等效电路


故有:

V 1  sCR 
Gidv  (8-117)
CLRs 2  Ls  RD2
根据式(8-116)和式(8-117),可得:
DI L R  V  sCRV
Gid  (8-118)
CLRs 2  Ls  RD2
又由于:

V2
Vg I L  (8-119)
R

利用式(8-119)求得 I L ,代入到式(8-118)中,可得:

2V  CRVs
Gid ( s)  (8-120)
CLRs 2  Ls  RD2
将式(8-120)写成标准形式,即:
 s 
Gd 0 1  
Gid ( s )   z  (8-121)
2
s  s 
1  
Q0  0 
根据式(8-120)和式(8-121)可得:

- 292 -
D
0 
CL
C
Q  DR
L (8-122)
2V
Gd 0 
RD2
2
z 
CR
(b) 将器件值代入到式(8-121),可得:
0.36s  600
Gid (s)  7 2
(8-123)
4.8 10 s  0.0004s  19.2
Gid ( s) 的 Bode 图如图 8-70 所示:

Bode Diagram
60
Gid
50
Magnitude (dB)

40

30

20

10
90
Gid
45
Phase (deg)

-45

-90

-135
2 3 4 5
10 10 10 10
Frequency (rad/s)

图 8-70 Gid ( s) 的 Bode 图

Problem 8.24

如图 8-71 所示的 Buck-Boost 变换器运行于 CCM 模式,其静态值为:


Vg  48V 、 V  24V ,在对数坐标系下,构建以下传递函数的 Bode 图:

(a) 从控制到输出的传递函数 Gvd ( s)

(b) 输出阻抗 Z out ( s )

- 293 -
vg L C R
5 v
50μH 220μF

f s  200kHz

图 8-71 Buck-Boost 变换器


(a) 当开关管导通时,电路如图 8-72 所示:
iC

vg L vL C R
5 v
50μH 220μF

f s  200kHz

图 8-72 开关管导通时的 Buck-Boost 变换器


故有:
vL  v g
v
iC   (8-124)
R
ig  iL

当开关管关断时,电路如图 8-73 所示:

iC
vg L C R
vL 5 v
50μH 220μF

f s  200kHz

图 8-73 开关管关断时的 Buck-Boost 变换器


故有:
vL  v
v
iC  iL  (8-125)
R
ig  0

使用周期平均进行近似后,可得到周期平均后的非线性方程:

- 294 -
d iL (t )
 d (t ) vg (t )  d (t ) v(t )
TS
L TS
dt TS

d v(t )  v(t )   v(t ) 


 d (t )     d (t )   iL (t )  
TS TS TS
C (8-126)
dt  R   TS
R 
   
ig (t )  d (t ) iL (t ) TS
TS

将周期平均近似的量表示成静态量与小信号交流变量的和,即:
vg (t )  Vg  vˆg (t )
TS

d (t )  D  dˆ (t )
d (t )  1  D  dˆ (t )
(8-127)
iL (t ) TS
 I L  iˆL (t )
v(t ) TS
 V  vˆ(t )
ig (t )  I g  iˆg (t )
TS

将式(8-127)代入到式(8-126),可得:

L

d I L  iˆL (t )    DV  DV 
dt  g 
  Dvˆ(t )  Vdˆ (t )  Dvˆg (t )  Vg dˆ (t ) 

   dˆ (t )vˆ(t )  dˆ (t )vˆg (t ) 


d VC  vˆC (t )   V
C    DI L   (8-128)
dt  R
 vˆ(t ) 
    DiˆL (t )  I L dˆ (t ) 
 R 
  dˆ (t )iˆL (t ) 

I g  iˆg (t )   DI L    DiˆL (t )  I L dˆ (t )    dˆ (t )iˆL (t ) 

根据式(8-128)可得 Buck-Boost 变换器的小信号方程为:


diˆL (t )
L  Dvˆ(t )  Vdˆ (t )  Dvˆg (t )  Vg dˆ (t )
dt
dvˆ (t ) vˆ(t )
C C   DiˆL (t )  I L dˆ (t ) (8-129)
dt R
iˆ (t )  Diˆ (t )  I dˆ (t )
g L L

根据式(8-129)可画出小信号等效电路如图 8-74 所示:

- 295 -
iˆL L V g  V  dˆ (t )

vˆg (t ) I L dˆ (t ) I L dˆ (t ) C R v

1: D D :1
图 8-74 Buck-Boost 变换器小信号交流等效电路

对于 Gvd ( s) 的推导,首先令 vˆg  0 ,则图 8-74 可简化为如图 8-75 所示:

iˆL L V g  V  dˆ (t )

I L dˆ (t ) C R v

D :1

图 8-75 vˆg  0 时的 Buck-Boost 变换器小信号交流等效电路

利用叠加定理,首先忽略电压源,并将原边侧等效至副边侧,则图 8-75 可
继续简化至如图 8-76 所示:

L
I L dˆ (t ) C R v
D 2

图 8-76 忽略电压源时的 Buck-Boost 小信号交流等效电路


故有:
1 1 1
  sC 
Z sL R
(8-130)
D 2

ˆ
vˆ  I dZ
L

根据式(8-130)可求得:
I L LRs
Gvdi  (8-131)
CLRs  Ls  RD2
2

而:
V
IL   (8-132)
DR
将式(8-132)代入到式(8-131)可得:
LV
s 
Gvdi  D (8-133)
CLRs 2  Ls  RD2
忽略掉电流源时,并将副边侧等效至原边侧时,图 8-75 可简化为如图 8-77

- 296 -
所示:

iˆL L V g  V  dˆ (t )

R
D2 Dv
1  RCs

图 8-77 忽略电流源时的 Buck-Boost 小信号交流等效电路


故有:

V g  V  dˆ

Dvˆ
(8-134)
sL  D2 Z1 D2 Z1
根据式(8-134),可求得:

 V  Vg  DR
Gvdv  (8-135)
CLRs 2  Ls  RD2
根据式(8-133)和式(8-135)可得:
  LV 
 D R V  Vg   D s 
Gvd   (8-136)
CLRs 2  Ls  RD2
将器件值代入到式(8-136),可得:
0.0018s  240
Gvd  (8-137)
5.5 10 s  5 105 s  2.222
8 2

Gvd 的 Bode 图如图 8-78 所示:

Bode Diagram
100
Gvd

50
Magnitude (dB)

-50
0
Gvd

-45
Phase (deg)

-90

-135

-180
3 4 5 6 7
10 10 10 10 10
Frequency (rad/s)

图 8-78 Gvd 的 Bode 图

(b) 对于输出阻抗的计算,令 vˆg  0 、 dˆ  0 ,则图 8-74 可简化为如图 8-79

所示:

- 297 -
sL Z out
C R v
D 2

图 8-79 输出阻抗的计算
则有:
1 1 1
  sC  (8-138)
Z out sL R
D 2

根据式(8-138)可得:
LRs
Zout  (8-139)
CLRs  Ls  RD2 2

将器件值代入到式(8-139),可得:
0.00025s
Zout  (8-140)
5.5 10 s  5 105 s  2.222
8 2

输出阻抗的 Bode 图如图 8-80 所示:


Bode Diagram
20
Zout
10
Magnitude (dB)

-10

-20

-30
90
Zout

45
Phase (deg)

-45

-90
3 4 5
10 10 10
Frequency (rad/s)

图 8-80 输出阻抗的 Bode 图

- 298 -
第9章 控制器设计

Problem 9.1

推导书中式 9.25 的两种形式,这里重新给出书中式 9.25 的表达式:


cos m
Q
sin m

1  1  4Q 4
m  tan 1

2Q 4

Solution:
书中式 9.25 的前提是环路增益 T ( s) 为:

1
T (s)  (9-1)
 s  s 
 1  
 0   2 

在反馈通路的传递函数 H ( s)  1 的条件下(此条件在一般情况下均成立),

闭环传递函数为:
T ( s) 1 1 1
   (9-2)
1  T ( s) 1  1  s  s  s s2
1   1   1  
T ( s)  0   2  0 02

将闭环传函写成标准形式:
1
(9-3)
s s2
1 
Qc c 2

根据式(9-2)和式(9-3),有:
Qc  0
c  02
0
Q (9-4)
2
c
2 
Q

令 s  j ,代入到开环传函式(9-1)中,有:

- 299 -
Qc 2
T ( )  (9-5)
Q 2  jc

开环传函,即式(9-1)的模函数(幅值函数)为:
Qc 2
T ( )  (9-6)
 Q 2   c 
2 2

令 T ( )  1 ,可求得开环传函的截止频率为:

c 4Q 4  1  1
s  (9-7)
Q 2
开环传函,即式(9-1)的相位函数为:
  
T ( )   tan 1   c  (9-8)
 Q 
根据式(9-7)可求得相位裕度为:
 c  1  c 
m  180  tan 1     tan  
 Qs   Qs 
(9-9)
c 4Q 4  1  1

Qs 2Q 4
根据式(9-9)可求得书中式 9.25 的第二种形式:
 4Q 4  1  1 
m  tan 1  (9-10)
 2Q 4 
 
根据式(9-10)可求得书中 9.25 的第一种形式,即:

1
1
1  tan m    cos m   cos m 
1
 1  tan m  
2 4 2 4
Q     (9-11)
 tan m  
4  sin m  sin m  sin m 
cos m  cos m 

Problem 9.2

如图 9-1 所示的反激变换器包含负反馈环路以调整主输出电压 v1 。一个辅助

绕组可提供输出电压 v2 ,直流输入电压 vg 的范围为 280V  vg  380V ,补偿网络

的传递函数为:

  
Gc ( s )  Gc 1  1 
 s 

- 300 -
其中 Gc  0.05 , f1  1 / 2  400Hz 。

(a) 误差电压 ve (t ) 的稳态值是多少?解释你的推导。

(b) 主输出电压 v1 的稳态值是多少?

(c) 估计辅助输出电压 v2 的稳态值。

64 : 3

C1 R1
v1
L  600μH 220μF 5Ω
vg :1
C2 R2
v2
100μF 2Ω
H (s)
f s  200kHz
隔离 H ( s)  0.2
驱动
vc ve
PWM Gc ( s )
Vm  4V
vref  3V
图 9-1 反激变换器
Solution:
(a) 误差电压 ve 的稳态值为 0。

(b) 主输出电压 v1 的稳态值为:

0.2V1  3  V1  15V (9-12)

(c) 辅助输出电压 v2 的稳态值为:

V1
V2   5V (9-13)
3

Problem 9.3

如图 9-2 所示的 Boost 变换器,其所有元器件均为理想器件。补偿器的增益


为 Gc ( s)  250 / s 。

- 301 -
(a) 构建环路增益 T ( s) 的 Bode 图。推导转折频率和 Q 值。

(b) 求解截止频率和相位裕度。
(c) 构建 1/ 1  T  的幅值 Bode 图。推导转折频率和 Q 值。

(d) 构建闭环的从交流输入到输出的传递函数 Bode 图。推导转折频率和 Q 值。

L
100μH
vg C R
v
48V 33μF 12Ω

1 H (s)
H (s) 
隔离 24
驱动
vc ve
PWM Gc ( s )
Vm  4V
vref  5V

图 9-2 Boost 变换器系统


Solution:
(a) 对于 Boost 变换器,从控制到输出的开环传递函数为:
 s 
Gd 0 1  
Gvd ( s )   z 
2
s  s 
1  
Q0  0 
V
Gd 0 
D
D
0  (9-14)
LC
C
Q  DR
L
D R
2
z 
L
其环路增益为:
1
T ( s)  Gvd HGc (9-15)
Vm

故有:

- 302 -
250Gd 0  s 
1  
24  4  z 
T ( s)  (9-16)
 s  s  
2

s 1    
 Q0  0  
将器件值代入到式(9-16)可得:
0.1628s  3125
T (s) 
s  8.25 108 s 2  0.0002083s  4 
Q  2.7574
0  6.96 103 (9-17)
z  1.92 104
Gd 0  300

环路增益 T ( s) 的 Bode 图如图 9-3 所示:

Bode Diagram
40
T
20
0
Magnitude (dB)

-20
-40
-60
-80
-100
-120
180
135 T

90
Phase (deg)

45
0
-45
-90
-135
-180
2 3 4 5 6
10 10 10 10 10
Frequency (rad/s)

图 9-3 环路增益 T ( s ) 的 Bode 图

(b) 根据式(9-17)可求得截止频率和相位裕度为:
Crossover  791.46rad/s
(9-18)
Pm85.25
(c) 1/ 1  T  的 Bode 图如图 9-4 所示:

其转折频率近似为式(9-17)中的 0 。

Q 值为式(9-17)中的 Q 的倒数。

- 303 -
Bode Diagram
10

-10

Magnitude (dB)
-20

-30

-40

-50
90

45
Phase (deg)

-45
1 2 3 4 5
10 10 10 10 10
Frequency (rad/s)

图 9-4 1/ 1  T  的 Bode 图

(d) 闭环从交流输入到输出的传递函数为:
1
Gcl _ vg  Gvg (9-19)
1 T
其 Bode 图如图 9-5 所示:
Bode Diagram
30

20

10
Magnitude (dB)

-10

-20

-30

-40
90

45

0
Phase (deg)

-45

-90

-135

-180
1 2 3 4 5
10 10 10 10 10
Frequency (rad/s)

图 9-5 闭环从交流输入到输出的 Bode 图

Problem 9.4

某一逆变器具有如下所述的环路增益:
 s 
1  
T ( s )  T0  z 
 s  s  s 
1  1   1  
 1  2   3 
其开环从输入到输出的传递函数为:
1
Gvg  Gg 0
 s  s 
1   1  
 1  3 

- 304 -
对于以上具体数值,有:
T0  1001  500rad/sec
2  1000rad/sec3  24000rad/sec
z  4000rad/secGg 0  0.5

反馈通路上的传递函数为 H ( s)  0.1 。

(a) 画出环路增益的 Bode 图,求解截止频率和相位裕度的数字值。


(b) 构建闭环从输入到输出的传递函数,标出其重要特性。
(c) 构建闭环从参考输入到输出的传递函数,标出其重要特性。
Solution:
(a) 环路增益的传递函数为:
0.025s  100
T ( s)  (9-20)
8.333 10 s  2.125 106 s 2  0.003042s  1
11 3

其 Bode 图如图 9-6 所示:


Bode Diagram
40
T
20

0
Magnitude (dB)

-20

-40

-60

-80
0
T

-45
Phase (deg)

-90

-135

-180
1 2 3 4 5 6
10 10 10 10 10 10
Frequency (rad/s)

图 9-6 环路增益的 Bode 图


其相位裕度和截止频率为:
Pm  52.37
(9-21)
crossover  1.1793 104 rad/s
(b) 闭环从输入到输出的传递函数为:
 s 
Gg 0  1  
Gcl _ vg  Gvg
1
  2  (9-22)
1 T  s  s  s   s 
1  1   1    T0 1  
 1  2   3   z 
其 Bode 图如图 9-7 所示:

- 305 -
Bode Diagram
-20

-40

Magnitude (dB)
-60

-80

-100

-120
45

0
Phase (deg)

-45

-90

-135

-180
1 2 3 4 5 6
10 10 10 10 10 10
Frequency (rad/s)

图 9-7 闭环从输入到输出的 Bode 图


(c) 闭环从参考输入到输出的传递函数为:
1 T
Gcl _ vd  (9-23)
H 1 T
将 H 和 T 的表达式带入到式(9-23),可得:
0.25s  1000
Gcl _ vd  (9-24)
8.333 10 s  2.125 106 s 2  0.02804s  101
11 3

其 Bode 图如图 9-8 所示:


Bode Diagram
40

20
Magnitude (dB)

-20

-40

-60
0

-45
Phase (deg)

-90

-135

-180
3 4 5 6
10 10 10 10
Frequency (rad/s)

图 9-8 闭环从参考输入到输出的 Bode 图

Problem 9.5

如图 9-9 所示的正激变换器,输入电压的静态值为 Vg  380V ,变压器的匝

数比为 n1 / n3  4.5 。占空比被 PWM 发生器限制在 0  d (t )  0.5 ,在此范围内,

占空比 d (t ) 和控制电压 vc (t ) 的关系为:

1 vc (t )
d (t ) 
2 Vm

- 306 -
其中 Vm  3V 。

(a) 推导静态值:占空比 D 、输出电压 V 、控制电压 Vc 。

(b) 运算放大器电路和反馈通路可以通过如图 9-10 所示的方框图进行建模,


其中 H ( s )  R2 /  R1  R2  ,推导传递函数 Gc ( s ) 和 Gr ( s ) 。

(c) 画出可代表整个系统小信号交流变化的方框图,并且推导每个框图的传递
函数。提示:变压器的激磁电感对变换器的动态特性可忽略不计,正激变换器的
小信号模型和 Buck 变换器的小信号模型类似。
(d) 构建环路增益的 Bode 图,求解截止频率和相位裕度。
(e) 构建闭环从输入到输出的 Bode 图,标出其重要特性。位于 120Hz 处的增
益是多少? vg 以什么频率扰动时对输出电压的影响最大?

ig n1 : n1 : n3 L
500μH
vg (t ) C R
v(t )
10μF 7Ω
Q1
f s  150kHz

C1 R3 R1
隔离 13nF 5.6kΩ 81.8kΩ
驱动
R2
vc 18.2kΩ
PWM
vref
5.1V
图 9-9 正激变换器
v

H (s)

vc vref
Gc ( s ) Gr ( s )

图 9-10 运算放大电路的方框图
Solution:
(a) 稳态时 vc 和 vref 应相等,故有:

- 307 -
Vc  vref  5.1V (9-25)

根据 R1 和 R2 的分压,可求得稳态时的输出电压为:

V v
 ref
R1  R2 R2
(9-26)
vref  R1  R2 
V  28V
R2
根据式(9-26)和式(8-108)可求出:
n3
DVg V  0
n1
(9-27)
V n1
D  0.3316
Vg n3

(b) 如图 9-11 所示定义支路电流:

i1 v(t )

C1 R3 R1
13nF 5.6kΩ v 81.8kΩ
ref
i3 i2
R2
vc 18.2kΩ

vref
5.1V

图 9-11 支路电流定义
故有:
v  vref
i1 
R1
vref
i2 
R2
(9-28)
v  vc
i3  ref
1
R3 
sC1
i1  i2  i3
根据式(9-28)可求出:

- 308 -
R2
H
R1  R2

Gr  HG1  H
 R1  R2  C1R1R2 s  C1R1R3 s  C1R2 R3s 
(9-29)
R2  C1 R2 R3 s

Gc 
 R2  C1R2 R3 s  R1  R2 
C1 R1 R2 2 s
(c) 描述整个系统交流小信号变化的方框图如图 9-12 所示:
vˆg
Gvg ( s )

1 d̂ v̂
Gvd ( s )
2Vm

vˆc
Gc ( s ) H (s)

vˆref  0
Gr ( s )

图 9-12 描述整个系统交流小信号变化的方框图

其中 Gvd 已在式(8-111)中求出,这里重新给出:

n3
RVg
vˆ n1
Gvd   (9-30)
dˆ vˆg  0 CLRs 2  Ls  R

(d) 环路增益为:
1
T (s)  Gvd HGc (9-31)
2Vm

将式(9-30)、式(9-29)和 Vm 的值带入到式(9-31)中,可得到环路增益数学表达

式为:

1.425 107 s  1.958 1011


T (9-32)
0.07397 s3  1057 s 2  1.479 107 s
环路增益的 Bode 图如图 9-13 所示:

- 309 -
Bode Diagram
30

20

10

Magnitude (dB)
0

-10

-20

-30

-40
-45

-90
Phase (deg)

-135

-180
3 4 5
10 10 10
Frequency (rad/s)

图 9-13 环路增益的 Bode 图


其相位裕度和截止频率为:
Pm32.54
(9-33)
crossover  1.6639 104 rad/s
(e) 闭环从输入到输出的传函为:
1
Gcl _ vg  Gvg (9-34)
1 T
Gvg 的表达式已在式(8-113)求出,这里重新给出:

n3
D R
n1
Gvg  (9-35)
CLRs 2  Ls  R
将式(9-35)和式(9-31)带入到式(9-34)中,可得:
Gcl _ vg 
0.039s 3  545s 2  7.6 106 s (9-36)
2.6 109 s 5  7.4 105 s 4  2.1s 3  2.9  104 s 2  3.0 108 s  1.4 1012
其 Bode 图如图 9-14 中的绿色线条所示:
Bode Diagram
50
T
Tcl_vg
System: Tcl_vg
0 Frequency (rad/s): 1.8e+04
Magnitude (dB)

Magnitude (dB): -18.9


System: Tcl_vg
Frequency (rad/s): 754
Magnitude (dB): -47.6

-50

-100
90
T
Tcl_vg
0
Phase (deg)

-90

-180
2 3 4 5 6
10 10 10 10 10
Frequency (rad/s)

图 9-14 环路增益和闭环从输入到输出的 Bode 图

- 310 -
从图 9-14 中可得出 120Hz 处的闭环输入到输出的增益为 47.6dB 。也可知
道当 vˆg 以 1.8 104 rad/s ,即 2864Hz 扰动时,对输出电压的影响最大。

Problem 9.6

对于 Problem 9.5,如果输入电压 vg 含有峰值 10V ,频率 120Hz 的扰动。

(a) 120Hz 的输入扰动会造成输出电压扰动,那么输出电压扰动的幅值是多


少?
(b) 修改补偿器网络,使得 120Hz 的输出电压扰动的幅值小于 25mV ,你的修
改不能改变稳态的输出电压值,此外截止频率不能超过 10kHz 。
Solution:
(a) 根据 Problem 9.5 知道,120Hz 处的闭环输入到输出的增益为 47.6dB 。故
对 120Hz 输入扰动的衰减倍数为:
20log R  47.6  R  0.00417 (9-37)

故当输入电压 vg 含有峰值 10V ,频率 120Hz 的扰动时,由于反馈的作用,输

出电压中 120Hz 扰动的峰值为:


Vpeak _120 Hz  R 10  0.0417  41.7mV (9-38)

(b) 要想保证 120Hz 的输出电压扰动幅值小于 25mV ,则可知闭环从输入到输


出的 Bode 图在 120Hz 处的增益要小于:
0.025
20log  52dB (9-39)
10
对于 Gc 的选择,如果只使用 PI 补偿,则会发现如果满足了 120Hz 处增益小

于 52dB 的条件,则相位裕度要小于 10 。


故必须再额外加入 PD 补偿,以提高相位裕度。
所设计的补偿网络为:
 s 
 1 
   z
Gc  Gc 0 1  L   
 s  1 s 
  
 p 
Gc 0  0.376
L  24000 (9-40)
z  4472
 p  89443
未补偿的环路增益、补偿过的环路增益、闭环从输入到输出的 Bode 图、闭
环从参数输入到输出的 Bode 图如图 9-15 所示:

- 311 -
Bode Diagram
50
T
T_ori
0 Tcl_vg
System: Tcl_vg

Magnitude (dB)
Frequency (rad/s): 754 Tcl_vd
Magnitude (dB): -52.6
-50

-100

-150
90
T
45 T_ori
Tcl_vg
0
Phase (deg)

Tcl_vd
System: T
-45 Phase Margin (deg): 48.8
Delay Margin (sec): 1.87e-05
-90 At frequency (rad/s): 4.56e+04
Closed loop stable? Yes

-135

-180
2 3 4 5 6 7
10 10 10 10 10 10
Frequency (rad/s)

图 9-15 增益 Bode 图
环路的截止频率为:
45600
crossover   7257Hz<10kHz (9-41)
2
闭环从参考输入到输出的 Bode 如图 9-15 中的浅蓝色线条所示,即 Tcl _ vd ,

其低频增益为 14.8dB ,即为改变稳态的输出电压值:


28
20log  14.8dB (9-42)
5.1

Problem 9.7

设计含有电流反馈 Boost 变换器的 PI 补偿器。在某些应用场合中,需要控


制变换器的输入电流波形(PFC 就属于这一种)。如图 9-16 所示的 Boost 变换器
包含一个反馈环路,使得变换器输入电流 ig (t ) 与 vref (t ) 成比例。反馈通道是一个

电流传感器,其增益为 H ( s)  0.2V/A 。PWM 发生器的锯齿波峰值为 Vm  3V 。

输入的静态值为: Vg  120V , Vref  2V 。所有的器件均为理想器件。

(a) 推导静态值 D 、 V 和 I g 。

(b) 推导小信号传递函数:
iˆg ( s )
Gid ( s ) 
dˆ ( s )
(c) 画出未补偿的环路增益 Bode 图:
(d) 如果需要环路增益在 120Hz 处的幅值要大于 35dB ,同时保持相位裕度大
于 72 。截止频率不超过 f s /10  10kHz 。设计 PI 补偿网络以满足这些需求,画

出补偿后的环路增益 Bode 图。

- 312 -
(e) 对于你在(d)小问的设计,画出闭环从参考输入到输出的传递函数 Bode
图:
iˆg ( s )
vˆref ( s )

ig L
400μH
vg (t ) C R
v(t )
10μF 120Ω

ig
f s  100kHz H (s)
隔离
驱动 ve
vc
PWM Gc ( s )
H ( s )ig
Vm  3V
vref

图 9-16 Boost 变换器


Solution:
(a) 稳态时的 I g 为:

vref 2
Ig    10A (9-43)
H 0.2
稳态时的输出电压 V 为:

V2
Vg I g   V  380V (9-44)
R
稳态时的占空比 D 为:
Vg
D 
V
D  0.316 (9-45)
D  0.684
(b) 对于 Boost 变换器,当开关管导通时如图 9-17 所示:

- 313 -
ig L
vL ic
vg (t ) C R
v(t )
10μF 120Ω

图 9-17 开关管导通时的 Boost 变换器


故有:
dig
L  vg
dt (9-46)
v
ic  
R
当开关管关断时,Boost 变换器如图 9-18 所示:
ig L
vL ic
vg (t ) C R
v(t )
10μF 120Ω

图 9-18 开关管关断时的 Boost 变换器


故有:
dig
L  vg  v
dt (9-47)
v
ic  ig 
R
使用周期平均后,有:

 
d ig
 d (t ) vg  d (t ) vg  v
Ts
L Ts
dt Ts Ts
(9-48)
 vT   v
 d (t )     d (t )  ig  
Ts
ic s
Ts  R   Ts R 
  
将周期平均近似的量表示成静态量与小信号交流变量的和,即:
vg  Vg  vˆg (t )
TS

d (t )  D  dˆ (t )
d (t )  1  D  dˆ (t )
(9-49)
ig  I g  iˆg (t )
TS

ic TS
 I c  iˆc (t )
v TS
 V  vˆ(t )

- 314 -
将式(9-49)代入到式(9-48)后,可得:

L

d I g  iˆg   V  DV   vˆg  Dvˆ  Vdˆ    dv
ˆ ˆ
dt  g 
(9-50)
 V   vˆ 
I c  iˆc     DI g      Diˆg  I g dˆ   di
ˆˆ 
g
 R   R 
故 Boost 变换器的小信号方程为:

diˆg
L  vˆg  Dvˆ  Vdˆ
dt (9-51)

iˆc    Diˆg  I g dˆ
R
根据式(9-51)可画出小信号等效电路如图 9-19 所示:

iˆg (t ) L Vdˆ (t )

vˆL (t )
vˆg (t ) C I g dˆ (t ) R vˆ(t )

D :1
图 9-19 Boost 变换器交流小信号等效模型

对于 Gid ( s) 的推导,首先令 vˆg  0 ,则图 9-19 可简化至如图 9-20 所示:

iˆg (t ) L Vdˆ (t )

vˆL (t )
C I g dˆ (t ) R vˆ(t )

D :1

图 9-20 vˆg  0 时的 Boost 变换器交流小信号等效模型

运用叠加定理,当忽略电压源的作用时,并将原边侧等效至副边侧,图 9-20
可继续简化至如图 9-21 所示:

Diˆg (t )

sL
C I g dˆ (t ) R vˆ(t )
D 2

图 9-21 忽略掉电压源时的 Boost 变换器交流小信号等效模型


故有:

- 315 -
1 1 1
  sC 
Z sL R
D  2

ˆ  sL Diˆ
I g dZ (9-52)
D 2
g

DI g R
Gidi 
CLRs 2  Ls  RD2
当忽略掉电流源时,并将副边侧等效至原边侧时,图 9-20 可简化至如图
9-22 所示:

iˆg (t ) L Vdˆ (t )

vˆL (t )
R
D2
1  RCs

图 9-22 忽略掉电流源时的 Boost 变换器交流小信号等效模型


故有:
V  CRVs
Gidv  (9-53)
CLRs 2  Ls  RD2
根据式(9-52)和式(9-53)可得:
V  DI g R  CRVs
Gid  Gidi  Gidv  (9-54)
CLRs 2  Ls  RD2
根据式(9-50)中的稳态量可得:
Vg  DV  0  Vg  DV
V (9-55)
  DI g  0  DI g R  V
R
将式(9-55)代入到式(9-54)可得:
2V  CRVs
Gid  (9-56)
CLRs 2  Ls  RD2
(c) 未补偿时的环路增益为:
1
Tuncompensated  Gid H (9-57)
Vm

将器件值带入到式(9-57),可得:
0.09107 s  151.8
Tuncompensated  (9-58)
1.44 106 s 2  0.0012s  36
未补偿时的环路增益 Bode 图如图 9-23 所示:

- 316 -
Bode Diagram
40
T_uncompensated
30

Magnitude (dB)
20

10

-10
90
T_uncompensated
45
Phase (deg)

-45

-90

-135
2 3 4 5
10 10 10 10
Frequency (rad/s)

图 9-23 未补偿时的环路增益 Bode 图


(d) PI 补偿器的形式为:

  
Gc _ pi  Gc 1  L  (9-59)
 s 

由于 PI 补偿器增益提高的斜率为 20dB/decade ,此外由于未补偿时的环路


增益在 120Hz=754rad/s 处的增益为 13.5dB ,为了将此频率处的增益提高到 35dB
以上,则 PI 补偿器中要满足下式:
L  11.885  2  120
(9-60)
20 log(11.885)  35  13.5

选取 L  18  2 120rad/s 、Gc  0.7 。注意: L 太接近 11.885  2 120 时,

你会发现很难同时满足增益和截止频率的要求,而  L 太大于 11.885  2 120 时,

你会发现很难同时满足增益和相位裕度的要求。
选取的 PI 补偿器为:
0.7s  9500
Gc _ pi  (9-61)
s
补偿后的环路增益表达式为:

0.06375s 2  971.5s  1.442 106


Tcompensated  (9-62)
1.44 106 s3  0.0012s 2  36s
补偿后的环路增益如图 9-24 所示:

- 317 -
Bode Diagram
80
T_uncompensated
60 System: T_compensated T_compensated
Frequency (rad/s): 753
Magnitude (dB): 35.5
40

Magnitude (dB)
20

-20

-40
90
T_uncompensated
45 T_compensated
0
Phase (deg)

-45

-90
System: T_compensated
-135 Phase Margin (deg): 72.8
Delay Margin (sec): 2.72e-05
-180 At frequency (rad/s): 4.67e+04
1 2 3 4 5
Closed loop stable? Yes 6
10 10 10 10 10 10
Frequency (rad/s)

图 9-24 补偿后的环路增益
(e) 闭环从参考输入到输出的传递函数为:
iˆg ( s) 1 Tcompensated
 (9-63)
vˆref ( s) H 1  Tcompensated
闭环从参考输入到输出传递函数的 Bode 图如图 9-25 所示:
Bode Diagram
100
T_uncompensated
T_compensated
T_cl_id
50
Magnitude (dB)

-50
90
T_uncompensated
T_compensated
T_cl_id
0
Phase (deg)

-90

-180
1 2 3 4 5 6
10 10 10 10 10 10
Frequency (rad/s)

图 9-25 闭环从参考输入到输出传递函数的 Bode 图

Problem 9.8

对 Buck 变换器进行设计,使其闭环输出阻抗满足指标要求,如图 9-26 所


示。闭环输出阻抗在 0 20kHz 的范围内要小于 0.2Ω 。为了保证瞬态响应良好,
闭环传递函数的极点,在截止频率的范围附近,其 Q 值不超过 1。静态负载电流

的变化范围为 5A 50A ,并且以上指标在此负载电流的范围内均满足。为了简


化考虑,可以假设输入电压的值固定不变。环路增益的截止频率 f c 不能超过

10kHz。所有元器件均为理想。PWM 发生器满足下式:

- 318 -
Vc
D
Vm
vˆ (t )
dˆ (t )  c
Vm

(a) 稳态输出电压 V 的值是多少?有效负载电阻 RLoad 的变化范围是多少?

(b) 构建开环输出阻抗 Z out ( s ) 的 Bode 图。在哪段频率范围内,开环的输出阻

抗不满足指标要求?为了使闭环输出阻抗满足指标要求,推导环路增益的最小值
应为多少?选择合适的截止频率。
(c) 设计补偿器网络 Gc ( s ) 以使所有指标均满足。此外环路增益 T ( s) 的直流增

益要不小于 20dB 。推导以下量:


i. 补偿器 Gc ( s ) 的传递函数。

ii. 最坏情况下的闭环 Q 值。

iii. 画出 5A 和 50A 这两种情况下的环路增益和闭环输出阻抗的 Bode 图。


RLoad 的变化对闭环特性有什么影响?

(d) 使用电阻、电容、放大器来实现你的 Gc ( s ) 。

L
1mH iLoad
C Z out
Vg v RLoad
100V 200μF

f s  100kHz
H ( s )  0.1 H ( s )

vc ve Hv
PWM Gc ( s )
Vm  4V
vref  5V

图 9-26 Buck 调整器


Solution:
(a) 稳态输出电压 V 的值为:
Vref
Vref  HV  V   50V (9-64)
H

有效负载电阻 RLoad 的范围为:

- 319 -
1Ω  RLoad  10Ω (9-65)

(b) Buck 变换器的交流小信号模型如图 9-27 所示:


iˆg (t ) iˆ(t ) L Vg dˆ (t )

vˆg (t ) Idˆ (t) C R vˆ(t )

1: D
图 9-27 Buck 变换器交流小信号模型

对于开环输出阻抗的推导,令 vˆg  0 、 dˆ  0 ,则图 9-27 可简化至如图 9-28

所示:
iˆ(t ) L

Z out
C R vˆ(t )

图 9-28 Buck 变换器输出阻抗的推导


故输出阻抗为:
1 1 1
  sC 
Z out R sL
(9-66)
LRs
Z out 
CLRs  Ls  R
2

式(9-66)的输出阻抗可写成标准形式,即:
Ls
Z out  2
s  s 
1  
Q0  0 
C
QR (9-67)
L
1
0 
CL

从式(9-67)可发现,0 固定不变, Q 的值会随着有效负载电阻的阻值变化而

变化,但 Q 的变化仅会影响输出阻抗 Bode 图峰值处的超调情况。

首先画出两种极端情况下的输出阻抗 Bode 图,即有效负载电阻的阻值最大


时和有效负载电阻的阻值最小时,如图 9-29 所示:

- 320 -
Bode Diagram
30
Zout_ol_R_min
20
Zout_ol_R_max
10

Magnitude (dB)
System: Zout_ol_R_max System: Zout_ol_R_max
0 Frequency (rad/s): 199 Frequency (rad/s): 2.52e+04
Magnitude (dB): -14 Magnitude (dB): -14
-10
-20
-30
-40
-50
90
Zout_ol_R_min
Zout_ol_R_max
45
Phase (deg)

-45

-90
1 2 3 4 5
10 10 10 10 10
Frequency (rad/s)

图 9-29 两种极端情况下的输出阻抗 Bode 图


对于开环的输出阻抗,如果也要求其在 0 20kHz 的频率范围内,输出阻抗
的值小于 0.2Ω ,那么就要要求开环输出阻抗的幅值 Bode 图要小于 0.2Ω 对应的
dB 值,即:
Tol _ zout  20log 0.2  14dB (9-68)

注:ol 代表 open-loop,cl 代表 close-loop。

从图 9-29 可以看出,199rad/s    2.52 104 rad/s 时,开环的输出阻抗不满


足小于 0.2Ω 的指标要求。
闭环输出阻抗的表达式为:
Z ol _ zout
Z cl _ zout 
1 T
(9-69)
GG H
T  c vd
Vm

从图 9-29 可以看出,开环输出阻抗在有效负载电阻阻值最大时,即负载电
流为最小值 5A 时取得最大值,开环输出阻抗的最大值为:
Zol _ zout _ max  20dB (9-70)

如果通过闭环使得开环输出阻抗的最大值对应的闭环输出阻抗值小于 0.2 ,
那么对于 0 20kHz 的频率范围内,无论有效负载电阻的阻值怎么变化,闭环的
输出阻抗值均会小于 0.2 。
即有:
Z ol _ zout _ max Z ol _ zout _ max
Z cl _ zout _ max   0.2 
1 T T (9-71)
Z ol _ zout _ max_dB  T_ dB  14dB  T_ dB  34dB

故环路增益在开环输出阻抗的谐振峰处的最小值为 34dB 。

环路增益的截止频率应稍大于 2.52 104 rad/s ,因为小于 2.52 104 rad/s 的频

- 321 -
率段内需要通过补偿器进行增益提升。而大于 2.52 104 rad/s 的频率段,其开环

输出阻抗的幅值增益已经满足小于 0.2 的指标要求,故大于 2.52 104 rad/s 的频


率段不需要额外补偿。
而对于超过截止频率以外的闭环输出阻抗,由于环路增益 T 1 ,故有:
Zol _ zout
Zcl _ zout   Zol _ zout  for f fc (9-72)
1 T

综上所述, 截止频率 要选得稍大 于 2.52 104 rad/s ,故将截止频率选在

4  104 rad/s左右。
(c) 选择完了截止频率后,会发现如不补偿,则相位裕度接近为 0 。为了使环
路稳定,且使闭环极点处的 Q 值小于 1,就要将相位裕度至少提高到 52 ,故需

加入 PD 补偿,选择的 Gc ( s ) 传递函数为:

 s 
1  
z 
Gc  Gc 0 
 s 
1  
 p  (9-73)
Gc 0  28.84
z  8.9443 103 rad / s
 p  1.7889 105 rad / s
使用此 PD 补偿后两种极端情况下的环路增益、闭环输出阻抗和未补偿时的
环路增益如图 9-30 所示:
Bode Diagram

50 T_uncompensated_max
System: T_compensated_max_pd
Frequency (rad/s): 2.2e+03 T_uncompensated_min
Magnitude (dB): 50.5 System: Zout_cl_R_min
T_compensated_max_pd
Frequency (rad/s): 2.22e+04
0
Magnitude (dB)

Magnitude (dB): -18.6 T_compensated_min_pd


Zout_cl_R_max
Zout_cl_R_min
-50

-100

90

45

0
Phase (deg)

-45

-90

-135 System: T_compensated_max_pd


Phase Margin (deg): 65.5
Delay Margin (sec): 2.83e-05
-180
1 2 3 4 At frequency 5(rad/s): 4.04e+04 6 7
10 10 10 10 10stable? Yes
Closed loop 10 10
Frequency (rad/s)

图 9-30 补偿后的 Bode 图


由图 9-30 可知,两种极端情况下相位裕度的最小值为 65.5 。闭环输出阻抗

- 322 -
在 0 20kHz 的频率段内其增益最大值为 18.6dB 。最恶劣情况下的环路增益在
谐振峰处的增益为 50.5dB ,远大于 34dB 。而且不论有效负载电阻的阻值如何变
化,其对闭环输出阻抗的影响均可以忽略不计,可以发现两种极端情况下的闭环
输出阻抗 Bode 图几乎一样。
(d) 将式(9-73)表达下式的形式:
1  Bs
Gc  A  
1   Bs
A  28.84  20  576.8
1 (9-74)
  0.05
20
1
B  1.118 104
z
1  Bs
其中  的电路形式如图 9-31 所示:
1   Bs
R1

u C R2 y

图 9-31 PD 补偿的电路实现形式
对于图 9-31 其中的参数有:
B  R1C
R2 (9-75)

R1  R2

对于 Gc 的实现还缺一个放大电路, 如图 9-32 所示:

u
y
R3 R4

图 9-32 放大电路的实现形式
故有:
R3  R4
A (9-76)
R4

故 Gc ( s ) 的电路实现形式如图 9-33 所示:

- 323 -
R1

u
R2 y
R3 R4 C

图 9-33 Gc ( s ) 的电路实现形式

Problem 9.9

对如图 9-34 所示的 Buck-Boost 电压调整器进行设计。变换器运行于 CCM


模式,元器件的值已在图 9-34 中所示。额定输入电压 Vg  48V ,需要将输出电

压整定到 15V ,设计一个你认为最好的补偿器,具有较高的截止频率(但不能


超过开关频率的 10% ),并且具有在反馈环的带宽内具有较大的环路增益,且要
保持 52 以上的相位裕度。
(a) 推导出所需要的 H 值,画出未补偿的环路增益 Bode 图和你所设计的补偿
器 Gc ( s ) 的 Bode 图。

(b) 构建经过补偿后的环路增益 Bode 图和 T / 1  T  及 1/ 1  T  的 Bode 图。

(c) 对你的设计进行讨论,什么阻止了你进一步增加截止频率?环路增益在
120Hz 处的幅值增益是多少?是否可以进一步提高?

L C R
vg v
50μH 220μF 5

f s  200kHz
H (s)

vc ve Hv
PWM Gc ( s )
Vm  3V
vref  5V

图 9-34 Buck-Boost 变换器


Solution:
(a) 所需要的 H 为:
vref 5 1
Hv  vref  H    (9-77)
v 15 3

- 324 -
Buck-Boost 变换器的小信号模型已在 Problem 8.24 推出,这里重新给出如图
9-35 所示:

iˆL L Vg  V  dˆ (t )

vˆg (t ) I L dˆ (t ) I L dˆ (t ) C R v

1: D D :1
图 9-35 Buck-Boost 变换器小信号交流等效模型

从控制到输出的传递函数 Gvd ( s) 已由式(8-136)推出,这里重新给出:

  LV 
 D R V  Vg   D s 
Gvd   (9-78)
CLRs 2  Ls  RD2
故其未补偿的环路增益为:
1
Tuncompensated  Gvd H (9-79)
Vm

将器件值代入到式(9-79),可得:
0.0003281s  80
Tuncompensated   (9-80)
1.65 107 s 2  0.00015s  8.707
未补偿的环路增益 Bode 图如图 9-36 所示:
Bode Diagram
40
T_uncompensated
20

0
Magnitude (dB)

-20

-40

-60

-80
180
T_uncompensated

135
Phase (deg)

90

45

0
3 4 5 6 7
10 10 10 10 10
Frequency (rad/s)

图 9-36 未补偿的环路增益
注意图 9-36 中相位坐标,首先环路的截止频率不能超过开关频率的 10% ,
即:
c  2  20 103  1.256 105 rad/s (9-81)

从图 9-36 可以看出,如果不补偿,环路增益的相位(非相位裕度)为 10 40


这个区间内,为了使环路稳定,且相位裕度要大于 52 ,则要将未补偿的环路增
益的相位曲线首先减去 180 ,然后再提升几十°即可。

将环路的截止频率设定在 1105 rad/s ,即 16kHz 左右。此时未补偿的环路增

- 325 -
益其相位为 23 。为了使相位裕度要大于 52 ,先使相位曲线减 180 之后,还需
再提高 29 以上。
根据书中的 Fig9.16,使用 PD 补偿,如想提高相位 40 ,则根据 Fig9.16 的
曲线,可知:
fp
 5.5
fz
(9-82)
1105
f crossover  f p fz 
2
根据式(9-82)可得:
 p  2.345 105 rad/s
(9-83)
z  4.264 104 rad/s
故所使用的 PD 补偿,其表达式为:
 s 
1  
z 
Gc _ pd  Gc 0  (9-84)
 s 
1  
 p 

但是 Gc 0 还未知,首先令 Gc 0  1 ,注意式(9-84)中 Gc 0 前面的负号可以使相位

曲线减 180°。画出此时的环路增益如图 9-37 所示:


Bode Diagram
40
T_compensated_pd
20
Magnitude (dB)

0 System: T_compensated_pd
Frequency (rad/s): 1.01e+05
Magnitude (dB): -18.3

-20

-40

-60
45
T_compensated_pd
0
Phase (deg)

-45
System: T_compensated_pd
Frequency (rad/s): 1.01e+05
-90 Phase (deg): -113

-135

-180
3 4 5 6 7
10 10 10 10 10
Frequency (rad/s)

图 9-37 Gc 0  1 时的 PD 补偿后的环路增益

为了使环路增益的截止频率为所设计的 1105 rad/s ,根据图 9-37 可知还需

将环路增益的幅值 Bode 图向上平移 18.3dB 。故可知 Gc 0 应为:

20log Gc 0  18.3dB  Gc 0  8.222 (9-85)

为了进一步提高低频增益,可以再加入 PI 补偿器,根据书中 Fig9.18 可知

- 326 -
PI 补偿器具有相位滞后的作用,为了不影响 PD 补偿器所提升的相位裕度。令
10 f L  f z
10L  z (9-86)
L  4264rad/s
则所加入的 PI 补偿器其表达式为:
s  L
Gc _ pi  (9-87)
s
综上所述,所设计的补偿器为:

   4.264 10 
3
s
 1  4 
1  
 4.264 10   s 
Gc _ pid  8.222 (9-88)
 s 
1  5 
 2.345 10 
补偿器的 Bode 图如图 9-38 所示:
Bode Diagram
55
50
45
Magnitude (dB)

40
35
30
25
20
15
180
135
90
Phase (deg)

45
0
-45
-90
-135
-180
2 3 4 5 6 7
10 10 10 10 10 10
Frequency (rad/s)

图 9-38 补偿器的 Bode 图

(b) 已 补 偿 过 的 环 路 增 益 T ( s) , 和 传 递 函 数 T ( s ) / 1  T ( s)  及 传 递 函 数

1 / 1 T s( )的 Bode 图如图 9-39 所示:

- 327 -
Bode Diagram
80
T
60
T / (1 + T)
40 1 / (1 + T)

Magnitude (dB)
20
0
-20
-40
-60
-80
180
135
90
Phase (deg)

45
System: T
0 Phase Margin (deg): 64.5
Delay Margin (sec): 1.11e-05
-45 At frequency (rad/s): 1.01e+05
-90 Closed loop stable? Yes

-135
-180
2 3 4 5 6 7
10 10 10 10 10 10
Frequency (rad/s)

图 9-39 已补偿过的环路增益 Bode 图


(c) 环路截止频率的提高受限于变换器的开关频率。由于环路的补偿依赖于所
建立的变换器的交流小信号模型,而此交流小信号模型使用了周期平均法,而周
期平均法对开关频率 1/10 以下的频率范围十分准确,当频率大于开关频率 1/10
以上时,周期平均法得到的交流小信号模型变得越来越不准确。
120Hz 处,即 2 120  754rad/s处的环路增益的幅值为 52.8dB 。在此频率

处,可以通过增大  L 的值来获得更高的环路增益值,但是增大  L 会使 PI 补偿

器的相位滞后频率段更靠近 PD 补偿器的相位超前频率段,也即增大  L 虽然会

提高环路增益值,但会抵消掉 PD 补偿所带来的相位裕度的提升,也即会降低相
位裕度。目前 120Hz 处的 52.8dB 几乎已经是在不影响相位裕度的情况下使用 PI
补偿能够能到的最大增益值了。

Problem 9.10

使用电压注射法对某反馈系统的环路增益进行测量。测量的数据如书中
Fig9.47 所示。推导出所需要的环路增益,并将其写成零极点形式。在哪段频率
范围内,所测量的数据是有效的?
Solution:
环路增益 T ( s) 为:

 Z2 
T  G2 HG1  
 Z1  Z 2 
Z1  R1
1 1
  sC (9-89)
Z 2 R2
R1  1103
R2  10 103
C  2 109

- 328 -
测量得到的增益 Tm ( s) 为:

vy
Tm  (9-90)
vx

由于:
ˆ 1
vˆ y  G1vˆe  iZ
vˆe  vˆxG2 H (9-91)

iˆ  x
Z2
根据式(9-91)可得:
vˆ y Z
 G1G2 H  1
vˆx Z2
(9-92)
vˆ  Z  Z
Tm  y  T 1  1   1
vˆx  Z2  Z2

Z1 、 Z 2 、 Z1 / Z 2 的 Bode 图如图 9-40 所示:

Bode Diagram
80
Z1
60 Z2
Z12
Magnitude (dB)

40

20 System: Z12
Frequency (rad/s): 5.04e+05
Magnitude (dB): 0.109
0

-20
90

45
Phase (deg)

-45

-90
3 4 5 6
10 10 10 10
Frequency (rad/s)

图 9-40 阻抗 Bode 图
从图 9-40 可知:
Z1  Z2  for  5 105 rad/s (9-93)

对于式(9-92), Tm  T 的前提条件为:

Z2 Z1
Z1 (9-94)
T
Z2

故有效测量增益如图 9-41 所示:

- 329 -
有效测
量增益

Z1
Z2

  5 105 rad / s
f  7.95 104 Hz

图 9-41 有效测量增益

对于环路增益的表达式,首先从图中找出转折频率和 Q 值,如图 9-42 所示:

图 9-42 获取环路增益表达式
根据图 9-42 可知:
9
Q  10 20  2.82
40
T0  10 20  100 (9-95)
f1  800Hz
f 2  3.5 103 Hz

- 330 -
再结合 Bode 图中的幅值渐近线和相位变化,可知环路增益的形式为:
s
1
2
T ( s)  T0 2
(9-96)
s  s 
1  
Q1  1 
将式(9-95)代入到式(9-96)可得:
0.004547 s  100
T (9-97)
3.958 108 s 2  7.055 105 s  1
推导出的环路增益 Bode 图如图 9-43 所示:
Bode Diagram
60

40
Magnitude (dB)

20

-20

-40
0

-45
Phase (deg)

-90

-135

-180
2 3 4 5 6
10 10 10 10 10
Frequency (Hz)

图 9-43 推导出的环路增益 Bode 图

- 331 -
第10章 输入滤波器设计

Problem 10.1

需要对如图 10-1 所示的反激变换器设计一个输入滤波器。输入电流 iin (t ) 的

开关频率上的谐波其幅值不能超过 10μA 的有效值。计算滤波器在开关频率处所

需要的衰减。
n  0.5
iin (t ) ig (t ) 1: n
D1
Lp C R
Vg v
输入 250μH 100μF 5
48V 滤波

Q1

D  0.3
f s  200kHz
图 10-1 反激变换器电路
Solution:
反激变换器的等效电路如图 10-2 所示:
n  0.5
iin (t ) ig (t ) 1: n
i ic
D1
Lp C R
Vg v v
输入 250μH L 100μF 5
48V 滤波 V1

Q1

D  0.3
f s  200kHz
图 10-2 反激变换器等效电路
当开关管导通时,有:

- 332 -
vL  V1
V
ic   (10-1)
R
ig  I

当开关管关断时,有:
V
vL  
n
I V
ic   (10-2)
n R
ig  0

故有:
 V
D V1   D     0
 n
 V I V
D     D     0 (10-3)
 R n R
D  I   D  0  I g
根据式(10-3)可推出:
nDV1 nDVg
V 
1 D 1 D
nV
I (10-4)
1  D  R
I g  DI

将元器件的值代入到式(10-4)可得:
V  10.286V
I  1.469A (10-5)
I g  0.441A

故 ig 的波形如图 10-3 所示:

ig (t )

1.469A I

0.441A Ig
0 t

图 10-3 ig (t ) 的波形

- 333 -
故图 10-3 中的电流波形经过傅里叶分解,在开关频率处的谐波电流,其幅
值为:
2I 2 1.469
I g1_ fs  sin  D   sin  0.3   0.756A (10-6)
 
故开关频率处的谐波电流,其有效值为:
0.756
I g1_ fs _ rms   0.535A (10-7)
2

其中,下标 g 后面的 1 代表的是基波,即 1 次谐波, f s 代表的是基波的频率

为开关频率。
为了将开关频率处的谐波电流衰减至 10μA 的有效值,故输入滤波器在开关

频率处的增益要小于:

10 106
20log  94.56dB (10-8)
0.535

Problem 10.2

在如图 10-4 所示的 Boost 变换器中。需要设计输入滤波器,使得 iin (t ) 的开

关频率处及更高次的谐波分量均不超过 10μA 的有效值。求解输入滤波器在开关

频率处及更高开关频率范围内所需要的衰减值。
iin (t ) ig (t ) L
100μH
Vg 输入 C R
v
48V 滤波 Q1 33μF 12

D  0.6
f s  200kHz
图 10-4 Boost 变换器
Solution:
稳态时的输出电压 V 为:
Vg 48
V   120V (10-9)
1 D 1  0.6
稳态时的输出功率 P 为:

V 2 1202
P   1200W (10-10)
R 12

- 334 -
故输入电流 I g 为:

P
P  Vg I g  I g   25A (10-11)
Vg

输入电流 ig 的纹波大小为:

2 i
Vg  L  i  0.72A (10-12)
DTs

输入电流 ig 的波形如图 10-5 所示:

ig (t )
i  0.72A
I g  25A

0 Ts t

图 10-5 输入电流 ig 的波形

输入电流 ig 的表达式为:

 480000t  24.28 for0  t  0.6Ts


f (t )    (10-13)
27.88  720000t for0.6T  t  Ts 

周期为 2l 的输入电流 ig 的傅里叶分解形式为:

a0   k t k t 
f (t )     ak cos  bk sin 
2 k 1  l l 
Ts
l
2 (10-14)
1 2l k t
ak   f (t ) cos dt
l 0 l
1 2l k t
bk   f (t )sin dt
l 0 l
故可求得:
a0  50
a1  0.55 (10-15)
b1  0.179

- 335 -
故输入电流 ig 的开关频率处的基波分量,其幅值和有效值为:

A1  a12  b12  0.578A


A1 (10-16)
A1_ rms   0.409A
2
由于随着谐波次数的增加,谐波的幅值逐渐递减,故幅值最大的谐波肯定是
基波。所以这里只需求出基波的幅值即可。
为了将基波电流的有效值衰减至 10μA ,故输入滤波器在开关频率处的增益

要小于:

10 106
20log  92.23dB (10-17)
0.409

Problem 10.3

推导书中表格 10.1 中 Z N 和 Z D 的表达式。

Solution:
(a) 对于 Buck 变换器,其交流小信号模型如图 10-6 所示:

Vg dˆ (t )
iˆg (t ) iˆ(t ) L

vˆg (t ) Idˆ (t) C R vˆ(t )

1: D
图 10-6 Buck 交流小信号模型

对于 Buck 变换器的 Z N ,有:

Z N  Z i ( s ) v ( s ) 0 (10-18)

故图 10-6 可简化为如图 10-7 所示:


iˆ(t )  0

ZN
Idˆ (t) Vg dˆ (t )

1: D

图 10-7 Buck 变换器 Z N 推导

- 336 -
故有:

Vg dˆ

ZN  D  R (10-19)
Idˆ D2

对于 Buck 的 Z D ,有:

Z D  Z i ( s ) d 0 (10-20)

故图 10-6 可简化为如图 10-8 所示:


iˆ(t ) L

ZD
C R vˆ(t )

1: D

图 10-8 Buck 的 Z D 的推导

故有:
1 1
  sC
Z1 R
Z 2  sL  Z1 (10-21)
1
ZD  Z2
D2
根据式(10-21)可求出:
L
1 s  CLs 2
R R
ZD  2 (10-22)
D 1  RCs 
(b) 对于 Boost 变换器,其交流小信号模型如图 10-9 所示:

iˆL (t ) L Vdˆ (t )

vˆL (t )
vˆg (t ) C I L dˆ (t ) R vˆ(t )

D :1
图 10-9 Boost 变换器交流小信号模型

对于 Z N 的推导,图 10-9 可以简化为如图 10-10 所示:

- 337 -
I L dˆ
Vdˆ (t )
D L
vˆL (t )
ZN
vˆ  0 C I L dˆ (t )

D :1

图 10-10 Boost 变换器 Z N 推导

故有:
V 1

Vg D
V2
Vg I L  (10-23)
R
Vd
Z N  sL 
ILd
D
由式(10-23)可推出:
Z N  sL  D2 R (10-24)

对于 Z D 的推导,图 10-9 可简化为如图 10-11 所示:

ZD
C R vˆ(t )

D :1

图 10-11 Boost 变换器 Z D 的推导

故有:
1 1
 sC 
Z1 R (10-25)
Z D  sL  D Z1 2

根据式(10-25)可求得:
L CL
s  2 s2 1
Z D  RD2 RD 2
D (10-26)
1  RCs
(c) Buck-Boost 变换器的交流小信号模型如图 10-12 所示:

- 338 -
L
V g  V  dˆ ( s )

vˆg (t ) Idˆ ( s ) Idˆ ( s ) C R

1: D D :1
图 10-12 Buck-Boost 变换器交流小信号模型

对于 Z N 推导,图 10-12 可以简化为如图 10-13 所示:

Idˆ ( s )
iˆ2 D L
V g  V  dˆ ( s )

ZN
v̂2 Idˆ ( s ) v̂1 vˆ  0 Idˆ ( s )

1: D D :1

图 10-13 Buck-Boost 变换器 Z N 推导

故有:
Id
i2  Id  D
D
1  Id 
v2   sL  Vg  V  d 
D  D 
V
I (10-27)
R  D  1
D
V Vg
D 1
v
ZN  2
i2
根据式(10-27)可求得:

D2 R DLs
ZN   (1  2 ) (10-28)
D 2
D R

对于 Z D 的推导,图 10-12 可简化为如图 10-14 所示:

- 339 -
L

ZD
C R

1: D D :1

图 10-14 Buck-Boost 变换器 Z D 推导

故有:
1 1
 sC 
Z1 R
Z 2  sL  D2 Z1 (10-29)
1
ZD  Z2
D2
由式(10-29)可推出:
L CL
1 s  2 s2
RD2 RD 2
D
ZD  (10-30)
D2 1  CRs 

Problem 10.4

如图 10-15 所示的反激变换器使用了一个单级 L f  C f 形式的输入滤波器。

通过一个十分大的阻断电容 Cb 与一个电阻 R f 串联来对滤波器进行衰减。

(a) 画出反激变换器的小信号模型,推导 Z N 和 Z D 的表达式,并画出它们的

Bode 图。
(b) 对滤波器进行设计,选择 L f 、C f 和 R f 的值,以保证滤波器在开关频率处

的增益至少 100dB ,此外还要保证滤波器的输出阻抗 Z o ( s ) 在所有的频率上满足

以下条件:
Z o ( j )  0.3 Z D ( j ) , Z o ( j )  0.3 Z N ( j )

注:此章的 Spice 暂时先不做,因为目前还不会用 Spice,待以后学会用 Spice


的时候再补上仿真。

- 340 -
n  0.5
iin (t ) ig (t ) 1: n
D1
Lp C R
Vg v
输入 250μH 100μF 5
48V 滤波

Q1

D  0.3
f s  200kHz
图 10-15 反激变换器
Solution:
(a) 反激变换器交流小信号模型已在 Problem 7.10 中推出,这里只需将 Problem
7.10 中得到的交流小信号模型理想化,即忽略绕组电阻、二极管正向压降及
MOSFET 导通电阻,如图 10-16 所示:

 Vˆ
 Vg   d (t )
iˆg (t ) iˆ(t ) LP  n

Idˆ (t )
vˆg (t ) Idˆ (t ) C R vˆ(t )
n

1: D D
:1
n
图 10-16 反激变换器交流小信号模型

对于 Z N 的推导,图 10-16 可简化为如图 10-17 所示:

Idˆ (t ) n  Vˆ
 Vg   d (t )
i2 n D LP  n

ZN Idˆ (t )
v2 Idˆ (t ) v1 vˆ  0
n

1: D D
:1
n

图 10-17 反激变换器 Z N 的推导

故有:

- 341 -
Idˆ (t ) n
i2  Idˆ  D
n D
v
v2  1
D
(10-31)
Idˆ (t ) n  V
v1  sLP   Vg   dˆ (t )
n D  n
v
ZN  2
i2
对于稳态量有:
V D
n
Vg D
(10-32)
nV
I
DR
根据式(10-31)和式(10-32),可得:

DLP n2 s  RD2
ZN  (10-33)
D2 n2

对于 Z D 的推导,图 10-16 可简化为如图 10-18 所示:

iˆ(t ) LP

ZD
C R vˆ(t )

1: D D
:1
n

图 10-18 反激变换器 Z D 的推导

故有:
1 1
 sC 
Z1 R
 D 
2

Z 2  sLP    Z1 (10-34)
 n 
Z
Z D  22
D
根据式(10-34)可推出:

LP n 2 CLP n 2 2
1  s  s
RD2 RD  2
D  2
ZD  2 2 (10-35)
Dn 1  CRs 

- 342 -
将参数值代入到式(10-33)和式(10-35),可得:

1.875 105 s  2.45


ZN 
0.0225
(10-36)
3.125 108 s 2  6.25 105 s  2.45
ZD 
1.125 105 s  0.0225

Z N 和 Z D 的 Bode 图如图 10-19 所示:

Bode Diagram
100
ZN
80 ZD
Magnitude (dB)

60

40

20

0
180
ZN
ZD
90
Phase (deg)

-90
2 3 4 5 6 7
10 10 10 10 10 10
Frequency (rad/s)

图 10-19 Z N 和 Z D 的 Bode 图

(b) 选择的输入滤波器的基本形式是 L f  C f 滤波器。如图 10-20 所示:

iin (t ) ig (t )

Vg Vo Z o ( s) Zi ( s)

图 10-20 输入滤波器的基本形式

如果希望 L f  C f 滤波器的滤波效果符合以下传递函数:

Vo 1
 (10-37)
Vg 1  LCs 2

从图 10-20 可知,式(10-37)成立的条件为 Zi   大,即 Z i 相当开路,如果

将图 10-20 表达成戴维宁等效电路的话,变可得知当式(10-37)成立时 Z i 与 Z o 的

关系应满足什么条件。图 10-20 的戴维宁等效电路如图 10-21 所示:

- 343 -
Z o ( s)

Vo Zi ( s)

图 10-21 图 10-20 的戴维宁等效电路

即如果希望式(10-37),那么就需要保证 Z o Zi 。
(事实上,此章的滤波器衰

减的思路也是如此)
对于图 10-20,基本滤波器的传递函数为:

iin iin iin


H  
ig ig ig
vg  0

1 1
  sC (10-38)
Z o sL
ig Z o  iin sL

根据式(10-38)可求出:
iin 1
H  (10-39)
ig 1  LCs 2

可以发现式(10-39)与式(10-37)一样。
但是图 10-20 的输出阻抗在谐振频率处是无穷大的,故不满足 Z o Zi 这个

条件,也即在谐振频率附近,输入滤波器和 Z i 相互起作用,导致了输入滤波器变

相得也成为了 Z i 的一部分,因此可能丧失滤波器的功能,甚至对 Z i 的传递函数

有影响。(事实也是如此,如果不对滤波器进行衰减,那么滤波器在谐振频率处
会对 Gvd 的传递函数有影响)

为了对滤波器的输出阻抗进行衰减,选择如图 10-22 所示的衰减滤波器形式:


iin (t ) ig (t )
Z o ( s)
Vg
Vg _ oc
48V Zi ( s)

图 10-22 输入滤波器的形式

此衰减输入滤波器的输出阻抗 Z o ( s ) 为:

- 344 -
1
Z o (s)  (10-40)
1 1
 sC f 
Lf s 1
Rf 
sCb
此衰减输入滤波器的传递函数为:

iin
H
ig
vg  0

ig Z o  iin sL (10-41)


R f Cb s  1
H
CbC f L f R f s 3   Cb L f  C f L f  s 2  Cb R f s  1

当频率很高时有:
R f Cb s  1 R f Cb s
H 
CbC f L f R f s 3   Cb L f  C f L f  s 2  Cb R f s  1 CbC f L f R f s 3
(10-42)
1

C f Lf s2

开关频率是 200kHz ,即 s  2  200 103  1.256 106 rad/s 。如果希望在开

关 频 率 处 有 至 少 100dB 的 衰 减 增 益 , 为 了 留 有 一 定 的 余 量 , 可 以 设 计 在

  1106 rad/s 处有 100dB 的衰减增益。故根据式(10-42)可有:

 
 1   100dB
20log
 C L  1106 2  (10-43)
 f f 
7
C f L f  10

对于输入滤波器的输出阻抗,式(10-43)可确定出输出阻抗的谐振频率(LC
滤波器输出阻抗的谐振频率即为滤波器传递函数增益渐近线的转折频率):
1
C f L f  107   c  3162rad/s (10-44)
c 2

为了使滤波器的输出阻抗满足题中的阻抗不等式要求,即使输出阻抗满足
Z o ( j )  0.3 Z D ( j ) , Z o ( j )  0.3 Z N ( j ) 。可以先将输出阻抗的增益渐近

线在转折频率处的增益设计为 20dB ,如果只有 L f  C f 做输入滤波器,那么在

转折频率处的输出阻抗的增益会趋于无穷大,为了对此转折频率处的输出阻抗的
增益进行衰减,需要加入 R f ,如果将转折频率处的输出阻抗的增益设计为 10dB ,

那么有:

- 345 -
20log  L f c   20dB
(10-45)
20log R f  10dB

为了使 R f 不流经直流电流,以免减少直流损耗,需要加入阻断电容 Cb 。为

了不影响 R f ,有:

1
Rf (10-46)
cCb

根据式(10-44)和式(10-45)可得:
L f  3.1623 105  31.623μH
C f  0.0032  3200μF (10-47)
R f  0.3162Ω

根据式(10-46)取 Cb  0.01  10mF 。

故输入滤波器的输出阻抗 Bode 图、 0.3Z N 和 0.3Z D 的 Bode 图和输入滤波器

的传递函数如图 10-23 所示:


Bode Diagram
100
ZN_03
50 ZD_03
Zo
Magnitude (dB)

H
0
System: Zo
Frequency (rad/s): 3.22e+03
-50 Magnitude (dB): -10.2

-100
System: H
Frequency (rad/s): 9.86e+05
-150 Magnitude (dB): -99.8
180
ZN_03
ZD_03
90
Zo
Phase (deg)

H
0

-90

-180
2 3 4 5 6 7
10 10 10 10 10 10
Frequency (rad/s)

图 10-23 阻抗 Bode 图

Problem 10.5

如图 10-24 所示的 Boost 变换器使用了一个单级 L f  C f 形式的输入滤波器。

通过一个十分大的阻断电容 Cb 与一个电阻 R f 串联来对滤波器进行衰减。

(a) 画出 Boost 变换器 Z N 和 Z D 的 Bode 图。

(b) 对滤波器进行设计,选择 L f 、C f 和 R f 的值,以保证滤波器在开关频率处

- 346 -
的衰减增益至少 80dB ,此外还要保证滤波器的输出阻抗 Z o ( s ) 在所有的频率上满

足以下条件:
Z o ( j )  0.2 Z D ( j ) , Z o ( j )  0.2 Z N ( j )

iin (t ) ig (t ) L
100μH
Vg 输入 C R
v
48V 滤波 Q1 33μF 12

D  0.6
f s  200kHz

图 10-24 带输入滤波的 Boost 变换器


Solution:
(a) Z N 和 Z D 的表达式已在 Problem 10.3 中求出,这里重新给出:

Z N  sL  D2 R
L CL
1 s  2 s2 (10-48)
Z D  RD 2 RD 2
D
1  RCs
将器件值代入到式(10-48)可得:
Z N  0.0001s  1.92
3.96 108 s 2  0.0001s  1.92 (10-49)
ZD 
0.000396 s  1
其 Bode 图如图 10-25 所示:
Bode Diagram
40
ZN
30 ZD

20
Magnitude (dB)

10

-10

-20
180
ZN
135 ZD
Phase (deg)

90

45

-45
2 3 4 5 6
10 10 10 10 10
Frequency (rad/s)

图 10-25 Z N 和 Z D 的 Bode 图

(b) 选择的滤波器形式如图 10-26 所示:

- 347 -
iin (t ) ig (t )

Vg Z o ( s)
48V

图 10-26 选择的滤波器形式
输入滤波器的输出阻抗为:
1
Z o (s)  (10-50)
1 1
 sC f 
Lf s 1
Rf 
sCb
输入滤波器的传递函数为:
R f Cb s  1
H (10-51)
CbC f L f R f s   Cb L f  C f L f  s 2  Cb R f s  1
3

当频率很高时有:
R f Cb s  1 R f Cb s
H 
CbC f L f R f s 3   Cb L f  C f L f  s 2  Cb R f s  1 CbC f L f R f s 3
(10-52)
1

C f Lf s2

开关频率是 200kHz ,即 s  2  200 103  1.256 106 rad/s 。如果希望在开

关 频 率 处 有 至 少 80dB 的 衰 减 增 益 , 为 了 留 有 一 定 的 余 量 , 可 以 设 计 在

  1106 rad/s 处有 120dB 的衰减增益。故根据式(10-52)可有:

 
 1   120dB
20log
 C L  1106 2  (10-53)
 f f 
6
C f L f  10

对于输入滤波器的输出阻抗,式(10-50)可确定出输出阻抗的谐振频率(LC
滤波器输出阻抗的谐振频率即为滤波器传递函数增益渐近线的转折频率):
1
C f L f  106   c  1000rad/s (10-54)
c 2

为了使滤波器的输出阻抗满足题中的阻抗不等式要求,即使输出阻抗满足
Z o ( j )  0.2 Z D ( j ) , Z o ( j )  0.2 Z N ( j ) 。可以先将输出阻抗的增益渐近

线在转折频率处的增益设计为 20dB ,如果只有 L f  C f 做输入滤波器,那么在

- 348 -
转折频率处的输出阻抗的增益会趋于无穷大,为了对此转折频率处的输出阻抗的
增益进行衰减,需要加入 R f ,如果将转折频率处的输出阻抗的增益设计为 20dB ,

那么有:
20log  L f c   20dB
(10-55)
20log R f  20dB

为了使 R f 不流经直流电流,以免减少直流损耗,需要加入阻断电容 Cb 。为

了不影响 R f ,有:

1
Rf (10-56)
c Cb

根据式(10-54)和式(10-55)可得:
L f  0.1mH
C f  10mF (10-57)
R f  0.1Ω

根据式(10-56)取 Cb  0.1  100mF 。

故输入滤波器的输出阻抗 Bode 图、 0.2Z N 和 0.2Z D 的 Bode 图如图 10-27 所

示:
Bode Diagram
50
ZN_02
ZD_02
0 Zo
Magnitude (dB)

H
System: Zo
Frequency (rad/s): 998
-50 Magnitude (dB): -20

-100

System: H
Frequency (rad/s): 1.02e+06
-150
Magnitude (dB): -120
180
135 ZN_02
ZD_02
90
Zo
Phase (deg)

45 H
0
-45
-90
-135
-180
1 2 3 4 5 6 7
10 10 10 10 10 10 10
Frequency (rad/s)

图 10-27 阻抗 Bode 图

Problem 10.6

使用书中 10.4.1 节中所述的输入滤波器优化设计方法重新对 Problem 10.4 进


行设计。找到 L f 、 C f 、 R f 和 Cb 的值。

- 349 -
Solution:
L f 、 C f 的值仍保持与 Problem 10.4 中选择的值一致,故有:

L f  3.1623 105  31.623μH


(10-58)
C f  0.0032  3200μF
则:
Lf
R0 f   0.1 (10-59)
Cf

输入滤波器输出阻抗的峰值也设定在 10dB ,故有:


20 log Z o mm
 10dB  Z o mm
 0.3162Ω (10-60)

故可求出系数 n 为:

R02 f  
2
Z o mm   0.7404
n 1 1 4 2 (10-61)
Z o mm  
2
R0 f
 

故阻断电容 Cb 的值为:

Cb  nC f  0.0023  2.3mF (10-62)

根据 n 的值可求出衰减电阻 R f 的值:

R f  R0 f
 2  n  4  3n   0.1811Ω
(10-63)
2n 2  4  n 
Problem 10.4 节中设计出的输入滤波器和使用优化方法设计出的滤波器的阻
抗如图 10-28 所示。其中红色线条的 Z o 为 Problem 10.4 节中设计的输入滤波器

输出阻抗的 Bode 图,浅蓝色线条的 Z o1 为使用优化方法进行设计的输入滤波器输

出阻抗 Bode 图。其中紫色线条的 H 为 Problem 10.4 节中设计的输入滤波器的传


递函数,金黄色线条 H1 为使用优化方法进行设计的输入滤波器的传递函数。

- 350 -
Bode Diagram
100
ZN_03
50 ZD_03
Zo

Magnitude (dB)
Zo1
0
H
System: Zo1 H1
Frequency (rad/s): 2.79e+03
-50 Magnitude (dB): -10.2

-100
System: H1
Frequency (rad/s): 9.86e+05
-150 Magnitude (dB): -99.8
180
ZN_03
ZD_03
90
Zo
Phase (deg)

Zo1
0 H
H1
-90

-180
2 3 4 5 6 7
10 10 10 10 10 10
Frequency (rad/s)

图 10-28 优化方法和原始方法得到的输出阻抗对比 Bode 图


从图 10-28 优化方法得到的输入滤波器输出阻抗 Bode 图和原始设计得到的
Bode 图几乎一模一样,但优化方法得到的 R f 和 Cb 的却相比原来小很多。

Problem 10.7

使用书中 10.4.1 节中所述的输入滤波器优化设计方法重新对 Problem 10.5 进


行设计。找到 L f 、 C f 、 R f 和 Cb 的值。

Solution:
L f 、 C f 的值仍保持与 Problem 10.5 中选择的值一致,故有:

L f  0.1mH
(10-64)
C f  10mF

则:
Lf
R0 f   0.1 (10-65)
Cf

输入滤波器输出阻抗的峰值也设定在 20dB ,故有:


20 log Z o mm
 20dB  Z o mm
 0.1 (10-66)

故可求出系数 n 为:

R02 f  
2
Z o mm   3.2361
n 1 1 4 2 (10-67)
Z o mm  
2
R0 f
 

故阻断电容 Cb 的值为:

Cb  nC f  0.0324  32.4mF (10-68)

- 351 -
根据 n 的值可求出衰减电阻 R f 的值:

R f  R0 f
 2  n  4  3n   0.0688 (10-69)
2n 2  4  n 
Problem 10.5 节中设计出的输入滤波器和使用优化方法设计出的滤波器的阻
抗如图 10-29 所示。其中紫色线条的 Z o 为 Problem 10.5 节中设计的输入滤波器

输出阻抗的 Bode 图,金黄色线条的 Z o1 为使用优化方法进行设计的输入滤波器输

出阻抗 Bode 图。红色线条的 H 为 Problem 10.5 节中设计的输入滤波器的传递函


数,浅蓝色线条 H1 为使用优化方法进行设计的输入滤波器的传递函数。

Bode Diagram
50
ZN_02
ZD_02
0 H
Magnitude (dB)

H1
System: Zo1
Frequency (rad/s): 698
Zo
-50 Magnitude (dB): -20.3 Zo1

-100

System: H1
Frequency (rad/s): 1.02e+06
-150
Magnitude (dB): -120
180
135 ZN_02
ZD_02
90
H
Phase (deg)

45 H1
0 Zo
-45 Zo1
-90
-135
-180
1 2 3 4 5 6 7
10 10 10 10 10 10 10
Frequency (rad/s)

图 10-29 优化方法和原始方法得到的输出阻抗对比 Bode 图


从图 10-29 优化方法得到的输入滤波器输出阻抗 Bode 图和原始设计得到的
Bode 图几乎一模一样,但优化方法得到的 R f 和 Cb 的却相比原来小很多。

Problem 10.8

使用书中 10.4.2 节中所述的输入滤波器优化设计方法重新对 Problem 10.4 进


行设计。找到 L f 、 C f 、 R f 和 Cb 的值。

Solution:
选择的输入滤波器形式如图 10-30 所示:

- 352 -
Rf Lb
iin (t ) ig (t )
Lf
Vg Cf Vo Z o ( s) Zi ( s)

图 10-30 输入滤波器
其输出阻抗和传递函数为:
1 1 1
 sC f  
Zo sL f R f  sLb
1 1 1
 
Z1 sL f R f  sLb
ig Z o  iin Z1 (10-70)
Lb L f s 2  L f R f s
Zo 
C f Lb L f s 3  C f L f R f s 2   L f  Lb  s  R f

H
L f  Lb  s  R f
C f Lb L f s  C f L f R f s 2   L f  Lb  s  R f
3

当  较大时,对于传递函数有:

H
L f  Lb  s  R f
C f Lb L f s 3  C f L f R f s 2   L f  Lb  s  R f


L f  Lb  s

1 (10-71)
C f Lb L f s 3 Lb L f
Cf s2
L f  Lb 

Lb L f Lb L f
由于   L f ,故当  较大时,对于同样的频率,此滤波器的
L f  Lb  Lb

衰减增益为:
 
 
H dB _ Lb  20 log  1  (10-72)
 Lb L f 
 Cf 2 

  L f  Lb  
对于 Problem 10.4 中的滤波器,其衰减增益为:
 1 
H dB _ Cb  20log 
 C f L f  
2
(10-73)
 

显然, H dB _ Lb  H dB _ Cb ,即当加入 R f  Lb 时, L f  C f 的滤波器的衰减倍数会

- 353 -
减小,即滤波性能会变差。变差的 dB 为:
 
 
 1   20 log  1 

 Lf 
20 log
 Lb L f   
 C L 2 
20 log 1   (10-74)
 Cf  
2  f f   Lb 

  L f  Lb  
根据书中 Fig10.23 中的图,可知当 Lb / L f  0.5 时,滤波器谐振频率处的衰

减与滤波器传递函数的滤波性能下降是一个比较好的折中点。
开关频率是 200kHz ,即 s  2  200 103  1.256 106 rad/s 。如果希望在开

关 频 率 处 有 至 少 100dB 的 衰 减 增 益 , 为 了 留 有 一 定 的 余 量 , 可 以 设 计 在
  1 106 rad/处有
s 100dB 的衰减增益。由于选定 Lb / L f  0.5 ,故根据式(10-72)

有:
 
 
H dB _ Lb  20 log  1   100dB
 
1106  
Lb L f 2
 Cf
 L f  Lb 
(10-75)
 
 
Lb
 0.5
Lf

对于 R f  Lb 优化设计法,当设定好滤波器最大的输出阻抗值 Z o mm
时,下式

中的 n 是两个电感的比值,不是匝数比。有:
L
n  b  0.5
Lf
Zo mm
 R0 f 2n 1  2n 
Lf (10-76)
R0 f 
Cf
Rf n  3  4n 1  2n 

R0 f 2 1  4n 

这里设定 Z o mm
 0.3162  10dB  。故根据式(10-75)和式(10-76)可得:

L f  122.47μH
C f  2.4mF
(10-77)
R f  0.2041
Lb  61.237μH

使用 R f  Lb 优化法设计出的输入滤波器的输出阻抗及传递函数如图 10-31

- 354 -
所示。其中红色线条 Z o 为 Problem 10.4 节设计的滤波器的输出阻抗,浅蓝色线

条 Z o1 为此节使用 R f  Lb 优化法 设计的滤波器的输 出 阻抗。紫色线条 H 为

Problem 10.4 节设计的滤波器的传递函数,金黄色线条 H1 为此节使用 R f  Lb 优

化法设计的滤波器的传递函数。
Bode Diagram
100
ZN_03
50 ZD_03
Zo
Magnitude (dB)

Zo1
0
H
System: Zo1 H1
Frequency (rad/s): 2.53e+03
-50 Magnitude (dB): -10

-100
System: H1
Frequency (rad/s): 1.02e+06
-150 Magnitude (dB): -100
180
ZN_03
ZD_03
90
Zo
Phase (deg)

Zo1
0 H
H1
-90

-180
2 3 4 5 6 7
10 10 10 10 10 10
Frequency (rad/s)

图 10-31 R f  Lb 优化法设计出的输入滤波器的输出阻抗及传递函数 Bode 图

Problem 10.9

使用书中 10.4.2 节中所述的输入滤波器优化设计方法重新对 Problem 10.5 进


行设计。找到 L f 、 C f 、 R f 和 Cb 的值。

Solution:
根据书中 Fig10.23 中的图,可知当 Lb / L f  0.5 时,滤波器谐振频率处的衰

减与滤波器传递函数的滤波性能下降是一个比较好的折中点。
开关频率是 200kHz ,即 s  2  200 103  1.256 106 rad/s 。如果希望在开

关 频 率 处 有 至 少 80dB 的 衰 减 增 益 , 为 了 留 有 一 定 的 余 量 , 可 以 设 计 在
  1106 rad/s 处有 120dB 的衰减增益。由于选定 Lb / L f  0.5 ,故根据式(10-72)
有:

- 355 -
 
 
H dB _ Lb  20 log  1   100dB
 
1106  
Lb L f 2
 Cf
 L f  Lb 
(10-78)
 
 
Lb
 0.5
Lf

对于 R f  Lb 优化设计法,当设定好滤波器最大的输出阻抗值 Z o mm
时,下式

中的 n 是两个电感的比值,不是匝数比。有:
L
n  b  0.5
Lf
Zo mm
 R0 f 2n 1  2n 
Lf (10-79)
R0 f 
Cf
Rf n  3  4n 1  2n 

R0 f 2 1  4n 

这里设定 Z o mm
 0.1  20dB  。故根据式(10-78)和式(10-79)可得:

L f  122.47μH
C f  2.4mF
(10-80)
R f  0.2041
Lb  61.237μH

使用 R f  Lb 优化法设计出的输入滤波器的输出阻抗及传递函数如图 10-32

所示。其中红色线条 Z o 为 Problem 10.5 节设计的滤波器的输出阻抗,浅蓝色线

条 Z o1 为此节使用 R f  Lb 优化法 设计的滤波器的输 出 阻抗。紫色线条 H 为

Problem 10.5 节设计的滤波器的传递函数,金黄色线条 H1 为此节使用 R f  Lb 优

化法设计的滤波器的传递函数。

- 356 -
Bode Diagram
50
ZN_02
ZD_02
0 Zo

Magnitude (dB)
Zo1
System: Zo1
Frequency (rad/s): 797
H
-50 Magnitude (dB): -20 H1

-100

System: H1
Frequency (rad/s): 9.97e+05
-150 Magnitude (dB): -120
180
135 ZN_02
ZD_02
90
Zo
Phase (deg)

45 Zo1
0 H
-45 H1
-90
-135
-180
1 2 3 4 5 6 7
10 10 10 10 10 10 10
Frequency (rad/s)

图 10-32 R f  Lb 优化法设计出的输入滤波器的输出阻抗及传递函数 Bode 图

Problem 10.10

对如图 10-33 所示的反激变换器设计级联的两级输入滤波器。每一级滤波器


都使用 R f  Cb 进行衰减。设计此级联的输入滤波器,要求滤波器在开关频率处

至少有 100dB 的衰减,此外滤波器的输出阻抗在所有的频率段上,要满足下式:


Z o ( j )  0.3 Z D ( j ) , Z o ( j )  0.3 Z N ( j )

n  0.5
iin (t ) ig (t ) 1: n
D1
Lp C R
Vg v
输入 250μH 100μF 5
48V 滤波

Q1

D  0.3
f s  200kHz
图 10-33 反激变换器
Solution:
开关频率是 200kHz ,即 s  2  200 10  1.256 10 rad/s 。如果希望在开
3 6

关 频 率 处 有 至 少 100dB 的 衰 减 增 益 , 为 了 留 有 一 定 的 余 量 , 可 以 设 计 在

  1 106 rad/处有
s 100dB 的衰减增益。
为了使两级滤波器之间不相互影响,使靠近变压器一端的第 1 级输入滤波器

- 357 -
在处   1106 rad/s 处有 60dB 的衰减,使靠近电源一端的第 2 级输入滤波器在

  1106 rad/s 处有 40dB 的衰减。


首先设计靠近变压器一端的第 1 级输入滤波器。其形式如图 10-34 所示:

Lf 1
iin (t ) ig (t )
Rf 1 Z o ( s)
Cf1
Cb1

图 10-34 第一级输入滤波器
则第 1 级输入滤波器的传递函数为:
R f 1Cb1s  1
H (10-81)
Cb1C f 1 L f 1 R f 1s   Cb1L f 1  C f 1L f 1  s 2  Cb1R f 1s  1
3

当  较大时,式(10-81)有:
R f 1Cb1s  1 1
H  (10-82)
Cb1C f 1L f 1R f 1s   Cb1L f 1  C f 1L f 1  s  Cb1R f 1s  1 C f 1 L f 1s
3 2 2

为了使第 1 级输入滤波器在   1106 rad/s 处有 60dB 的衰减,对于式(10-82)


有:

 
1
20 log    60dB
 C L 1106 2 
 f1 f1  (10-83)
1
C f 1L f 1  9
10
故可求出第 1 级输入滤波器的输出阻抗的谐振频率为:
1 1
c1   9
 3.16 104 rad/s (10-84)
L f 1C f 1 10

为了使第 1 级输入滤波器的输出阻抗满足阻抗不等式,根据 Problem 10.4 节


的图 10-23 中的 0.3Z N 和 0.3Z D ,可以将第 1 级输入滤波器的输出阻抗增益渐近

线在谐振频率处的增益设计为 0dB ,将第 1 级输入滤波器在谐振频率处的输出阻


抗设置在 Z o mm
 20 log 0.3162  10dB 。

故有:
20log  L f 1c1   0dB
(10-85)
20log R f 1  10dB

- 358 -
对于阻断电容 Cb1 的选择,有:

1
Cb1 (10-86)
R f 1c1

根据式(10-84)、式(10-85)和式(10-86)可得:

L f 1  3.1623 105  31.623μH


C f 1  3.1623 105  31.623μF
(10-87)
R f 1  3.162Ω
Cb1  1104  0.1mF

第 1 级输入滤波器的输出阻抗和传递函数的 Bode 图如图 10-35 所示:


Bode Diagram
100
ZN_03
ZD_03
50 Zo1
Magnitude (dB)

H1

0 System: Zo1
Frequency (rad/s): 3.15e+04
Magnitude (dB): 10

-50
System: H1
Frequency (rad/s): 1.01e+06
-100 Magnitude (dB): -60.1
180
ZN_03
ZD_03
90
Zo1
Phase (deg)

H1
0

-90

-180
2 3 4 5 6 7
10 10 10 10 10 10
Frequency (rad/s)

图 10-35 第 1 级输入滤波器的输出阻抗和传递函数的 Bode 图


级联的输入滤波器如图 10-36 所示:

Lf 2 Lf 1
iin (t ) ig (t )
Rf 2 Rf 1 Z o ( s)
Cf 2 Z a Z i1 Cf1
Cb 2 Cb1

图 10-36 级联的输入滤波器
接下来设计第 2 级输入滤波器,第 2 级输入滤波器的输出阻抗也应该满足相
应的阻抗不等式。即 Z a Z N 1 , Z a Z D1 。

对于 Z N 1 的推导,如图 10-37 所示:

故有:
Z N1  sL f 1 (10-88)

- 359 -
Lf 1

Rf 1
Z N1 Cf1
Cb1

图 10-37 Z N 1 的推导

对于 Z D1 的推导,如图 10-38 所示:

Lf 1

Rf 1
Z D1 Cf1
Cb1

图 10-38 Z D1 的推导

故有:
Cb1C f 1L f 1R f 1s 3   Cb1L f 1  C f 1L f 1  s 2  Cb1R f 1s  1
Z D1  (10-89)
Cb1C f 1R f 1s 2   Cb1  C f 1  s

Z D1 和 Z N 1 的 Bode 图如图 10-39 所示:

Bode Diagram
40
ZN1
20 ZD1
Magnitude (dB)

-20

-40

-60
90
ZN1
ZD1
45
Phase (deg)

-45

-90
2 3 4 5 6
10 10 10 10 10
Frequency (rad/s)

图 10-39 Z D1 和 Z N 1 的 Bode 图

为了使第 2 级输入滤波器在   1106 rad/s 处有 40dB 的衰减,对于式(10-90)


有:

- 360 -
 
1
20 log    40dB
 C L 1106 2 
 f2 f2  (10-90)
1
C f 2 L f 2  10
10
故可求出第 2 级输入滤波器的输出阻抗的谐振频率为:
1 1
c 2   10
 1105 rad/s (10-91)
L f 2C f 2 10

为了使第 2 级输入滤波器的输出阻抗满足阻抗不等式,即满足:
Za Z N 1 , Z a Z D1 (10-92)

将第 2 级输入滤波器输出阻抗的增益渐近线在其谐振频率 c 2 处的增益设定

在 10dB ,将第 2 级输入滤波器输出阻抗在其谐振频率 c 2 处的增益设定在 0dB 。

故有:
20log  L f 2c 2   10dB
(10-93)
20log R f 2  0dB

对于阻断电容 Cb 2 的选择,要满足:

1
Cb 2 (10-94)
R f 2c 2

根据式(10-91)、式(10-93)和式(10-94),可得:

L f 2  3.1623 106  3.1623μH


C f 2  3.1623 105  31.623μF
(10-95)
R f 2  1Ω
Cb 2  110 4  0.1mF

第 2 级输入滤波器的输出阻抗和传递函数的 Bode 图如图 10-40 所示:

- 361 -
Bode Diagram
60
ZN1
40
ZD1
20 Zo2

Magnitude (dB)
H2
0
System: Zo2
-20 Frequency (rad/s): 1.01e+05
Magnitude (dB): -0.0557

-40
System: H2
-60 Frequency (rad/s): 1.01e+06
Magnitude (dB): -40

-80
90
ZN1
45 ZD1
Zo2
0
Phase (deg)

H2
-45

-90

-135

-180
2 3 4 5 6 7
10 10 10 10 10 10
Frequency (rad/s)

图 10-40 第 2 级输入滤波器的输出阻抗和传递函数的 Bode 图


第 1 级输入滤波器和第 2 级输入滤波器级联后的输出阻抗和传递函数如图
10-41 所示:
Bode Diagram
100 ZN_03
System: Zo_all ZD_03
50 Frequency (rad/s): 3.04e+04 Zo1
Magnitude (dB): 9.63
Zo_all
0
Magnitude (dB)

H1
H_all
-50
System: H1
-100 Frequency (rad/s): 9.96e+05
System: H_all Magnitude (dB): -59.9
Frequency (rad/s): 1.01e+06
-150 Magnitude (dB): -100

-200
180

0
Phase (deg)

-180

-360
2 3 4 5 6 7
10 10 10 10 10 10
Frequency (rad/s)

图 10-41 级联后的输出阻抗与传递函数
从图 10-41 可知,第 2 级的输入滤波器的输出阻抗已经很好得满足了阻抗不
等式的要求。级联后的输入滤波器的传递函数所需要的衰减增益已经实现。级联
后的输入滤波器的输出阻抗也满足其阻抗不等式要求。

- 362 -
第11章 DCM 模式下的 AC 和 DC 等效电路建模

Problem 11.1

反激变换器的平均开关模型。如图 11-1 所示的反激变换器运行于 DCM 模


式。两绕组的电感,其匝数比为 1: n ,漏感可以忽略不计,可以使用一个理想变
压器并在原边侧并联一个激磁电感 Lp 进行建模。

(a) 画出晶体管和二极管的电压电流波形,推导它们的平均值表达式。
(b) 画出含有无损电阻网络的变换器平均模型,给出 Re (d ) 的表达式。

(c) 当变换器处于 DCM 模式时,推导电压转换比率 V / Vg 。

(d) 负载电流处于什么范围时,
(c)部分的推导是有效的?推导变换器运行于
DCM 模式的边界条件,以 I  I crit ( D, Re ,Vg , n) 形式表达。

(e) 推导从控制到输出的交流小信号传递函数 Gvd ( s) ,你可以忽略电感的动态

特性。
1: n D1

Lp v
C R
vg

Q1

图 11-1 反激变换器
Solution:
(a) 反激变换器的等效电路如图 11-2 所示:

1: n i2 D1
iL v2

Lp C R v
vg

i1
Q1 v1

图 11-2 反激变换器的等效电路

- 363 -
当开关管导通时,即 0  t  d1Ts 时,有:

v1  0
i1  iL
(11-1)
v2  nvg  v
i2  0

当开关管关断,且激磁电感中的电流在断续之前时,即 d1Ts  t  d 2Ts ,有:

v
v1   vg
n
i1  0
(11-2)
v2  0
iL
i2 
n

当开关管关断,且激磁电感中的电流在断续之后时,即 d 2Ts  t  Ts ,有:

v1  vg
i1  0
(11-3)
v2  v
i2  0
激磁电感的电压电流波形如图 11-3 所示:

vL

vg

t
v

n
iL
vg
i pk  d1Ts
Lp

0 d1Ts  d1  d 2  Ts Ts t

图 11-3 激磁电感的电压电流波形
故二极管和晶体管的电压电压波形如图 11-4 所示:

- 364 -
v1 v2
v
 vg
n
vg nvg  v
v
t t

i1 i2
vg vg
i1 pk  d1Ts i2 pk  d1Ts
Lp nLp

0 d1Ts  d1  d 2  Ts Ts t 0 d1Ts  d1  d 2  Ts Ts t

图 11-4 二极管和晶体管的电压电压波形
二极管和晶体管电压电流的周期平均值为:

1  vT  
v1  
d T
2 s 
 n
s
 v g T
s 
  1  d1  d 2  Ts v g T

Ts
Ts    

s

 vg 
1 1
i1 T  d1Ts 
Ts
d1Ts
s
Ts  2 Lp 
  (11-4)

s
Ts 

v2 T   d1Ts n vg  v T  1  d1  d 2  Ts v T 
1
Ts s
 s  
 
 vg 
1 1
 d1Ts 
Ts
i2 d 2Ts
Ts
Ts  2 nLp 
 

由于激磁电感的伏秒积为 0,故可求出 d 2 的值为:

 vT 
vg d1Ts    s
d 2Ts   0
Ts  n 
 
(11-5)
d1n vg
d2 
Ts

v Ts

将式(11-5)求出的 d 2 代入到式(11-4)中可得:

- 365 -
v1 Ts
 vg
Ts
2
Td
i1 Ts
 s 1
vg
2 Lp Ts

(11-6)
v2 Ts
 v Ts
2
Ts d12 vg

Ts
i2 Ts
2 Lp v Ts

注意:书中的公式 11.14 有误,不应该有负号。


(b) 根据式(11-6)可首先画出开关网络的含有无损电阻网络的平均模型如图
11-5 所示:

i1 Ts
i2 Ts

v1 Ts
Re v2 Ts

图 11-5 开关网络的无损电阻平均模型

其中无损电阻 Re 为:

2 Lp
Re  (11-7)
Ts d12

首先将如图 11-2 所示反激变换的等效模型进行变形,如图 11-6 所示:


1: n
iL

Lp C R v
vg
i1
v1 v2
i2

图 11-6 变形后的反激变换器电路
将图 11-6 中的开关网络用如图 11-5 所示无损电阻网络进行代替,如图
11-7 所示:

- 366 -
1: n
iL

Lp C R v
vg
i1 Ts

v1 Re v2
Ts Ts
i2 Ts

图 11-7 含有无损电阻网络的反激变换器 DCM 模式平均开关模型


(c) 当只考虑稳态量时,图 11-7 可简化为如图 11-8 所示:

I1 Ts
I2 Ts

Vg V1 Re V2 R V
Ts Ts

图 11-8 只考虑稳态量时的反激变换器 DCM 模式平均开关模型


故有:
Vg2 V2
 (11-8)
Re R

将式(11-7)代入到式(11-8)可得:
V D1
 (11-9)
Vg 2 Lp
RTs

(d) 当变换器运行于 CCM 和 DCM 之间的临界状态时, i2 的电流波形如图

11-9 所示:
i2
Vg
i2 pk  D1Ts
nLp

0 D1Ts Ts t

图 11-9 临界导通模式下的 i2 电流波形

根据图 11-9 可知,DCM 模式发生的条件为( I 为负载电流):

- 367 -
I   i2
i2 pk (11-10)
 i2 
2
即有:
Vg
I D1Ts (11-11)
n2 Lp

将式(11-11)进行变形,可得:
Vg
I (11-12)
nD1 Re

(e) 图 11-5 的平均端口方程为:


Ts d12
i1 Ts
 v1 Ts
2 Lp
2 (11-13)
Ts d12 vg

Ts
i2 Ts
2 Lp v2 Ts

图 11-5 的小信号交流等效电路的形式如图 11-10 所示:


iˆ1 iˆ2

v̂1 r1 j1dˆ g1vˆ2 g 2 vˆ1 j2 dˆ r2 v̂2

图 11-10 DCM 下开关网络的小信号交流模型


故对于左侧有:
Ts d12
f1 (v1 , v2 , d1 )  v1
2 Lp
Ts d12v12
f 2 (v1 , v2 , d1 ) 
2 Lp v2
1 f1 (v1 , V2 , D1 ) 1
  (11-14)
r1 v1 v V
Re ( D1 )
1 1

f1 (V1 , V2 , d1 ) DTV 2V1


j1   1 s 1
d1 d D
Lp Re ( D1 ) D1
1 1

f1 (V1 , v2 , D1 )
g1  0
v2 v V 2 2

对于右侧有(注意 r2 的计算要添个负号):

- 368 -
1 f (v ,V , D ) 1
 2 1 2 1  2
r2 v2 v V
M Re ( D1 )
2 2

f 2 (V1 ,V2 , d1 ) DTV 2 2V1


j2   1 s 1  (11-15)
d1 d D
LpV2 MD1 Re ( D1 )
1 1

f 2 (V1 , v2 , D1 ) 2
g2  
v1 v V
MRe ( D1 )
1 1

将图 11-7 中的开关网络用图 11-10 取代,如图 11-11 所示:


1: n
iL

Lp C R v
vg

v̂1 r1 j1dˆ g1vˆ2 g 2 vˆ1 j2 dˆ r2 v̂2

图 11-11 DCM 模式下反激变换器交流小信号模型


对于图 11-11,为了便于分析,忽略激磁电感的作用,则图 11-11 可以简化
至如图 11-12 所示:

vg v̂1 r1 j1dˆ g1vˆ2 g 2 vˆ1 j2 dˆ r2 v̂2 C R v

图 11-12 简化的 DCM 模式下反激变换器交流小信号模型

对于 Gvd ( s) 推导,令图 11-12 中的 vˆg  0 ,则图 11-12 还可进一步简化为如

图 11-13 所示:

v̂1 r1 j1dˆ j2 dˆ r2 v̂2 C R v

图 11-13 Gvd 的推导

故有:
1 1 1
  sC 
Z r2 R (11-16)
ˆ
vˆ  j dZ
2

根据式(11-16)可得:

- 369 -
1
Gvd  j2 Z  Gd 0
s
1
p
Gd 0  j2  r2 R  (11-17)
1
p 
C  r2 R 
将式(11-15)代入到式(11-17)可得:
V
Gd 0 
D1
(11-18)
2
p 
CR

Problem 11.2

非隔离 Watkins-johnson 变换器的平均开关模型。如图 11-14 所示的变换器


运行于 DCM 模式。两绕组的电感具有 1:1 的匝数比和可忽略的漏感,其可通过
使用一个理想变压器并联一个激磁电感进行建模。
(a) 画出晶体管和二极管的电压电流波形,推导它们的周期平均值。
(b) 画出包含无损电阻网络的变换器的平均开关模型,给出无损电阻 Re (d ) 的

表达式。
(c) 求解平均开关模型,以获得稳态下的 DCM 模式下的电压转换比率
M ( D)  V / Vg 。负载电流在什么范围内,你所推导的表达式有效。

Q1

vg C R v
L D1

图 11-14 非隔离 Watkins-johnson 变换器


Solution:
(a) 等效电路如图 11-15 所示
vL iL
L i1 Q1
1:1
v1
vg i2 C R v
v2 D1

图 11-15 非隔离 Watkins-johnson 变换器等效电路

- 370 -
激磁电感的电压电流波形如图 11-16 所示:
vL
vg  v

vg
iL
vg  v
i pk  d1Ts
L

d1Ts
0  d1  d 2  Ts Ts t

图 11-16 激磁电感电压电流波形
晶体管和二极管的电压电流波形如图 11-17 所示:
v1 v2
2v g  v 2v g  v
vg  v vg
t t

i1 i2
vg  v vg  v
i pk  d1Ts i pk  d1Ts
L L

d1Ts d1Ts
0  d1  d 2  Ts Ts t 0  d1  d 2  Ts Ts t

图 11-17 晶体管和二极管的电压电流波形
二极管和晶体管的电压电流周期平均值为:

v1 Ts

1
Ts 

d 2Ts 2 vg
Ts
 v Ts   1  d  d T  v
1 2 s g T
s
 v Ts 
 vg  v T 
1 1
i1 T  d1Ts 
Ts s
d1Ts
s
Ts  2 L 
 
(11-19)
s
Ts  Ts
v2 T   d1Ts 2 vg  v T  1  d1  d 2  Ts
1
s
 v 
g T
s


 vg  v 
1 1
d1Ts 
Ts

Ts
i2 d 2Ts
Ts
Ts  2 L 
 

- 371 -
由于激磁电感的伏秒积为 0,故可求出 d 2 的值为:

v g T
s
 d T   v
 v Ts 1 s g T
s
d 2Ts  0 
d  v1  v  g T Ts
(11-20)
d2  s

vg
Ts

将式(11-20)代入到式(11-19)中可得:
v1 Ts
 vg  v Ts
Ts

v1

Ts
i1 Ts
Re
v2 Ts
 vg (11-21)
Ts
2
v1

Ts
i2 Ts
Re v2 Ts

2L
Re 
d12Ts
(b) 根 据式 (11-21)可首 先 画出开关网络的含 有 无损电阻的平均模 型 如 图
11-18 所示:

i1 Ts
i2 Ts

v1 Ts
Re v2 Ts

图 11-18 开关网络的含有无损电阻的平均模型
将图 11-15 所示的反激变换器等效电路进行变形,如图 11-19 所示:
vL iL
L
1:1 i1

vg i2 Q1 C R v
v1
v2 D1

图 11-19 非隔离 Watkins-johnson 等效电路的变形


用图 11-18 所述的开关网络的平均模型替代图 11-19 中的开关网络,替代后
如图 11-20 所示:

- 372 -
L
1:1 i1 Ts

vg i2 Re v
Ts C R Ts
Ts v1 Ts
v2 Ts

图 11-20 非隔离 Watkins-johnson 平均开关网络模型


(c) 为了求解 DCM 模式下稳态时的电压转换比率,则图 11-20 可简化至如图
11-21 所示:

i1 Ts

vg i2 Re v
Ts R Ts
Ts v1 Ts
v2 Ts

图 11-21 稳态下的非隔离 Watkins-johnson 平均开关网络模型


故有:
Vg  V V

Re R
(11-22)
V R

Vg R  Re
将式(11-21)代入到式(11-22)可得:

V D12
 (11-23)
Vg D 2  2 L
1
RTs

以上推导仅在 DCM 模式下有效,为了保证 DCM 模式的运行,有:


Vg  V
I  i1  I  D1Ts (11-24)
2L

Problem 11.3

画出 Buck-Boost 变换器的稳态输出特性:不同占空比 D 下的输出电压 V 与


负载电流 I 的关系。要包含 CCM 模式和 DCM 模式,标出 CCM 模式和 DCM 模
式的边界。
Solution:
Buck-Boost 变换器如图 11-22 所示:

- 373 -
Q1 D1
i (t )
Vg L C R V

图 11-22 Buck-Boost 变换器

对于 CCM 模式下的 Buck-Boost 变换器,当开关管导通时,即 0  t  D1Ts 时,

电路如图 11-23 所示:

i (t ) iC
Vg vL L C R V

图 11-23 CCM 模式下,开关管导通时的 Buck-Boost 变换器


故有:
vL  Vg
V (11-25)
ic  
R

当开关管关断时,即 D1Ts  t  Ts 时,电路如图 11-24 所示:

D1
i (t ) iC
Vg vL L C R V

图 11-24 CCM 模式下,开关管关断时的 Buck-Boost 变换器


故有:
vL  V
V (11-26)
ic  iL 
R
根据式(11-25)和式(11-26)可得:
V D1

Vg D1  1
(11-27)
V
IL 
1  D1  R
式(11-27)的前提条件为变换器处于 CCM 模式,电感电流断续,即电感电流

- 374 -
的直流分量要大于电感电流的纹波。

1  D1 
2
VTs 2L
I L  iL I   1  D1 
2
(11-28)
2L RTs
对于 DCM 模式,当开关管导通时,电路如图 11-25 所示:

i (t ) iC
Vg vL L C R V

图 11-25 DCM 模式下,开关管导通时的 Buck-Boost 变换器


故有:
vL  v g
v (11-29)
iC  
R
当开关管关断时,且电感电流在断续之前时,电路如图 11-26 所示:

D1
i (t ) iC
Vg vL L C R V

图 11-26 DCM 模式下,开关管关断且断续之前时的 Buck-Boost 变换器


故有:
vL  v
v (11-30)
iC  iL 
R
当开关管关断时,且电感电流在断续之后时,电路如图 11-27 所示:

i (t ) iC
Vg vL L C R V

图 11-27 DCM 模式下,开关管关断且断续之后的 Buck-Boost 变换器


故有:
vL  0
v (11-31)
iC  
R
根据式(11-29)、式(11-30)和式(11-31)可得:

- 375 -
1 g  D2V  1  D1  D2  0  0
DV (11-32)

由式(11-32)可求出:
DV
D2   1 g
(11-33)
V
二极管的电流波形如图 11-28 所示:
iD (t )
i pk

iD

0 D1TS D2TS D3TS t

图 11-28 二极管的电流波形
由于二极管电流的直流分量等于负载电流值,故有:
1 1
I  D2Ts i pk
Ts 2
(11-34)
Vg
i pk  D1Ts
L
由式(11-34)可得:
D1D2TsVg
I  (11-35)
2L
将式(11-33)代入到式(11-35)中,可得:
Vg2
VI  (11-36)
2L
D12Ts

式(11-36)的前提条件为:

1  D1 
2
VTs 2L
I L  iL I    1  D1 
2
(11-37)
2L RTs

Problem 11.4

如图 11-29 所示的网络。功率源 p(t ) 的表达式为:

p(t )  1000cos 2 377t

电路处于稳态,推导电阻上的电压有效值 VR,rms 。

- 376 -
C2
300μF

L1 L2
7mH 7mH
p(t ) C1 R vR
30μF 20

图 11-29 功率源网络
Solution:
功率源的周期为:
  377rad/s
2 (11-38)
T

故一个周期内,功率源所提供的功为:
T T
W   p(t )dt   1000cos2 377tdt  8.333W (11-39)
0 0

由于一个周期内电感电容消耗的平均功率为 0,故有:

v 
2

T W
R , rms
(11-40)
R
将式(11-39)代入到式(11-40),可得:
vR,rms  100V (11-41)

Problem 11.5

验证书中 Table11.3 中 Gd 0 和  p 的表达式。

Solution:
(a) Buck 变换器的电路图如图 11-30 所示:
i1 i2 L iL

vg (t ) v1 v2 C R v(t )

图 11-30 Buck 变换器


端口电压电流波形如图 11-31 所示:

- 377 -
v1 v2

vg vg

v
t t

i1 i2
vg  v vg  v
i1 pk  d1Ts i2 pk  d1Ts
L L

0 d1Ts Ts t 0 d1Ts  d1  d 2  Ts Ts t

图 11-31 端口电压电流波形
端口电压电流的周期平均值为:
v1 Ts
 vg
Ts

 vg  v T 
1 1
 d1Ts 
Ts s
i1 d1Ts
Ts
Ts  2 L 
 
(11-42)
  d1Ts vg  1  d1  d 2  Ts v 
1
v2 Ts 
Ts
Ts  Ts 
 vg  v 
1 1
 d1  d 2  Ts d1Ts 
Ts
i2 T 
Ts
s
Ts  2 L 
 
由于电感的伏秒积为 0 可得:

  v 
  
d1 vg
d1 vg
Ts
 v Ts
 d2  v Ts   0  d2 
v
Ts

Ts
Ts
(11-43)

将式(11-43)代入到式(11-42)中,可得:
v1 Ts
 vg
Ts

v1  v2

Ts Ts
i1 Ts 2L
d12Ts (11-44)
v2 Ts
 v Ts

i1 v1

Ts Ts
i2 Ts
v2 Ts

则开关网络的小信号等效电路,其形式如图 11-32 所示:

- 378 -
iˆ1 iˆ2

v̂1 r1 j1dˆ g1vˆ2 g 2 vˆ1 j2 dˆ r2 v̂2

图 11-32 DCM 模式下,Buck 开关网络的交流小信号模型


故对于左边侧有:
v1  v2
f1 (v1 , v2 , d1 ) 
2L
d12Ts
v1  v1  v2 
f 2 (v1 , v2 , d1 ) 
2L
v2
d12Ts
2L
Re 
D12Ts (11-45)
1 f1 (v1 ,V2 , D1 ) 1
 
r1 v1 v V
Re
1 1

f1 (V1 ,V2 , d1 ) 2 1  M  V1


j1  
d1 d D
D1 Re
1 1

f1 (V1 , v2 , D1 ) 1
g1  
v2 v V
Re
2 2

对于右边侧有:
1 f (v ,V , D ) 1
 2 1 2 1  2
r2 v2 v V
M Re
2 2

f 2 (V1 ,V2 , d1 ) 2 1  M  V1
j2   (11-46)
d1 d D
MRe D1
1 1

f 2 (V1 , v2 , D1 ) 2M
g2  
v1 v V
Re M
1 1

注意:书中 Table11.2 的 Buck 一行中的 g1 写错了,应该有负号的。

将图 11-32 替换至图 11-30 中,可有:


iˆ1 iˆ2 L

vg (t ) v̂1 r1 j1dˆ g1vˆ2 g 2 vˆ1 j2 dˆ r2 v̂2 C R v(t )

图 11-33 DCM 模式下 Buck 变换器的交流小信号模型

- 379 -
对于 Gvd 的推导,忽略掉电感的动态特性,图 11-33 可简化为如图 11-34 所

示:
iˆ1 iˆ2

v̂1 r1 j1dˆ g1vˆ2 j2 dˆ r2 v̂2 C R v(t )

图 11-34 DCM 模式下,Buck 变换器 Gvd 的推导

故有:
 
 
1
Gvd  j2 Z  Gd 0  
 1 s 
   (11-47)
 p 
1 1 1
  sC 
Z r2 R
由式(11-47)可得:
Gd 0  j2  R r2 
1 (11-48)
p 
C  R r2 

将式(11-46)代入到式(11-48)中可得:
2MRV1 1  M 
Gd 0 
D1 Re M 2  D1R
(11-49)
R M2 R
p  e 2
CM RRe
由于:
V2
V1 
M
RK
Re  2 (11-50)
D1
2L
K
RTs

将式(11-50)代入到式(11-49)中可得:

1 2 1  M 
2 DV
Gd 0 
D12  KM 2
(11-51)
D 2  KM 2
p  1
CKM 2 R

- 380 -
又因为:
2 D12  D12 M
M K  (11-52)
K M2
1 1 4 2
D1
将式(11-52)代入到式(11-51)中可得:
2V2  M  1 2V  M  1
Gd 0  
D1  M  2  D1  M  2 
(11-53)
2M
P 
1  M  CR
(b) Boost 变换器电路如图 11-35 所示:

L i1 i2

vg (t ) C
v1 v2 R v(t )

图 11-35 Boost 电路
端口电压电流波形如图 11-36 所示:
v1 v2

v
v
vg

t t

i1 i2
vg vg
i1 pk  d1Ts i2 pk  d1Ts
L L

0 d1Ts  d1  d 2  Ts Ts t 0 d1Ts  d1  d 2  Ts t

图 11-36 Boost 变换器端口电压电流波形


端口电压电流的周期平均值为:

v1 
1 d T v  1  d1  d 2  Ts vg 
Ts
Ts  2 s Ts Ts 
 vg 
1 1
i1   1 2 s
d  d T
Ts
d T 
Ts  2 
Ts 1 s
L
  (11-54)
v2 Ts
 vT
s

 vg 
1 1
 d1Ts 
Ts
i2 d 2Ts
Ts
Ts  2 L 
 

- 381 -
由于电感的伏秒积为 0,故有:

 
d1 vg
 d 2 vg  v  0  d2 
Ts
d1 vg (11-55)
Ts Ts Ts
v Ts
 vg
Ts

将式(11-55)代入到式(11-54)中,可得:
v1 Ts
 vg
Ts

v1 v2

Ts Ts

v R
i1
 v1
Ts
2 Ts Ts e

v2 Ts
 v Ts
(11-56)
2
v1

Ts

v R
i2 Ts
2 Ts  v1 Ts e

2L
Re 
d12Ts
Boost 的开关网络的交流小信号等效电路形式如图 11-37 所示:
iˆ1 iˆ2

v̂1 r1 j1dˆ g1vˆ2 g 2 vˆ1 j2 dˆ r2 v̂2

图 11-37 Boost 开关网络的交流小信号等效电路


对于左侧有:
v1v2
f1 (v1 , v2 , d1 ) 
2L
 v2  v1 
d12Ts
v12
f 2 (v1 , v2 , d1 ) 
2L
 v2  v1 
d12Ts
R  M  1
2
1 f1 (v1 ,V2 , D1 )
  r1  e (11-57)
r1 v1 v V
M2
1 1

f1 (V1 ,V2 , d1 ) 2MV1


j1  
d1 d D
Re  M  1 D1
1 1

f1 (V1 , v2 , D1 ) 1
g1  
v2 Re  M  1
2
v V 2 2

2L
其中 Re  。
D12Ts

注意:书中 Table11.2 中 Boost 一行的 g1 少了个负号。

- 382 -
对于右侧有:
1 f (v ,V , D )
 2 1 2 1  r2  Re  M  1
2

r2 v2 v V 2 2

f 2 (V1 , V2 , d1 ) 2V1
j2   (11-58)
d1 d D
1
 M  1 Re D1
1

g2 
f 2 (V1 , v2 , D1 )

 2M  1
v1 Re  M  1
2
v1 V1

故 DCM 模式下 Boost 变换器的交流小信号模型如图 11-38 所示:


L iˆ1 iˆ2

C
vg (t ) v̂1 r1 j1dˆ g1vˆ2 g 2 vˆ1 j2 dˆ r2 v̂2 R v(t )

图 11-38 DCM 模式下 Boost 变换器的交流小信号模型

对于 Gvd 的推导,首先令 vˆg  0 ,并忽略电感的动态特性,图 11-38 可进一

步简化为如图 11-39 所示:


iˆ1 iˆ2

C
v̂1 r1 j1dˆ g1vˆ2 j2 dˆ r2 v̂2 R v(t )

图 11-39 DCM 模式下,Boost 变换器 Gvd 的推导

故有:
 
 
1
Gvd  j2 Z  Gd 0  
 1 s 
   (11-59)
 p 
1 1 1
  sC 
Z r2 R
由式(11-59)可得:
Gd 0  j2  R r2 
1 (11-60)
p 
C  R r2 

将式(11-58)代入到式(11-60)中可得:

- 383 -
2 RV1  M  1
Gd 0 
D1  Re M 2  2 Re M  R  Re 
(11-61)
Re M 2  2 Re M  R  Re
p 
CRRe  M  1
2

由于有:
V2
V1 
M
RK
Re  2 (11-62)
D1
2L
K
RTs

将式(11-62)代入到式(11-61)中,可得:
1 2  M  1
2 DV
Gd 0 
D12 M  KM 3  2 KM 2  KM
(11-63)
D12  KM 2  2 KM  K
p 
CKR  M  1
2

对于 Boost 变换器有:

D12
1 1 4
K K 4 D12
M (11-64)
 2M  1 1
2
2

将式(11-64)代入到式(11-63)中可得:
2V2  M  1 2V  M  1
Gd 0  
D1  2M  1 D1  2M  1
(11-65)
2M  1
p 
CR  M  1
(c) Buck-Boost 变换器的电路如图 11-40 所示:
i1 i2

v1 Q1 D1 v2

i (t )
Vg vL L C R V

图 11-40 Buck-Boost 变换器电路


端口电压电流波形如图 11-41 所示:

- 384 -
v1 v2
vg  v
vg  v
vg
v
t t

i1 i2
vg vg
i1 pk  d1Ts i2 pk  d1Ts
L L

0 d1Ts  d1  d 2  Ts Ts t 0 d1Ts  d1  d 2  Ts Ts t

图 11-41 Buck-Boost 变换器端口电压电流波形


故端口电压电流周期平均值为:

v1 Ts

1
Ts
d T
 2 s v g T
s
 v Ts   1  d  d T 1 2 s vg
Ts


 vg 
1 1
i1 T  d1Ts 
Ts
d1Ts
s
Ts  2 L 
 
(11-66)
s
1
Ts  Ts 
v2 T   d1Ts vg  v T  1  d1  d 2  Ts  v
s
  Ts  

 vg 
1 1
 d1Ts 
Ts
i2 d 2Ts
Ts
Ts  2 L 
 
由于电感的伏秒积为 0 可得:
d1 vg
 d2 v  0  d2  
Ts
d1 vg Ts
(11-67)
Ts v Ts

将式(11-67)代入到式(11-66),可得:
v1 Ts
 vg
Ts

v1

Ts
i1 Ts
Re
v2 Ts
 v Ts
(11-68)
2
v1

Ts
i2 Ts
v2 Ts
Re
2L
Re 
d12Ts
Buck-Boost 的开关网络的交流小信号等效电路形式如 图 11-42 所示:

- 385 -
iˆ1 iˆ2

v̂1 r1 j1dˆ g1vˆ2 g 2 vˆ1 j2 dˆ r2 v̂2

图 11-42 Buck-Boost 的开关网络的交流小信号等效电路


对于图 11-42 的左侧有:
v1
f1 (v1 , v2 , d1 ) 
2L
d12Ts
v12
f 2 (v1 , v2 , d1 ) 
2L
v2
d12Ts
1 f1 (v1 ,V2 , D1 )
  r1  Re (11-69)
r1 v1 v V 1 1

f1 (V1 ,V2 , d1 ) 2V1


j1  
d1 d D
Re D1
1 1

f1 (V1 , v2 , D1 )
g1  0
v2 v V 2 2

对于图 11-42 的右侧有:


1 f (v ,V , D )
 2 1 2 1  r2  Re M 2
r2 v2 v V 2 2

f 2 (V1 ,V2 , d1 ) 2V1


j2   (11-70)
d1 d D
MRe D1
1 1

f 2 (V1 , v2 , D1 ) 2
g2  
v1 v V
Re M
1 1

故 DCM 模式下 Buck-Boost 变换器的交流小信号模型如图 11-43 所示:

iˆ1 iˆ2

v1 r1 j1dˆ g1vˆ2 g 2 vˆ1 j2 dˆ r2 v̂2

i (t )
vg vL L C R v

图 11-43 DCM 模式下 Buck-Boost 变换器的交流小信号模型

对于 Gvd 的推导,首先令 vˆg  0 ,并忽略电感的动态特性,则图 11-43 可简

- 386 -
化为如图 11-44 所示:
iˆ1 iˆ2

C
v̂1 r1 j1dˆ j2 dˆ r2 v̂2 R v(t )

图 11-44 DCM 模式下 Buck-Boost 变换器 Gvd 的推导

故有:
 
 
1
Gvd  j2 Z  Gd 0  
 1 s 
   (11-71)
 p 
1 1 1
  sC 
Z r2 R
由式(11-71)可得:
Gd 0  j2  R r2 
1 (11-72)
p 
C  R r2 

将式(11-70)代入到式(11-72)中,可得:
2MRV1
Gd 0 
D1 Re M 2  D1R
(11-73)
Re M 2  R
p 
CM 2 RRe
又由于有:
V2
V1 
M
RK
Re  2 (11-74)
D1
2L
K
RTs

将式(11-74)代入到式(11-73)中,可得:
2 D1V2
Gd 0 
D  KM 2
1
2
(11-75)
D 2  KM 2
p  1
CKM 2 R
对于 Buck-Boost 变换器,有:

- 387 -
D1 D12
M  K  2 (11-76)
K M
将式(11-76)代入到式(11-75)中,可得:
V2 V
Gd 0  
D1 D1
(11-77)
2
p 
CR

Problem 11.6

某 Buck 变换器的输入电压 Vg  28V ,输出电压 V  15V ,负载电阻 R  10Ω ,

其他元器件的值为 L  8μH 、 C  220μF 、 f s  150kHz 。

(a) 推导无损电阻 Re 的值。

(b) 推导静态占空比 D 的值。


(c) 画出 Gvd 的传递函数,你可以忽略电感的动态特性。

Solution:
(a) 系数 K 的值为:
2L
K  0.24 (11-78)
RTs

如果 Buck 变换器运行于 CCM 模式,则有:


V
Kcrit ( D)  1  D  1   0.464 (11-79)
Vg

为了使变换器运行于 CCM 模式,则必须满足:


K  K crit ( D) (11-80)

通过式(11-78)可知,式(11-80)不满足,故变换器运行于 DCM 模式。


而对于 Buck 变换器,其 DCM 模式下的电压转换比率 M 为:
V 2
M  (11-81)
Vg K
1 1 4 2
D
根据式(11-81)可求出:
D  0.385 (11-82)
故无损电阻 Re 的值为:

- 388 -
2L
Re   16.178 (11-83)
D 2Ts

(b) 静态占空比 D 的值为:


D  0.385 (11-84)
(c) Gvd 的传递函数已在 Problem 11.5,这里重新给出:

 
 
1
Gvd  Gd 0  
 1 s 
  
 p 
2V  M  1
Gd 0  (11-85)
D1  M  2 
2M
P 
1  M  CR
将器件值代入到式(11-85)中可得:
15.47
Gvd  (11-86)
6957 s  1
其 Bode 图如图 11-45 所示:
Bode Diagram
30
Gvd
20
Magnitude (dB)

10

-10

-20
0
Gvd
Phase (deg)

-45

-90
-5 -4 -3 -2
10 10 10 10
Frequency (rad/s)

图 11-45 DCM 模式下,Buck 变换器 Gvd 的 Bode 图

Problem 11.7 (推导的结果可能有问题)

使用书中 11.3 节中的方法,推导 Boost 变换器的 Gvd 的传递函数,不能忽略

电感的动态特性。
(a) 推导 DC 增益 Gd 0 和右半平面零点 z 的表达式,用 M 、Re 、D 、Vg 、 L 、

C 和 R 来表示。

- 389 -
(b) 假设 C 足够大, L 足够小,则 Gvd 的极点可以使用低 Q 近似,使用低 Q 近

似,将两个极点表达成 M 、D 、L 、C 和 R 的形式。证明低频极点如书中 Table11.3


所述,高频极点如书中 Table11.4 所述。
Solution:
(a) Boost 变换器如图 11-46 所示:

L i1 i2

vg (t ) C
v1 v2 R v(t )

图 11-46 Boost 变换器


对于电感电流有:

1  d1  d 2  vg d1Ts
 i pk  d1  d 2  
Ts
iL Ts
(11-87)
2 2L
由式(11-87)可得:
 Re iL 
d2   Ts
 1 d1 (11-88)
 vg 
 Ts 
Boost 变换器的端口平均值已在(11-54)求出,这里重新给出:

v1 
1 d T v  1  d1  d 2  Ts vg 
Ts
Ts  2 s Ts Ts 
 vg 
1 1
i1 T   d1  d 2  Ts Ts
d1Ts 
s
Ts  2 L 
  (11-89)
v2 T  v T
s s

 vg 
1 1
 d1Ts 
Ts
i2 d 2Ts
Ts
Ts  2 L 
 
将式(11-88)代入到式(11-89),可得:

v1 
vg
2

Ts
 d1 v Ts
vg
Ts
 Re d1 iL Ts v Ts
 vg
Ts 
Ts
vg
Ts (11-90)
vg  Re iL Ts

Ts
i2 Ts
Re
故开关网络的交流小信号模型如图 11-47 所示:

- 390 -
iL  i1 i2

k g vg

kv v
v1 v2
j2 d1 g g vg h2iL
r1iL

f1d1

图 11-47 Boost 开关网络交流小信号等效电路


用图 11-47 所示的开关网络等效电路去替代图 11-46 中的开关网络,替换后
如图 11-48 所示:
L iL  i1 i2

k g vg i2

kv v
vg (t ) v1 v2 R v(t )
j2 d1 g g vg h2iL
r1iL

f1d1

图 11-48 DCM 模式下 Boost 交流小信号等效电路

对于 Gvd 推导,图 11-48 可简化为如图 11-49 所示:

L iL  i1 i2

i2

kv v
v1 v2 R v(t )
j2 d1 h2iL
r1iL

f1d1

图 11-49 Boost 变换器 Gvd 的推导

故有:

- 391 -
f1 (vg , v, d1 ,i L ) 
vg
2

Ts
 d1 v Ts
vg
Ts
 Re d1 iL Ts v Ts
 vg
Ts 
vg
Ts

vg  Re iL Ts
f 2 (v1 , v2 , d1 )  
Ts
(11-91)
Re
2L
Re 
d12Ts
根据式(11-91)可得:
f1 (Vg , v, D1 , I L ) 2I L L
kv    D1
v v V
D1TsVg
f1 (Vg ,V , D1 , iL ) 2 LV  2 LVg
r1  
iL iL  I L
D1TsVg
f1 (Vg ,V , d1 , I L ) TsVVg D12  2 I L LV  2 I L LVg
f1   (11-92)
d1 d1  D1
D12TsVg
f 2 (Vg , V , d1 , I L ) D1TsVg
j2  
d1 d1  D1
L
f 2 (Vg , V , D1 , iL )
h2  1
iL iL  I L

对于 Gvd 的推导,可采用叠加定理,首先考虑电压源 f1d1 单独起作用时,则

电路如图 11-50 所示:


L iL  i1 i2

i2

kv v
v1 v2 R v(t )
h2iL
r1iL

f1d1

图 11-50 Gvd 的推导, f1d1 单独起作用时

故有:
iL sL   f1d1  r1iL  kv v
1 1
 sC  (11-93)
Z R
h2iL Z  v

- 392 -
由式(11-93)可得:
 Rf1h2
Gvd 1  (11-94)
CLRs   CRr1  L  s  r1  Rh2 kv
2

当考虑 j2 d1 单独作用时,则电路如图 11-51 所示:

L iL  i1 i2

i2

kv v
v1 v2 R v(t )
j2 d1 h2iL
r1iL

图 11-51 Gvd 的推导, j2 d1 单独起作用时

故有:
iL sL  ri
1 L  kv v

1 1
 sC  (11-95)
Z R
v   j2 d1  h2iL  Z
根据式(11-95)可得:
LRj2 s  Rj2 r1
Gvd 2  (11-96)
CLRs   CRr1  L  s  r1  Rh2 kv
2

由式(11-94)和式(11-96)可得:
R  j2 r1  f1h2  Lj2 s 
Gvd  (11-97)
CLRs   CRr1  L  s  r1  Rh2 kv
2

式(11-97)可变形为:
s
1
f1h2  j2 r1
R  j2 r1  f1h2  Lj2
Gvd  (11-98)
r1  Rh2 kv s 2
s
 1
r1  Rh2 kv r1  Rh2 kv
CLR CRr1  L
则有:

- 393 -
R  j2 r1  f1h2 
Gd 0 
r1  Rh2 kv
(11-99)
fh jr
z  1 2 2 1
Lj2
将式(11-92)代入到式(11-99)中可得:
RVg  RRe M 2  RRe M 3  M  2 
Gd 0 
D1 R  D1 Re M  D1Re M 2 R 2  D1Re
(11-100)
RR M 2  RRe M 3  M  2
z   e
D1Ts

- 394 -
第12章 电流型控制模式

Problem 12.1

某非理想 Buck 变换器运行于 CCM 模式,其输入电压 Vg  10V ,f s  100kHz ,

L  4μH , C  75μF , R  0.25 。所需要的负载输出是 5V , 20A 。功率部分包括

以 下 损 耗 因 素 : MOSFET 导 通 电 阻 Ron  0 . 1 , 肖 特 基 二 极 管 的 正 向 压 降

VD  0 . 5 V,电感绕组的电阻 RL  0.03 。

(a) 稳态分析:推导变换器稳态占空比 D ,电感电流纹波斜率 m1 和 m2 、无量

纲参数 K  2 L / RTs 。

(b) 推导变换器的小信号方程。
一个电流型控制器应用于此变换器中,并添加一个人工斜坡,其斜率固定为
M a  0.5M 2 ,其中 M 2 是斜率 m2 在输出为 5V , 20A 时的稳态值。

(c) 占空比 D 在什么范围时,电流型控制器是稳定的?额定输出时是否是稳定


的。注意非理想特性对于稳定边界的影响。
(d) 推导从控制到输出的传递函数 Gvc ( s) ,使用简单的近似条件: iL Ts
 ic (t ) 。

推导转折频率和直流增益的表达式,并画出 Gvc ( s) 的 Bode 图。

Solution:
(a) Buck 变换器如图 12-1 所示:
L

vg C R

图 12-1 Buck 变换器


当开关管导通时,电路如图 12-2 所示:

- 395 -
IL Ron RL L
VL
Vg C R V

图 12-2 开关管导通时的 Buck 变换器


故有:
VL  Vg  I L  Ron  RL   V
V (12-1)
IL 
R
当开关管关断时,电路如图 12-3 所示:

RL I L L
VL
vg VD C R V

图 12-3 开关管关断时的 Buck 变换器


故有:
VL   I L RL  VD  V
V (12-2)
IL 
R
根据伏秒平衡可得:
D Vg  I L  Ron  RL   V   D   I L RL  VD  V   0
V (12-3)
IL 
R
由式(12-3)可得:
RV  RVD  RLV
D  0.7176 (12-4)
RVD  RVg  RonV

故:
Vg  I L  Ron  RL   V
m1   6 105
L
(12-5)
 I L RL  VD  V
m2   15.25 105
L

对于无量纲参数 K ,其值为:

- 396 -
2L
K  3.2 (12-6)
RTs

(b) 变换器的周期平均方程为:
d iL
 d  vg  iL  Ron  RL   
Ts
L v 
dt  Ts Ts Ts

d   VD  iL RL  v  (12-7)


 Ts Ts 
d vT vT
C s
 iL T  s

dt s
R
将周期平均的量表达成稳态量与小信号交流扰动量:
iL Ts
 I L  iˆL
vg  Vg  vˆg (12-8)
Ts

v Ts
 V  vˆ

将式(12-8)代入到式(12-7)中,可得:

L

d I L  iˆL    DV  V  DVg  I L RL  DI L Ron  
dt  D

 vˆ  VD dˆ  Dvˆg  Vg dˆ  RLiˆL  DRoniˆL  I L Ron dˆ  


(12-9)
ˆ ˆ  R di
 dv ˆˆ 
g on L 

d V  vˆ   V  vˆ 
L   I L     iˆL  
dt  R  R
故非理想 Buck 变换器的小信号方程为:

diˆL
L  vˆ  VD dˆ  Dvˆg  Vg dˆ  RLiˆL  DRoniˆL  I L Ron dˆ
dt (12-10)
dvˆ vˆ
L  iˆL 
dt R
(c) 当加入人工斜坡后,有:
ˆ m  m 
iˆL (0)  dTs 1 a
(12-11)
ˆ m  m 
iˆL (Ts )  dTs a 2

由式(12-11)可得:
iˆL (Ts ) ma  m2
 (12-12)
iˆL (0) m1  ma

当 ma  0.5M 2 ,则式(12-12)为:

- 397 -
iˆL (Ts ) 0.5M 2  m2
 (12-13)
iˆL (0) m1  0.5M 2

为了使电流型控制稳定,则有:
0.5M 2  m2
1 (12-14)
m1  0.5M 2

故可知当变换器处于额定工况时,式(12-14)可以简化为:
0.5M 2  M 2 0.5M 2
 1 (12-15)
M 1  0.5M 2 M 1  0.5M 2

由于式(12-15)肯定满足,故额定工况下电流型控制是稳定的。
当变换器不运行在额定工况下时,即负载电阻 R 是变化的,那么为了维持固
定的输出电压,占空比 D 随着 R 的变化而变化。
则式(12-14)可简化为:
0.5M 2  m2
 1  m2  0.5M 2  m1  0.5M 2 (12-16)
m1  0.5M 2

其中 m2  0.5M 2 。

用式(12-4)和式(12-5)代入到式(12-16)中并消去负载电阻 R ,可得:
 2D  1  RonV  RLVD  RLVg  RonVD 
 M2 (12-17)
LRL  DLRon
将参数值代入式(12-17)可得:
1.73D  0.865
 15.25 105  D  0.936 (12-18)
 4D  1.2 107

(d) 当 iˆL  iˆC 时,则小信号方程得出的拉普拉斯方程为:

sLiˆc  vˆ  VD dˆ  Dvˆg  Vg dˆ  RLiˆc  DRoniˆc  I L Ron dˆ



sLvˆ  iˆc  (12-19)
R
V
iˆg  Diˆc  dˆ
R
根据式(12-19)中的第一个子式可求得:
Rvˆ  DRvˆg  RRLiˆc  DRRoniˆc  LRiˆc s
dˆ  (12-20)
RVD  RVg  RonV
将式(12-20)代入到式(12-19)中的第 3 子式,可有:

- 398 -

sLvˆ  iˆc 
R
(12-21)
V Rvˆ  DRvˆg  RRLiˆc  DRRoniˆc  LRiˆc s
iˆg  Diˆc 
R RVD  RVg  RonV

将式(12-21)变形有:

sLvˆ  iˆc 
R
vg
iˆg  f1iˆc  g1vˆ 
r1
V  RRL  DRRon  LRs 
f1  D    (12-22)
R  RVD  RVg  RonV 

V R 
g1   
R  RVD  RVg  RonV

R  RV  RVg  RonV 
r1    D 
V DR 
故可画出电流型控制的小信号交流等效电路如图 12-4 所示:

iˆg

vˆg r1 f1iˆc g1vˆ iˆc C R v̂

图 12-4 电流型控制 Buck 变换器交流小信号等效电路


故有:
R
Gvc (s)  (12-23)
CRs  1
将参数值代入到式(12-23)中,可得:
0.25
Gvc (s)  (12-24)
186s  1
其 Bode 图如图 12-5 所示:

- 399 -
Bode Diagram
-10

-15

-20

Magnitude (dB)
-25

-30

-35

-40
0
Phase (deg)

-45

-90
-4 -3 -2 -1
10 10 10 10
Frequency (rad/s)

图 12-5 Gvc ( s) 的 Bode 图

Problem 12.2

使用平均开关周期建模法对 CCM 模式下电流型控制的 Boost 变换器进行建


模。
(a) 如书中 Fig7.46(a)那样定义开关网络的端口量。使用 iL (t ) Ts
 ic (t ) 近似

条件,推导端口波形的平均值表达式,你应该得到如书中 Fig12.18(a)一样的
等效电路。
(b) 扰动并线性化你得到的模型,获取如书中 Fig12.22 所示的等效电路。
(c) 对(b)中得到的模型进行求解,推导从控制到输出的传递函数 Gvc ( s) 和

从交流输入到输出的传递函数 Gvg ( s) 。将这两个传递函数表达成标准形式。

Solution:
(a) Boost 变换器及开关网络端口定义如图 12-6 所示:
iL L i1 i2

C
vg v1 v2 R v

图 12-6 Boost 变换器及开关网络定义


故有:
v1 Ts
 d  v2 Ts
(12-25)
i2 Ts
 d  i1 Ts

根据式(12-25)可得:
v1 Ts
i1 Ts
 v2 Ts
i2 Ts
 p Ts
(12-26)

- 400 -
当 iL Ts
 i1 Ts
 ic Ts
时,CPM 模式下的 Boost 变换器的平均开关周期模型

如图 12-7 所示:

iL Ts L
p Ts

C
vg ic Ts R v
Ts Ts

图 12-7 CPM 模式下的 Boost 变换器平均开关周期模型


(b) 将周期平均的量表达成稳态量与交流小信号扰动量,即:
v1 Ts
 V1  vˆ1
ic Ts
 I c  iˆc
(12-27)
v2 Ts
 V2  vˆ2
i2 Ts
 I 2  iˆ2

将式(12-27)代入到式(12-26)中可得:

V1  v1   Ic  iˆc   V2  v2   I 2  iˆ2  (12-28)

则式(12-28)的可变形为:
V1I c    I c vˆ1  V1iˆc   iˆc vˆ1   V2 I 2    I 2vˆ2  V2iˆ2   iˆ2vˆ2  (12-29)

则交流小信号方程为:

I c vˆ1  V1iˆc  I 2vˆ2  V2iˆ2 (12-30)

根据式(12-30)可求出:
I I V
iˆ2  c vˆ1  2 vˆ2  1 iˆc (12-31)
V2 V2 V2

由于有:
V1
 1 D
V2
I2 1

V2 R (12-32)
I c I1 I2 1
  
V2 V2 1  D  V2 1  D  R
vˆ1  vˆg  sLiˆc
将式(12-32)代入到式(12-31)中可得:
vˆg  Dvˆ2   D2 R  sL  iˆc 
1
iˆ2  (12-33)
DR  

- 401 -
故根据式(12-33)可画出 CPM 模式下 Boost 变换器的交流小信号模型如图
12-8 所示:

iˆL L iˆ2
 sL 
D 1  2  iˆc
 D R  1 C
vˆg iˆc R vˆg R v
DR
Ts

图 12-8 CPM 模式下 Boost 变换器的交流小信号模型

(c) 对于 Gvc ( s) 的推导,图 12-8 可简化为如图 12-9 所示:

 sL 
D 1  2  iˆc
 D R  C
R R v Ts

图 12-9 Gvc ( s) 的推导

有:
1 1 1
  sC 
Z R R
(12-34)
 sL 
vˆ  ZD 1  2  iˆc
 D R 
故可求出:
vˆ D2 R  sL
Gvc ( s )   (12-35)
iˆc 2 D  CDRs

将 Gvc ( s) 变形为标准形式:

s
1
z
Gvc ( s )  Gvc 0
s
1
p
2
p  (12-36)
CR
D2 R
z 
L
DR
Gvc 0 
2

对于 Gvg ( s) 的推导,图 12-8 可简化为如图 12-10 所示:

- 402 -
iˆ2

1 C
R vˆg R v
DR
Ts

图 12-10 Gvg ( s) 的推导

故有:
1 1 1
  sC 
Z R R (12-37)
1
vˆ  vˆg Z
DR
根据式(12-37)可求得:
1
Gvg (s)  (12-38)
2D  CDRs
将 Gvg ( s) 写成标准形式为:

1
Gvg ( s)  Gvg 0
s
1
p
1
Gvg 0  (12-39)
2 D
2
p 
CR

Problem 12.3

使用平均开关周期建模法对 CCM 模式下电流型控制的 Cuk 变换器进行建模。


Cuk 变换器如书中 Fig2.20 所示。
(a) 如果想要使用书中 12.2.2 节中所述的方法对开关网络进行建模,那么如
何定义端口的电压和电流?
(b) 画出开关网络的端口电压电流波形。假设有 i1 (t ) Ts
 i2 (t ) Ts
 ic (t ) 时(此

式电流方向的定义对应书中图 Fig2.20,与图 12-11 定义的方向正好相反),推导


端口波形的周期值表达式。推导类似书中 Fig12.18 所示的等效电路。
(c) 扰动并线性化(b)中的模型,来获取类型书中 Fig12.19 所示的等效电路。
Solution:
(a) Cuk 变换器的端口定义如图 12-11 所示:

- 403 -
vc1
iL1 L1 iL 2 L2

i1 C1

Vg C2 R v
v1 v2

i2

图 12-11 Cuk 变换器的端口定义


(b) 开关网络的端口波形如图 12-12 所示:
v1 v2
vc1 vc1

t t

i1 i2
iL1  iL 2 iL1  iL 2

0 dTs Ts t 0 dTs Ts t

图 12-12 开关网络的端口波形
故端口波形的周期平均值为:
v1 Ts
 d  vc1 Ts

v2 Ts
 d vc1 Ts
(12-40)
i1 Ts
 d iL1  iL 2 Ts

i2 Ts
 d  iL1  iL 2 Ts

由于书中 Fig2.20 所定义的电流方向与图 12-11 所定义的方向正好相反,故


有:
iL1  iL 2 Ts
 ic Ts
(12-41)

根据式(12-40)可得:
d
v1 Ts
 v2 T
d s
(12-42)
d
i2 Ts
 i1 T
d s

根据式(12-42)可得:

- 404 -
v1 Ts
i1 Ts
 v2 Ts
i2 Ts
 p(t ) Ts
(12-43)
v1 Ts
d ic Ts
 v2 Ts
i2 Ts
 p(t ) Ts

故可画出如书中 Fig12.18 的等效电路,如图 12-13 所示:


vc1
iL1 L1 iL 2 L2
i1 C1

Vg C2 R v
v1 v2
d ic Ts

i2

图 12-13 CPM 模式下的 Cuk 变换器平均周期模型


(c) 将周期平均量表达成稳态量与交流小信号扰动的和,即:
v1 T  V1  vˆ1
s

ic Ts
 I c  iˆc
v2 Ts
 V2  vˆ2 (12-44)
i2 Ts
 I 2  iˆ2

d (t )  D  dˆ
将式(12-44)代入到式(12-43)中,并提取线性的小信号交流分量,可得:
DI vˆ  I V dˆ  DV iˆ  I vˆ  V iˆ
c 1 C 1 1c 2 2 2 2

DI I I V DV
iˆ2  c vˆ1  2 vˆ2  C 1 dˆ  1 iˆc (12-45)
V2 V2 V2 V2
iˆ  Diˆ  I dˆ
1 c c

根据式(12-45)可画出 CPM 模式下 Cuk 变换器的小信号交流模型,如图


12-14 所示:
vc1
iˆL1 L1 C1 iL 2 L2

iˆ1 i2

vˆg C2
DI c V2
v̂1 Diˆc I c dˆ vˆ1 I CV1 ˆ
d
DV1 ˆ
ic v̂2
R v
V2 I2 V2 V2

图 12-14 CPM 模式下 Cuk 变换器交流小信号模型

- 405 -
Problem 12.4

如图 12-15 所示的全桥变换器,其输入电压 Vg  320V ,提供 1000 W 的功

率输出给 42V 的阻性负载。损耗可以全部忽略,占空比 D  0.7 ,开关周期


Ts  10μs ,电感 L  50μH ,电容 C  100μF 。此变换器使用电流型控制策略,你

可以忽略变压器的激磁电感。
(a) 能使变换器在给定运行点处稳定运行的最小人工载波斜率 ma 为多少?用

m2 来表达。

(b) 当 ma  m2 时,画出电流环路增益 Ti ( s ) 的 Bode 图,推导转折频率 f c 。

(c) 当 ma  m2 时,画出从控制到输出的传递函数 Gvc ( s) 和从交流输入到输出

的传递函数 Gvg ( s) 的 Bode 图,标出转折频率和 DC 增益。

Q1 D1 Q3 D3
1: n D5 L i (t )
vL
vg vT vs C R v
D6
:n
Q2 D2 Q4 D4

图 12-15 全桥变换器
Solution:
(a) 电感电流 i (t ) 的波形如图 12-16 所示:

其中电感电流上升下降的斜率为:
nvg  v
m1 
L (12-46)
v
m2 
L
当加入人工载波后,对于电感电流的任一扰动,经过一个开关周期后,有:
 m  m2 
iL (Ts )   a  iL (0) (12-47)
 m1  ma 

- 406 -
vL
nvg  v
0 t

v
iL
m1 m2

0 DTs Ts t

图 12-16 电感电流波形
为了使电流型控制的全桥变换器在给定点处稳定运行,故有:
ma  m2
1 (12-48)
m1  ma

当人工载波的斜率小于 m2 时,即:

ma  m2 (12-49)

则根据式(12-48)可得:
m2  m1
 ma (12-50)
2
m  m1
故人工载波的斜率最小值为 2 。
2
(b) 当考虑到电感电流纹波和人工载波的影响后,电感电流的周期平均值为:
m dT m d T
iL T  ic T  dTs ma  d 1 s  d  2 s (12-51)
s s
2 2
将周期平均量表达成稳态量与交流小信号扰动量的和,即:

iL Ts
 I L  iˆL
ic Ts
 I c  iˆc

d (t )  D  dˆ
(12-52)
d (t )  1  D  dˆ
m1 Ts
 M 1  mˆ 1
m2 Ts
 M 2  mˆ 2

将式(12-52)代入到式(12-51),并提取出线性的 1 阶分量,可得:

- 407 -
D Ts 2
D Ts 2
iˆL  iˆc   M aTs  DM 2Ts  DM1Ts  dˆ  mˆ 1  mˆ 2 (12-53)
2 2
对于稳态量,即当电感电流在一个周期没有净变化时,有:
M 1 D  M 2 D (12-54)

将式(12-54)代入到式(12-53)中,可有:

D Ts 2
D Ts 2
iˆL  iˆc  M aTs dˆ  mˆ 1  mˆ 2 (12-55)
2 2
通过式(12-55)可求出:
1  D 2Ts D2Ts 
dˆ  c L
i  i  ˆ
m 1  mˆ 2  (12-56)
M aTs  2 2 
将式(12-46)代入到式(12-56)中,可得:
dˆ  Fm  ic  iL  Fg vˆg  Fv vˆ 
1
Fm 
M aTs
(12-57)
nD 2Ts
Fg 
2L

Fv 
1  2 D  Ts
2L
根据式(12-57)可画出 CPM 模式下的控制器等效电路框图,如图 12-17 所示:

vˆg

Fg

iˆc d̂
Fm

iˆL Fv

图 12-17 CPM 控制等效电路框图

接下来推导 Gvd 、 Gvg 、 Gid 和 Gig 。

当 0  t  dTs 时,有:

- 408 -
diL
L  nvg  v
dt
di v
C c  iL  (12-58)
dt R
ig  niL

当 dTs  t  Ts 时,有:

diL
L  v
dt
di v
C c  iL  (12-59)
dt R
ig  0

故有(下式中的 ic 是电容电流,不是控制信号):

   d v 
d iL
 d n vg  v
Ts
L Ts Ts
dt Ts

d vT v
 iL 
Ts
C s
Ts
(12-60)
dt R
ig  dn iL Ts
Ts

将周期平均形式的量表达成稳态量与交流小信号扰动量的和:
iL Ts
 I L  iˆL
ic Ts
 I c  iˆc

d (t )  D  dˆ
(12-61)
d (t )  1  D  dˆ
vg  Vg  vˆg
Ts

v Ts
 V  vˆ

将式(12-61)代入到式(12-60)中,有:


d I L  iˆL   nDV   ˆ ˆ ˆ   ˆˆ 
L  g  V    Dnvg  v  nVg d    ndvg 
dt
d V  vˆ  V  vˆ
C  I L  iˆL  (12-62)
dt R
I g  iˆg   nDI L    nDiˆL  nI L dˆ    ndi
ˆˆ 
L

故全桥变换器的交流小信号方程为:

- 409 -
diˆL
L  Dnvˆg  vˆ  nVg dˆ
dt
dvˆ ˆ vˆ
C  iL  (12-63)
dt R
iˆg  nDiˆL  nI L dˆ

根据式(12-63)可画出全桥变换器的交流小信号等效电路如图 12-18 所示:


iˆg nVg dˆ L iˆL
vL

vˆg nI L dˆ C R v̂

1: nD
图 12-18 全桥变换器交流小信号等效电路

对于 Gvd 的推导,令 vˆg  0 ,则图 12-18 可简化为如图 12-19 所示:

nVg dˆ L iˆL
vL

C R v̂

图 12-19 Gvd 的推导

故有:
nVg dˆ vˆ

sL  Z1 Z1
(12-64)
1 1
 sC 
Z1 R
根据式(12-64)可求得:
nRVg
Gvd  (12-65)
CLRs 2  Ls  R

对于 Gvg 的推导,令 dˆ  0 ,则图 12-18 可简化为如图 12-20 所示:

- 410 -
L iˆL
vL

nDvˆg C R v̂

图 12-20 Gvg 的推导

故有:
nDvˆg vˆ
 (12-66)
sL  Z1 Z1

根据式(12-66)可得:
nDR
Gvg  (12-67)
CLRs 2  Ls  R
对于 Gid 的推导,根据图 12-19 可得:

nVg dˆ
iˆL  (12-68)
sL  Z1
根据式(12-68)可得:

nVg  CRs  1
Gid  (12-69)
CLRs 2  Ls  R

对于 Gig 的推导,根据图 12-20 可得:

nDvˆg
iˆL  (12-70)
sL  Z1

故根据式(12-70)可得:

nD  CRs  1
Gig  (12-71)
CLRs 2  Ls  R
CPM 模式下,全桥变换器的交流小信号等效电路框图如图 12-21 所示:

- 411 -
vˆg

Fg
Gig

iˆc d̂ iˆL
Fm Gid

iˆL Fv Gvd
v̂ v̂
Gvg

图 12-21 CPM 模式下全桥变换器的交流小信号等效电路框图

故电流环路增益 Ti ( s ) 为:

Ti ( s)  FmGid (12-72)

其 Bode 图如图 12-22 所示:


Bode Diagram
30
Ti
25

20
Magnitude (dB)

15

10

0
45
Ti

0
Phase (deg)

-45

-90
2 3 4 5
10 10 10 10
Frequency (rad/s)

图 12-22 电流环路增益 Ti ( s )

(c) 对于 Gvc 的推导,令 vˆg  0 ,则图 12-21 可简化为如图 12-23 所示:

- 412 -
iˆc d̂ iˆL
Fm Gid

iˆL Fv v̂
Gvd

图 12-23 Gvc 的推导

将图 12-23 进行变形,如图 12-24 所示:


iˆc d̂ iˆL
Fm Gid

iˆL Fv v̂
Gvd

iˆc d̂ iˆL
Fm Gid
iˆL
Fv v̂
Gvd

iˆc d̂ iˆL
Fm Gid
iˆL
Fv Gvd v̂
Gid

iˆc d̂ iˆL Gvd v̂


Fm Gid
Gid
iˆL
Fv

图 12-24 Gvc 的变形推导

根据图 12-24 可求得:


FmGvd
Gvc  (12-73)
FmGid  Fm FvGvd  1

- 413 -
Gvc 的 Bode 图如图 12-25 所示:

Bode Diagram
50
Gvc

0
Magnitude (dB)

-50

-100
0
Gvc

-45
Phase (deg)

-90

-135

-180
2 3 4 5 6 7
10 10 10 10 10 10
Frequency (rad/s)

图 12-25 Gvc 的 Bode 图

对于 Gvg cpm 的推导,根据式(12-57)及下式:

vˆ  Gvd dˆ  Gvg vˆg


(12-74)
iˆL  Gid dˆ  Gig vˆg
可求出:
Gvg  Fg FmGvd  FmGid Gvg  FmGig Gvd
Gvg cpm  (12-75)
1  Fm Fv Gvd  FmGid

Gvg cpm 的 Bode 图如图 12-26 所示:

Bode Diagram
0
Gvg_cpm

-50
Magnitude (dB)

-100

-150
0
Gvg_cpm

-45
Phase (deg)

-90

-135

-180
2 3 4 5 6 7
10 10 10 10 10 10
Frequency (rad/s)

图 12-26 Gvg cpm 的 Bode 图

- 414 -
Problem 12.5

在一个 CCM 模式下的电流型控制的 Buck 变换器中,通过 ma  0.5m2 来使从

交流输入到输出的传递函数的增益 Gvg 最小,然而,由于器件值存在误差,电感

值 L 相对于其标称值 100μH 会有 10% 的偏离。因此, m2 是变化的,但是 ma 确

是固定的, ma 的取值为 0.5m2 ,这里的 m2 是在 L 为其标称值时取得的。开关频

率为 100kHz ,输出电压为 15V ,负载电流的变换范围为 2A 4A ,输入电压的


变化范围为 22V 32V 。你可以忽略全部损耗,需要推导出最恶劣情况下的从交
流输入到输出的传递函数的 DC 增益,即 Gvg (0) 。

Solution:
从交流输入到输出的传递函数 Gvg 的 DC 增益为:

Gvg (0)  lim Gvg cpm ( s ) (12-76)


s 0

而对于 Gvg cpm (s) ,根据书中 Table12.3 有:

1
Gvg cpm ( s )  Gg 0 2
 s 
s
1  
Qcc  c 
 Fm FgV  (12-77)
1  
 D2 
Gg 0  D
 FmV Fm FvV 
1   
 DR D 
根据式(12-76)和式(12-77)可得:
Gvg (0)  Gg 0 (12-78)

对于 CPM 模式下的 Buck 变换器,有:

- 415 -
1
Fm 
M aTs
D 2Ts
Fg 
2L

Fv 
1  2 D  Ts (12-79)
2L
M a  0.5M 2
V
M2 
L
将式(12-79)代入到式(12-77)中,可得:
TsV 3  L  Lm 
Gg 0  (12-80)
2 I Load LLmVg2  LTsV 2Vg  LmTsVVg2  2 LmTsV 2Vg

将元器件的值代入到式(12-80),可得:
675 10000 L  1
Gg 0  (12-81)
Vg  450000 L  3Vg  40000 I Load LVg  90 

通过式(12-81)可知,最恶劣情况下的 Gg 0 发生在负载电流最小时,即发生在:

I Load  2A (12-82)

将式(12-82)代入到式(12-81)中,可得:
675 10000 L  1
Gg 0  (12-83)
Vg  450000 L  3Vg  80000 LVg  90 

根据式(12-83)可知,最恶劣情况下的 Gg 0 发生在输入电压处于最小值时,即:

Vg  22V (12-84)

将式(12-84)代入到式(12-83)中,可得:
6750000L  675
Gg 0  (12-85)
48620000L  528
式(12-85)如图 12-27 所示:

- 416 -
0.015

0.0106

3
6.2510

3
1.87510

6750000 t  675 3
 2.510
48620000 t  528
3
 6.87510

 0.0113

 0.0156

 0.02
5 5 5 5 4 4 4 4 4
910 9.2510 9.510 9.7510 110 1.02510 1.0510 1.07510 1.110
t

图 12-27 Gg 0 的图形

故可知最恶劣情况下的 Gvg (0) 在 L  90μH 处获得,其值为:

Gvg cpm (0)  0.018  34.89dB (12-86)

Problem 12.6

如图 12-28 所示的使用 CPM 控制的非理想反激变换器,其人工载波斜率为


ma 。MOSFET 的导通电阻为 Ron 。针对原边侧电流使用 CPM 控制。

(a) 推导类似书中 Fig12.24 所示的 CPM 模式下的方框图,并给出方框图中的


传递函数的表达式。
(b) 将变换器的小信号模型和(a)得到的框图结合起来,推导 Gvc (s) 的表达式。

ig (t ) 1: n D1

iC
L
C R v(t )
vg (t )

Q1

图 12-28 非理想反激变换器
Solution:
(a) 对于非理想反激变换器,其等效电路如图 12-29 所示:

- 417 -
ig (t ) 1: n D1
iL iC

vT L C R v(t )
vg (t )

Q1

图 12-29 非理想反激变换器的等效电路
当考虑到电感电流纹波和人工载波的影响后,电感电流的周期平均值为:
m dT m d T
iL T  ic T  dTs ma  d 1 s  d  2 s (12-87)
s s
2 2
式(12-87)的交流小信号方程为:

D Ts 2
D Ts 2
iˆL  iˆc  M aTs dˆ  mˆ 1  mˆ 2 (12-88)
2 2
其中:

vˆg  iˆL Ron


mˆ 1 
L (12-89)

mˆ 2 
nL
将式(12-89)代入到式(12-88)中,可得:
1   D2 RonTs  D 2Ts D2Ts 
dˆ  ˆ 
c 
i 1  Lˆ
i  ˆ
v g  vˆ  (12-90)
M aTs   2L  2L 2Ln 
将式(12-90)进行变形,可有:

dˆ  Fm iˆc  FLiˆL  Fg vˆg  Fv vˆ 


1
Fm 
M aTs
 D 2 RonTs 
FL  1   (12-91)
 2L 
D 2Ts
Fg 
2L
D2Ts
Fv 
2 Ln
故考虑了 MOSFET 导通电阻影响的 CPM 控制器的框图如图 12-30 所示:

- 418 -
vˆg

Fg

iˆc d̂
Fm

FL Fv

iˆL v̂

图 12-30 考虑了 MOSFET 导通电阻影响的 CPM 控制器的框图


(b) 非理想反激变换器的小信号等效电路已由书中 Fig7.27 所示,这里重新给
出,如图 12-31 所示:

 V ˆ
 Vg   Ron I  d (t )
iˆg (t ) iˆ(t ) L  n 

Idˆ (t )
vˆg (t ) Idˆ (t ) C R vˆ(t )
DRon n

1: D D
:1
n
图 12-31 非理想反激变换器的小信号等效电路
则 CPM 控制下的反激变换器的小信号等效电路框图如图 12-32 所示:

vˆg

Fg
Gig

iˆc d̂ iˆL
Fm Gid
iˆL
FL Fv Gvd
v̂ v̂
Gvg

图 12-32 CPM 控制下的反激变换器的小信号等效电路框图


根据式(12-91)和下式:

- 419 -
vˆ  Gvd dˆ  Gvg vˆg
(12-92)
iˆL  Gid dˆ  Gig vˆg
可得:
FmGvd
vˆ  iˆc 
FL FmGid  Fm FvGvd  1
(12-93)
Gvg  Fg FmGvd  FL FmGid Gvg  FL FmGig Gvd
 vˆg
FL FmGid  Fm FvGvd  1

故 Gvc ( s) 为:

FmGvd
Gvc ( s)  (12-94)
FL FmGid  Fm FvGvd  1

下面推导 Gvd 和 Gid ,首先令 vˆg  0 ,则图 12-31 可简化为如图 12-33 所示:

 V ˆ
 Vg   Ron I  d (t )
 n 
iˆ(t ) L

Idˆ (t )
C R vˆ(t )
DRon n

D
:1
n

图 12-33 vˆg  0 时的非理想反激变换器的小信号等效电路

根据叠加定理,当单独考虑电压源时,并将副边侧等效至原边时,图 12-33
可简化为如图 12-34 所示:
 V ˆ
 Vg   Ron I  d (t )
iˆ(t ) L  n 

 D 
2
D
  Z vˆ(t )
DRon  n  n

图 12-34 vˆg  0 时且只考虑电压源时的非理想反激变换器的小信号等效电路

其中:
R
Z (12-95)
CRs  1
故有:

- 420 -
 V 
 Vg   Ron I  dˆ
D   D  Z
2

ˆv(t )  
n
 
 D   n 
2
n
sL    Z  DRon
 n 
(12-96)
 V ˆ
 Vg   Ron I  d
iˆ   
n
 D 
2

sL    Z  DRon
 n 
根据式(12-96)可求出:

DR V  nVg  IRon n 


Gvdv 
CLRn 2 s 2   Ln2  CDRRon n2  s  D2 R  DRon n 2
(12-97)
n  CRs  1 V  nVg  IRon n 
Gidv 
CLRn 2 s 2   Ln2  CDRRon n2  s  D2 R  DRon n 2

当单独考虑电流源时,并将原边侧等效至副边时,图 12-33 可简化为如图


12-35 所示:

iˆ2

Idˆ (t )
2
 n 
Z1   C R vˆ(t )
 D  n

图 12-35 vˆg  0 时且只考虑电流源时的非理想反激变换器的小信号等效电路

其中:
Z1  sL  DRon (12-98)

故有:
1 1 1
 2
 sC 
Z2  n  R
Z1  
 D 
Idˆ
Z 2  vˆ
n (12-99)
iˆ n

iˆ2 D
 n  ˆ Idˆ
2

Z1   i2  Z 2
 D  n
根据式(12-99)可求得:

- 421 -
 ILRns  DIRRon n
Gvdi 
CLRn s   Ln 2  CDRRon n 2  s  D2 R  DRon n 2
2 2

(12-100)
DIR
Gidi 
CLRn s   Ln  CDRRon n 2  s  D2 R  DRon n 2
2 2 2

根据式(12-100)和式(12-97)可求得:
Gvd  Gvdv  Gvdi 
 R  DIRon n  DVg n  DV  DIRon n  ILns 

CLRn 2 s 2   Ln 2  CDRRon n 2  s  D2 R  DRon n 2
(12-101)
Gid  Gidv  Gidi 


V ng
2
 Vn  IRon n 2  DIR  CRVg n 2 s  CRVns  CIRRon n 2 s 
CLRn 2 s 2   Ln 2  CDRRon n 2  s  D2 R  DRon n 2

对于稳态分量有:
 V
D Vg  IRon   D     0
 n
(12-102)
 V I V
D     D     0
 R n R
根据式(12-102)可得:
nV
I
DR
(12-103)
 nV DV 
Vg  Ron   
 DR DRon n 
将式(12-103)代入到式(12-101)中,可得:
Gvd 
RVD2  RonVD 2 n 2  LVDn 2 s
(12-104)
CLRDDn 2 s 2   LDDn 2  CRRon D 2 Dn 2  s  Ron D 2 Dn 2  RDD3

Gid 
1  D Vn  CRVns
CLRDn 2 s 2   LDn 2  CRRon D 2 n 2  s  Ron D 2 n 2  RDD2

将式(12-91)和式(12-104)代入到式(12-94)中,即可求出 Gvc 的解析表达式。

Problem 12.7

使用 CPM 控制策略的 Buck 变换器,其器件参数值为:

- 422 -
Vg  120VD  0.6
R  10 f s  100kHz
L  550μHC  100μF

添加的人工载波的斜率为 0.15A/μs 。

(a) 构建占空比控制模式下的从控制到输出的传递函数 Gvd ( s) 的 Bode 图。在

此 Bode 图上,构建 CPM 模式下,从控制到输出的传递函数 Gvc ( s) 的 Bode 图。

(b) 构建占空比控制模式下的从交流输入到输出的传递函数 Gvg ( s) 的 Bode 图。

在此 Bode 图上,构建 CPM 模式下,从交流输入到输出的传递函数 Gvg cpm (s) 的

Bode 图。
Solution:
(a) Gvd ( s) 和 Gvc ( s) 的表达式如书中 Table12.3 所述,这里重新给出:

V 1
Gvd 
D 1  s L  s 2 LC
R
1
Gvc  Gc 0 2
s  s 
1  
Qcc  c 
V Fm
Gc 0 
D  FmV Fm FvV 
1  DR  D 
 
FmV Fm FvV
1 
C DR D
Qc  R
L  RCFmV 
1  DL 
 
1 F V F FV
c  1 m  m v
LC DR D (12-105)
对于 Buck 变换器,根据书中 Table12.2 有:
D 2Ts
Fg 
2L

Fv 
1  2 D  Ts (12-106)
2L
1
Fm 
M aTs

- 423 -
将器件参数值和式(12-106)代入到式(12-105)中可得:
120
Gvd 
5.5 10 s  5.5 105 s  1
8 2
(12-107)
9.035
Gvc 
6.21110 s  9.097 104 s  1
9 2

其 Bode 图如图 12-36 所示:


Bode Diagram
100
Gvd
Gvc
50
Magnitude (dB)

-50

-100
0
Gvd
Gvc
-45
Phase (deg)

-90

-135

-180
1 2 3 4 5 6 7
10 10 10 10 10 10 10
Frequency (rad/s)

图 12-36 Gvd 和 Gvc 的 Bode 图

(b) Gvg ( s) 和 Gvg cpm (s) 的表达式如书中 Table12.3 所述,这里重新给出:

1
Gvg  D
L 2
1 s  s LC
R
1
Gvg cpm  Gg 0 2
(12-108)
 s  s
1  
Qcc  c 
F FV
1  m 2g
Gg 0  D D
 FmV Fm FvV 
1  DR  D 
 
将器件参数值和式(12-106)代入到式(12-108)中,可得:
0.6
Gvg 
5.5 10 s  5.5 105 s  1
8 2
(12-109)
0.03819
Gvg cpm 
6.21110 s  9.097 104 s  1
9 2

其 Bode 如图 12-37 所示:

- 424 -
Bode Diagram
50
Gvg
Gvg_cpm
0

Magnitude (dB)
-50

-100

-150
0
Gvg
Gvg_cpm
-45
Phase (deg)

-90

-135

-180
1 2 3 4 5 6 7
10 10 10 10 10 10 10
Frequency (rad/s)

图 12-37 Gvg 和 Gvg cpm 的 Bode 图

Problem 12.8

某 Buck-Boost 变换器运行于 DCM 模式,使用 CPM 控制方式,并且不添加


人工载波,即 ma  0 。

(a) 使用 L  0 这个近似条件,推导从控制到输出的传递函数 Gvc ( s) 的表达式,

给出转折频率和 DC 增益的表达式。
(b) 当考虑电感值的影响后,再次重复(a)。证明当电感值足够小时,电感对
Gvc ( s) 的影响仅仅是添加了一个高频极点和高频零点,此外(a)小问推导出的

低频极点并不会受到电感值的影响。
(c) 当变换器运行于 CCM 模式和 DCM 模式的边界时,RHP 平面的零点频率
的最小值是多少?
Solution:
(a) Buck-Boost 变换器如图 12-38 所示:

i1 Q1 D1 i2

v2
v1
Vg L C R v

图 12-38 Buck-Boost 变换器

对于 Buck-Boost 变换器的控制信号 ic 有:

ic  i pk  m1dTs (12-110)

根据式(12-110)可求得:

- 425 -
ic
d (12-111)
m1Ts

而:
vg
m1  (12-112)
L

晶体管电流的平均值 i1 Ts
为:

1
 i pk d i1 Ts
(12-113)
2
将式(12-111)和式(12-112)代入到式(12-113)可得:
Lic2 f s
i1 Ts
 (12-114)
2 vg
Ts

对于始终处于 DCM 模式下的 Buck-Boost 变换器,由于电感电流在开关周期


的起点和末尾点均为 0,故电感两端电压的周期平均值为 0,即 vL Ts
 0 。故开

关管两端电压 v1 的周期平均值为:

v1 Ts
 vg  vL Ts
 vg (12-115)
Ts Ts

将式(12-115)代入到式(12-114)中,可得:
Lic2 f s
i1 Ts
 (12-116)
2 v1 T
s

故有:

Lic2 f s
p Ts
 v1 Ts
i1 Ts
 (12-117)
2

对于二极管电流的周期平均值 i2 Ts
,有:

1
i2 Ts
 i pk d 2 (12-118)
2
而对于 d 2 有:

vg  v vg
d d2  d2  
Ts Ts Ts
d (12-119)
L L v Ts

由于:
v2 Ts
 vL Ts
 v Ts
 v Ts
(12-120)

- 426 -
将式(12-120)代入到式(12-119)中,可得:
v1
d2 
Ts
d (12-121)
v2 Ts

将式(12-113)和式(12-121)代入到(12-118)中,可得:
v2 Ts
i2 Ts
 v1 Ts
i1 Ts
 p Ts
(12-122)

故晶体管可被一个功率吸收源等效,而二极管可被一个功率输出源等效。
根据式(12-122)和式(12-116)可得:
v1 i1 Lic2 f s
 
Ts Ts
i2 Ts
(12-123)
v2 Ts
2 v2 T
s

根据式(12-116)和式(12-123)可有:
Lic2 f s
i1 Ts
 f1 (ic , v1 ) 
2 v1 T
s

2
(12-124)
Li f
i2 Ts
 f 2 (ic , v2 )  c s

2 v2 T
s

故根据式(12-124)可画出开关网络的交流小信号等效电路形式,如图 12-39
所示:
iˆ1 iˆ2

v̂1 r1 f1iˆc f 2iˆc r2 v̂2

图 12-39 开关网络的交流小信号等效电路形式
故有:

1 f1 ( I c , v1 ) 2V 2
  r1   1 2
r1 v1 v V
Lf s I c
1 1

f1 (ic ,V1 ) I Lf


f1   f1  c s
ic i I
V1
c c
(12-125)
1 f 2 ( I c , v2 ) 2V22
   r2 
r2 v2 v V
Lf s I c2
2 2

f 2 (ic ,V2 ) I Lf
f2   f2  c s
ic i I
V2
c c

而对于稳态量有:

- 427 -
V22 LI c2 f s

R 2
V2
 M
V1 (12-126)
2
LI f
V1 I1  V2 I 2  c s

2
I1   MI 2
将式(12-126)代入到式(12-125)中,可得:
R
r1  
M2
2I
f1  1
Ic (12-127)
r2  R
2I2
f2 
Ic
故可画出 DCM 模式下,使用 CPM 控制策略的 Buck-Boost 变换器的交流小
信号等效电路如图 12-40 所示:

iˆ1 iˆ2

v̂1 r1 f1iˆc f 2iˆc r2 v̂2

vˆg
C R v̂
L

图 12-40 DCM 模式下,使用 CPM 控制策略的 Buck-Boost 变换器的交流小信号等效电路


当使用 L  0 这个近似条件时,则图 12-40 可继续简化为如图 12-41 所示:
iˆ1 iˆ2

vˆg v̂1 r1 f1iˆc f 2iˆc r2 v̂2 C R v̂

图 12-41 L  0 时的 Buck-Boost 变换器交流小信号等效电路


故有:
1 1 1
  sC 
Z r2 R (12-128)
f 2iˆc Z  vˆ

- 428 -
根据式(12-128)可得:
vˆ Rf 2 r2 Rf r 1 1
Gvc    2 2  Gvc 0
iˆc R  r2  CRr2 s R  r2 1
s
1
s
R  r2 p
CRr2
Rf 2 r2
Gvc 0   (12-129)
R  r2
R  r2
p 
CRr2
将式(12-127)代入到式(12-129)可得:
I2
Gvc 0   R
Ic
(12-130)
2
p 
CR
故根据式(12-129)可得:
I2
 R
Ic
Gvc  (12-131)
s
1
2
CR
(b) 当考虑电感值的影响时,对于 Gvc 的推导,首先令 vˆg  0 ,则图 12-40 可

简化为如图 12-42 所示:

iˆ1 iˆ2

v̂1 r1 f1iˆc f 2iˆc r2 v̂2

C R v̂
L

图 12-42 vˆg  0 的 Buck-Boost 变换器交流小信号等效电路

再根据叠加定理,首先仅考虑左边的电流源起作用时,则图 12-42 可继续简


化为如图 12-43 所示:

- 429 -
iˆ1 iˆ2

v̂1 r1 f1iˆc r2 v̂2

C R v̂
L

图 12-43 仅考虑 f1iˆc 时的 Buck-Boost 变换器交流小信号等效电路

将图 12-43 进行变形,如图 12-44 所示:

r2

f1iˆc r1 L C R v̂

图 12-44 仅考虑 f1iˆc 时的 Buck-Boost 变换器交流小信号简化等效电路

故有:
1 1 1
 
Z1 r1 sL
1 1
 sC 
Z2 R
(12-132)
1 1 1
 
Z 3 Z1 r2  Z 2

 r2  Z 2   f1iˆc Z3
Z2
根据式(12-132)可得:
LRf1r1s
Gvc1  (12-133)
 CLRr1  CLRr2  s   Lr1  Lr2  LR  CRr1r2  s  Rr1  r1r2
2

当仅考虑右边的电流源 f 2iˆc 时,图 12-42 可简化为如图 12-45 所示:

- 430 -
iˆ1 iˆ2

v̂1 r1 f 2iˆc r2 v̂2

C R v̂
L

图 12-45 仅考虑 f 2iˆc 时的 Buck-Boost 变换器交流小信号等效电路

将图 12-45 进行变形,如图 12-46 所示:

C R v̂

f 2iˆc r2
r1 L

图 12-46 仅考虑 f 2iˆc 时的 Buck-Boost 变换器交流小信号简化等效电路

故有:
1 1 1
 
Z1 r1 sL
1 1
 sC 
Z2 R
(12-134)
1 1 1
 
Z 4 r2 Z1  Z 2

 f 2ic Z 4   Z1  Z 2 
Z2
根据式(12-134)可求得:
 Rf 2 r2  r1  Ls 
Gvc 2  (12-135)
 CLRr1  CLRr2  s   Lr1  Lr2  LR  CRr1r2  s  Rr1  r1r2
2

根据式(12-133)和式(12-135)可得:
Gvc  Gvc1  Gvc 2 
 R  f 2 r1r2  Lf1r1s  Lf 2 r2 s  (12-136)
 CLRr1  CLRr2  s 2   Lr1  Lr2  LR  CRr1r2  s  Rr1  r1r2
将式(12-127)代入到式(12-136)可得:

- 431 -
Gvc 
 2I LM R  2I LR  s  2I R
2
2
1 2
2

(12-137)
CI LR  CI LRM  s  CI R  2I LM  I L  s  2I R
c c
2 2
c
2
c
2
c c

将式(12-137)进行变形,可得:
s
1
2I2 R2

Gvc 
I2 R  2I 2 LM 2 R  2I1LR  (12-138)
Ic s2 s
 1
2Ic R 2Ic R
 CI c LR  CI c LRM 2  CI c R 2  2I c LM 2  I c L 
故有:
2I2 R2
z 
 2I 2 LM 2 R  2I1LR 
2Ic R
Q0  (12-139)
 CI c R  2I c LM 2  I c L 
2

2Ic R
02 
 CI c LR  CI c LRM 2 
根据式(12-139)可求得:

R 2CL 1  M 2 
Q (12-140)
CR 2  2 LM 2  L

根据式(12-140)可知,当电感值足够小时,可知 Q 值也很小,即有:

R 2CL 1  M 2  2L 1  M 2 
Q  (12-141)
CR 2  2 LM 2  L CR

当 Q 值很小时,即可使用小 Q 近似来获得两个近似的极点,即:

2Ic R

 p1 
0

 CI c LR  CI c LRM 2 

 CI c R 2  2 I c LM 2  I c L 
Q 2Ic R  CI c LR  CI c LRM 2 
 CI c R 2  2I c LM 2  I c L  (12-142)

2Ic R
 p 2  Q0 
 CI c R 2  2I c LM 2  I c L 
对于 DCM 模式的 Buck-Boost 变换器有:

- 432 -
D
M 
K
2L
K (12-143)
RTs
1
Ts 
fs
将式(12-143)代入到式(12-142)中的第 1 子式中,可得:
0 2Cf s R 2  2 D 2 R  2 Lf s
 p1  
Q CD 2 R 2  2CLRf s
(12-144)
2Ic R
p2  Q0 
 CIc R  2I c LM 2  Ic L 
2

当 L 足够小时,式(12-144)可近似为:
2Cf s R 2  2 D 2 R  2 Lf s 2 D 2  2Cf s R 2 2
 p1      2 fs
CD R  2CLRf s
2 2
CD R 2
CR D
(12-145)
2Ic R 2Ic R 2
p2   
 CIc R  2I c LM  Ic L  CI c R  CR
2 2 2

故可知当电感值足够小时,电感对 Gvc ( s) 极点的影响是额外增加了一个高频

极点。此外,电感对(a)小问推导出的低频极点并没有影响。
对于零点,将式(12-143)和式(12-126)的第 4 子式代入到式(12-139)中的第 1
子式,可得:

2 2 fs 2 fs
z   (12-146)
Lf s D2
2D2  2D
R

故可知当电感值足够小时,电感对 Gvc ( s) 零点的影响是额外增加了一个高频

零点。
(c) 当变换器处于 CCM 和 DCM 的边界时,有:
K  K crit ( D )
2L (12-147)
 1  D 
2

RTs

将式(12-147)代入到式(12-146)中可得:
2 2 fs
z  (12-148)
D2 1
2D  D   2D2
2 2

- 433 -
D2 1
为了求得 z 的最小值,只需求出 2 D  D   2 D 2 的最大值,将此式
2 2
的图形画出,如图 12-47 所示:
1.6

1.4

1.2

2
D 1 2
2 D D  2 D 0.8
2 2

0.6

0.4

0.2

0
0 0.125 0.25 0.375 0.5 0.625 0.75 0.875 1
D

D2 1
图 12-47 2 D  D   2 D 2 的图形
2 2

D2 1
故可知 2 D  D   2 D 2 的最大值在 D  1 处取得。令 D  1 ,则对于式
2 2
(12-148)有:
z _ min  2 f s (12-149)

Problem 12.9

某电流型控制的 Boost 变换器,使用 3V 的电池作为输入,向 5V 的小便携负


载提供能量。变换器运行于 DCM 模式,晶体管的导通时间 ton 为定值,其关断时

间可变,因此开关管的开关频率是变化的,故可作为控制变量。此变换器不添加
人工载波,晶体管的峰值电流 ic 等于固定值 I c 。实际中, I c 的选取以总损耗最小

为目的。
(a) 画出晶体管和二极管的电压电流波形,推导其平均值的表达式,故可获得
此变换器的大信号模型。
(b) 扰动和线性化(a)的结果,以获得小信号模型,注意开关频率 f s 也应该

扰动。
(c) 求解(b)的结果,以获得适用于低频段的从控制到输出的传递函数
Gvf ( s)  vˆ / fˆs 的表达式,并将其表达成标准形式,给出转折频率和 DC 增益的表

- 434 -
达式,你可以忽略电感值来进行简化。
Solution:
(a) Boost 变换器电路如图 12-48 所示:
L v2
i1 i2
Vg v1 C R v

图 12-48 Boost 变换器


晶体管和二极管的电压电流波形如图 12-49 所示:
v1 v2

v v
vg v  vg

0 t 0 t
i1 i2

Ic Ic

0 t 0 t
ton toff t0 ton toff t0
图 12-49 晶体管和二极管的电压电流波形
根据图 12-49 可求得晶体管电压电流的周期平均值表达式:
1 1
i1 Ts
 ton I c
Ts 2
(12-150)
1 1
i2 Ts
 toff I c
Ts 2
又由于:
vg
ton  I c
Ts

L (12-151)
vg  v Ts
toff   I c
Ts

L
根据式(12-151)可求得:
Ic L
ton 
vg
Ts
(12-152)
Ic L
toff 
v Ts
 vg
Ts

又由于变换器处于 DCM 模式,即 vL Ts


 0 ,因此有:

- 435 -
vg  v1 Ts
Ts
(12-153)
v Ts
 vg  v2 Ts
Ts

将式(12-153)代入到式(12-152)中,可得:
Ic L
ton 
v1 T
s
(12-154)
IL
toff  c
v2 T
s

将式(12-154)代入到式(12-150)可得:
1 1 Ic L
i1 Ts
 Ic
Ts 2 v1 T
s
(12-155)
1 1 Ic L
i2 Ts
 Ic
Ts 2 v2 T
s

根据式(12-155)可得:

f s I c2 L
v1 Ts
i1 Ts
 v2 Ts
i2 Ts
  p Ts
(12-156)
2
根据式(12-156)可得:
f s I c2 L
i1 Ts
  f1  f s , v1 
2 v1 T
s

2
(12-157)
fI L
i2 Ts
  f 2  f s , v2 
s c

2 v2 T
s

根据式(12-156)可画出 DCM 模式下,使用 CPM 控制策略的 Boost 变换器的


大信号模型,如图 12-50 所示:
v2
L
i1 i2
Vg v1 C R v
p Ts

图 12-50 DCM 模式下,使用 CPM 控制策略的 Boost 变换器的大信号模型


(b) 根据式(12-157)可画出开关网络的交流小信号等效电路的形式如图 12-51
所示:

- 436 -
iˆ1 iˆ2

v̂1 r1 f1 fˆs f 2 fˆs r2 v̂2

图 12-51 DCM 模式下,使用 CPM 控制策略的 Boost 开关网络的交流小信号模型


根据图 12-51 和式(12-157)可有:

1 f1 ( f s , v1 ) 2V 2
  r1   12
r1 v1 v V
Fs I c L
1 1

f ( f , v ) I c2 L
f1  1 s 1  f1 
f s f s  Fs
2V1
(12-158)
1 f ( f , v ) 2V22
  2 s 2  r2 
r2 v2 v V
Fs I c2 L
2 2

f 2 ( f s , v2 ) I c2 L
f2   f2 
f s f s  Fs
2V2

故 DCM 模式下,使用 CPM 控制策略的 Boost 变换器的交流小信号模型如


图 12-52 所示:

L
f 2 fˆs
i1
vˆg r1 f1 fˆs r2
C R v̂

图 12-52 DCM 模式下,使用 CPM 控制策略的 Boost 变换器的交流小信号模型


(c) 当认为电感值 L 很小时,即忽略电感的作用,则图 12-52 可简化为如图
12-53 所示:

f 2 fˆs
i1
vˆg r1 f1 fˆs r2
C R v̂

图 12-53 忽略电感时的 Boost 变换器的交流小信号模型

对于从控制到输出的传递函数 Gvf (s) 的推导,首先令 vˆg  0 ,则图 12-53 可

进一步简化为如图 12-54 所示:

- 437 -
f 2 fˆs

r2
C R v̂

图 12-54 忽略电感,且 vˆg  0 时的 Boost 变换器的交流小信号模型

根据图 12-54 可有:


1 1 1
  sC 
Z r2 R (12-159)
vˆ  f fˆ Z 2 s

根据式(12-159)和式(12-158)可求得:
I c2 LRV2
Gvf  (12-160)
2V22  Fs I c2 LR  2CRV22 s

将式(12-160)进行变形,可得:

I c2 LRV2 1
Gvf  (12-161)
2V22  Fs I c2 LR 1  s
2V2  Fs I c2 LR
2

2CRV22

Problem 12.10

某 Boost 变换器应用在低谐波的整流器系统,此系统的输入电压为整流后的
正弦波: vg  VM sin t  ,直流输出电压 v  V  VM 。输出电容的容值很大,使

得输出电压几乎不含有交流分量。需要对变换器进行控制,使得输入电流 ig (t ) 与

输入电压 vg (t ) 成正比:ig (t )  vg (t ) / Re ,这里称 Re 为模拟电阻。书中 Fig12.18(a)

表明可以让 ic (t ) 简单得与 vg (t ) 成正比,即 ic (t )  vg (t ) / Re ,以此达到输入电流 ig (t )

与输入电压 vg (t ) 成正比。你可以假设变换器始终处于 CCM 模式。

(a) 求解书中 Fig12.18(a)中的模型,推导平均功率 p Ts


的表达式,再求出 p Ts

在输入电压 vg 的一个周期内的平均值。

(b) 在某些运行点处,需要添加人工载波以使得系统稳定,那么使得系统稳定

- 438 -
的人工载波斜率 ma 的最小值为多少?以 V 、 L 为参数表达结果。

(c) 人工载波和电感电流纹波的存在会使得平均电流不等于控制电流 ic (t ) ,推

导 ig (t ) 的表达式,以 ic 、 ma 、 vg 、 V 、 L 和 Ts 的形式进行表达,你可以忽略
Ts

电感的动态影响。
(d) 将 vg (t )  VM sin t  和 ic (t )  vg (t ) / Re 代入到(c)中,推导 ig (t ) 的表达式。

ig (t ) 和整流后的正弦波有什么区别?

Solution:
(a) Boost 变换器电路及端口电量方向定义如图 12-55 所示:
L i1 i2

Vg v1 v2 C R v

图 12-55 Boost 变换器电路及端口电量方向定义


端口波形如图 12-56 所示:
v1 v2

v v
t t

i1 i2

0 dTs Ts t 0 dTs Ts t

图 12-56 端口电压电流波形
故端口电压电流波形周期平均值为:
v1 Ts
 d v Ts

v2 Ts
 v Ts
(12-162)
i1 Ts
 iL Ts

i2 Ts
 d  iL Ts

根据式(12-162)可得:

- 439 -
v1 Ts
 d  v2 Ts
(12-163)
i2 Ts
 d  i1 Ts

根据式(12-163)可得:
i2 Ts
v2 Ts
 v1 Ts
i1 Ts
 p Ts
(12-164)

而端口功率的周期平均值为:

p Ts
 vg i1 Ts
 vg iL Ts
 vg ic Ts

Ts Ts Ts
(12-165)

/ Re  VM sin t  
2 2
 vg / Re
Ts

端口功率的周期平均值 p Ts
在输入电压的一个周期内的平均值为:

 V 
1 2 2
VM2
p  sin t  d t   (12-166)
2
Ts _ vg M
0 2
(b) 当加入人工载波后,为了使系统始终稳定,根据书中公式 12.24 有:
m2  ma
1 (12-167)
m1  ma

为了求出 ma 的最小值,当 m2  ma 时,式(12-167)可化简为:

m2  m1
 ma (12-168)
2
对于 m1 、 m2 有:

vg
m1 
L
(12-169)
v  vg
m2 
L
将式(12-169)代入到式(12-168)中,可得:

m2  m1 v  2vg
  ma (12-170)
2 2L
根据式(12-170)可得:
 v  2vg  v V
ma _ min      (12-171)
 2 L max 2 L 2 L
(c) 当考虑人工载波和电感电流纹波的影响后,有:
1 1
ig  iL T  I c  ma dTs  m1d 2Ts  m2d 2Ts (12-172)
Ts s
2 2

- 440 -
将式(12-169)代入到式(12-172)中,可得:

ig 
Ts

I 
v Ts
 vg
Ts T  2T d  v
s s Ts
 vg
Ts T d
s
2
v Ts
 Ts dma
(12-173)
c
2L

(d) 将 vg (t )  VM sin t  和 ic (t )  vg (t ) / Re 代入到式(12-173)中,可得:

VM sin t  Ts v TsVM sin t 


ig   Ts dma 
Ts
d 2  1  2d  (12-174)
Ts Re 2L 2L

通过式(12-174)可发现,当考虑人工载波和电感纹波时,输入电流 ig 会与
Ts

VM sin t 
控制电流 ic  发生偏离,其中人工载波造成偏离的影响为 Ts dma ,而
Re

Ts v TsVM sin t 


电感纹波造成偏离的影响为 
Ts
d 2  1  2d  。
2L 2L

Problem 12.11

如图 12-57 所示,某 Buck 变换器带有一个充电控制器。充电控制器的运行


原理类似 CPM 控制器。在每个开关周期的起点处会有一个短暂的时钟脉冲置位

SR 锁存器,以此使功率 MOSFET 导通,同时 SR 锁存器的 Q 输出端会输出逻辑

低电平使得开关 S s 断开。正比于功率 MOSFET 导通电流的 K s is 会向电容 Cs 充电。

当 t  dTs 时,电容电压 vq (t ) 与控制电压 R f ic 相等,此时模拟比较器会输出高电平,

并将此高电平信号送入 SR 锁存器,使得 SR 锁存器复位,故 SR 锁存器的 Q 端

会输出低电平,使 MOSFET 断开,同时 SR 锁存器的 Q 端会输出高电平使开关 S s

导通,对电容 Cs 的电压进行放电,以便下一个开关周期重新充电。

变换器和控制器的参数为: Vg  24V 、 f s  1/ Ts  100kHz 、 L  60μH 、

C  100μF 、R  3 、KTs / Cs  R f  1 ,你可认为变换器始终处于 CCM 模式。

- 441 -
开关网络
i1 (t ) is (t ) i2 (t ) L

vg v1 (t ) v2 (t ) C R v

Clock S Q

R Q
vq (t )
Cs Ss
K s is

R f ic 充电控制器

控制输入端

图 12-57 带有充电控制器的 Buck 变换器


(a) 推导开关网络端口波形的周期平均值表达式,以获得带有充电控制器的
Buck 开关网络的大信号周期平均模型,模型的控制输入为控制电流 ic 。平均周

期模型应包含一个电流源和一个功率源,占空比 d 不应该出现在模型之中。
(b) 使用(a)中推导的大信号周期平均模型,推导稳态输出电压 V 的表达式,
使用 Vg 、 I c 和 R 进行表达。当 I c  2A 时,推导 I1 、 I 2 和占空比 D 的数值。对于

此静态工作点,画出一个开关周期内的 i1 (t ) 、 i2 (t ) 和 vq (t ) 波形。

(c) 对(a)中推导的结果进行扰动和线性化,以获得小信号交流模型,并推
导模型中的解析表达式,画出完整的小信号交流模型。

(d) 求解(c)中的模型,以获得 Gvc  vˆ / iˆc 。使用(b)中的静态工作点数据,

画出 Gvc 的 Bode 图。

(e) 与占空比控制或者电流型控制相比,此充电控制有什么优点?
Solution:
(a) CCM 模式下,开关网络的端口波形如图 12-58 所示:

- 442 -
v1 v2

vg vg
t t

i1 i2

0 dTs Ts t 0 dTs Ts t

图 12-58 Buck 开关网络的端口波形


故根据图 12-58 可求出端口波形的周期平均值为:
v1 Ts
 vg
Ts

v2 Ts
 d vg
Ts
(12-175)
i1 Ts
 d iL Ts

i2 Ts
 iL Ts

根据式(12-175)可得:
v2 Ts
 d v1 Ts
(12-176)
i1 Ts
 d i2 Ts

根据式(12-176)可推出:
i2 Ts
v2 Ts
 v1 Ts
i1 Ts
(12-177)

又由于有:
i1 (t ) Ts
 is (t ) Ts

K s is (t ) (12-178)
 vq (t ) 
Ts
R f ic (t ) Ts
dTs
Ts Cs
根据式(12-178)可求出 d ,再代入到式(12-176)中的第 1 子式,可得:
Cs R f ic v1
 is 
Ts Ts
i1 Ts Ts
(12-179)
K sTs v2 Ts

根据式(12-177)和式(12-179)可得:
2
Cs R f ic v1

Ts Ts
i2 Ts 2
(12-180)
K sTs v2 Ts

故根据式(12-179)和式(12-180)可画出开关网络的大信号模型如图 12-59 所
示:

- 443 -
i1 Ts
i2 Ts

Cs R f ic Ts
v1 Ts
v1 Ts v2 Ts
K sTs v2 Ts

图 12-59 开关网络的大信号等效电路
(b) 带有充电控制器的 Buck 变换器的大信号模型如图 12-60 所示:

i1 Ts
i2 Ts L

Cs R f ic v1
vg v1 Ts Ts
v2 C R v
Ts Ts Ts Ts
K sTs v2 Ts

图 12-60 带有充电控制器的 Buck 变换器的大信号模型


当变换器处于稳态时,图 12-60 可简化为如图 12-61 所示:

i1 Ts
i2 Ts

Cs R f ic v1
vg v1 Ts Ts
v2 R v
Ts Ts Ts Ts
K sTs v2 Ts

图 12-61 稳态时的带有充电控制器的 Buck 变换器的大信号模型


故有:

V2
V1I1  V2 I 2  (12-181)
R
将式(12-179)代入到式(12-181)中可得:
1
 C R I RV 2  3
V  s f c g  (12-182)
 K sTs 
 
根据式(12-181)和式(12-182),将稳态时的参数值代入可得:
V  3 3456  15.12V
I1  3.17A (12-183)
I 2  5.04A
根据式(12-183)和式(12-176)的第 1 子式可求出:
D  0.63 (12-184)
对于此运行点,一个周期内 i1 (t ) 、 i2 (t ) 和 vq (t ) 的波形如图 12-62 所示:

- 444 -
i1 i2 vq
5.04A 2V

0 dTs Ts 0 dTs Ts 0 dTs Ts

图 12-62 i1 (t ) 、 i2 (t ) 和 vq (t ) 的波形

(c) 对式(12-179)和式(12-180)进行扰动和线性化,可知开关网络的交流小信号
等效电路的形式如图 12-63 所示:
iˆ1 iˆ2

v̂1 r1 f1iˆc g1vˆ2 g 2 vˆ1 f 2iˆc r2 v̂2

图 12-63 开关网络的交流小信号等效电路
根据式(12-179)和式(12-180)有:
Cs R f ic v1
  T1  ic , v1 , v2 
Ts Ts
i1 Ts
K sTs v2 Ts
2 (12-185)
Cs R f ic v1
  T2  ic , v1 , v2 
Ts Ts
i2 Ts 2
K sTs v2 Ts

故有:

1 T1 ( I c , v1 ,V2 ) K TV
  r1  s s 2
r1 v1 v V
Cs I c R f
1 1

T1 (ic ,V1 ,V2 ) CRV


f1   s f 1
ic i I
K sTsV2
c c

T1 ( I c ,V1 , v2 ) CIRV


g1    s c f 21
v2 v V
K sTsV2
2 2
(12-186)
1 T ( I ,V , v ) K sTsV23
  2 c 1 2  r2 
r2 v2 v V
2Cs I c R f V12
2 2

T2 (ic ,V1 ,V2 ) Cs R f V12


f2  
ic i I
K sTsV22
c c

T2 ( I c , v1 ,V2 ) 2C I R V
g2   s c f2 1
v1 v V
K sTsV2
1 1

将求得的稳态值代入到式(12-186)中,可得:

- 445 -
r1  7.56
f1  1.587
g1  0.21
(12-187)
r2  1.5
f 2  2.52
g 2  0.42
根据式(12-186)可画出完整的小信号等效电路,如图 12-64 所示:
iˆ1 iˆ2 L

vˆg v̂1 r1 f1iˆc g1vˆ2 g 2 vˆ1 f 2iˆc r2 v̂2 C R v̂

图 12-64 带有充电控制器的 Buck 变换器完整的小信号等效电路

(d) 对于 Gvc 的推导,首先令 vˆg  0 ,则图 12-64 可简化为如图 12-65 所示:

iˆ2 L

f 2iˆc r2 v̂2 C R v̂

图 12-65 vˆg  0 时的带有充电控制器的 Buck 变换器小信号等效电路

故有:
1 1
 sC 
Z1 R
Z 2  sL  Z1
1 1 1 (12-188)
 
Z 3 r2 Z 2

f 2iˆc Z 3  Z 2
Z1
根据式(12-188)可推出:
vˆ Rf 2 r2
Gvc   (12-189)
ˆic R  r2  Ls  CRr2 s  CLRs 2

将稳态值和参数值代入到式(12-189)中,可得:
11.34
Gvc  8 2
(12-190)
1.8 10 s  0.00051s  4.5
其 Bode 图如图 12-66 所示:

- 446 -
Bode Diagram
20
Gvc
0

Magnitude (dB)
-20

-40

-60

-80
0
Gvc

-45
Phase (deg)

-90

-135

-180
2 3 4 5 6
10 10 10 10 10
Frequency (rad/s)

图 12-66 Gvc 的 Bode 图

(e) 此变换器的控制方式,其原理和 CPM 控制是一样的,唯一的区别也就是


使用采样电流对电容充电,然后将电容电压和控制信号相比较来控制开关管的导
通与关断。这种方式和 CPM 方式,相对于占空比控制策略来说,更容易补偿,
这点从图 12-66 可以看出,其相位是缓慢下降到 180 的,即使使用 PI 控制器
来提升低频增益时,也有足够的相位裕度用来弥补 PI 控制器对相位裕度的负面
影响。此外这种控制电路实现相比占空比控制来说更为简单。

Problem 12.12

如图 12-67 所示,某 Buck 变换器带有一个单周期控制器。其运行原理类似


CPM 控制器。在每个开关周期的起点处会有一个短暂的时钟脉冲置位 SR 锁存器,

以此使功率 MOSFET 导通,同时 SR 锁存器的 Q 输出端会输出逻辑低电平使得

开关 S s 断开。正比于电压 v2 (t ) 的电流 Gs v2 会向电容 Cs 充电。当 t  dTs 时,电容

电压 vs ( s ) 与控制电压 vc (t ) 相等,此时模拟比较器会输出高电平,并将此高电平

信号送入 SR 锁存器,使得 SR 锁存器复位,故 SR 锁存器的 Q 端会输出低电平,

使 MOSFET 断开,同时 SR 锁存器的 Q 端会输出高电平使开关 S s 导通,对电容 Cs

的电压进行放电,以便下一个开关周期重新充电。
变换器和控制器的参数为: Vg  24V 、 f s  1/ Ts  100kHz 、 L  60μH 、

C  100μF 、 R  3 、 GsTs / Cs  1 。你可认为变换器始终处于 CCM 模式。

- 447 -
i1 i1 L

vg v1 v2 C R v

Clock S Q

R Q
vs (t )
Cs Ss
Gs v2

vc 充电控制器

控制输入端

图 12-67 带有单周期控制的 Buck 变换器


(a) 推导开关网络端口波形的周期平均值表达式,以获得带有单周期控制器的
Buck 开关网络的大信号周期平均模型,模型的控制输入为控制电压 vc 。占空比 d

不应该出现在模型之中。
(b) 使用(a)中推导的大信号周期平均模型,推导稳态输出电压 V 的表达式,
使用 Vc 进行表达。当 Vc  10V 时,推导 I1 、 I 2 和 V 的数值。对于此静态工作点,

画出一个开关周期内的 i1 (t ) 、 i2 (t ) 和 vs (t ) 的波形。

(c) 对(a)中推导的结果进行扰动和线性化,以获得小信号交流模型,并推
导模型中的解析表达式,画出完整的小信号交流模型。
(d) 求解(c)中的模型,以获得 Gvc  vˆ / vˆc 和 Gvg  vˆ / vˆg 。使用(b)中的静态

工作点数据,画出 Bode 图。
(e) 与占空比控制相比,此充电控制有什么优点?
Solution:
(a) CCM 模式下,开关网络的端口波形如图 12-68 所示:

- 448 -
v1 v2

vg vg
t t

i1 i2

0 dTs Ts t 0 dTs Ts t

图 12-68 开关网络端口电压电流波形
故端口电压电流的周期平均值为:
v1 Ts
 vg
Ts

v2 Ts
 d vg
Ts
(12-191)
i1 Ts
 d iL Ts

i2 Ts
 iL Ts

根据式(12-191)可求出:
v2 Ts
 d v1 Ts
(12-192)
i1 Ts
 d i2 Ts

根据式(12-192)可求得:
i2 Ts
v2 Ts
 v1 Ts
i1 Ts
(12-193)

又由于:
Gs v2 (t )

Ts
vc (t ) Ts
dTs (12-194)
Cs
根据式(12-194)求出 d ,再代入到式(12-192)的第 2 子式中,可求得:
Cs i2 vc

Ts Ts
v2 Ts
(12-195)
GsTs i1 Ts

根据式(12-195)和式(12-193)可求出:
2
Cs i2 vc

Ts Ts
v1 Ts 2
(12-196)
GsTs i1 Ts

根据式(12-195)和式(12-196)可画出开关网络的大信号模型,如图 12-69 所示:

- 449 -
i1 Ts
i2 Ts

2
Cs i2 vc

Ts Ts
v1 Ts v1 2
v2 Ts
Ts
GsTs i1 Ts

图 12-69 开关网络的大信号模型
(b) 带有单周期控制器的 Buck 变换器的大信号模型如图 12-70 所示:

i1 Ts
i2 Ts L
2
Cs i2 vc
vg v1 Ts Ts
v2 C R v
Ts Ts 2 Ts Ts
GsTs i1 Ts

图 12-70 带有单周期控制器的 Buck 变换器的大信号模型


当变换器处于稳态时,图 12-70 可简化为如图 12-71 所示:

i1 Ts
i2 Ts

2
Cs i2 vc
vg v1 Ts Ts
v2 R v
Ts Ts 2 Ts Ts
GsTs i1 Ts

图 12-71 稳态时的带有单周期控制器的 Buck 变换器的大信号模型


故根据式(12-195)有:
Cs I 2Vc
V  V2  (12-197)
GsTs I1

根据式(12-193)可得:
I 2 V1
 (12-198)
I1 V2

将式(12-198)代入到式(12-197)中可得:
CsV1Vc
V (12-199)
GsTs

当 Vc  10V ,将参数值代入到式(12-199)、式(12-198)和式(12-192)中可得:

V  15.49V
I 2  5.16A
(12-200)
I1  3.33A
D  0.645

- 450 -
一个开关周期内的 i1 (t ) 、 i2 (t ) 和 vs (t ) 的波形如图 12-72 所示:

i1 i2 vs
5.16A 10V

0 dTs Ts 0 dTs Ts 0 dTs Ts

图 12-72 一个开关周期内的 i1 (t ) 、 i2 (t ) 和 vs (t ) 的波形

(c) 根据式(12-195)和式(12-196)可画出开关网络的交流小信号等效电路如图
12-73 所示:
iˆ1 iˆ2

r1 r2

v̂1 f1vˆc f 2 vˆc v̂2

g1iˆ2 g 2iˆ1

图 12-73 开关网络的交流小信号等效电路
根据式(12-195)和式(12-196)有:
2
Cs i2 vc
  T1  i1 , i2 , vc 
Ts Ts
v1 Ts 2
GsTs i1 Ts
(12-201)
Cs i2 vc
  T2  i1 , i2 , vc 
Ts Ts
v2 Ts
GsTs i1 Ts

其中:

T1 (i1 , I 2 ,Vc ) 2C I 2V


r1   r1   s 2 3c
i1 i I
GsTs I1
1 1

T1 ( I1 , I 2 , vc ) C I2
f1   s 22
vc v V
Gs I1 Ts
c c

T1 ( I1 , i2 ,Vc ) 2C I V
g1   s 22 c
i2 i I
Gs I1 Ts
2 2
(12-202)
T ( I , i ,V ) CV
r2  2 1 2 c  r2   s c
i2 i I
GsTs I1
2 2

T2 ( I1 , I 2 , vc ) CI
f2   s 2
vc v V
GsTs I1
c c

T2 (i1 , I 2 , Vc ) CIV


g2    s 22 c
i1 i I
Gs I1 Ts
1 1

- 451 -
将稳态值代入到式(12-202)中可得:
r1  14.4
f1  2.4
g1  9.30
(12-203)
r2  3
f 2  1.55
g 2  4.65
根据式(12-201)可画出带有单周期控制器的 Buck 变换器的交流小信号等效
电路模型,如图 12-74 所示:

iˆ1 iˆ2
L
r1 r2

vˆg v̂1 f1vˆc f 2 vˆc v̂2 C R v̂

g1iˆ2 g 2iˆ1

图 12-74 带有单周期控制器的 Buck 变换器的交流小信号等效电路

(d) 对于 Gvc 的推导,首先令 vˆg  0 ,则图 12-74 可简化为如图 12-75 所示:

iˆ1 iˆ2
L
r1 r2

v̂1 f1vˆc f 2 vˆc v̂2 C R v̂

g1iˆ2 g 2iˆ1

图 12-75 vˆg  0 时的带有单周期控制器的 Buck 变换器的交流小信号等效电路

故有:
f1vˆc  g1iˆ2  r1iˆ1  0
1 1
 sC 
Z R (12-204)
f 2 vˆc  g 2iˆ1  r2iˆ2  sLiˆ2  vˆ
vˆ ˆ
 i2
Z
根据式(12-204)可求得:
vˆ Rf 2 r1  Rf1 g 2
Gvc   (12-205)
vˆc CLRr1s   CRg1 g 2  CRr1r2  Lr1  s  Rr1  g1 g 2  r1r2
2

- 452 -
对于 Gvg 的推导,令 vˆc  0 ,则图 12-75 可简化为如图 12-76 所示:

iˆ1 iˆ2
L
r1 r2

vˆg v̂1 v̂2 C R v̂

g1iˆ2 g 2iˆ1

图 12-76 vˆc  0 时的带有单周期控制器的 Buck 变换器的交流小信号等效电路

故有:
vˆg  iˆ1r1  g1iˆ2
g 2iˆ1  iˆ2 r2  sLiˆ2  vˆ
1 1 (12-206)
 sC 
Z R

iˆ2 
Z
根据式(12-206)可求出:
Rg 2
Gvg  (12-207)
CLRr1s   CRg1 g 2  CRr1r2  Lr1  s  Rr1  g1 g 2  r1r2
2

将式(12-203)代入到式(12-205)和式(12-207)中,可得:
33.48
Gvc  7 2
2.592 10 s  0.0008775s  43.25
(12-208)
13.95
Gvg  7 2
2.592 10 s  0.0008775s  43.25
Gvc 的 Bode 图如图 12-77 所示:

Bode Diagram
10
Gvc
0
Magnitude (dB)

-10

-20

-30

-40
0
Gvc

-45
Phase (deg)

-90

-135

-180
3 4 5
10 10 10
Frequency (rad/s)

图 12-77 Gvc 的 Bode 图

- 453 -
Gvg 的 Bode 图如图 12-78 所示:

Bode Diagram
20
Gvg

0
Magnitude (dB)

-20

-40

-60
0
Gvg

-45
Phase (deg)

-90

-135

-180
3 4 5
10 10 10
Frequency (rad/s)

图 12-78 Gvg 的 Bode 图

(f) 这种单周期控制的原理和 CPM 控制基本一样,唯一的区别也就是使用和


采样电压成正比的电流对电容充电,然后将电容电压和控制信号相比较来控制开
关管的导通与关断。这种单周期控制相比于占空比控制的优势在于实现的电路简
单。然而在控制层面上,相比于占空比控制来说,至少从 Bode 图中并没有看出
什么优势。

- 454 -
第13章 磁学的基本理论

Problem 13.1

如书中 Fig13.51(a)所示的铁芯,其厚度为 1cm 。除了右侧垂直方向的臂宽


为 0.5cm ,其余臂宽均为 1cm 。你可以忽略由铁芯转角处引起的磁力线分布不一
致性。
(a) 推导此器件的磁路模型,并标出模型中的各磁阻值。
(b) 推导绕组的电感值。
如书中 Fig13.51(b)所示,当在此铁芯的右侧添加第 2 个绕组时:
(c) 修改(a)中的模型,以将第 2 个绕组包含进去。
(d) 此模型的电路方程可以表达成如下形式:
 v1   L11 L12  d  i1 
v    L L22  dt i2 
 2   12

使用叠加定理来推导此方程,并给出 L11 、 L12 、 L22 的表达式。

Solution:
(a) 磁路模型如图 13-1 所示:

Rm1 Rm3

n1i1 (t ) Rm 2 Rm5
Rm 4

图 13-1 单绕组时的磁路模型
l1 9 102
Rm1    7.162 105 H -1
u0ur Ac1 u0ur 1104
l2 3 102
Rm 2   4
 2.387 105 H -1
u0ur Ac 2 u0ur 110
各磁阻值为: (13-1)
l3 3 102
Rm3  Rm 4   4
 2.387 105 H -1
u0ur Ac 3 u0ur 110
l5 3 102
Rm5   4
 4.775 105 H -1
u0ur Ac 5 u0ur  0.5 10
(b) 根据法拉第定理有:
d
v  n1 (13-2)
dt
将图 13-1 进行简化,如图 13-2 所示:

- 455 -
Rm1

n1i1 (t ) Rm 6

图 13-2 简化的磁路模型
故有:
1 1 1
 
Rm 6 Rm 2 Rm3  Rm 4  Rm5 (13-3)
n1i1    Rm1  Rm6 
根据式(13-3)可求出穿过绕组的磁通为:
n1i1
 (13-4)
Rm1  Rm 6

将式(13-4)代入到式(13-2)中可得:
n1 di1
v  n1 (13-5)
Rm1  Rm 6 dt

故电感值为:
n12
L (13-6)
Rm1  Rm 6

将(13-1)代入到式(13-3)和式(13-6)中可得:

L  1.102 104  110.2μH (13-7)

(c) 当将第 2 个绕组考虑进来时,磁路模型如图 13-3 所示:

Rm1 Rm3
Rm5
n1i1 (t ) Rm 2
n2i2 (t )
Rm 4

图 13-3 同时考虑 2 个绕组时的磁路模型


(d) 当同时考虑 2 个绕组时,有:
d 1
v1  n1
dt
(13-8)
d 2
v2  n2
dt
其中对于 1 和  2 ,如图 13-4 所示:

- 456 -
Rm1 Rm3
1
Rm5
n1i1 (t ) Rm 2
n2i2 (t )
Rm 4
2

图 13-4 两磁通的定义

接下来通过叠加定理求出 1 和  2 关于两电流的表达式,当只考虑左侧的磁

动势 n1i1 时,图 13-4 可简化为如图 13-5 所示:

Rm1 Rm 3
11
Rm 5
n1i1 (t ) Rm 2
Rm 4
 21

图 13-5 只考虑 n1i1 时的磁路模型

故有:
1 1 1
 
Rm 6 Rm 2 Rm3  Rm 4  Rm5
n1i1
11  (13-9)
Rm1  Rm 6
11 Rm 6   21  Rm3  Rm 4  Rm5 
根据式(13-9)可推出:
n1i1
11 
Rm1  Rm 6
(13-10)
n1i1 Rm 6
 21 
 Rm1  Rm6  Rm3  Rm4  Rm5 
当只考虑右侧的磁动势 n2i2 时,则图 13-4 可简化为如图 13-6 所示:

Rm1 Rm 3

12 Rm 5
Rm 2
n2i2 (t )
Rm 4
 22

图 13-6 只考虑 n2i2 时的磁路模型

- 457 -
故有:
1 1 1
 
Rm 7 Rm1 Rm 2
n2i2
 22  (13-11)
Rm3  Rm 4  Rm5  Rm 7
 22 Rm 7  12 Rm1
根据式(13-11)可求出:
n2i2
 22 
Rm3  Rm 4  Rm5  Rm 7
(13-12)
n2i2 Rm 7
12 
Rm1  Rm3  Rm 4  Rm5  Rm 7 
根据式(13-10)和式(13-12)可求得:
1  11  12 
n1i1 n2i2 Rm 7
 
Rm1  Rm 6 Rm1  Rm3  Rm 4  Rm5  Rm 7 
(13-13)
 2   21   22 
n1i1 Rm 6 n2i2
 
 Rm1  Rm6  Rm3  Rm 4  Rm5  Rm3  Rm 4  Rm5  Rm7
将式(13-13)代入到式(13-8)中,可有:
d 1
v1  n1 
dt
n12 di1 n1n2 Rm 7 di2
 
Rm1  Rm 6 dt Rm1  Rm3  Rm 4  Rm5  Rm 7  dt
(13-14)
d 2
v2  n2 
dt
n1n2 Rm 6 di1 n22 di2
 
 Rm1  Rm6  Rm3  Rm 4  Rm5  dt Rm3  Rm 4  Rm5  Rm7 dt
故有:
n12
L11 
Rm1  Rm 6
n1n2 Rm 7
L12 
Rm1  Rm3  Rm 4  Rm5  Rm 7 
(13-15)
n1n2 Rm 6
L21 
 Rm1  Rm6  Rm3  Rm 4  Rm5 
n22
L22 
Rm3  Rm 4  Rm5  Rm 7
将(13-1)中的各磁阻值代入到式(13-15)中,可得:

- 458 -
L11  110.2μH
L12  L21  44.09μH (13-16)
L22  352.7μH

Problem 13.2

如书中 Fig13.52(a)所示,在一个固定横截面积 Ac  1cm2 的铁芯上放置两

个绕组,每一个绕组的匝数为 50。铁芯的相对磁导率为 μ r  104 。

(a) 画出等效磁路,推导各磁阻的数值。
(b) 推导每个绕组的自感。

(c) 当两个绕组的同名端正向串联时,推导此时的电感值 L 。

(d) 当两个绕组的同名端反向串联时,推导此时的电感值 L 。

Solution:
(a) 等效磁路如图 13-7 所示:

Rm1 Rm 2
Rm3
ni1 (t ) ni2 (t )

图 13-7 等效磁路
各磁阻的阻值为:
l1 15 102
Rm1  Rm3    1.194 105 H -1
0 r Ac 4 10 10 10
7 4 4

(13-17)
l2 5 102
Rm 2    3.979 104 H -1
0 r Ac 4 107 104 104
(b) 定义两磁通如图 13-8 所示:

Rm1 Rm 2
1 2 Rm3
ni1 (t ) ni2 (t )

图 13-8 两磁通的定义
对于两绕组有:
d 1
v1  n
dt
(13-18)
d 2
v2  n
dt
对于两绕组的自感,可以像 Problem 13.1 那样通过叠加定理求得,也可以直

- 459 -
接罗列方程求出,这里通过罗列方程求出。根据图 13-8 有:
ni1  Rm11  ni2  Rm 2 2   1   2  Rm3 (13-19)

根据式(13-19)可求得:

1 
 nRm 2  nRm3  i1  nRm3i2
Rm1 Rm 2  Rm1Rm3  Rm 2 Rm3
(13-20)
nRm3i1   nRm1  nRm3  i2
2 
Rm1Rm 2  Rm1Rm3  Rm 2 Rm3
将式(13-20)代入到式(13-18)可得:
d 1
v1  n 
dt
n 2  Rm 2  Rm3  di1 n 2 Rm3 di2
 
Rm1 Rm 2  Rm1 Rm 3  Rm 2 Rm 3 dt Rm1 Rm 2  Rm1Rm 3  Rm 2 Rm 3 dt
(13-21)
d 2
v2  n 
dt
n 2 Rm3 di1 n 2  Rm1  Rm 3  di2
 
Rm1 Rm 2  Rm1 Rm 3  Rm 2 Rm 3 dt Rm1 Rm 2  Rm1Rm 3  Rm 2 Rm 3 dt
故两绕组的自感可根据式(13-21)求出:
n 2  Rm 2  Rm3 
L11 
Rm1 Rm 2  Rm1Rm3  Rm 2 Rm3
(13-22)
n 2  Rm1  Rm3 
L22 
Rm1 Rm 2  Rm1Rm3  Rm 2 Rm3
将式(13-17)代入到式(13-22)中可得求出两绕组的自感值为:
L11  17mH
(13-23)
L22  25mH

(c) 当两绕组的同名端正向串联时,即有:
i2  i1 (13-24)

将式(13-24)代入到式(13-21)中可得:
d 1 n 2 Rm 2 di1
v1  n 
dt Rm1 Rm 2  Rm1Rm3  Rm 2 Rm3 dt
(13-25)
d 2 n 2 Rm1 di1
v2  n 
dt Rm1 Rm 2  Rm1Rm3  Rm 2 Rm3 dt

根据式(13-25)可求出电感 L 为:

- 460 -
n 2  Rm1  Rm 2  di1
v  v1  v2 
Rm1 Rm 2  Rm1Rm3  Rm 2 Rm3 dt
(13-26)
n 2  Rm1  Rm 2 
L   17mH
Rm1 Rm 2  Rm1Rm3  Rm 2 Rm3
(d) 当两同名端反向串联时,此时有:
i2  i1 (13-27)

将式(13-27)代入到式(13-21)中,可得:
d 1 n 2  Rm 2  2 Rm3  di1
v1  n 
dt Rm1 Rm 2  Rm1Rm3  Rm 2 Rm3 dt
(13-28)
d 2 n 2  Rm1  2 Rm3  di1
v2  n 
dt Rm1 Rm 2  Rm1Rm3  Rm 2 Rm3 dt

根据式(13-28)可求出电感 L 的值为:

n 2  Rm1  Rm 2  4 Rm3  di1


v  v1  v2 
Rm1 Rm 2  Rm1Rm3  Rm 2 Rm3 dt
(13-29)
 n 2  Rm1  Rm 2  4 Rm3 
L   67mH
Rm1 Rm 2  Rm1Rm3  Rm 2 Rm3

Problem 13.3

如书中 Fig13.53 所示的铁芯,其 3 个铁芯臂具有同样的横截面积 A c ,铁芯

臂 1 和铁芯臂 2 的磁路长度为 3l ,铁芯臂 3 的磁路长度为 l ,两个绕组的匝数均


为 n 匝。铁芯的磁导率  0 。

(a) 画出磁路模型,并给出磁阻与磁动势的表达式。
当绕组 1 连接了一个电压源时,此电压源 v1 (t ) 的波形为方波,方波的峰值为

Vma ,方波的周期为 Ts ,绕组 2 为开路。

(b) 画出 i1 (t ) ,并求解其峰值。

(c) 推导铁芯臂 2 的磁链 2 (t ) ,画出 2 (t ) 的波形,并标出其峰值。

(d) 画出 v2 (t ) 的波形,并标出其峰值。

Solution:
(a) 磁路模型如图 13-9 所示:

- 461 -
Rm1 Rm1
1 Rm 2 2
ni1 (t ) ni2 (t )

图 13-9 磁路模型
各磁阻的表达式为:
3l
Rm1 
 Ac
(13-30)
l
Rm 2 
 Ac
根据式(13-30)可求出:
Rm1  3Rm 2 (13-31)

(b) 电压源 v1 (t ) 的波形如图 13-10 所示:

v1 (t )

Vma

0 t
Ts
Vma

图 13-10 电压源 v1 (t ) 的波形

当绕组 1 连接此电压源,绕组 2 开路时,则图 13-9 可简化为如图 13-11 所


示:

Rm1 Rm1
1 Rm 2
ni1 (t ) 2

图 13-11 绕组 2 开路时的磁路模型
故有:
d 1
v1  n
dt
1 1 1
  (13-32)
Z Rm 2 Rm1
ni1  1  Rm1  Z 

根据式(13-32)可求出:

- 462 -
ni1  Rm1  Rm 2 
1 
Rm1  Rm1  2 Rm 2 
(13-33)
n 2  Rm1  Rm 2  di1
v1 
Rm1  Rm1  2 Rm 2  dt
将式(13-31)代入到式(13-33)中,可得:
4n 2 di1
v1 
15Rm 2 dt
(13-34)
4n 2
L1 
15Rm 2

故根据式(13-34)可画出 i1 (t ) 的波形如图 13-12 所示:

i1 (t )
Vma 15Rm 2Ts
16n 2 Ts
0 t

图 13-12 i1 (t ) 的波形

i1 (t ) 的峰值为:

Vma Ts 1 15Rm 2VmaTs


i1 pk   (13-35)
L1 2 2 16n 2

(c) 对于图 13-11 中的磁通  2 ,有:

1Z   2 Rm1 (13-36)

根据式(13-36)可求出:
Z
2  1 (13-37)
Rm1

将式(13-33)代入到式(13-37)中可得:
n
2  i1 (13-38)
15Rm 2

故磁通  2 的波形如图 13-13 所示:

- 463 -
2

 2 pk
Ts
0 t

图 13-13 磁通  2 的波形

将式(13-35)代入到式(13-38)可求出磁通  2 的峰值为:

VmaTs
2 pk  (13-39)
16n
(d) 对于 v2 (t ) 有:

d 2
v2  n (13-40)
dt
将式(13-38)代入到式(13-40)中,可得:
n 2 di1
v2  (13-41)
15Rm 2 dt

根据式(13-34)和式(13-41)可得:
v1
v2  (13-42)
4
故感应出来的 v2 (t ) 的波形如图 13-14 所示:

v2 (t )
Vma
4
0 t
Ts
V
 ma
4

图 13-14 v2 (t ) 的波形

Problem 13.4

如书中 Fig13.54(a)所示的磁性器件含有两个绕组,此磁性器件可用来替
代 Cuk、Sepic 变换器中的两个电感。对于此题,所有的 3 个铁芯臂具有同样的
横截面积 Ac 。铁芯臂上的气隙长度分别为 g1 、 g 2 和 g 3 。铁芯的磁导率足够大。

你可以不考虑杂散磁通的影响。铁芯臂 1 和铁芯臂 2 上的绕组匝数分别为 n1 和 n2 。

- 464 -
(a) 推导此磁性器件的磁路模型,并给出各磁阻的表达式,需要标出磁动势的
极性。
(b) 以如下所示的矩阵形式,推导此磁性器件的端口方程。并给出 L11 、 L12 和

L22 的表达式。

 v1   L11 L12  d  i1 
v    L L22  dt i2 
 2   12
(c) 推导此器件的电路模型。用(a)中的匝数和磁阻来表示匝数比和各自电
感值。
如书中 Fig13.54(b)所示,这个单独的器件被用来替代 Cuk 变换器中的两
个电感。
(d) 使用线性纹波近似的方法,画出电压波形 v1 (t ) 和 v2 (t ) 。你可以认为变换

器工作在 CCM 模式。


(e) 当(d)中的电压波形应用在(b)和(c)中时,求解模型来推导 DTs 和 DTs

的子区间内的电感电流纹波斜率。画出稳态时,电感电流波形 i1 (t ) 和 i2 (t ) 。

(f) 通过有技巧的选择 n1 / n2 和气隙长度,有可能使电感电流 i1 (t ) 或 i2 (t ) 中的纹

波 i 为 0。推导使电感电流 i2 (t ) 的纹波为 0 时的 n1 / n2 值和气隙长度值 g1 、 g 2 和

g 3 。画出此时的 i1 (t ) 和 i2 (t ) 波形。

当变换器中的两电感电压成比例,通过这种方式将两电感相互耦合,是可以
使电感电流纹波趋近于 0 的。
Solution:
(a) 此磁性器件的磁路模型如图 13-15 所示:

Rm1 Rm 2
1 Rm 3 n2i2 (t )
n1i1 (t )
2

图 13-15 磁路模型
各磁阻的表达式为:

- 465 -
g1
Rm1 
0 Ac
g2
Rm 2  (13-43)
0 Ac
g3
Rm 3 
0 Ac
(b) 端口方程可以通过叠加定理求出,也可以直接通过罗列方程求出,这里通
过罗列方程求出,根据图 13-15 有:
n1i1  1 Rm1   1   2  Rm3   2 Rm 2  n2i2 (13-44)

根据式(13-44)可求出:

1 
 Rm 2  Rm3  n1i1  Rm3n2i2
Rm1 Rm 2  Rm1Rm3  Rm 2 Rm3
(13-45)
R n i   Rm1  Rm3  n2i2
 2  m3 1 1
Rm1Rm 2  Rm1Rm3  Rm 2 Rm3
根据法拉第定理有:
d 1
v1  n1
dt
(13-46)
d 2
v2  n2
dt
将式(13-45)代入到式(13-46)中,可得:

v1 
 Rm 2  Rm3  n12 di1

Rm3n1n2 di2
Rm1Rm 2  Rm1Rm3  Rm 2 Rm3 dt Rm1Rm 2  Rm1Rm 3  Rm 2 Rm 3 dt
(13-47)
v2 
Rm3n1n2 di1

 Rm1  Rm3  n22 di2
Rm1Rm 2  Rm1Rm3  Rm 2 Rm3 dt Rm1Rm 2  Rm1Rm 3  Rm 2 Rm 3 dt
根据式(13-47)有:

L11 
 Rm 2  Rm3  n12
Rm1 Rm 2  Rm1 Rm 3  Rm 2 Rm3
Rm3n1n2
L12  (13-48)
Rm1 Rm 2  Rm1 Rm 3  Rm 2 Rm3

L22 
 Rm1  Rm3  n22
Rm1 Rm 2  Rm1 Rm 3  Rm 2 Rm3
(c) 此磁性器件的等效电路如图 13-16 所示:

- 466 -
L12
i1 (t ) i2 (t )

v1 L11 L22 v2

图 13-16 磁性器件的等效电路

(d) 电压波形 v1 (t ) 和 v2 (t ) 如图 13-17 所示:

v1 (t ) v2 (t )

Vg VC1  V

0 0
DTs Ts t DTs Ts t
Vg  VC1 V

图 13-17 电压波形 v1 (t ) 和 v2 (t )

(e) 根据式(13-47)和式(13-48)可求出电感电流纹波的斜率为:
di1 L12v2  L22v1
 2
dt L12  L11 L22
(13-49)
di2 L12v1  L11v2
 2
dt L12  L11L22

当 0  t  DTs 时,对于电压 v1 和 v2 有:

v1  Vg
(13-50)
v2  Vc1  V  Vg

将式(13-50)代入到式(13-49)中,可得 0  t  DTs 时的电流纹波斜率:

di1  L12  L22  Vg


 2
dt L12  L11 L22
(13-51)
di2  L12  L11  Vg
 2
dt L12  L11 L22

当 DTs  t  Ts 时,对于电压 v1 和 v2 有:

v1  Vg  Vc1  V
(13-52)
v2  V

- 467 -
将式(13-52)代入到式(13-49)中,可得 DTs  t  Ts 时的电流纹波斜率:

di1   L12  L22 V


 2
dt L12  L11L22
(13-53)
di2   L12  L11 V
 2
dt L12  L11L22
稳态时,两电感电流的波形如图 13-18 所示:
i1 i2

0 DTs Ts t 0 DTs Ts t

图 13-18 稳态时,两电感电流的波形

(f) 为了使电感电流 i2 (t ) 的纹波为 0,只需令式(13-53)的第 2 子式为 0 即可,

故有:
L12  L11 (13-54)

将式(13-48)代入到式(13-54)可得:
 Rm 2  Rm3  n12  Rm3n1n2 (13-55)

将式(13-43)代入到式(13-55)中,可得:
n1 g3
 (13-56)
n2 g 2  g3

Problem 13.5

某永磁材料具有如书中 Fig13.55 中的实线所示的 BH 曲线,此永磁材料的长


度为 lm  0.5cm ,横截面积为 4cm2 , Bm  1T 。推导此永磁体的磁路模型。

Solution:
根据法拉第定理,有:
d dB
vn  nAc (13-57)
dt dt
根据书中 Fig13.55 可知:
B  Bm   H (13-58)

根据安培定理,有:

- 468 -
ni  Hlm (13-59)

联立式(13-57)、式(13-58)和式(13-59),可得:
 
  Bm 
ni  lm
 B  Bm   l  Ac   lm   lm Bm  R   lm Bm (13-60)
 m
  Ac  m

故可根据式(13-60)可画出磁路模型,如图 13-19 所示:
lm lm Bm
Rm 
 Ac 

图 13-19 磁路模型
将参数值代入到式(13-60)可得:
Rm  9.384 106 H 1
lm Bm (13-61)
 3.754 103

Problem 13.6

如书中 Fig6.27 所示的双管正激变换器,其输入电压为 Vg  300V ,输出电压

为 V  28V ,开关频率为 f s  100kHz ,匝数比为 n  0.25 。直流负载功率为 250W 。

变压器使用型号为 EC41 的铁氧体磁芯,该铁芯的相关数据列在书中的附录 D 中。


铁芯损耗如书中 Fig13.20 所示。原边绕组线圈使用 44 匝的 #21AWG 线缆,副边
线圈使用 11 匝的 #15AWG 线缆。 AWG 线缆的数据也列在书中的附录 D 中。
(a) 评估此变压器的铁芯损耗。
(b) 推导此变压器的铜耗,你可以忽略临近效应。
Solution:
(a) 当开关管导通时,有:
d dB B
Vg  n p  n p Ac  n p Ac
dt dt DTs (13-62)
vL  nVg  V

当开关管关断时,有:
vL  V (13-63)

由电感的伏秒平衡,根据式(13-62)的第 2 子式和式(13-63)可求得:

D  nVg  V   1  D  V   0  D 
V
 0.373 (13-64)
nVg

- 469 -
根据书中附录 D 中的 EC41 铁芯数据,可知:
Ac  1.21
(13-65)
lm  8.93

将式(13-64)代入到式(13-62)中,可求得(注意,B 是磁密总变化量的一半,
类型电流纹波 i ,做到 Problem 14.5 时,才反应过来这里搞错了。。。):
DTsVg
B   0.105T (13-66)
2 Ac n p

根据书中 Fig13.20 所示的铁耗曲线,可知当开关频率为 100kHz 时,单位体

积(以 cm3 为单位)的铁耗为:

p fe  0.06W/cm3 (13-67)

根据式(13-67)和式(13-65)可求得变压器的铁芯损耗为:
Pfe  p fe Aclm  0.648W (13-68)

(b) 根据书中附录 D.3 中的 EC41 铁芯数据,每匝线圈的平均长度为:


MLT  5.30cm (13-69)
由于原边线圈为 44 匝的 #21AWG 线缆,根据书中附录 D.6(注意中间那列
表示的是每 cm 长度的电阻,而非电阻率)中的数据可求得其直流电阻为:
R p  n p  MLT  Rcm p  44  5.30  418.9 106  0.098 (13-70)

同理,对于副边有:
Rs  ns  MLT  Rcms  11 5.30 104.3106  6.081103  (13-71)

对于双管正激变换器,其副边电感电流的直流分量为:
P 250
Is    8.929A (13-72)
V 28
故开关管导通时的原边电流直流分量为:
I p  nI s  0.25I s  2.232A (13-73)

故忽略临近效应时的变压器的铜耗为:
Pcu  D  I p2 R p  I s2 Rs   0.363W (13-74)

Problem 13.7

如书中 Fig6.27 所示的双管正激变换器,其输入电压为 Vg  300V ,输出电压

为 V  28V ,开关频率为 f s  100kHz ,匝数比为 n  0.25 。直流负载功率为 250W 。

- 470 -
变压器使用型号为 EC41 的铁氧体磁芯,该铁芯的相关数据列在书中的附录 D 中。
铁芯的窗口高度为 lw  2.78cm 。原边绕组线圈使用 44 匝的 #24AWG 线缆,副边

线圈使用 11 匝的 #14AWG 线缆。 AWG 线缆的数据也列在书中的附录 D 中。绕


组工作在室温。
(a) 推导由绕组电流中的直流分量引起的原副边绕组的铜耗。
(b) 推导由绕组电流中的基波分量引起的原副边绕组的铜耗。
(c) 推导由绕组电流中的二次谐波分量引起的原副边绕组的铜耗。
Solution:
(a) 占空比 D 为:

D  nVg  V   1  D  V   0  D 
V
 0.373 (13-75)
nVg

副边电感电流的直流分量为:
P 250
 Is 
 8.929A (13-76)
V 28
根据书中附录 D.6 的数据可知,副边绕组的直流电阻为:
Rdcs  ns  MLT  Rcms  11 5.30  82.8 106  4.827 103  (13-77)

副边绕组电流的直流分量为:
I dcs  DI s  3.331A (13-78)

故副边电流中的直流分量产生的铜耗为:
Pdcs  I dcs
2
Rdcs  0.054W (13-79)

开关管导通时的原边电流的为:
I p  nI s  0.25I s  2.232A (13-80)

故原边绕组电流的直流分量为:
I dcp  DI p  0.833A (13-81)

根据书中附录 D.6 的数据可知,原边绕组的直流电阻为:


Rdcp  n p  MLT  Rcmp  44  5.30  842.1106  0.196 (13-82)

故原边电流中的直流分量产生的铜耗为:
Pdcp  I dcp
2
Rdcp  0.136W (13-83)

故由直流分量引起的原副边总铜耗为:
Pdc  Pdcp  Pdcs  0.19W (13-84)

(b) j 次谐波产生的铜耗为:

- 471 -

 
Pj  I 2j Rdc j1 G1 j1   M 2  1 G1

2
3
  
j1  2G2  
j1 

sinh  2   sin  2 
G1    (13-85)
cosh  2   cos  2 
sinh   cos    cosh   sin  
G2   
cosh  2   cos  2 
对于副边绕组电流的 1 次的基波有:

2I s
I1s  sin  D   3.704A

h  ds
1   s  7.5
 4 s
 ns
s  ds  0.6 (13-86)
4 lw
s Rdcs Aws
s    0.016
0 f s 0 f s
M 1
将式(13-86)和式(13-77)代入到式(13-85)中可算得副边绕组电流中的 1 次基
波产生的铜耗为:
P1s  0.497W (13-87)

对于原边绕组电流中的 1 次基波有:
2I p
I1 p  sin  D   0.926A

h  dp
1   p  1.401
 4 p
 np
p  dp  0.794 (13-88)
4 lw
p Rdcp Awp
p    0.032
0 f s 0 f s
M 1
将式(13-88)和式(13-82)代入到式(13-85)中,可算得原边绕组电流中的 1 次基
波产生的铜耗为:
P1 p  0.219W (13-89)

故 1 次基波在原副边绕组上产生的铜耗为:
P1  P1 p  P1s  0.716W (13-90)

- 472 -
(c) 对于副边绕组电流的 2 次的谐波有:

2I s
I2s  sin  2 D   1.439A (13-91)
2
将式(13-91)、式(13-86)和式(13-77)代入到式(13-85)中,可算得副边绕组电流
中的 2 次谐波产生的铜耗为:
P2 s  0.106W (13-92)

对于原边绕组电流中的 2 次谐波有:

2I p
I2 p  sin  2 D   0.36A (13-93)
2
将式(13-93)、式(13-88)和式(13-82)代入到式(13-85)中,可得原边绕组电流中
的 2 次谐波产生的铜耗为:
P2 p  0.048W (13-94)

故 2 次谐波在原副边绕组上产生的铜耗为:
P2  P2 s  P2 p  0.154W (13-95)

Problem 13.8 (待完成)

应用于某高压逆变器中的变压器的绕组电流为正弦波形,其谐波可以忽略不
计,并且不存在直流分量。原边绕组为 1 层 10 匝的圆形铜导线,副边为 250 匝的
10 层圆形铜导线。开关频率为 50kHz 。绕组的孔隙率为 0.8 。决定原副边导线的
直径及导线型号,以使得铜耗最小。
Solution:

Problem 13.9 (待完成)

Problem 13.10 (待完成)

Problem 13.11 (待完成)

Problem 13.12

如书中 Fig6.37 和书中 Fig6.38 所示的隔离型 SEPIC 变换器,其工作在 CCM


模式。
(a) 你认为 SEPIC 变换器中的变压器是否要含有气隙?为什么?
(b) 画出 CCM 模式下,变压器的 B-H 环型的运行轨迹。
(c) 对于以 CCM 模式运行的 SEPIC 变换器,你认为铁耗是主要损耗么?

- 473 -
(d) 对于以 CCM 模式运行的 SEPIC 变换器,你认为绕组的临近损耗是主要损
耗么?
Solution:
(a) 此隔离型的 SEPIC 变换器中的变压器应该含有气隙,因为此变换器使用
激磁电感传递能量,为了使能量有效的传递,应该使激磁电感的感值在电流变化
的范围内保持固定。如果不使用气隙,当电流较大时,铁芯极易饱和,因此会使
电感值急剧下降,进而使激磁电感存储的能量较少,为了使激磁电感的感值不随
电流变化而发生较大改变,需要加入气隙。
(b) CCM 模式下的 B-H 运行轨迹如书中 Fig13.48 所示。
(c) 由于激磁电感中的电流为直流基础上叠加一个开关频率的小纹波电流,故
其直流分量占据主要地位,而产生铁耗的开关频率的小纹波电流,其幅值相对直
流分量来说很小,故产生的铁耗也很小,这点可从 B-H 的运行轨迹看出。故铁
耗不是主要损耗,在设计的过程中,只要使开关频率的电流纹波幅值很小,就可
以忽略铁耗的影响。
(d) 临近损耗是由交流电流引起,首先激磁电感中的交流电流为其开关频率的
小纹波电流,故此交流分量的幅值很小,当开关频率不是很高的时候,并且激磁
电感中的电流纹波也较小的时候,就可以忽略临近损耗的影响。

- 474 -
第14章 电感设计

Problem 14.1

某 Buck 变换器的开关频率为 50kHz ,输入电压为 Vg  40V ,输出电压为

V  20V ,负载电阻 R  4 (书中的符号可能有误,如果负载电阻  4 ,并且


还要保持电流纹波为其直流分量的 10% ,那么需要使电感值随着负载电阻的变大
而变大,因此电感值不是固定的数值,故这里使用 R  4 条件)。
(a) 推导输出滤波电感的电感值,使得电感电流的纹波 i 为其直流分量的
10% 。
(b) 推导稳态时电感电流的最大值 I max 。

(c) 对(a)和(b)得到的电感值和最大电流进行电感设计。使用铁氧体 EE
铁芯,其 Bmax  0.25T 。设计绕组的电阻值,使得其铜耗在室温下小于 1W 。你可

以认为 K u  0.5 ,需要指定铁芯型号、气隙长度、线缆型号(AWG)和匝数。

Solution:
(a) 电感电流纹波  i 为其直流分量的 10% ,故此 Buck 变换器运行于 CCM 模

式。其直流分量为:
V
I
R
(14-1)
V
D
Vg

对于电感电流纹波 i 有:
2i
Vg  V  L (14-2)
DTs

根据题中的设计目的,有:
i  0.1I (14-3)
根据式(14-1)、式(14-2)和式(14-3),可得:
5R
L  1  D   0.2mH (14-4)
fs

(b) 电感电流的最大值为:

V TsV 1  D 
I max  I  i    5.5A (14-5)
R 2L

- 475 -
(c) 首先,决定铁芯的型号,即:
 L2 I max
2
Kg  2
108 (14-6)
Bmax RK u

这里的 R 切记不是负载电阻,是电感绕组的电阻!
根据题意,设定电感绕组的电阻上的铜耗在室温下为 1W ,即:
I 2 R  1W  R  0.04 (14-7)
将式(14-7)代入到式(14-6)中,可得:
K g  0.167cm5 (14-8)

根据书中的附录 D.2 可知,型号为 EE40 的铁氧体铁芯其 K g  0.209cm5 ,刚

好满足式(14-8),因此铁芯型号选择 EE40。对于 EE40 型号的铁芯,根据书中附


录 D.2 中的数据,有:

Ac  1.27cm 2
(14-9)
MLT  8.5cm
其次,决定气隙的长度,即:
0 LI max
2
lg  2
104  9.578 104 m  0.9578mm (14-10)
B max Ac

再次,决定电感绕组的匝数,即:
LI max
n 104  34.646  35 (14-11)
Bmax Ac

最后,关于绕组的线缆型号,有:
KW
Aw  u A  0.0157cm2 (14-12)
n
根据书中的附录 D.6,选择型号为 AWG #16 的线缆,其横截面积为:
Aw  13.07 103 cm2 (14-13)

故电感绕组的电阻为:
 MLT
Rn  0.039 (14-14)
Aw

而式(14-14)刚好满足式(14-7)。

Problem 14.2

某 Boost 变换器,其静态运行点为: Vg  28V , V  48V , PLoad  150W ,

f s  100kHz 。对此变换器的电感进行设计。设计电感的值,使得电感电流纹波

- 476 -
为其直流分量的 10% 。最大磁密为 0.225T ,填充系数 K u  0.5 。室温下,电感绕

组的铜耗允许为负载功率的 0.5% 。铁芯的型号为 PQ 铁芯,需要指定铁芯型号、


气隙长度、线缆型号(AWG)和匝数。
Solution:
电感电流的直流分量为:

V2
Vg I   PLoad  I  5.357A (14-15)
R
对于电感电流的纹波 i ,有:
2 i
Vg  L
DTs
(14-16)
Vg
V
1 D
而在设计上,还需:
i  0.1I (14-17)
根据式(14-15)、式(14-16)和式(14-17)可得:
L  0.1089mH (14-18)
电感电流的最大值为:
I max  I  i  5.893A (14-19)

首先,决定 PQ 类型铁芯的具体型号,即:
 L2 I max
2
Kg  2
108 (14-20)
Bmax RK u

由于室温下电感绕组的阻值产生的铜耗,允许为其负载功率的 0.5% ,故有:


0.5%  PLoad
I 2 R  0.5%  PLoad  R   0.026 (14-21)
I2
将式(14-21)代入到式(14-20)中,可得:
K g  0.107cm5 (14-22)

根据书中附录 D.5 可知,型号为 PQ26 / 25 的铁芯满足式(14-22),其几何系

数 K g 为:

K g  0.125cm5 (14-23)

对于 PQ26 / 25 的铁芯,根据书中附录 D.5,有:

- 477 -
Ac  1.18cm 2
WA  0.503cm 2 (14-24)
MLT  5.62cm
其次,决定气隙长度,即:
0 LI max
2
lg  2
104  7.954 104 m  0.7954mm (14-25)
B max Ac

再次,决定电感绕组的匝数,即:
LI max
n 104  24.168  25 (14-26)
Bmax Ac

最后,关于电感绕组上的线缆型号,有:
KW
Aw  u A  0.01cm2 (14-27)
n
根据书中的附录 D.6,可知型号为 AWG #18 的线缆满足式(14-27)的要求。对
于 AWG #18 型号的线缆,有:
Aw  8.228 103 cm2 (14-28)

故电感绕组的阻值为:
 MLT
Rn  0.024 (14-29)
Aw

式(14-29)和式(14-21)十分接近,故不需要再对绕组线缆的横截面积进行调整。

Problem 14.3

使用扩展的 K g 法对两绕组变压器进行设计,此变压器的匝数比为 1: n 。由

于此变压器几乎不存储能量,因此不需要气隙,故两绕组的电流比 i2 (t ) / i1 (t ) 本

质上等于匝数比 n 。原边上的伏秒积 1 由书中 Fig13.45(b)所示的 PWM 电压波

形 v1 (t ) 产生。此伏秒积使得最大磁密 Bmax  B ,也即磁密 B(t ) 不含有直流分量,

如书中 Fig13.46 所示。你应该分配半个铁芯窗口面积给各个绕组,总的铜耗 Pcu 已

经指定,你可以忽略临近效应。
(a) 给定以下设计变量:总的铜耗 Pcu 、最大磁密 Bmax 、填充系数 Ku 、绕组电

阻率  、原边绕组电流有效值 I1 、原边伏秒积 1 和匝数比 1: n 。推导此变压器的

- 478 -
设计步骤,你推导的设计步骤应该可以决定以下值:所需要的铁芯几何常数 K g 、

原副边匝数 n1 和 n2 、原副边绕组的横截面积 Aw1 和 Aw 2 。

(b) 对于如书中 Fig6.40C(应该是 Fig6.40C,而不是 Fig6.41C)所示的 Cuk


变换器,其中变压器的原边电压波形为:当开关管导通时,原边电压为输入电压
Vg ,当二极管导通时,原边电压为 Vg D / 1  D  。此变换器的开关频率为 100kHz ,

稳态占空比的值为 D  0.4 。输入电压 Vg  120V ,输出电压 V  24V ,负载功率

为 200W ,你可以将填充系数设定在 K u  0.3 ,使用(a)中的设计步骤来设计此

变压器。最大磁密设定在 Bmax  0.15T ,100 时的总的铜耗为 Pcu  0.25W 。使用

PQ 铁芯,需指定铁芯型号、原副边匝数和绕组导线的型号。

Solution:
(a) 总的铜耗为:
k k n j MLT
Pcu   I 2j R j   I 2j  (14-30)
j 1 j 1 Awj

对于各个绕组的横截面积,有:
n j Awj   j KuWA (14-31)

将式(14-31)代入到式(14-30)中,可得:
k n 2j I 2j  MLT
Pcu   (14-32)
j 1 K uWA j

为了使铜耗最小,有:
nm I m
m  k
(14-33)
n I
j 1
j j

将式(14-33)代入到式(14-32)中,可得:
2
 MLT   k
Pcu    njI j  (14-34)
KuWA  j 1 
而:
k
 n nk 
n I j j  n1  I1  2 I 2  I k   n1 I tot (14-35)
j 1  n1 n1 

将式(14-35)代入到式(14-34)可得:

- 479 -
 MLTn12 I tot2
Pcu  (14-36)
K uWA

对于原边的伏秒积,又有:
d dB B
V1  n1  n1 Ac  n1 Ac  V1DTs  n1 Ac B
dt dt DTs (14-37)
1  n1 Ac Bmax
将式(14-37)的第 2 子式代入到式(14-36)中,可得:
2
I tot MLT 12 Ac2WA 2
I tot 12
Pcu   K g   (14-38)
Ac2 Bmax
2
KuWA MLT Pcu Ku Bmax 2

根据式(14-38)可确定铁芯的型号,进而确定 Ac 、 WA 和 MLT 。

接下来可确定原边的匝数,根据式(14-37)可得:
1
n1  (14-39)
Ac Bmax

再根据匝数比可确定其余绕组的匝数。
最后,可确定各绕组导线的型号,即有:
 j KuWA
n j Awj   j KuWA  Awj  (14-40)
nj

其中  j 由式(14-33)给出。

(b) 书中 Fig6.40(c)所示的 Cuk 变换器,这里重新画出,如图 14-1 所示:


L1 C1a C1b L2

Vg Q1 D1 C2 R v

1: n
图 14-1 隔离型 Cuk 变换器
当开关管导通时,Cuk 变换器如图 14-2 所示:
iL1 L1 iC1 C1a i1 i2 C1b iL 2 L2
vL1 vC1 vC 2 vL 2 iC 2
Vg Q1 v1 v2 C2 R v

1: n
图 14-2 开关管导通时的 Cuk 变换器
故有:

- 480 -
vL1  Vg
vL 2  Vc 2  v2  V
v1 1

v2 n
v1  Vc1 (14-41)
i1
 n
i2
V
i2  I L 2 
R
当开关管关断时,电路如图 14-3 所示:
iL1 L1 iC1 C1a i1 i2 C1b iL 2 L2
vL1 vC1 vC 2 vL 2 iC 2
Vg v1 v2 D C2 R v
1

1: n
图 14-3 开关管关断时的 Cuk 变换器
故有:
vL1  Vg  v1  Vc1
i1  I L1
v1 1

v2 n
(14-42)
vL 2  V
v2  Vc 2
i1
 n
i2
根据式(14-41)和式(14-42),可得:
 V  nVg  nVc1 
DVg  1  D   c 2 0
 n 
D  nVc1  Vc 2  V   1  D  V   0
 V 
D  Vc1   1  D    c 2   0 (14-43)
 n 
D  nI L 2   1  D  I L1   0
V
I L2 
R
根据式(14-43)可求出:

- 481 -
n
1  D V
DVg
V
I L2  (14-44)
R
nDI L 2
I L1 
1 D
将已知参数值代入到式(14-44)中,可得:
n  0.3
I L 2  8.333A (14-45)
I L1  1.667A

故可画出两绕组的电流波形如图 14-4 所示:

i2 (t )
i1 (t )
8.333A

1.667A
0
DTs Ts t DTs Ts t
2.50A

5.556A

图 14-4 两绕组的电流波形
根据图 14-4 可求出两绕组电流的有效值为:

I1  D   2.5   1  D   1.667   2.041A


2 2

(14-46)
I 2  D   8.333  1  D    5.556   6.804A
2 2

故根据式(14-46)可求出:
I tot  I1  nI 2  4.083A (14-47)

注意式(14-38)的单位为标准单位,即和长度、面积相关的单位为 m ,如使用
cm 为单位时,则式(14-38)则为:
Ac2WA I 2  2
K g  cm5    tot 12 108 (14-48)
MLT Pcu Ku Bmax

将已知参数值和式(14-47)代入到式(14-48),可得:
K g  0.523cm5 (14-49)

根据式(14-49)和书中附录 D.5 可知,型号为 PQ 35/35 的铁芯满足式(14-49)的

- 482 -
要求,故铁芯型号选择 PQ 35/35 ,则有:

Ac  1.96cm2
WA  1.61cm2 (14-50)
MTL  7.52cm
接下来,要确定绕组的匝数,将式(14-50)代入到式(14-39)中,注意式(14-39)

的单位为 m 2 ,故有:

n1  16.327  16 (14-51)

则第 2 个绕组的匝数为:
n2  n  n1  4.898  5 (14-52)

最后再根据式(14-40)确定绕组的导线型号,同样要注意式(14-40)的单位,故
有:
1 KuWA
Aw1 
n1
 2 KuWA
Aw 2  (14-53)
n2
1   2  0.5
根据式(14-53)可得:
Aw1  0.015cm 2
(14-54)
Aw 2  0.048cm 2
根据书中附录 D.6,可知当原边绕组导线型号选择为 AWG#16 ,副边绕组导
线型号选择为 AWG#11 时,可满足式(14-54)的要求。
故当原边绕组导线选择 AWG#16 型号时,副边绕组导线选择 AWG#11 型号
时,此时的铜耗为:
 
Pcu  n1MLT I12  n2 MLT I 22  0.184W (14-55)
Aw1 Aw 2

注意式(14-55)中的电阻率为 100 时的数据。

Problem 14.4

耦合电感设计。如书中 Fig13.47(a)所示的两输出正激变换器的副边使用
耦合电感。此耦合电感需要使用气隙。
此耦合电感的应用条件为:V1  5V 、V2  15V 、I1  20A 、I 2  4A 、D  0.4 。

相对于 5V 的原边绕组来说,激磁电感的电感值为 8μH 。填充系数你可以设定为

- 483 -
K u  0.5 。 100 时的总铜耗允许为 1W 。最大磁密设定在 Bmax  0.2T 。该耦合电

感使用 EE 类型的铁芯,需要指定铁芯的具体型号、气隙长度、匝数和绕组导线
的型号。
注意:我认为此题缺少开关频率这个给定条件,故接下来的求解以
f s  2 0 0 k H z为前提条件!

Solution:
绕组 1 的激磁电流纹波 im 为:

V1 DTs
im   0.938A (14-56)
2 Lm

激磁电流的直流分量为:
n2
I m  I1  I 2  32A (14-57)
n1

故激磁电流的最大值为:
I m max  I m  im  32.938A (14-58)

而折算到原边绕组的总的电流有效值为:
n2
I tot  I1  I 2  32A (14-59)
n1

根据式(14-58)和式(14-59)算出的值,接下来可确定铁芯的几何常数:
 L2m I tot2 I m2 max
Kg  2
108  0.818 (14-60)
B P Ku
max cu

根据书中附录 D.2,可知 EE50 型号的铁芯满足式(14-60)要求,对于 EE50 型


号的铁芯,有:
Ac  2.26cm2
WA  1.78cm2 (14-61)
MLT  10.0cm
接下来,确定气隙长度:
0 Lm I m2 max
lg  2
104  1.206 103 m  1.206mm (14-62)
Bmax Ac

然后,确定两绕组的匝数,即:

- 484 -
Lm I m max 4
n1  10  5.83  6
Bmax Ac
(14-63)
V
n2  2 n1  17.489  18
V1
对于两绕组的窗口分配系数,有:
n1 I1
1   0.625
n1I1  n2 I 2
(14-64)
n2 I 2
1   0.375
n1I1  n2 I 2
最后,确定绕组导线的具体型号,故有:
1KuWA
Aw1   0.093cm2
n1
(14-65)
 2 KuWA
Aw2   0.019cm 2

n2
根据书中附录 D.6,可知当绕组 1 的导线型号选择为 AWG#8 时,绕组 2 的
导线型号选择为 AWG#15 时,可满足式(14-65)要求。
最后,再核算一下总的铜耗,即:
n1MLT  2 n2 MLT  2
Pcu  I1  I 2  1.061W
Aw1 Aw2
Aw1  83.67 103 cm 2 (14-66)
Aw 2  16.51103 cm 2

Problem 14.5

反激变压器设计。反激变换器的输入为 Vg  160V 、其输出为 V  28V 。最

大的负载电流为 I max  2A 。变压器的匝数比为 8 :1 。开关频率为 f s  100kHz 。变

换器在满载时处于 DCM 模式运行。总的铜耗应该小于 0.75W 。


(a) 设计激磁电感的值,使得满载时 D3  0.1 。指出此激磁电感的值是相对原

边还是副边的。此时晶体管的峰值电流是多少?二极管的峰值电流是多少?
(b) 对此变换器中的变压器进行设计,使用 POT 类型的铁氧体铁芯,最大磁
密为 Bmax  0.25T ,填充系数为 K u  0.4 。需指定铁芯的具体型号,原副边的匝数,

气隙长度和绕组导线的具体型号。
(c) 对于(b)部分的设计,计算原副边绕组的铜耗,你可以忽略临近效应。
(d) 对于(b)部分的设计,计算铁耗,铁芯材料的损耗数据已在书中 Fig13.20

- 485 -
中给出,铁耗是否小于(c)部分计算的铜耗?
Solution:
(a) 反激变换器的电路如图 14-5 所示:
1: n D1

Lm C V
Vg
Q1

图 14-5 反激变换器
当开关管导通时,反激变换器的电路如图 14-6 所示:
ig i1 1: n
i iC

vT Lm C V
Vg
Q1

图 14-6 开关管导通时的反激变换器
故有:
vT  Vg (14-67)

当开关管关断时,且激磁电感中的电流未断续之前时,反激变换器的电路如
图 14-7 所示:
ig i1 1: n i2 D1
i iC

vT Lm C V
Vg

图 14-7 开关管关断且激磁电流未断续时的反激变换器
故有:
V
vT   (14-68)
n

- 486 -
当开关管关断时,且激磁电感直流断续之后的反激变换器如图 14-8 所示:
ig i1 1: n
i iC

vT Lm C V
Vg

图 14-8 开关管关断,且激磁电感电流断续之后时的反激变换器
故有:
vT  0 (14-69)

根据式(14-67)、式(14-68)和式(14-69)可得:

 V
1 g  D2  
DV 0 (14-70)
 n

激磁电感中的电流 i 和二极管电流 i2 的波形如图 14-9 所示:

i2
i
i2 pk
i pk
I2
0 t 0 t
D1TS D2TS D3TS
D1TS D2TS D3TS

图 14-9 激磁电感电流和二极管电流波形
根据图 14-9 可有:
I 2  I max
1 1
I2  D2Ts i2 pk (14-71)
Ts 2
i pk Vg
i2 pk   D1Ts
n nLm
根据式(14-71)和式(14-70)可求出:
V 1  D3 
D1 
V  nVg
(14-72)
D12TsVg2
Lm 
2 I maxV

- 487 -
将已知的参数值代入到式(14-72)中,(注意 n  1/ 8 )可得:
D1  0.525
D2  0.375 (14-73)
Lm  0.63mH
此激磁电感的值是相对于原边的。
晶体管的峰值电流为:
Vg
i1 pk  D1Ts  1.333A (14-74)
Lm

二极管的峰值电流为:
i1 pk
i2 pk   10.667A (14-75)
n
(b) 原副边绕组的电流波形如图 14-10 所示:
is
ip
i2 pk
i pk
I2
0 t 0 t
D1TS D2TS D3TS
D1TS D2TS D3TS

图 14-10 原副边绕组的电流波形
其中:
i pk  i1 pk  1.333A
(14-76)
i2 pk  10.667A

根据书中附录 A 中的式(A.8),可求出原副边绕组电流有效值为:
D1
I1  i1 pk  0.558A
3
(14-77)
D2
I 2  i2 pk  3.771A
3
故折算到原边的总的电流有效值为:
I tot  I1  nI 2  1.029A (14-78)

接下来,可确定出铁芯的几何常数,即:
 L2m I tot2 I m2 max
Kg  2
108  6.872 103 cm5 (14-79)
B P Ku
max cu

根据书中附录 D.1,可知型号为 POT1811 的铁芯满足式(14-79)要求,但是此

- 488 -
题要选择 POT2213 的铁芯,因为选择 POT1811 铁芯的话,会发现最后算出的导
横截面积过小,以至于在书中附录 D.6 中没有合适的。故选择 POT2213 铁芯:
Ac  0.635cm2
WA  0.297cm2 (14-80)
MLT  4.42cm
接下来,可确定气隙长度:
0 Lm I m2 max
lg  2
104  0.3546mm (14-81)
Bmax Ac

然后,确定原副边绕组的匝数,即:
Lm I m max 4
n1  10  52.913  53
Bmax Ac (14-82)
n2  n  n1  6.614  7

对于原副边绕组的铁芯窗口分配系数,有:
n1 I1
1   0.542
n1I1  n2 I 2
(14-83)
n2 I 2
2   0.484
n1 I1  n2 I 2

此次会发现 1   2  1 ,这是由于绕组匝数理论上应为小数,而实际进行了

取整,而取整会导致铁芯窗口的分配系数产生微小的误差。
最后,确定绕组导线的具体型号,故有:
1 KuWA
Aw1   1.215 103 cm 2
n1
(14-84)
 2 KuWA 3
Aw 2   8.214 10 cm 2

n2
根据书中附录 D.6 和式(14-84)可知,绕组 1 的导线型号应选为 AWG#27 ,
绕组 2 的导线型号应选为 AWG#19 。
(c) 原边绕组的铜耗为:
Pcu _ p  n1MLT  p I12  0.123W
(14-85)
 p  1687.6 106 Ω/cm
副边绕组的铜耗为:
Pcu _s  n2 MLT s I 22  0.116W
(14-86)
s  263.9 106 Ω/cm
故总铜耗为:
Pcu  Pcu _ p  Pcu _ s  0.239W (14-87)

- 489 -
式(14-87)算出的结果和设计值 Pcu  0.75W 相差较大,是因为选择了几何常

数较大的铁芯(选择几何常数较大的铁芯是因为导线的型号可以找到,而如果选
择几何常数与理论计算接近的铁芯,则最后在选择导线型号时,会发现没有合适
的导线),选择几何常数较大的铁芯,会降低铜耗,这与式(14-87)的计算结果相
符。
(d) 磁密的变化量 B (注意是磁密的总变化量的一半,类似 CCM 电流波形
的纹波)为:
Vg D1Ts
B  104  0.125T (14-88)
2n1 Ac

根据书中 Fig13.20 可知,铁耗为:


Pfe   0.1W/cm3   Aclm   0.2W
Ac  0.635cm 2 (14-89)
lm  3.15cm
根据式(14-89)可知,铁耗和铜耗差不多。这是因为 DCM 模式下的激磁电流
的变化量要远大于 CCM 模式下激磁电流变化量,进而使 DCM 模式下的 B 要远
大于 CCM 模式下的 B ,进而造成较大的铁耗,这也是 CCM 模式下可以忽略铁
耗,而 DCM 模式下最好不要忽略铁耗的原因。

- 490 -
第15章 变压器设计

Problem 15.1

正激变换器的电感和变压器设计。此题的任务是设计如图 15-1 所示的双管


两输出正激变换器的两个电感和变压器。三个磁性器件均使用铁氧体磁芯,其饱
和磁密在 120 时近似为 0.3T 。为了提供一定的安全裕度,最大磁密 Bmax 不应超

过饱和磁密的 75% 。当开关频率为 100kHz 时,铁耗可由书中式 15.1 描述,其中


参数为 β  2.6 , K fe  50 ,需计算 100 时的铜耗。

稳态变换器分析和设计。你可以假设变换器具有 100% 的效率,且元器件均


为理想器件。
(a) 选择变压器的匝数比使得当占空比 D  0.4 时,可获得所需要的输出电压。
(b) 指定电感 L1 和 L2 的值,使得它们的纹波 i1 和 i2 是它们各自满载电流直

流分量的 10% 。
(c) 给出变压器绕组和电感电流的有效值和峰值。
电感设计。
运行电感 L1 的铜耗为 1W ,电感 L2 的铜耗为 0.4W 。填充系数 K u  0.5 。使

用 EE 类型的铁氧体磁芯,对这两个电感进行设计,需要指定具体的铁芯型号、
气隙长度、匝数、导线型号。
变压器设计。
允许总的损耗为 1W ,填充系数 Ku  0.35 (考虑绕组之间的绝缘影响,此填

充系数会比滤波电感的填充系数要小)。使用 EE 类型铁氧体磁芯,你可以忽略
由集肤效应和临近效应产生的铜耗,但是要考虑到铁耗和直流铜耗。对此变压器
进行设计,需要指定具体的铁芯型号、匝数 n1 、 n2 和 n3 、绕组导线的型号。

核对变压器的设计。
需要计算最大磁密,此最大磁密是否会使铁芯饱和?计算铁耗和各个绕组的
铜耗及总损耗。

- 491 -
L1 i1
vL1
n2 5V
Vg n1 V1
30A
325V

L2 i2
n3 vL 2
15V
V2
1A

图 15-1 双管两输出正激变换器
Solution:
(a) 当开关管导通时,即 0  t  DTs 时,有:

n2
vL1  Vg  V1
n1
(15-1)
n
vL 2  Vg 3  V2
n1

当开关管关断时,即 DTs  t  Ts 时,有:

vL1  V1
(15-2)
vL 2  V2

故根据式(15-1)和式(15-2)有:
 n2 
D  Vg  V1   1  D  V1   0
 n1 
(15-3)
 n3 
D  Vg  V2   1  D  V2   0
 n1 
根据式(15-3)可得到匝数比为:
n2 V
 1
n1 DVg
(15-4)
n3 V
 2
n1 DVg
将已知参数代入到式(15-4)中,可得:

- 492 -
n2
 0.0385
n1
(15-5)
n3
 0.1154
n1

两电感的电流纹波 i1 和 i2 为:

2i1
V1  L1
1  D  Ts
(15-6)
2i2
V2  L2
1  D  Ts
设计上又要求:
i1  0.1I1
(15-7)
i2  0.1I 2

根据式(15-6)和式(15-7)可得:
L1  5μH
(15-8)
L2  0.45mH

两电感电流的有效值和峰值为:
I1rms  30A
I1 pk  I1  i1  33A
(15-9)
I 2 rms  1A
I 2 pk  I 2  i2  1.1A
式(15-9)中有效值的计算没有考虑纹波的影响,这是因为根据书中附录 A 中
的式(A.2)可知, 10% 的电流纹波对其有效值的影响几乎可以忽略。
根据书中附录 A 中的式(A.6)可算出变压器副边绕组电流的有效值和峰值:
2
1  i 
I s1rms  I1 D 1   1   I1 D  18.974A
3  I1 
2
1  i  (15-10)
I s 2 rms  I 2 D 1   2   I 2 D  0.632A
3  I2 
I s1 pk  I1 pk  33A
I s 2 pk  I 2 pk  1.1A
原边绕组电流的有效值和峰值为:

- 493 -
I prms  I p D  0.803A
n2 n
Ip  I1  3 I 2  1.269A (15-11)
n1 n1
n2 n
I ppk  I1 pk  3 I 2 pk  1.396A
n1 n1
对于两电感的设计,如果使电感 1 的铜耗为 1W ,电感 2 的铜耗为 0.4W 。
故有:
I12rms R1  1W
(15-12)
I 22rms R2  0.4W
根据式(15-9)和式(15-12)可求得:
R1  1.111103 
(15-13)
R2  0.4

首先确定两电感的铁芯型号:
 L12 I12pk
K g1  2
108
Bmax R1 Ku
(15-14)
 L22 I 22pk
Kg2  2
108

Bmax R2 Ku
将式(15-13)和已知参数值代入到式(15-14)中,可得:
K g1  0.223cm5
(15-15)
K g 2  5.566 103 cm5
根据书中附录 D.2 可知,电感 1 的铁芯型号选择 EE50 时,电感 2 的铁芯型
号选择 EE22 时,可满足式(15-15)。对于电感 1 的 EE50 型号的铁芯和电感 2 的
EE22 型号的铁芯有:
Ac1  2.26cm 2
WA1  1.78cm 2
MLT 1  10.0cm
(15-16)
Ac 2  0.41cm 2
WA 2  0.196cm 2
MLT 2  3.99cm
接下来,可确定电感 1 和电感 2 的气隙长度,即:
0 L1 I12pk
lg1  2
104  0.598mm
Bmax Ac1
(15-17)
0 L2 I 22pk
lg 2  2
104  0.3297mm
Bmax Ac 2
然后,可确定电感 1 和电感 2 的匝数,即:

- 494 -
L1 I1 pk
n1  104  3.245  4
Bmax Ac1
(15-18)
L2 I 2 pk
n2  104  53.659  54
Bmax Ac 2
最后,可确定电感 1 和电感 2 的导线型号,即:
KuWA1
Aw1   223 103 cm 2
n1
(15-19)
KW
Aw2  u A2  1.815 103 cm2
n2
根据书中附录 D.6 可知,电感 1 的导线型号选择 AWG#4 时,电感 2 的导线
型号选择 AWG#25 时,式(15-19)可满足。
接下来设计变压器,首先确定变压器的铁芯型号,即:
12 Itot2 K fe2/  
K gfe      2 /  
108 (15-20)
4 Ku Ptot
其中:
1  Vg DTs  1.3 103 V  s
n2 n (15-21)
I tot  I prms  I s1rms  3 I s 2 rms  1.605A
n1 n1
将式(15-21)代入到式(15-20)中,可得:
K gfe  0.0145 (15-22)

根据书中附录 D.2 可知,EE50 型号的铁芯满足式(15-22)要求。对于 EE50 型


号的铁芯,有:
Ac  2.26cm 2
WA  1.78cm 2
(15-23)
MLT  10.0cm
lm  9.58cm
根据式(15-23)可确定交流磁密的峰值为:
 1 
 
  2 I 2 MLT 1   2 
B  108 1 tot   0.044T (15-24)
 2 Ku WA Ac3lm  K fe 
接下来可确定原副边各绕组的匝数,即:

- 495 -
1
n1  104  65.767  66
2BAc
n 
n2  n1  2   2.53  3 (15-25)
 n1 
n 
n3  n1  3   7.589  8
 n1 
原副边绕组的窗口分配系数为:
n1 I prms
1   0.5
n1 I tot
n2 I s1rms
2   0.455 (15-26)
n1 I tot
n3 I s 2 rms
3   0.045
n1 I tot
接下来,确定原副边绕组的导线型号:
1 KuWA
Awp   4.72 103 cm 2
n1
 2 KuWA
Aws1   94 103 cm 2 (15-27)
n2
 3 KuWA
Aws 2   3.54 103 cm 2
n3
根据书中附录 D.6 可知,原边绕组导线型号选择 AWG#21 ,副边绕组 1 导线
型号选择 AWG#8 ,副边绕组 2 导线型号选择 AWG#22 。
根据式(15-24)可知,交流磁密的峰值远小于饱和磁密,故不会导致变压器的
铁芯饱和。
最后,对于铁耗的核算有:
Pfe  K fe  B  Aclm  0.317W

(15-28)

对于原副边绕组的铜耗,有:
n1MLT 2
Pcup   I prms  0.238W
Awp
n2 MLT 2
Pcus1   I s1rms  0.297W (15-29)
Aws1
n3 MLT 2
Pcus 2   I s 2 rms  0.023W
Aws 2
故总的损耗为:
P  Pcup  Pcus1  Pcus 2  Pfe  0.874W (15-30)

- 496 -
Problem 15.2

某单管正激变换器其输入电压为 Vg  160V ,具有两路输出,一路输出电流

2A ,输出电压 24V ;另一路输出电流 6A ,输出电压 15V 。占空比为 D  0.4 。


原边绕组和复位绕组的匝数比为 1:1 ,开关频率为 100kHz 。铁耗方程的参数为
β  2.7 ,K fe  50 。你可以假设填充系数 K u  0.25 。铁芯最大磁密不能超过 0.3T 。

对此应用场合的变压器进行设计,其总损耗在 100 时不能超过 1.5W 。可以


忽略临近效应造成的损耗,也可以忽略复位绕组。此变压器使用 PQ 类型的铁芯,
需要指定具体的铁芯型号、交流磁密的峰值、各绕组的匝数和绕组导线的具体型
号。最后再核算铜耗和铁耗。
Solution:
此正激变换器如图 15-2 所示:
n1 : n1 : n2 L2 iL 2
i1 i2 vL 2

v1 v2 R2 V2
Vg
D1
Q1 L3 iL 3
i3 vL 3

v3 R3 V3

n1 : n1 : n3
图 15-2 单管双路输出正激变换器
当开关管导通时,有:
n2
vL 2  Vg  V2
n1
(15-31)
n
vL 3  3 Vg  V3
n1
当开关管关断时,有:
vL 2  V2
(15-32)
vL 3  V3

此题认为电感 L2 和 L3 足够大,使得变换器工作在 CCM 模式,故有:

- 497 -
n 
D  2 Vg  V2   1  D  V2   0
 n1 
(15-33)
n 
D  3 Vg  V3   1  D  V3   0
 n1 
根据式(15-33)可求得:
n2 V
 2
n1 DVg
(15-34)
n3 V
 3
n1 DVg
对于两电感电流,有:
V2
I L2 
R2
(15-35)
V
I L3  3
R3
将已知的参数值代入到式(15-34)和式(15-35)中,可得:
n2
 0.375
n1
n3
 0.234 (15-36)
n1
I L 2  2A
I L 3  6A

副边绕组 i2 (t ) 和 i3 (t ) 的波形如图 15-3 所示:

i2 (t ) i3 (t )

I L2 I L3

0 DTS TS t 0 DTS TS t

图 15-3 副边绕组的两电流波形
根据书中附录 A 中的式(A.5)可知,副边绕组两电流的有效值为:
I 2 rms  I L 2 D  1.265A
(15-37)
I3rms  I L3 D  3.795A
原边绕组的电流波形和图 15-3 一样,其有效值为:
n I n I 
I1rms   2 L 2 3 L 3  D  1.364A (15-38)
 n1 

- 498 -
将副边电流有效值折算到原边,可得总的原边电流有效值为:
n2 n
I tot  I1rms  I 2 rms  3 I 3rms  2.727A (15-39)
n1 n1

原边绕组的伏秒积为:
1  Vg DTs  6.4 104 V  s (15-40)

首先,确定铁芯的具体型号,即:
12 Itot2 K fe2/  
K gfe      2 /  
108 (15-41)
4 Ku Ptot
如果在设计总损耗时,不考虑留有一些裕度的话,即将总损耗设置在 1.5W ,
则将已知的参数值代入到式(15-41)中,可得:
K g  6.2744 103 cm5 (15-42)

则根据书中附录 D.5 可知,变压器铁芯型号选择 PQ 26/20 可满足式(15-42)要

求,如果铁芯型号选择 PQ 26/20 的话,根据附录 D.5 中的数据,可有:

Ac  1.19cm 2
WA  0.333cm 2
(15-43)
MLT  5.62cm
lm  4.63cm
接下来确定交流磁密的峰值,即:
 1 
 
  2 I 2 MLT 1   2 
B  108 1 tot   0.103T (15-44)
 2 Ku WA Ac3lm  K fe 
根据式(15-44)可知,交流磁密的峰值远小于饱和磁密,故铁芯达不到饱和状
态。
接下来确定原副边绕组的匝数,即:
1
n1  104  26.222  26
2BAc
n 
n2  n1  2   9.833  10 (15-45)
 n1 
n 
n3  n1  3   6.146  6
 n1 
原副边各绕组在铁芯窗口内所占面积的分配系数为:

- 499 -
n1 I prms
1   0.5
n1 I tot
n2 I s1rms
2   0.174 (15-46)
n1 I tot
n3 I s 2 rms
3   0.326
n1 I tot
接下来,可确定原副边绕组的导线型号,有:
1 K uWA
Awp   1.601103 cm 2
n1
 2 KuWA
Aws1   1.448 103 cm 2 (15-47)
n2
 3 K uWA
Aws 2   4.524  103 cm 2
n3
根据书中附录 D.6 可知,原边绕组导线型号选择 AWG#26 ,副边绕组 1 导
线型号选择 AWG#26 ,副边绕组 2 导线型号选择 AWG#21 。
最后要核算总的损耗,首先核算铁耗,即:
Pfe  K fe  B  Aclm  0.588W

(15-48)

对于已选择的原副边绕组导线的型号,有:
Awp  1.28 103 cm 2
Aws1  1.28 103 cm 2 (15-49)
Aws 2  4.116 103 cm 2
对于原副边绕组的铜耗,有:
n1MLT 2
Pcup   I1rms  0.488W
Awp
n2 MLT 2
Pcus1   I 2 rms  0.162W (15-50)
Aws1
n3 MLT 2
Pcus 2   I 3rms  0.271W
Aws 2
故总的损耗为:
P  Pcup  Pcus1  Pcus 2  Pfe  1.51W (15-51)

Problem 15.3 (题中数据有问题?)

反激或 SEPIC 变换器中的变压器设计。对于反激或 SEPIC 变换器中的变压


器,其实属于能量存储器件,可以用多绕组电感来更准确得描述它。变压器中激
磁电感的作用类似一般变换器中的存储能量的电感,因此反激或 SEPIC 变换器

- 500 -
中的变压器需要含有气隙。变换器也许会设计工作在 CCM 模式,也许会设计工
作在 DCM 模式。变压器的铁耗是较大的,故不能忽略。此外,也要确保激磁电
流的峰值不能过大,以使得铁芯发生饱和。
需要对某两输出反激变换器设计变压器,具体参数如下:
输入电压: Vg  1 6 0 V

输出 1: I 2 dc  1 0 A、 V2  5V

输出 2: I 3dc  1A 、 V3  15V

开关频率: fs  1 0 0 k H z

激磁电感(相对原边): Lp  1.33mH

匝数比: 160 : 5 :15


变压器允许的总损耗: 1W
(a) 变换器运行于 CCM 模式还是 DCM 模式?相对于原边绕组,激磁电流的
纹波 i 为多少?激磁电流的直流分量为多少?激磁电流的峰值 I pk 是多少?

(b) 推导各绕组电流的有效值。所施加的原边绕组伏秒积 1 。 1 是否与 I pk 成

正比?
(c) 修改变压器和 ac 电感的设计步骤,来获得反激变换器中变压器的推导步
骤,此步骤应考虑到铁耗和铜耗,此外通过选择合适的交流磁密使得总损耗最小。
(d) 给出一步一步的设计步骤,要注明指标和单位。
(e) 使用(d)中的步骤对此变压器进行设计,此变压器使用 EE 类型的铁芯,
β  2.7 、 Kfe  50W/Tβcm3 ,需要指定铁芯的具体型号、气隙长度、原副边匝数

和原副边绕组导线型号。
(f) 对于(e)中的设计,核算铁耗、铜耗和磁密峰值。
Solution:
(a) 反激变换器如图 15-4 所示:
ig i1 i2
i
C R2 V2
vT Lp
Vg
i3
Q1
C R3 V3

160 : 5 :15
图 15-4 反激变换器

- 501 -
假设变换器工作在 CCM 模式,则当开关管导通时,有:
vT  Vg
2i
vT  Lp (15-52)
DTs
n1i  n2i2  n3i3
当开关管关断时,有:
n1 n
vT  V2  V3 1
n2 n3
(15-53)
2i
vT  Lp
DTs
根据式(15-52)可算出稳态时的占空比为:
V2 n1
D (15-54)
V2 n1  Vg n2

CCM 模式下, i2 (t ) 和 i3 (t ) 的波形如图 15-5 所示:

i2 (t ) i3 (t )

I2 I3
I 2dc I 3dc

0 DTS TS 0 DTS TS

图 15-5 i2 (t ) 和 i3 (t ) 的波形

根据图 15-5 则有:


I 2 dc
1  D  I 2  I 2 dc  I 2 
1  D 
(15-55)
n n 2i2 T V n 1  D 
2

V2 1  2 Lp  i2  s 2 1 2
n2 n1 1  D  Ts 2 Lp n2
故变换器运行于 CCM 模式的前提条件为:
I 2  i2 (15-56)

将已知参数值代入到式(15-54)和式(15-55)中,可得:
D  0.5
I 2  20 (15-57)
i2  9.624

如果 D  0.5 ,则 5V 输出会工作在 CCM 模式,15V 输出会工作在 DCM 模


式,那么也就是第 1 路输出可以达到 5V,而第 2 路输出达不到 15V,也即两路

- 502 -
输出电压只能满足一个,是不是题中的数据出问题了?此题暂时先放这里吧。

Problem 15.4

当运行频率超出原来设计的频率范围时,对于确定的某个铁氧体铁芯,其铁
耗系数与频率的关系可以使用一个单调递增的 4 次方程近似。即:
  f   f 
2
 f 
3
 f  
4

K fe ( f )  K fe 0 1  1     2     3     4   
  f0   f0   f0   f 0  

其中 K fe 、 a1 、  2 、  3 、  4 和 f 0 为常数。在某个典型变换器的变压器应用

中,原边绕组的伏秒积与开关周期 Ts  1/ f s 直接相关联。需要选择合适的开关频

率,使得 K gfe 、变压器体积最小。

(a) 证明最优的开关频率是以下多项式的根:
2 3 4
  1   f     2  f     3  f     4  f 
1  1      2      3      4   
    f0      f0      f0      f0 
接下来,使用参数如下所示的某铁芯材料:
  2.7K fe 0  7.6
f 0  100kHz
1  1.3 2  5.3
 3  0.5 4  0.075

以上系数适用于 10kHz  f  1MHz 。

(b) 画出 K fe 关于 f 的曲线。

(c) 确定频率 f 使得 K gfe 最小。

(d) 当 100kHz  f  1MHz 时,画出 K gfe  f  / K gfe 100kHz  。变压器的体积对

运行频率有多敏感?
Solution:
(a) 对于变压器的设计,有:
12 I tot2 K fe2/  
K gfe     2/  
 c112 K fe2/  
4 K u Ptot
(15-58)
 I tot2
c1 
4 K u Ptot
   2/  

对于原边绕组的伏秒积有:

- 503 -
c0
1  c0Ts  (15-59)
f

而铁耗系数 K fe 的表达式为:

  f   f 
2
 f 
3
 f  
4

K fe ( f )  K fe 0 1  1     2     3     4    (15-60)
  f0   f0   f0   f 0  

将式(15-59)和式(15-60)代入到式(15-58)中,可得:
2
 K fe 0  4 f 4   3 f 3 f 0   2 f 2 f 02  1 ff 03  f 04   
c0 c1 
2

 f 04 
K gfe  2
(15-61)
f

由于 c0 、 c1 、 K fe 0 为常数,为了求出式(15-61)的最小值,只需求出下式的最

小值:
2

K gfe1 
 4 f   3 f f 0   2 f f 0  1 ff 0  f 0 
4 3 2 2 3 4 

f2
2
(15-62)
 K 
K gfe  c02 c1  fe40  K gfe1
 f0 
对式(15-62)求导,然后令导数等于 0,可得:
(2  f 04  8 f 4 4  4 f 2 f 02 2  2  f 4 4  2 ff 031
(15-63)
 6 f 3 f 0 3  2 ff 031  2 f 3 f 0 3  2 f 2 f 02 2 )  0
将式(15-63)化简,可得:
    4   f 4    3  f 
3
   2  f 
2

  4      3      2    
     f 0      f0      f0 
(15-64)
  1   f  
1      1  0
    f0  

即令 K gfe1 也即 K gfe 取得最小值的最优频率 f 是(15-64)的根。

(b) K fe 关于频率 f 的曲线如图 15-6 所示:

- 504 -
3
610

3
5.2510

3
4.510

3
3.7510
 f
2
f
3
f 
4
Kfe0 1  1   2     3     4   
f 3
310
 f0 f0 f0 f0 
      
3
2.2510

3
1.510

750

0
4 5 5 5 5 5 5 5 6
110 1.33710 2.57510 3.81310 5.0510 6.28710 7.52510 8.76210 110
f

图 15-6 K fe ( f ) 关于 f 的曲线

(c) 首先将参数值代入到式(15-64),然后画出式(15-64)的曲线,如图 15-7 所


示:
180

155

130

105
   4    f 
4
  3    f 
3
  2    f 
2
  1    f   1
  4      3      2      1    80
     f0      f0      f0      f0  

55

30

 20
4 5 5 5 5 5 5 5 6
110 1.33710 2.57510 3.81310 5.0510 6.28710 7.52510 8.76210 110
f

图 15-7 K gef ( f ) 关于 f 的导数

求解式(15-64)可得:
f  6.76 105  0.676MHz (15-65)

根据图 15-7 同样可知,K gfe ( f ) 在 10kHz  f  1MHz 范围内,先单调递减至

极小值(也即最小值)处,然后再单调递增。
故使 K gfe 取得最小值的最优频率为 f  0.676MHz 。

(d) 对于 K gfe  f  / K gfe 100kHz  有:

1  2/  
K fe
K gfe  f  f2
 (15-66)
K gfe 100kHz   1  
2 2

 3 
K (100  10 3
)  
 100 10   
fe

式(15-66)的图形如图 15-8 所示:

- 505 -
1.2

1.1

2 0.9

  2 3
f 
4
Kfe01 1   2    3    4   
f f f
  f0
 f0   f0   f0   0.8
2
f
1.3855847801016840065e-9
0.7

0.6

0.5

0.4
4 5 5 5 5 5 5 5 6
110 1.33710 2.57510 3.81310 5.0510 6.28710 7.52510 8.76210 110
f

图 15-8 K gfe  f  / K gfe 100kHz  的图形

从图 15-8 可以看出,频率对铁芯大小的影响,足足有 3 倍之多。

Problem 15.5

设计变压器以获得给定的温升。铁氧体铁芯中心臂的温升 T 与变压器总损
耗 Ptot 成正比,即 T  Rth Ptot ,这里的 Rth 为给定环境下变压器的热阻。你可以认

为变压器的温升和变压器内损耗的分布无关。故可以对设计变压器的 K gfe 法进行

修改,以便使用温升而不是总损耗 Ptot 作为设计指标。你可以认为温升和导线的

电阻率无关。
(a) 修改 n 绕组变压器的 K gfe 设计法,定义含有热阻 Rth 的新的铁芯几何常数

K th 。

(b) 书中附录 D.3 所示的 EC 类型的铁芯数据已含有热阻数据,如果认为


β  2.7 ,制作书中附录 D.3 所示的铁芯的 K th 的表格。

(c) 某 750W 单输出的全桥隔离 Buck 变换器的运行频率为 f s  200kHz ,直流

输入电压为 Vg  400V ,输出电压 V  48V ,匝数比为 6 :1 。铁耗方程的参数在

100kHz 时的 K fe  10W/Tβ cm3 , β  2.7 。填充系数为 K u  0.3 ,可以忽略临近效

应产生的损耗。使用(a)和(b)中推导的步骤来设计此应用中的变压器,设计
指标的温升要限制在 20 。需要给出 EC 类型铁芯的的具体型号,原副边绕组的
匝数,绕组导线的型号,交流磁密的峰值。

- 506 -
Solution:
(a) 对于设计变压器的 K gfe 法,有:

12 Itot2 K fe2/  


K gfe  (15-67)
4 Ku Ptot
  2 /  

对于热阻 Rth 、温升 T 和总损耗 Ptot 的关系,有:

T  Rth Ptot (15-68)

将式(15-68),可得:
K gfe 12 I tot2 K fe2/  
Kth   (15-69)
Rth
  2 /       2 /  
4 Ku  T 

(b) 使用书中附录 D.3 的数据及式(15-69),可制作 K th 的表格如表 15-1 所示,

其中数据的单位和书中附录 D.3 中数据的单位一致,故表格中不再标明单位:


表 15-1 K th 表格数据

铁芯型号 K gfe 几何常数 热阻 Rth K th 几何常数

EC35 9.9 103 18.5 6.163 105


3
EC41 19.5 10 16.5 1.482 104
3
EC52 31.7 10 11.0 4.878 104
3
EC70 56.2 10 7.5 1.685 103
(c) 单输出全桥隔离型 Buck 变换器如图 15-9 所示:

n1 : n2 i2 IL
vL

Vg i1
v1 C R V

i3

图 15-9 单输出全桥隔离型 Buck 变换器


当开关管导通时,有:
n2
vL  Vg V (15-70)
n1

当开关管关断时,有:

- 507 -
vL  V (15-71)

故有:
n 
D  2 Vg  V   1  D  V   0 (15-72)
 n1 
根据式(15-72)可推得:
V n1
D (15-73)
Vg n2

将已知参数值代入到式(15-73)中,可得:
D  0.72 (15-74)
故原边绕组的伏秒积为:
1  Vg DTs  1.44 103 V  s
1 1 (15-75)
Ts  
f s 200 103

原边绕组电流有效值 I1 为:

n2
I1  I L D  2.21A
n1
(15-76)
P
I L   15.625A
V

副边绕组的电流有效值 I 2 为:

1
I2  I L 1  D  10.246A (15-77)
2
将副边绕组电流有效值折算到原边,可得原边总电流的有效值 Itot 为:

n2
I tot  I1  2 I 2  5.625A (15-78)
n1

接下来,可确定铁芯的具体型号,即根据式(15-69),注意单位的换算,使用

cm 为单位时,需要乘以 108 ,即

12 Itot2 K fe2/  


Kth      2 /  
108  2.82 104 cm5 (15-79)
4 Ku  T 
故根据表表 15-1 选择 EC52 型号的铁芯,根据书中附录 D.3 中的数据,可
知此型号的铁芯参数为:

- 508 -
Ac  1.80cm 2
WA  2.12cm 2
(15-80)
MLT  7.5cm
lm  10.5cm
然后,可求出交流磁密的峰值为:
 1 
 
  2 I 2 MLT 1   2 
B  108 1 tot   0.116T (15-81)
 2 Ku WA Ac3lm  K fe 
接下来,可确定原副边的匝数,即:
1
n1  104  34.436  35
2BAc
(15-82)
n
n2  2 n1  5.739  6
n1
对于原副边绕组在铁芯窗口内的面积分配系数,有:
n1 I1
1   0.393
n1 I tot
n2 I 2
2   0.304 (15-83)
n1 I tot
1  2 2  1
根据式(15-83)可确定原副边绕组的导线型号,即:
1 KuWA
Aw1   7.138 103 cm2
n1
(15-84)
 2 KuWA 3
Aw2   32 10 cm 2

n2
故根据式(15-84)和书中附录 D.6 可知,原边绕组的导线型号应选 AWG#19 ,
副边绕组的导线型号应选 AWG#13 。

- 509 -
第16章 非线性系统中的功率和谐波

Problem 16.1

如图 16-1 所示的无源整流电路中, L 足够大使得电感电流 i (t ) 为直流。所

有的器件均为理想器件。
(a) 推导输出电压、输出电流和输出功率。
(b) 画出交流输入电流 ig (t ) 的波形,和整流后输出电压 vR (t ) 的波形。

(c) 推导交流输入电流的有效值、基波有效值、三次谐波有效值。此整流器是
否符合 IEC-1000 中的谐波电流限值标准?
(d) 推导在 S1 和 S 2 表面测量到的功率因数。

ig (t ) i (t ) L
vL (t )
vg (t )
230Vrms vR (t ) C V R
50Hz 40

S1 S2

图 16-1 无源整流电路
Solution:
(a) 如果电感 L 足够大使得电感电流为直流,则输出电压 V 为直流,故电感电
压 vL (t ) 如图 16-2 所示:

vR (t )
Vm
V

0 t

图 16-2 电感电压示意图
稳态时,电感电流在一个周期内的净增量为 0,即一个周期内的电感电压的
伏秒积为 0,则有:

 V
0
2
m sin(t )  V  dt  0 (16-1)
根据式(16-1)可得:

- 510 -
2
V Vm (16-2)

将已知参数值代入到式(16-2)中,可得:
2 2
V  Vm   230 1.414  207.04V
 
V
I  5.176A (16-3)
R
P  VI  1071.647W
(b) 交流输入电流 ig (t ) 的波形和整流后电压 vR (t ) 的波形如图 16-3 所示:

vR (t )
Vm

0
t
ig (t )

0
 2 t

I

图 16-3 交流输入电流波形和整流后电压波形

(c) 输入电流 ig (t ) 的函数表达式为:

 I 0  t  
ig (t )   (16-4)
 I   t  2

输入电流 ig (t ) 故可写成如下形式的傅里叶级数,即:

a0  
ig (t )   an cos nt   bn sin nt
2 n 1 n 1 (16-5)
  2 f  2  50rad/s
对于系数的求解,有:

- 511 -
2 a0 1 2
0
ig (t )dt 
2
2  a0   ig (t )d t
 0
2 2
0 ig (t ) cos  nt  dt  0 an cos  nt  dt 
2

1 2
an   ig (t ) cos  nt  d t (16-6)
 0

2 2
 ig (t ) sin  nt  dt   bn sin  nt   d t 
2

0 0

1 
bn   ig (t ) sin  nt  dt
2

 0

根据式(16-6)可求得:
b1  6.579
b3  2.193
(16-7)
an  0 n  0,1, 2... 
b2 n  0 n  1, 2,3... 

对于 ig (t ) 的有效值,根据书中附录 A 中的式(A.3)可知:

I g _ rms  I (16-8)

故有:
I g _ rms  5.176A
b1
I g1   4.653A (16-9)
2
b
Ig3  3  1.551A
2
根据书中的表 16.1 可知,式(16-9)满足 IEC-1000-3-2 标准。
(d) 功率因数的定义为:
Pav
PF  (16-10)
Vrms I rms

对于测量面 S1 来说,有:

1  2Vm I
IVm sin t dt 

Pav 
0 
Vm
Vrms  (16-11)
2
I rms I

故 S1 面测量到的功率因数为:

- 512 -
2 2
PF1   0.9 (16-12)

对于测量面 S 2 来说:

Pav  VI
Vrms  V (16-13)
I rms  I

故 S 2 面测量到的功率因数为:

PF2  1 (16-14)

Problem 16.2

如图 16-4 所示的三相整流器连接到线电压有效值为 480V 的三相 60Hz 电


网上,所有的元器件均为理想器件,电感值 L 足够大,使得电流 i (t ) 为直流,其

360Hz 的纹波可忽略不计。
L
 a ia (t ) i (t )

ib (t ) R
b vd (t ) C V
20

 c ic (t )
图 16-4 三相整流器

(a) 画出 vd (t ) 的波形。

(b) 推导直流输出电压 V 。
(c) 画出线电流 ia (t ) 、 ib (t ) 和 ic (t ) 的波形。

(d) 推导 ia (t ) 的傅里叶级数。

(e) 推导畸变因数、相移因数、功率因数和线电流的 THD。


Solution:
(a) vd (t ) 的波形如图 16-5 所示:

- 513 -
VPhase Gnd
 a b c
Va

0 t

Vlineline
abac bc ca abac bc cacb
3Va ba cb ba

0 t

vd (t )
abac bc ca abac bc cacb
ba cb ba
3Va

图 16-5 vd (t ) 的波形

(b) vd (t ) 的周期为 360Hz ,当电感值 L 足够大时,输出电压和输出电流 i (t ) 均

为直流,即电感电压 vL (t ) 的伏秒积在 vd (t ) 的一个周期内为 0,当 vd (t ) 的波形由

vab (t ) 产生时,即:

vd (t )  vab (t )  Vm cos(t  60)30  t  90


(16-15)
Vm  2  480  678.82V
故有:
90
 Vm cos t  60  V dt  0 (16-16)
30

根据式(16-16)可得:
3
V Vm  648.23V (16-17)

(c) ia (t ) 、 ib (t ) 和 ic (t ) 的波形如图 16-6 所示:

- 514 -
V ia (t ) ib (t ) ic (t ) ia (t ) ib (t ) ic (t )
 32.41A
R

图 16-6 ia (t ) 、 ib (t ) 和 ic (t ) 波形

(d) ia (t ) 的函数表达式为:

  5
 I  6  t 6
ia (t )   (16-18)
 I  7  t 11
 6 6

电流 ia (t ) 可进行傅里叶分解,即:

a0  
ia (t )   an cos nt   bn sin nt
2 n 1 n 1 (16-19)
  2 f  2  60rad/s
对于系数的求解,有:
2 a0 1 2
0
ia (t )dt 
2
2  a0   ia (t )d t
 0
2 2
 ia (t ) cos  nt  dt   an cos  nt   d t 
2

0 0

1 
an   ia (t ) cos  nt  d t
2
(16-20)
 0

2 2
 ia (t ) sin  nt  dt   bn sin  nt   d t 
2

0 0

1 
bn   ia (t ) sin  nt  dt
2

 0

将式(16-18)代入到式(16-20)中,可得:
a0  0
an  0 (16-21)
4I n 2n
bn  sin( ) sin( )
n 2 3

(e) 电流 ia (t ) 的畸变因数为:

- 515 -
I1 I1 b1
DistortionFactor  2  2  2 (16-22)
 2  
In I n2 bn2
I0  
2
 
n 1 2 n 1 2 n 1 2


bn2
对于无穷级数 
n 1 2
的和,我还不会求它的解析式,这里只能借助 Matlab

程序求此级数的数值了,这里就计算到 n  1000 为止,故有:



bn2 1000
bn2

n 1 2
 
n 1 2
 26.46
(16-23)
b1  35.73

1000
ia (t )   bn sin  nt  的波形如图 16-7 所示:
n 1

40

30

20

10

-10

-20

-30

-40
0 1 2 3 4 5 6 7

图 16-7 ia (t ) 的波形,考虑前 1000 次谐波

根据式(16-22)和式(16-23)可得:
DistortionFactor0.9551 (16-24)
由于 ia (t ) 的基波的相位与电压 va (t ) 同相,故相移因数为 1。

故三相电网中的 a 相的功率因数为:
PF  DistortionFactor DisplacementFactor  0.9551 (16-25)

电流 ia (t ) 的 THD 为:

 1000 1000

 I n2
n2
 I n2
n2
b
n2
2
n
THD     31.03% (16-26)
I1 I1 b1

- 516 -
Problem 16.3

谐波污染的监测问题。如图 16-8 所示的网络,在测量面 S 处可观测到电压


谐波。此问题的任务是确定谐波污染的源头是由电源引起的还是由负载引起的?
源或负载包含有一个非线性器件,此非线性器件是谐波产生的原因,令一个器件
则是理想的。
i (t )
Z1
vs (t ) v(t ) Z2

S
Source Load
图 16-8 单相功率系统

(a) 考虑第一种情况:当负载为无源线性阻抗 Z 2 ( s ) 时,即对于所有的  ,其

相位区间为 90  Z 2 ( j )  90 ,故谐波由电源产生,平均功率表达成:


P   Pn
n 0

其中 Pn 是源传递给负载的 n 次谐波的平均功率。你可以判断出 Pn 的极性

么?
(b) 当负载为非线性负载时,而源是线性的,则源可以使用戴维宁定理等效成
一个理想正弦电压源和一个线性阻抗 Z1 ( s ) 相串联。平均功率仍可以写成如(a)

中的形式,这时,你可以判断出 Pn 的极性么?

(c) 当通过测量,得到以下的傅里叶级数时:
电压 电流
谐波次数
幅值 相位 幅值 相位
1 230V 0° 6A -20°
3 20V 180° 4A 20°
5 8V 60° 1A -110°
你认为谐波是由源产生的还是由负载引起的?为什么?
Solution:
(a) 当 90  Z 2 ( j )  90 时,那么对于任意的  ,其功率因数为:

PF  cos  90 90   0 (16-27)

- 517 -
故 Pn  0 ,即能量是从源流入到负载中。

(b) 当负载为非线性负载时,此时 Pn 的极性判断不出来。

(c) 谐波是由源引起的,因为对于表中任一次数的电压谐波,都有电流谐波与
其对应,且在此次谐波频率处的阻抗值是固定不变的,另外没有单独出现的电压
谐波或电流谐波,因此负载是线性的。

Problem 16.4

对于 Problem 16.3 中的网络和波形,推导指定平面的功率因数和流向负载的


平均功率,5 次以上的谐波可忽略不计。
Solution:
对于各次谐波,流向负载的平均功率为:
VI
P1  1 1 cos  0  20   648.388W
2
VI
P3  3 3 cos 180  20   37.588W (16-28)
2
P5  5 5 cos  60   110    3.939W
VI
2
负载的电压有效值为:

Vn2
V 3 5 2  163.346V
n 1、、
(16-29)

负载的电流有效值为:

I n2
I 3 5 2  5.148A
n 1、、
(16-30)

故功率因数为:
P1  P3  P5
PF   0.722 (16-31)
VI

Problem 16.5

当所测量到傅里叶级数如下表所示时,重复 Problem 16.3 的(c)小问。


电压 电流
谐波次数
幅值 相位 幅值 相位
1 120V 0° 5A 25°
3 4V 60° 0.5A 40°
5 2V -160° 0.2A -100°
Solution:
谐波是由源引起的,因为对于表中任一次数的电压谐波,都有电流谐波与其

- 518 -
对应,且在此次谐波频率处的阻抗值是固定不变的,另外没有单独出现的电压谐
波或电流谐波,因此负载是线性的。

Problem 16.6

某平衡的三相星形连接的负载由每相 20 的负载电阻构成。此平衡负载连接


至星形连接的三相电压源,电压源的基波分量为线电压 380V 有效值。此外,各
相电压源也产生三次和五次谐波,各次谐波的幅值均为 20V 有效值,并且其相位
和相电压基波同相。
(a) 将源的中性点和负载的中性点相连接,形成四线系统,推导线电流和中性
线电流的傅里叶级数。
(b) 当断开中性线时,形成三线系统,推导此时线电流的傅里叶级数,此外,
推导源和负载两中性点之间电压的傅里叶级数。
Solution:
(a) 四线系统的电路如图 16-9 所示:
ic

vc va 20
ia 20

vb 20
i0

ib
图 16-9 四线系统
其中各相电压源的电压为:
va  Vm1 sin t   Vm 3 sin  3t   Vm 5 sin  5t 
vb  Vm1 sin t  120   Vm3 sin 3 t  120   
Vm5 sin 5 t  120   (16-32)
vc  Vm1 sin t  120   Vm3 sin 3 t  120   
Vm5 sin 5 t  120  
对于式(16-32)中的系数,有:
380
Vm1   2  310.2V
3 (16-33)
Vm3  Vm5  20  2  28.28V
故各线电流和中性线电流为:

- 519 -
va
ia 
R
v
ia  b
R (16-34)
v
ia  c
R
i0  ia  ib  ic
根据式(16-32)、式(16-33)和式(16-34),可得到线电流和中性线电流的傅里叶
级数为:
ia  310.2  sin t   28.28  sin  3t   28.28  sin  5t 
(16-35)
i0  4.24  sin  3t 

其余两线电流的波形和 ia 一样,只是相位不同。

(b) 三线系统如图 16-10 所示:


ic

vc va 20
ia 20
0 0
vb 20

ib
图 16-10 三线系统
故有:
va  vb   ia  ib  R
va  vc   ia  ic  R (16-36)
ia  ib  ic  0
根据式(16-36)可求出:
2va  vb  vc
ia  (16-37)
3R
将式(16-32)和式(16-33)代入到式(16-37)中,可得:
ia  1.414  sin  5t   15.51sin t  (16-38)

其余两线电流的波形和 ia 一致,只是相位不同,相差 120 。

两中性点间的电压为:
V00  va  ia R  Vm3 sin  3t   28.28sin  3t  (16-39)

- 520 -
第17章 线间换流的整流器

Problem 17.1

如图 17-1 所示的单相半控桥式整流电路包含一个感值足够大的电感 L ,其
电流 iL (t ) 的纹波可以忽略不计。晶闸管的延迟角为  (求解中认为  为弧度)。

ia (t ) iL (t ) L
Q1 D1 vL (t )
va (t ) vd (t ) C V R
Q2 D2

图 17-1 半控桥式整流电路

(a) 画出 vd (t ) 和 ia (t ) 的波形,标出各晶闸管和二极管的导通区间。

(b) 推导输出电压 V 的表达式,用输入电压有效值和延迟角表达。


(c) 推导功率因数的表达式。
(d) 延迟角在什么范围时,(b)和(c)的推导有效?
Solution:
(a) vd (t ) 和 ia (t ) 的波形如图 17-2 所示:

va (t )

0 t

Q1 Q2 Q1 Q2
vd (t )
D2 D1 D2 D1

0
  t
ia (t )
Ia
0
  t

图 17-2 vd (t ) 和 ia (t ) 的波形

(b) 稳态时,电感电压波形一个周期内的值为:

- 521 -
 V 0  t  
vL (t )   (17-1)
vd (t )  V   t  
由于稳态时,一个周期内,电感电流的净增量为 0,故电感一个周期内的伏
秒积为 0,故有:

 v t d t   0


0
L (17-2)

根据式(17-2)有:
 

 V d t    
0
2Varms sin(t )  V d t   0 (17-3)

根据式(17-3)可推出:

2Varms
V 1  cos   (17-4)

(c) 功率因数的定义为:
average power
PF  (17-5)
 rmsvoltage  rmscurrent 
输入电流的有效值为:

 
I arms  I a
 (17-6)
V
Ia 
R
在一个电网周期内,输入的平均功率为:

2 2Varms sin t  I a d t 
2 2Varms I a 1  cos  
average power  
 (17-7)
2 2
将式(17-6)和式(17-7)代入到式(17-5)中,可得:

2 2 1  cos  
PF  (17-8)
 
2

(d) 延迟角    时,(b)和(c)的推导有效。

Problem 17.2

如图 17-3 所示的三相半控桥式整流电路包含一个感值足够大的电感 L ,其
电流 iL (t ) 的纹波可以忽略不计。晶闸管的延迟角为  (求解中认为  为弧度)。

- 522 -
iL (t ) L
 a ia (t )
Q1 Q2 Q3 vL (t )
ib (t )
b vd (t ) C V R

 c ic (t ) D1 D2 D3

图 17-3 三相半控桥式整流电路

(a) 画出 vd (t ) 和 ia (t ) 的波形,标出各晶闸管和二极管的导通区间。

(b) 推导输出电压 V 的表达式,用输入电压有效值和延迟角表达。


(c) 推导功率因数的表达式。
(d) 延迟角在什么范围时,(b)和(c)的推导有效?
Solution:
(a) 不同延迟角  下的 vd (t ) 波形和 ia (t ) 波形如图 17-4 所示:

vd (t ) vd (t )


 0 
ia (t ) 6 i (t )
a

ga ga
gb gb
gc gc
vd (t )
vd (t )

 
 
3 2
ia (t ) ia (t )

ga ga
gb gb
gc gc

图 17-4 不同延迟角  下的 vd (t ) 波形和 ia (t ) 波形


(b) 当 0    时, vd (t ) 波形是连续的,当变换器处于稳态时,且电感电流
3
的纹波可忽略不计时,则可认为电感电压为 0,故输出电压 V 等于 vd (t ) 的平均值,

即有:
vd (t ) T
V (17-9)

- 523 -
线电压 vab (t ) 、 vac (t ) 为:


vab (t )  2Vrms cos(t  )
3
(17-10)
2
vac (t )  2Vrms cos(t  )
3
故 vd (t ) 的平均值为:

1  2 
vd (t ) T    2Vrms cos(t  )d t  
2  6 
 3
3
5
 2 
 6 2Vrms cos(t  )d t   (17-11)
2 3 
2Vrms 1  cos  

2
3

当     时, vd (t ) 波形是断续的,当变换器处于稳态时,且电感电流的
3
纹波可忽略不计时,则可认为电感电压为 0,故输出电压 V 等于 vd (t ) 的平均值,

即有:
vd (t ) T
V (17-12)

故 vd (t ) 的平均值为:

1  76 2  2Vrms 1  cos  


vd (t )    2Vrms cos(t  )d t    (17-13)
T 2  6 

3  2
3 3
注意,式(17-11)和式(17-13)的结果相同,故输出电压 V 的值为:

2Vrms 1  cos  
V (17-14)
2
3
(c) 功率因数的定义为:
average power
PF  (17-15)
 rmsvoltage  rmscurrent 

当0   时,A 相的平均功率(一个相电压周期内)为:
3

- 524 -
 56 
1  2Vrms V
average power   sin(t ) d t  
2  3 R
 6 
11

 V 
6
2Vrms
  sin(t )    d t   (17-16)
7 3  R
6

2VrmsV
 1  cos  
2 R
将式(17-14)代入到式(17-16)中,可得:
2
3Vrms
average power 2 1  cos  
2
(17-17)
2 R
A 相电流的有效值为:

3
I arms  Vrms 1  cos   (17-18)
R

将式(17-16)和式(17-18)代入到式(17-15)中,可得到当 0    时的 A 相功
3
率因数为:
2
3Vrms
1  cos  
2

PF  2 R
2

3
1  cos   (17-19)
Vrms 3 2
 Vrms 1  cos  
3 R

当     时,A 相的平均功率为:
3
 7
1  6 2Vrms V
average power   sin(t ) d t  
2  3 R
 6 
11

 V 
6
2Vrms
  sin(t )    d t   (17-20)
5 3  R
6
 
2VrmsV
 1  cos  
2 R
A 相电流的有效值为:

V  
2
I arms 2  I a2 2      I arms  (17-21)
R 

- 525 -

将式(17-20)和式(17-21)代入到式(17-15)中,可得当     时的 A 相功率
3
因数为:

6
1  cos  
PF  2 (17-22)
 

(d) (b)和(c)中推导的前提条件已在前面列出。

Problem 17.3

如图 17-5 所示的三相可控硅整流电路直接与负载电阻相连接。当延迟角 
较小时,此电路工作在连续模式,当延迟角  较大时,此电路工作在断续模式。

 a ia (t )
Q1 Q2 Q3
ib (t )
b v(t ) R

 c ic (t ) Q4 Q5 Q6

图 17-5 三相可控硅整流电路

(a) 画出 CCM 模式和 DCM 模式下的输出电压 v(t ) 的波形,并标出每个可控

硅导通的区间。
(b) 变换器运行在 CCM 模式和 DCM 模式的前提条件是什么?
(c) 推导 CCM 模式下,输出电压直流分量的表达式。
(d) 推导 DCM 模式下,输出电压直流分量的表达式。
Solution:
 
(a) 当延迟角 0    时,输出电压波形连续。当延迟角   时,输出电压
3 3
波形断续。两种情形下的输出电压波形如图 17-6 所示。
v(t ) v(t )

 
 
6 2
1 1 2 2 3 3
1 1 2 2 3 3
5 6 6 4 4 5
5 6 6 4 4 5

图 17-6 三相全控整流电路输出电压波形
 
(b) 当延迟角 0    时,变换器运行于 CCM 模式。当延迟角   时,变
3 3
换器运行于 DCM 模式。

- 526 -
(c) 当变换器处于 CCM 模式时,输出电压的直流分量为:


1 2
  2Vrms
2Vrms cos  t  d t  
 
V cos  (17-23)
 3 

3 6 3
(d) 当变换器处于 DCM 模式时,输出电压的直流分量为:
5

1 6
  2Vrms    
V  2Vrms cos  t  d t   1  cos    3   (17-24)
   3    

3 6 3

Problem 17.4

某整流器连接至 60Hz 的电网系统。需要设计一个谐波陷波滤波器,其对


60Hz 的电流既不衰减也不放大,但对 5 次和 7 次谐波衰减 10 倍,即 20dB 。交
流线的电感 Ls 为 500μH 。

(a) 当 5 次谐波陷波滤波器的电感值为 500μH 时,7 次谐波陷波滤波器的电感

值为 250μH 时,在不考虑并联谐振的前提下,给出 5 次和 7 次谐波陷波滤波器的

电阻和电容值。
(b) 画出此谐波陷波滤波器的频率响应。你所设计的滤波器是否满足衰减指标?
并联谐振的影响是否很大?滤波器对 3 次谐波的衰减或增益为多少?
(c) 修改滤波器的参数值,使得你认为已设计出效果最好的滤波器为止。除了
交流线的电感值外,你可以改变任何参数。画出你所设计的改善后的滤波器的频
率响应。效果最好意味着滤波器对于 5 次谐波和 7 次谐波的衰减为 20dB ,对于
60Hz 信号的衰减应为 0dB 。此外并联谐振的参数 Q 应越小越好。

Solution:
(a) 此谐波陷波滤波器的电路图如图 17-7 所示:
Ls
is
L5 L7 ir

C5 C7
R5 R7

图 17-7 谐波陷波滤波器
为了对 5 次和 7 次谐波进行衰减,故有:

- 527 -
1
2  5  60  L5 
2  5  60  C5
(17-25)
1
2  7  60  L7 
2  7  60  C7
为了使衰减的增益为 20dB ,故有:
 R5 
20 log    20dB
 2  5  60  Ls 
(17-26)
 R7 
20 log    20dB
 2  7  60  Ls 
根据式(17-25)和式(17-26)可得:
C5  0.5629mF
C7  0.5744mF
(17-27)
R5  0.094
R7  0.132
(b) 如图 17-7 所示的谐波陷波滤波器的传递函数为:
is ( s)  sLs   Z 5   Z7 
H ( s)  
ir ( s ) sLs
1
Z 5  R5  sL5  (17-28)
sC5
1
Z 7  R7  sL7 
sC7
将已知的参数值和式(17-27)代入到式(17-28)中,可得:
H ( s) 
4.042 1014 s 4  2.894 1011 s 3  4.291107 s 2  1.287 10 4 s  1 (17-29)
1.617 1013 s 4  6.547 1011 s 3  9.977 10 7 s 2  1.287 10 4 s  1
此滤波器的频率响应如图 17-8 所示:

- 528 -
Bode Diagram
System: H
30 Frequency (Hz): 180
Magnitude (dB): 19.8
20
System: H
10

Magnitude (dB)
Frequency (Hz): 60.1
Magnitude (dB): 0.765

-10

-20
System: H System: H
Frequency (Hz): 301 Frequency (Hz): 420
-30 Magnitude (dB): -20.4 Magnitude (dB): -20.5
0

-45
Phase (deg)

-90

-135

-180
1 2 3 4
10 10 10 10
Frequency (Hz)

图 17-8 滤波器的频率响应
从图 17-8 可以看出,此滤波器满足对 5 次和 7 次谐波的 20dB 衰减指标,
但是其并联谐振的影响很大,即对 3 次谐波有 20dB 的放大效果,即放大 10 倍。
(c) 对于此滤波器的优化,如果要满足 5 次和 7 次谐波处的 20dB ,且将两串
联谐振频率设定在 5 次和 7 次谐波上时,此外还希望满足 60Hz 处几乎 0dB 的衰
减,计算加调试了一下午就没发现比(b)中的频率响应更好的参数了。如果不
将两串联谐振频率设定在 5 次和 7 次上,可能会有更好的频率响应,即保持同样
的衰减,但可获得更小的 Q 值。

Problem 17.5

某整流器连接至 50Hz 的电网系统,需要设计一个谐波陷波滤波器使得对


50Hz 电流几乎不衰减或放大,但对 5、7 和 11 次谐波具有 14dB 的衰减,此外
此滤波器还必须还有一个单极点响应,可对 13 次及更高次谐波具有 5n /13 衰减
倍数。交流线电感值为 100μH 。

设计可满足以上指标的谐波陷波滤波器,尽你所能使得滤波器在 50Hz 处的
增益为 0  1dB ,并且对于 3 次和 9 次的增益尽可能小,给出滤波器的参数和频
率响应图。
Solution:
此谐波陷波滤波器的电路图如图 17-9 所示:
Ls
is
L5 L7 L11 L13 ir
Rbp
R5 R7 R11 R13
C5 C7 C11 C13

图 17-9 谐波陷波滤波器
首先,考虑对 5、7 和 11 次谐波的 14dB 衰减,故有:

- 529 -
1
5 L5 
5C5
1
7 L7 
7C7
1
11 L11 
11C11
(17-30)
R
20 log 5  14dB
5 Ls
R7
20 log  14dB
7 Ls
R11
20 log  14dB
11 Ls

根据式(17-30)可推出:
R5  0.0314
R7  0.0440 (17-31)
R11  0.0691
根据式(17-30)可知,各次谐波陷波滤波器中的 L 和 C 只能调节其中一个参数,
令一个参数由式(17-30)中的前三个子式确定。 对于 5、7 和 11 次陷波滤波器的
设计,其阻抗形式为:
2
 
s  s 
1  
1  1 
1 RC  LC 

Z  R  sL  
sC sC
1
 (17-32)
LC
1
Q
CR
对于式(17-32)可知, R 的值已在(17-31)确定,  的也已确定(比如对于 5
次陷波滤波器,  5  51  5  2  50 ),故只剩下 C 一个参数可以调节,这里

将 Q 作为可调参数,即对于某一个调节的 Q 值,Q 值确定则电容 C 的值便可确定,

然后根据(17-32)中的第 2 子式可确定电感值 L ,这样便可确定陷波滤波器中的 R 、


L 和 C 的值。
对于 5、7 和 11 次陷波滤波器的 Q 值的选择,这里选取为:

- 530 -
Q5  60
Q7  60 (17-33)
Q11  70

接下来讲下自己选择 Q 值的心得和原因。对于式(17-33)的 Q 值,5、7 和 11

次陷波滤波器的阻抗(阻抗由式(17-32)得出)的 Bode 图如图 17-10 所示:


Bode Diagram
60
Z5
40 Z7
Z11
Magnitude (dB)

20

0 System: Z11
Frequency (Hz): 550
Magnitude (dB): -23.2
-20
System: Z7
System: Z5
-40 Frequency (Hz): 250
Frequency (Hz): 350
90 Magnitude (dB): -30.1
Magnitude (dB): -27.1
Z5
Z7
45
Z11
Phase (deg)

-45

-90
1 2 3 4
10 10 10 10
Frequency (Hz)

图 17-10 5、7 和 11 次陷波滤波器的阻抗 Bode 图


各次阻抗的最低点,即陷波处的增益是由电阻值确定的,对于已确定好的电
阻值,随着 Q 值的增加,各次阻抗的两条渐近线( 20dB/decade 的区段)会往上

抬高,并且两条渐近线的交点距最低点的距离会增大,也即陷波的深度越深。
由于最终的传递函数为:
Z s Z 5 Z 7 Z11 Z13
H
Zs
Z s  sLs (17-34)
Ls  100μH

阻抗的并,比如说 Z5 Z7 Z11 在 Bode 图上反映为取 Z5 和 Z 7 和 Z11 的最小值,

也即三条曲线的最小区段,这种方法是面向设计的,即不是完全精确的,但可以
给出直观的结果,可以自己想象一下应该是什么样子的,之后会给出精确的 Bode
图。

- 531 -
Bode Diagram
60
Z5
40 Z7
Z11

Magnitude (dB)
Z_5_7_11
20

-20

-40
90

45
Phase (deg)

-45

-90
2 3 4 5
10 10 10 10
Frequency (rad/s)

图 17-11 Z5 Z7 Z11 的精确 Bode 图

可以发现当 Q 值较大时, Z5 Z7 Z11 的 Bode 图在陷波处可以说是精确的重

合,也即 Z5 Z7 Z11 的 Bode 图可以说是和自己直观想象出来的 Bode 图在陷波

频率处完全一致。这也是 Q 选取较大值的原因,网上查阅的说是 Q 一般选取在

30 60 之间。当 Q  5 时,会发现一方面陷波没有那么深,甚至没有陷波。令一

方面并之后的总阻抗在陷波处重合得没那么好。
接下来是 Z13 的设计,即考虑对 13 次及更高次谐波的衰减,根据题意,对于

13 次谐波,其衰减倍数为 5 13 /13  5 ,用 dB 来表示,即 20 log(1/ 5)   14 dB,

这和之前的 5、7 和 11 次谐波的衰减倍数是一样的。再根据题意,对于 13 次以


上的谐波,其衰减系数为 5  n /13 ,这意味着对于 13 次及 13 次以上的谐波,传
递函数的 Bode 图在 13 次谐波后应该是 20dB/decade 的直线。
根据式(17-34)可知,为了保证传递函数的 Bode 图在 13 次谐波以后为
20dB/decade 的直线,那么 Z s Z5 Z 7 Z11 Z13 的 Bode 图在 13 次谐波以后应为

水平线,因为水平线的斜率为 0dB/decade ,除以 Z s 后( Z s 的斜率为 20dB/decade ),

传递函数在 13 次谐波以后的斜率自然为 20dB/decade 。


至此, Z13 的形式已确定为如图 17-9 所示, Z13 只能加入一个旁路电路,不

能添加旁路电容,因为旁路电容的添加会使得 Z13 的 Bode 图在 13 次谐波以后不

是水平线。这样会使 Z s Z5 Z 7 Z11 Z13 的 Bode 图在 13 次谐波以后不为水平线。

Z13 的 Bode 图的水平线的位置,可由 13 次以后的高次谐波的衰减系数确定,

- 532 -
这样进而可以确定旁路电阻 Rbp 的值,这里选取 15 次谐波作为考虑,即:

Rbp 1

15 Ls 5 15 (17-35)
13
15  15  2  50

根据式(17-35)可确定 Rbp 的值为:

Rbp  0.0817 (17-36)

对于 R13 的确定,有:

R13
20 log  14dB
13 Ls (17-37)
R13  0.0817

接 下 来 确 定 Z13 的 Q 值 , 根 据 题 意 13 次 谐 波 以 后 的 Bode 图 应 为

2 0 d B / d e c a的直线,那么如果
de Q 值过大,也即有了陷波的波形,那么 13 次谐

波附近的直线将会被深深的影响。基于此考虑选取 Q 值在 1 附近,因为 Q  1 时,

阻抗的 Bode 图最接近渐近线,反复选取调试,最后确定 Q13  0.8 。 Q13 确定后,

L13 和 C13 也就确定了。

综上所有的参数已确定:
R5  0.0314L5  1.2mHC5  337.7μF
R7  0.0440L7  1.2mHC7  172.3μF
R11  0.0691L11  1.4mHC11  59.8μF (17-38)
R13  0.0817L13  16μHC13  3.7mF
Rbp  0.0817
最终的谐波陷波滤波器的 Bode 图如图 17-12 所示:

- 533 -
Bode Diagram
20
H
10

Magnitude (dB)
-10

-20

-30

-40
0

-45
Phase (deg)

-90

-135

-180
1 2 3 4
10 10 10 10
Frequency (Hz)

图 17-12 最终谐波陷波滤波器的频率响应 Bode 图


对于最终谐波陷波滤波器 Bode 图上关键频率处的增益值如图 17-13 所示:
Bode Diagram
20
System: H
10 Frequency (Hz): 49.9
Magnitude (dB): 0.381
0
Magnitude (dB)

System: H
Frequency (Hz): 350
-10 Magnitude (dB): -14.7

System: H
-20 Frequency (Hz): 250 System: H
System: H
Magnitude (dB): -14 Frequency (Hz): 5e+03
Frequency (Hz): 550
Magnitude (dB): -32.1
-30 Magnitude (dB): -17.8

-40
0

-45
Phase (deg)

-90

-135

-180
1 2 3 4
10 10 10 10
Frequency (Hz)

图 17-13 最终谐波陷波滤波器的关键频率处的增益值

Problem 17.6

某单相整流器连接至有效值为 230V ,频率为 50Hz 的欧洲单相电源上。此整


流器为 1000W 直流负载供电,需要满足 IEC-1000 标准中的 CLASS A 或 D 的谐
波电流限值。此整流器的电路如图 17-14 所示。直流负载会存在 100Hz 的电压纹
波,此纹波的峰峰值不能超过负载电压直流分量的 5% 。
(a) 评估满足题意要求的电感最小值。
(b) 给出电感值和电容值,以满足题意要求。并通过仿真证明。
ia (t ) iL (t ) L
Q1 D1 vL (t )
va (t ) vd (t ) C V R
Q2 D2

图 17-14 单相整流器
Solution:

- 534 -
(a) 负载电压中的 100Hz 纹波的峰峰值不超过直流分量的 5%,这说明负载电
压的波形已经很接近直流,根据题意负载电阻要消耗 1000W 的功率。接下来确
定大概的电阻值:
由于负载电压波形接近直流,那么电感值 L 就相对较大,根据式(17-4)可求
得输出电压的直流分量为:

2 2Vrms
V  207.07V (17-39)

故电阻值为:

V2
R  42.88 (17-40)
P
接下来估算下 IEC-1000 标准中的 CLASS A 的 THD 大概是多少,根据书中
Table16.1,有:
 13

 I n2
n2
I
n2
2
n
THD   
I1 P
Vrms (17-41)
2.32  1.142  0.77 2  0.42  0.332  0.212
  63%
1000
230
式(17-41)只是一个参考值,对于完全的 CCM 模式,其 THD 在 48% 左右,
但是通过仿真可知其 THD 满足式(17-41)要求,但是 9、13 次谐波稍微比标准中
要求的要高,也即不满足标准要求。故变换器需要工作在 CCM 和 DCM 的交界
处,以继续降低 THD 值。
令 K L  0.07 ,可知电感值为:

L  0.030H=30mH (17-42)
电容值要取得足够大,这里电容值取:
C  4700μH (17-43)
通过仿真可以得知 THD  37.65% ,各次谐波也都满足 IEC-1000 标准中的
CLASS A 要求。负载电压中 100Hz 的纹波也不超过其直流分量的 5%。

Problem 17.7

如图 17-15 所示的 12 脉冲整流器,其含有 6 个可控硅器件和 6 个二极管。


直流滤波电感的值足够大以至于其电流纹波可忽略不计。可控硅器件的延迟角为
 。可控硅桥臂通过 wye-wye 形式的三相变压器相连接,而二极管桥臂通过
wye-delta 形式的三相变压器相连接。由于两个变压器均含有 wye 形式的原边,
故在电路的实现上,仅用一个 wey 形式的原边即可。
(a) 推导变压器副边电压 va1b1 和 va 2b 2 的有效值和相位,用原边线电压的有效

- 535 -
值 vab 来表达。

(b) 画出 vd 1 (t ) 和 vd 2 (t ) 的波形。标出各晶闸管和二极管的导通区间。

(c) 推导输出电压的直流分量,用输入线电压有效值、延迟角  和匝数比 n 来


表达。
(d) 延迟角  在什么范围内,
(c)的推导有效。此整流器可产生什么样的输出
电压?

ia (t ) a1 IL L
1: n
a
T2 T3
T1 T1 b1
b o o1 vd 1 (t )
T3 T2 c1
c
R
a2
1: 3n

T1 T2 b2
vd 2 (t )
T3
c2

图 17-15 12 脉冲整流器
Solution:
(a) 变压器原边线电压为:
vab  2Vrms sin t 
vbc  2Vrms sin t  120  (17-44)
vca  2Vrms sin t  120 
则变压器原边相电压为:
2
vao  Vrms sin t  30 
3
2
vbo  Vrms sin t  30  120  (17-45)
3
2
vco  Vrms sin t  30  120 
3
则副边线电压为:

- 536 -
n 2
va1b1  nvao  nvco  Vrms sin t  30   sin t  30  120  
3
 n 2Vrms sin(t  60)
(17-46)
2
va 2b 2   3nvco   3n Vrms sin t  30  120 
3
 n 2Vrms sin t  90 

(b) vd 1 (t ) 和 vd 2 (t ) 的波形如图 17-16 所示(不同延迟角下, vd 1 (t ) 的波形会有

所不同):
Vlineline
abac bc ca abac bc cacb
2Vrms ba cb ba

0 t

vd 1 (t ) 1 2 3 4 5 6



6
t
vd 2 (t )

图 17-16 vd 1 (t ) 和 vd 2 (t ) 的波形

(c) vd 1 (t ) 的直流分量已在式(17-23)求出,这里重新给出(注意匝数比和电压

幅值的不同):

n 2Vrms
Vd 1  cos  (17-47)

3
故输出电压 V 为:

n 2Vrms
V  Vd 1  Vd 2 

1  cos   (17-48)

3
(d) (c)中推导的前提为 0     。故此整流器产生的电压范围为:

- 537 -
n2 2Vrms
0 V  (17-49)

3

Problem 17.8

对于如书中 Fig17.27(a)所示的 12 脉冲整流器,推导原边电流 ia1 (t ) 、ia 2 (t )

和 ia (t ) 的傅里叶系数(从基波至 13 次谐波)。用负载侧的直流电流和匝数比来表

达结果。你可以认为直流侧滤波电感值足够大,并且变压器也是理想的。
Solution:
原边电流 ia1 (t ) 的表达式为:

  5
 nI L  6  t  6
ia1 (t )   (17-50)
nI  7  t 11
 L
6 6

电流 ia1 (t ) 可写成傅里叶级数形式,即:

a0  
ia1 (t )    an cos nt   bn sin nt (17-51)
2 n 1 n 1

对于系数的求解,有:
2 2
a 1
0 ia1 (t )dt  20 2  a0   i
0
a1 (t )d t

2 2

 ia1 (t ) cos ntdt   an  cos nt  dt 


2

0 0
2
1
an 
 i
0
a1 (t ) cos ntd t (17-52)

2 2

 ia1 (t )sin ntdt   bn  sin nt  dt 


2

0 0
2
1
bn 
 i
0
a1 (t )sin ntd t

将式(17-50)代入到式(17-52)中,可求得电流 ia1 (t ) 的傅里叶系数为:

- 538 -
a1  0
2 3nI L
b1 

a5  0
2 3nI L
b5  
5
a7  0
2 3nI L
b7  
7
a11  0
2 3nI L
b11 
11
a13  0
2 3nI L
b13 
13 (17-53)

电流 ia 2 (t ) 的表达式为:

 nI L 
 3 0  t  3

 2 3nI L  2
   t 
 3 3 3
 nI L 2
 3  3  t 
ia 2 (t )   (17-54)
 nI L   t  4
 3 3

 2 3nI L  4  t  5
 3 3 3

 nI L  5  t 2
 3 3

ia 2 (t ) 的傅里叶系数求解有:

- 539 -
2 2
a 1
0 ia 2 (t )dt  20 2  a0   i
0
a2 (t )d t

2 2

 ia 2 (t ) cos ntdt   an  cos nt  dt 


2

0 0
2
1
an 
 i
0
a2 (t ) cos ntd t (17-55)

2 2

 ia 2 (t ) sin ntdt   bn  sin nt  dt 


2

0 0
2
1
bn 
 i
0
a2 (t ) sin ntd t

将式(17-54)代入到式(17-55)中,可得 ia 2 (t ) 的傅里叶系数为:

a1  0
2 3nI L
b1 

a5  0
2 3nI L
b5 
5
a7  0
2 3nI L
b7 
7
a11  0
2 3nI L
b11 
11
a13  0
2 3nI L
b13 
13 (17-56)

由于总的线电流 ia (t ) 为两个分支线电流 ia1 (t ) 和 ia 2 (t ) 的和,故总的线电流

ia (t ) 的傅里叶系数为两个分支线电流傅里叶系数的和,根据式(17-53)和式(17-56)

可得总的线电流 ia (t ) 的傅里叶系数为:

- 540 -
a1  0
4 3nI L
b1 

a5  0
b5  0
a7  0
b7  0 (17-57)
a11  0
4 3nI L
b11 
11
a13  0
4 3nI L
b13 
13

由上式可知总的线电流 ia (t ) 中 5 次和 7 次谐波消失了,故总的线电流的 THD

降低了。

Problem 17.9

如图 17-17 所示的单相可控桥式整流器运行在 CCM 模式。当正弦输入电压


vg (t ) 的幅值发生缓慢变化时,需要不断调整负载电压 v(t ) 。因此需要一个控制器

来改变延迟角以保持负载的电压 v(t ) 恒定。故需要推导整流器直流侧的小信号交

流模型。
(a) 画出 vs (t ) 和 vg (t ) 的波形,标出延迟角  。

(b) 使用电路平均方法推导小信号传递函数 vˆ(s) / ˆ (s) 和 vˆ(s) / vˆg ( s) 。此外需要

推导稳态关系 V  f (Vg , A) 。这里有:

 (t )  A  ˆ (t )
v(t )  V  vˆ(t )
vg (t )  Vg  vˆg (t )  sin t 

你可以认为延迟角  、输出电压 v 和输入电压 vg 的变换频率远远慢于交流输

入的频率  。此外也可以认为电感电流纹波极小。

- 541 -
i (t ) L
Q1 Q2

vg (t ) vs (t ) C R v(t )

Q3 Q4

图 17-17 单相可控桥式整流器
Solution:
(a) vs (t ) 和 vg (t ) 的波形如图 17-18 所示:

vg (t )

0 t

Q1 Q2 Q1 Q2
vs (t )
Q4 Q3 Q4 Q3

0
  t

图 17-18 vs (t ) 和 vg (t ) 的波形

(b) 对 vs (t ) 进行周期平均,这里的平均周期为电网的半个周期内,故有:

1  vg (1cos  )
 sin t d t  
TL

TL
vs (t ) vg (17-58)
TL
 TL
TL 
将式(17-58)中的周期平均量表示成静态量与交流小信号扰动的和,故有:

Vs  vˆs 
V g  vˆg  1  cos  A  ˆ  
(17-59)

将式(17-59)展开可有:
1
Vs  vˆs  (Vg  vˆg  Vg cos A cos ˆ  Vg sin A sin ˆ 
 (17-60)
vˆg cos A cos ˆ  vˆg sin A sin ˆ )

对于式(17-60),当延迟角  的扰动很小时,有:
1
Vs  vˆs  (Vg  vˆg  Vg cos A  Vgˆ sin A  vˆg cos A  vˆgˆ sin A) (17-61)

- 542 -
忽略二次非线性部分,可得:
Vs  vˆs  Vg  Vg cos A   vˆg  Vgˆ sin A  vˆg cos A
1
(17-62)

对于整流器直流侧的稳态关系,有:
Vs  V (17-63)

根据式(17-62)和式(17-63),可得稳态关系 V  f (Vg , A) 为:

Vg 1  cos A 
V (17-64)

根据式(17-62)和整流器的直流侧部分,可画出变换器交流小信号模型如图
17-19 所示:
iˆ(t ) L
vˆg
1  cos A

vˆs (t ) C R vˆ(t )
ˆVg sin A

图 17-19 变换器交流小信号模型
当考虑从控制输入到输出的传递函数时,图 17-19 可简化为如图 17-20 所
示:
iˆ(t ) L

ˆVg sin A
C R vˆ(t )

图 17-20 考虑从控制输入到输出时的简化模型
根据图 17-20 可得:
vˆ( s) RVg sin A
 (17-65)
ˆ ( s)   CLRs 2  Ls  R 
当考虑从交流输入到输出的传递函数时,图 17-19 可简化为如图 17-21 所
示:
iˆ(t ) L

vˆg
1  cos A C R vˆ(t )

图 17-21 考虑从交流输入到输出时的简化模型
故有:

- 543 -
vˆ( s) R 1  cos A
 (17-66)
vˆg ( s)   CLRs 2  Ls  R 

- 544 -
第18章 PWM 变换器

Problem 18.1

用 Buck-Boost 变换器替代如书中 Fig18.5 中的 Boost 变换器。电感的能量存


储对变换器波形的低频分量的影响可忽略不计。平均负载功率为 PLoad 。直流输出

电压为 V 。交流正弦输入的幅值为 Vm 。

(a) 推导占空比变量 d (t ) 的表达式和电感电流变量 i (t ) 的表达式。假设变换器

运行在 CCM 模式。


(b) 推导变换器运行在 CCM 模式下的前提条件,证明当模拟电阻 Re 的值小于

Re,crit ( L, Ts , vg (t ),V ) 时,变换器运行在 CCM 模式,并给出 Re,crit 的表达式。

(c) 对于模拟电阻 Re ,其取何值时,变换器始终运行在 CCM 模式或运行在

DCM 模式?
(d) 交流输入电压的幅值处于 108V~132V 的范围。最大负载功率为 100W ,最
小负载功率为 10W 。直流输出电压为 120V 。开关频率为 75kHz 。电感 L 取何值
时可以保证变换器始终运行在 CCM 模式或 DCM 模式?
Solution:
(a) 使用 Buck-Boost 变换器作为整流器的电路如图 18-1 所示:
iac (t ) ig (t )

vac (t ) vg (t ) vL L C R V

图 18-1 使用 Buck-Boost 变换器作为整流器


由于可以认为电感的能量存储对变换器波形中的低频分量影响可忽略不计,
故相对于 vg (t ) 的变化来说,Buck-Boost 电路可以认为始终处于稳态模式。

假设变换器运行在 CCM 模式,且当开关管导通时,对于 Buck-Boost 变换器,


如图 18-2 所示:

- 545 -
ig (t )

vg (t ) vL L C R V
iL (t ) ic (t )

图 18-2 当开关管导通时的 Buck-Boost 电路


故有:
vL  vg (t )
V
ic   (18-1)
R
ig  iL

当开关管关断时,电路如图 18-3 所示:

ig (t )

vg (t ) vL L C R V
iL (t ) ic (t )

图 18-3 当开关管关断时的 Buck-Boost 电路


故有:
vL  V
V
ic  iL  (18-2)
R
ig  0

根据式(18-1)和式(18-2),有:

d vg (t )   1  d V   0
 V  V
d     1  d   iL    0 (18-3)
 R  R
ig  d  iL 
Ts

根据式(18-3)可推出:
V V
d (t )  
V  vg (t ) V  Vm sin t 
 V   V  (18-4)
PLoad   1 PLoad   1 
V  vg (t )  Vm sin t  
iL      
R 1  d (t )  V V
(b) 为了使变换器运行在 CCM 模式,故有:

- 546 -
iL Ts
 iL (18-5)

其中电感电流的周期平均值 iL Ts
已由式(18-3)给出,对于电感电流的纹波,

有:
2iL
vg (t )  L (18-6)
d (t )Ts

对于模拟电阻,有:
vg (t )
 Re (18-7)
ig (t )

根据式(18-3)、式(18-6)和式(18-7)可推出:
Re  Re,crit  L, Ts ,V , vg (t ) 
2
2 L V  vg (t )  (18-8)
Re,crit 
TsV 2
(c) 为了使整流器始终运行在 CCM 模式,根据式(18-8)有(注意 V 的符号为
负):

Re  min  Re,crit  
2L
(18-9)
Ts

为了使整流器始终运行在 DCM 模式,根据式(18-8)有:

2 L V  Vm 
2

Re  max  Re,crit   (18-10)


TsV 2
(d) 对于模拟电阻,有:
Vm2
Re  (18-11)
2 PLoad

为了使整流器始终运行在 CCM 模式,对于电感值 L 的选择,将式(18-11)代


入到式(18-9)中,有:
TsVm2
L (18-12)
4 PLoad

由于交流输入电压的幅值和负载功率都是在一定范围内变化,故为了保证整
流器始终运行在 CCM 模式,式(18-12)应进一步变形为:
Ts  max Vm  
2

L (18-13)
4 min  PLoad 
为了使整流器始终运行在 DCM 模式,对于电感值 L 的选择,将式(18-11)代
入到式(18-10)中,有:

- 547 -
TsV 2
L 2
(18-14)
V 
4 PLoad   1
 Vm 
由于交流输入电压的幅值和负载功率都是在一定范围内变化,故为了保证整
流器始终运行在 DCM 模式,式(18-14)应进一步变形为:
TsV 2
L 2
(18-15)
 V 
4 max  PLoad    1
 min(Vm ) 
将题中已知的参数值代入到式(18-13)中,可知为了保证整流器始终处于
CCM 模式,电感值 L 应满足:
L  5.808mH (18-16)
将题中已知的参数值代入到式(18-15)中(注意,电压 V 的符号应为负数!),
可知为了保证整流器始终处于 DCM 模式,电感值 L 应满足:
L  0.1077mH (18-17)

Problem 18.2

推导 Buck-Boost 变换器输入特性的表达式,类似书中的式(18.25)和式
(18.33)。画出变换器的输入特性,并标出 CCM 和 DCM 的边界。
Solution:
CCM 模式下的输入特性已在 Problem 18.1 中的式(18-4)中求出,这里重新给
出,即:
V 1
d (t )   (18-18)
V  vg (t ) vg (t )
1
V
为了给出统一的图形,这里进行标幺化,即:
vg (t )
mg (t ) 
V
(18-19)
2L
jg (t )  ig (t )
VTs

将式(18-19)代入到式(18-18)中,可得:
1
d (t )  (18-20)
1  mg (t )

接下来推导 DCM 模式下 Buck-Boost 变换器的输入特性。


当开关管导通时,变换器如图 18-4 所示:

- 548 -
ig
i (t ) iC
vg (t ) vL L C R V

图 18-4 开关管导通时的 DCM 模式下的 Buck-Boost 变换器


故有:
vL  vg (t )
V
ic   (18-21)
R
ig  i

当开关管关断时且电感电流断续之前,变换器如图 18-5 所示:

D1
i (t ) iC
vg (t ) vL L C R V

图 18-5 开关管关断时且电感电流断续之前的 Buck-Boost 变换器


故有:
vL  V
V
ic  i  (18-22)
R
ig  0

当开关管关断时且电感电流断续之后,变换器如图 18-6 所示:

i (t ) iC
vg (t ) vL L C R V

图 18-6 开关管关断时且电感电流断续之后的 Buck-Boost 变换器


故有:
vL  0
V
ic   (18-23)
R
ig  0

对电感使用伏秒平衡,有:
d1 vg (t )   d 2 V   0 (18-24)

- 549 -
根据式(18-24)可得:
Vd 2
d1   (18-25)
vg (t )

这里的式(18-25)为输入特性方程,但是占空比 d 2 未知。故接下来求占空比 d 2 。

DCM 模式下,输入电流 ig (t ) 和二极管电流 iD (t ) 的波形如图 18-7 所示:

ig (t ) iD (t )
IM IM

0 t 0 t
d1TS d 2TS d3TS d1TS d 2TS d3TS

图 18-7 DCM 模式下,输入电流和二极管电流波形


即有:
1 1
ig (t )  d1Ts I M
Ts Ts 2
(18-26)
I
vg (t )  L M
d1Ts
根据式(18-26)可求出:

Ts d12vg (t )
ig (t )  (18-27)
Ts 2L
将式(18-19)代入到式(18-27)中,可得:
jg  d12 mg (18-28)

接下来需要找出 CCM 和 DCM 的边界。


为了使变换器始终运行在 CCM 模式,将式(18-19)代入到式(18-8)中,可得:
mg
jg  (18-29)
 mg  1
2

根据式(18-20)、式(18-28)和式(18-29)可画出 Buck-Boost 变换器的静态输入


特性如图 18-8 所示:

- 550 -
0
CCM
d  0.3
-0.5

-1

jg -1.5

-2 DCM
d  0.9
-2.5

d 1
-3
-3 -2.5 -2 -1.5 -1 -0.5 0
mg
图 18-8 Buck-Boost 变换器静态输入特性

Problem 18.3

对于使用 Cuk 变换器作为单相 CCM 模式的整流器,推导晶体管和二极管电


流的有效值表达式,将表达式变换成类似书中 Table18.3 的形式。
Solution:
Cuk 变换器如图 18-9 所示:

ig L1 iL1 iC1 C1 iL 2 L2
vL1 iQ vC1 iD vL 2 iC 2
vg Q1 D1 vC 2 C2 V R

图 18-9 Cuk 变换器


当变换器运行在 CCM 模式,且开关管导通时,有:
vL1  vg
vL 2  V  vC1  V  VC1
iC1  iL 2   I L 2 (18-30)
iQ  iL1  iC1  I L1  iC1
iD1  0
当开关管关断时,有:

- 551 -
vL1  vg  vC1  vg  VC1
vL 2  V
iC1  iL1  I L1 (18-31)
iQ1  0
iD1  iL1  iL 2
根据式(18-30)和式(18-31),使用电感的伏秒平衡和电容的电荷平衡,有:
I L1d 
I L2 
d
V d
 (18-32)
vg d
ig  I L1
根据式(18-30)和式(18-32),一个开关周期内,晶体管的电流为:
 ig
 0  t  dTs
iQ   d (18-33)
0dT  t  T
 s s

根据式(18-31)和式(18-32),一个开关周期内,二极管的电流为:
 00  t  dTs

iD1   ig (18-34)
 dTs  t  Ts
d
故有:
t Ts t  dTs 2
 ig 
2
1 1 1 ig ig2
  i dt    
2 2
iQ T Q   dt dTs
s Ts t
Ts t d  Ts d 2 d
(18-35)
t Ts dTs Ts
ig2 d 
2
 ig 
2
1 1 1 ig
iD2 1  i dt     
2
D1   dt d T s
Ts Ts t
Ts dTs d Ts d 2 d2

接下来求占空比 d 在半个个交流电周期内的表达式。
根据式(18-32)可得:
V V
d (t )   (18-36)
V  vg (t ) V  VM sin t 

对于输入电流有:
vg Vm sin t 
ig   (18-37)
Re Re
将式(18-36)和式(18-37)代入到式(18-35)中,可得:

- 552 -
VM2

sin t  V  VM sin t  
2
iQ2  2
Ts Re V
(18-38)
V3
 2M 2 sin t  V  VM sin t  
3
iD2 1
Ts Re V
故晶体管电流和二极管电流在半个交流电周期内的有效值为:
Tac
1
I Q _ rms 
Tac 0
iQ2
Ts
dt
(18-39)
Tac
1
I D1_ rms 
Tac 0
iD2 1
Ts
dt

将式(18-38)代入到式(18-39)可得:
VM2  V 4VM 
I Q _ rms    
Re2V  2 3 
(18-40)
V  4V 3VM 
3
I D1_ rms  M
  
R V  3
2 2
e 8 
对于输入电流的有效值来说,有:
Vm
I ac _ rms  (18-41)
2 Re

将式(18-41)代入到式(18-40)中,可得:
8 Vm
I Q _ rms  I ac _ rms 1 
3 V
(18-42)
16 Vm 3 Vm2
I D1_ rms  I ac _ rms 
6 V 4 V 2

Problem 18.4

为了获取隔离型的直流输出,使用如书中 Fig6.35(实际上是书中 Fig6.34)


所示的全桥带变压器隔离的 CCM boost 变换器来代替书中 Fig18.5 中的 boost 变
换器。推导晶体管电流有效值的表达式,用 I ac _ rms 、 n 、 V 和 Vm 表示。

Solution:
变换器如图 18-10 所示:

- 553 -
iL L
1: n D
iac (t ) vL 1

Q1 Q3
vac (t ) vT
vg C R V
D2
Q2 Q4

图 18-10 使用全桥带变压器隔离的 Boost 整流器

vT (t ) 的波形如图 18-11 所示:

vT (t )

d Ts
0 t
dTs d Ts dTs

图 18-11 电压 vT (t ) 波形

当 0  t  dTs 时,有:

vL  vg (18-43)

当 dTs  t  Ts 时,有:

V
vL  vg  (18-44)
n
对电感使用伏秒平衡,有:

 V
dvg  1  d   vg    0 (18-45)
 n

由式(18-45)可得:
V  nvg
d (18-46)
V
对于电感电流,又有:
ig  iL  I L (18-47)

对于一个完整的开关周期(指 2Ts ),对于各晶体管电流有:

- 554 -
iQ1 iQ 2 iQ 3 iQ 4
ig ig ig ig
0  t  dTs
2 2 2 2
dTs  t  Ts ig 0 0 ig (18-48)
ig ig ig ig
Ts  t  1  d  Ts
2 2 2 2
1  d  Ts  t  2Ts 0 ig ig 0
故有:
1
2Ts
1  ig2 ig2   2  d  ig2
  i dt   dTs  ig 1  d  Ts  dT  
2 2 2
iQ1 2T Q1 (18-49)
s 2Ts 0
2Ts  4 4  4

故晶体管电流在半个交流电周期内的有效值为:
Tac
1
I Q1_ rms 
Tac 0
iQ21
2Ts
dt (18-50)

对于输入电流 ig 有:

vg Vm sin t 
ig   (18-51)
Re Re
根据式(18-51)、式(18-49)和式(18-50)可得:

Vm2  3 V  8nVm 
Tac
1
I Q1_ rms 
Tac 0
iQ21
2Ts
dt 
24 Re2V
(18-52)

对于输入电流的有效值来说,又有:
Vm
I ac _ rms  (18-53)
2 Re

将式(18-53)代入到式(18-52)中,可得:

1 2nVm
I Q1_ rms  I ac _ rms  (18-54)
4 3 V

Problem 18.5

对使用 CCM boost 变换器和隔离型 SEPIC 变换器作为通用输入的单相整流


器进行对比。直流输出电压为 V  400V ,负载功率 P  500W ,输入电压有效值
的变化范围为 90V~270V ,也即交流输入电压峰值 Vm 的变化范围为 Vm _ min  127V

至 Vm _ max  382V 。将晶体管的应力定义为最坏情况下晶体管所受的峰值电压与最

- 555 -
坏情况下晶体管的电流有效值的乘积。应尽可能使晶体管的应力 S 最小。
(a) 推导 boost 变换器的应力 S 表达式。
(b) 如果不需要通用输入性,即 Vm  382V ,此时晶体管的应力会如何?

对于隔离型的 SEPIC 变换器,变压器的匝数比为 n :1 ,通过对匝数比进行设


计来优化应力。
(c) 推导隔离型 SEPIC 变换器的晶体管应力,用 n 、V 、P 、Vm _ min 和 Vm _ max 进

行表示。
(d) 选择匝数比使得隔离型 SEPIC 变换器的晶体管应力最小,并与 CCM boost
变换器进行对比。
Solution:
(a) 通用输入 CCM boost 整流器如图 18-12 所示:
iac (t ) ig (t ) L
vL
vac (t ) vg (t ) C R v

图 18-12 通用输入 CCM boost 整流器

首先计算出最坏情况下晶体管所受的峰值电压 Vs ,当晶体管关断时,晶体管

所受的电压为恒定的输出电压 V ,故有:
Vs  V  400V (18-55)

接下来计算最坏情况下晶体管电流的有效值 I s 。

对于一个开关周期内,晶体管电流 iQ 为:

 i 0  t  dTs
iQ   g (18-56)
0dTs  t  Ts
故有:
t Ts
1 1
  i dt  T ig dTs  ig2 d
2 2
iQ T Q (18-57)
s Ts t s

而对于占空比 d 有:
vg Vm
d  1  1 sin t  (18-58)
V V

对于输入电流 ig 有:

- 556 -
vg Vm
ig   sin t  (18-59)
Re Re

将式(18-58)、式(18-59)代入到式(18-57)中,可得:
Vm2
 2 sin t  V  Vm sin t  
2 2
i
Q T (18-60)
s VRe

晶体管电流有效值 IQ1_ rms 为:

Tac
1 8Vm
I Q1_ rms 
Tac 0
iQ21
Ts
dt  I ac _ rms 1 
3 V
(18-61)

故最坏情况下的晶体管电流发生在输入电压的有效值最低时,故根据式
(18-61)可得:

I s  max  I Q1_rms  
P 8Vm _ min
1  4.759A (18-62)
Vm _ min 3 V
2
故最坏情况下的晶体管应力为:
S  Vs I s  400  4.759  1904VA (18-63)

(b) 如果不需要输入的通用适应性,则此时晶体管电流的有效值为:

I s  max  I Q1_ rms  


P 8V
1  m  0.806 A
Vm 3 V
(18-64)
2
Vm  382V
此时晶体管的应力为:
S  Vs I s  322.4VA (18-65)

(c) 使用隔离型 SEPIC 拓扑作为单相通用输入整流器,电路如图 18-13 所示:


iac (t ) ig (t ) L C1 n :1
vL
vac (t ) vg (t ) C2 R v

图 18-13 隔离型 SEPIC 单相整流器


其等效电路如图 18-14 所示:

- 557 -
iac (t ) ig L iC1 C1 ip n :1 is
vL vC1
vac (t ) vg (t ) vT L C2 R v
m

im iC 2
图 18-14 隔离型 SEPIC 单相整流器的等效电路
当开关管导通时,有:
vL  v g
vT  vC1  VC1
iC1  im   I m (18-66)
v V
iC 2   
R R
当开关管关断时,有:
vL  vg  vT  vC1  vg  vT  VC1
vT  nv  nV
iC1  ig (18-67)
v V
iC 2  is   ni p 
R R
i p  ig  im  ig  I m
使用电感的伏秒平衡和电容的电荷平衡,可得:
dvg
V
n 1  d 
nV
d
nV  vg
(18-68)
nV  d  1
VC1 
d
i  d  1
Im  g
d
根据式(18-68),当晶体管关断时,其所承受的电压为:
nV
Vs  Vc1  nV  (18-69)
d
对于单相隔离型 SEPIC 整流器来说,占空比的表达式可根据式(18-68)为:
nV nV
d  (18-70)
nV  vg nV  Vm sin t 

将式(18-70)代入到式(18-69)中,可得:
Vs  nV  Vm sin t  (18-71)

故最坏情况下,晶体管所受到的电压应力根据式(18-71)可求出:

- 558 -
Vs  nV  Vm _ max (18-72)

根据式(18-68)可求得一个开关周期内,晶体管的电流为:
 ig
 ig  I m  0  t  dTs
iQ   d (18-73)
0dT  t  T
 s s

故有:
t Ts 2
1 1 i 
  i dt   g  dTs
2 2
iQ T Q (18-74)
s Ts t
Ts  d 
对于整流器的输入电流来说,有:
Vm
ig  sin t  (18-75)
Re

将式(18-75)和式(18-70)代入到式(18-74)中,可得:


Vm2 sin t   V sin t   nV 
2
m
iQ2  2
(18-76)
Ts nVR e

晶体管电流在半个交流电周期内的有效值为:
Tac
1
I Q1_ rms 
Tac 0
iQ21
Ts
dt (18-77)

将式(18-76)代入到式(18-77)中,可得:
Vm2 8Vm  3 nV 
I Q _ rms  (18-78)
6 nVRe2
对于交流电流有效值来说,有:
Vm
I ac _ rms  (18-79)
2 Re

将式(18-79)代入到式(18-78)中,可得:

8Vm
I Q _ rms  I ac _ rms 1  (18-80)
3 nV
由于输出功率恒定,故有:
Vm
P  I ac _ rms (18-81)
2
将式(18-81)代入到式(18-80)中,可得:
8Vm  3n V 8 1
I Q _ rms  6 P  6P  2 (18-82)
9 nVVm 2
9 nVVm 3Vm

- 559 -
故最坏情况下的晶体管电流有效值为:

I s  max  I Q _ rms   6 P
8 1
 2 (18-83)
9n VVm _ min 3Vm _ min

根据式(18-72)和式(18-83)晶体管的应力为:

S  Vs I s  6 P  nV  Vm _ max 
8 1
 2 (18-84)
9 nVVm _ min 3Vm _ min

(d) 将各参数值代入到式(18-84)中,可得:

1016  10000n 9550 


6 1200    
n  381 381 
S (18-85)

开关管的应力 S 关于匝数比 n 的图形如图 18-15 所示:
4
2.510

4
2.18810

4
1.87510

4
1.56310
1016  10000 n  9550 

6 1200   
 381 381  4
n 1.2510
 3
9.37510

3
6.2510

3
3.12510

0
0 1.25 2.5 3.75 5 6.25 7.5 8.75 10
n

图 18-15 开关管的应力 S 关于匝数比 n 的图形


为了求出当匝数比 n 为何值时,开关管应力 S 最小,故对式(18-85)中的匝数
比 n 求导,并令导数等于 0,可求出:
n  0.298 (18-86)
故当匝数比为 0.298 :1 时,开关管所受的到应力最小。
如果只比较开关管应力的话,则隔离型 SEPIC 整流器的应力要远大于 CCM
boost 整流器的开关管应力。

Problem 18.6

如图 18-16 所示的三相 boost 型整流器,交流侧电感的值不能被忽略,即交


流侧电感在电网频率处的阻抗不能被忽略。三相交流电压可由书中式(18.150)
表示。占空比由书中式(18.156)表示。变换器运行在 CCM 模式。
(a) 推导从开关网络的交流线输入端到中性点的平均电压的幅值与相位,并用
Dm 、 V 和  进行表示。

(b) 推导变换器从电网吸收的有功功率 P 和无功功率 Q ,用 Vm 、 V 、 Dm 、 

- 560 -
和  L 进行表示。
(c) 如何选择  以实现单位功率因数?

Solution:
(a) 三相 Boost 型整流器如图 18-16 所示:

v AN
iA L
A
iB L
N B R V
iC L
C

O
图 18-16 三相 Boost 型整流器
调制的占空比表达式如书中式(18.156)所示,这里重新给出:
1
d1 (t )  D0  Dm sin t   
2
1
d 2 (t )  D0  Dm sin t    120  (18-87)
2
1
d3 (t )  D0  Dm sin t    240 
2
故从开关网络的交流线输入端到中性点的平均电压为:
 1 
vAO  d1 (t )V   D0  Dm sin t     V
 
Ts
2
 1 
vBO  d 2 (t )V   D0  Dm sin t    120   V (18-88)
 
Ts
2
 1 
vC O  d3 (t )V   D0  Dm sin t    240   V
 
Ts
2
(b) 列写线间方程,有:
diAB
v AB  L  v AB
dt
di
vBC  L BC  vBC 
dt
(18-89)
di
vCA  L CA  vC A
dt
dv v
d1 (t )iA  d 2 (t )iB  d3 (t )iC  C 
dt R
其中:

- 561 -
vAB  vAN  vBN
iAB  iA  iB (18-90)
vAB  vAO  vBO
相电压的表达式如书中式(18.150)所示,这里重新给出:
vAN  Vm sin t 
vBN  Vm sin t  120  (18-91)
vCN  Vm sin t  240 

根据式(18-91)、式(18-90)和式(18-89)可求出电流 iAB (此非线电流!)为:

2 3Vm sin t   6Vm cos t   3DmV cos t     3DmV sin t   
iAB 
4 L
(18-92)
故整流器从电网吸收到的有功功率 P 为:
1 T
P   vABiAB dt (18-93)
T 0
将式(18-90)、式(18-91)和式(18-92)代入到式(18-93)中,可得:
3D VV sin 
P m m (18-94)
4 L
为了求出无功功率 Q ,首先计算电流 iAB 的幅值,式(18-92)可变形为:

3   DV 3   
iAB  Vm sin  t    m sin  t     (18-95)
L  3 2 L  3 

故 iAB 的幅值为:

3Dm2V 2  12cos   DmVVm  12Vm2


iAB  (18-96)
2 L
故视在功率 S 为:

3Vm iAB
S (18-97)
2 2
由于无功功率为:

Q  S 2  P2 (18-98)
根据式(18-96)、式(18-97)和式(18-98)可得:

3 Vm2  2Vm  DmV cos   


2

Q (18-99)
4 L

- 562 -
(c) 为了实现单位功率因数,即无功功率 Q  0 ,令式(18-99)等于 0,可得:

 2Vm 
   arccos   (18-100)
 DmV 
为了使有功功率 P 为正数,则式(18-100)中负数要舍弃掉,故有:
 2Vm 
  arccos   (18-101)
 DmV 

Problem 18.7

如书中 Fig18.39 所示的 boost 型三相整流器,开关的调制占空比波形如书中


Fig18.42 所示。当电感值 L 足够小时,在保证输入电流波形无畸变的情况下,可
以使得直流输出电压与交流输入线电压峰值相等。当 0  t  60 时, d1 (t )  1 。

当 t  0 时, v12 (t ) Ts
V 。

(a) 当 0  t  60 时,推导 d 2 (t ) 和 d3 (t ) 的表达式。

(b) 展示 d1 (t ) 、 d 2 (t ) 和 d3 (t ) 在各 60 区间内如何变化。

Solution:
(a) 这里重画三相 boost 型整流器,如图 18-17 所示:

v AN
iA L
A
iB L
N B R V
iC L
C

图 18-17 三相 boost 型整流器


线间方程为:
diAB
vAB  L  v AB
dt
di
vBC  L BC  vBC  (18-102)
dt
diCA
vCA  L  vC A
dt
其中:

- 563 -
vAB  vAN  vBN
vAB  vAO  vBO  d1 (t )V  d 2 (t )V (18-103)
iAB  iA  iB
当电感值很小时,则式(18-102)可简化为:
v AB  vAB
vBC  vBC  (18-104)
vCA  vC A

根据 v12 (t ) 的波形,可知:

vAB  V cos t 


vBC  V cos t  120  (18-105)
vCA  V cos t  240 
根据式(18-104)和式(18-105),当 0  t  60 时,有:
vAB  V cos t   vAB   d1  d 2 V
vBC  V cos t  120   vBC    d 2  d3 V
(18-106)
vCA  V cos t  240   vC A   d3  d1 V
d1  1
根据式(18-106)可求得,当 0  t  60 时:
d1  1
d 2  1  cos t  (18-107)
d3  1  cos t  60 
(b) 当 60  t  120 时,根据式(18-104)和式(18-105)有:
vAB  V cos t   vAB   d1  d 2 V
vBC  V cos t  120   vBC    d 2  d3 V
(18-108)
vCA  V cos t  240   vC A   d3  d1 V
d3  0
由式(18-108)可得当 60  t  120 时:
d1  sin t  30 
d 2  sin t  30  (18-109)
d3  0
同理,当 120  t  180 时,有:

- 564 -
vAB  V cos t   vAB   d1  d 2 V
vBC  V cos t  120   vBC    d 2  d3 V
(18-110)
vCA  V cos t  240   vC A   d3  d1 V
d2  1
由式(18-110)可得当 120  t  180 时:
d1  1  cos t 
d2  1 (18-111)
d3  1  sin t  30 
当 180  t  240 时,有:
vAB  V cos t   vAB   d1  d 2 V
vBC  V cos t  120   vBC    d 2  d3 V
(18-112)
vCA  V cos t  240   vC A   d3  d1 V
d1  0
由式(18-112)可得:
d1  0
d 2   cos t  (18-113)
d3   sin t  30 
当 240  t  300 时,有:
vAB  V cos t   vAB   d1  d 2 V
vBC  V cos t  120   vBC    d 2  d3 V
(18-114)
vCA  V cos t  240   vC A   d3  d1 V
d3  1
由式(18-114)可得:
d1  1  sin t  30 
d 2  1  sin t  30  (18-115)
d3  1
当 300  t  360 时,有:
vAB  V cos t   vAB   d1  d 2 V
vBC  V cos t  120   vBC    d 2  d3 V
(18-116)
vCA  V cos t  240   vC A   d3  d1 V
d2  0
由式(18-116)可得:
d1  cos t 
d2  0 (18-117)
d3   sin t  30 

- 565 -
综上,可画出 3 相占空比的波形如图 18-18 所示:
1
d1
0.9 d2
d3
0.8

0.7

0.6

0.5

0.4

0.3

0.2

0.1

0
0 1 2 3 4 5 6 7

图 18-18 三相占空比波形

Problem 18.8

如图 18-19 所示的 Buck 型三相整流器工作在 CCM 模式。晶体管 Q1 至 Q6 的

占空比为 d1 至 d 6 。

(a) 推导晶体管运行的限制条件,即哪些晶体管不能同时导通?哪些晶体管的
占空比为 1?
(b) 平均三相开关桥臂,推导交流侧电流的周期平均值 ia (t ) Ts
、 ib (t ) Ts

ic (t ) Ts

(c) 证明直流侧开关电压的周期表达式为:
vd (t ) Ts
  d1 (t )  d 4 (t )  van (t ) Ts
  d 2 (t )  d 5 (t )  vbn (t ) Ts
  d 3 (t )  d 6 (t )  vcn (t ) Ts

(d) 当交流输入电压如书中式(18.150)所示时,且占空比的变化如下所示时,
1 1
d1 (t )   Dm sin t   
3 2
1 1
d 2 (t )   Dm sin t    120 
3 2
1 1
d3 (t )   Dm sin t    240 
3 2
1 1
d 4 (t )   Dm sin t   
3 2
1 1
d5 (t )   Dm sin t    120 
3 2
1 1
d 6 (t )   Dm sin t    240 
3 2

- 566 -
输入滤波对于变换器波形的低频分量的影响可忽略不计。推导稳态直流输出电压
V 的表达式,并用 Vm 、 Dm 和  表示。

(e) 推导功率因数的表达式。你可以认为输入滤波已将电流 ia (t ) 、ib (t ) 和 ic (t )

中的开关纹波完全滤除。然而,输入滤波器却几乎不消耗无功功率。
iL (t ) L

Q1 Q2 Q3
ia
a D1 D2 D3
ib
b C v(t )
c
ic
Q4 Q5 Q6
D4 D5 D6

图 18-19 三相 Buck 型整流器


Solution:
(a) 任意时刻,上桥臂或下桥臂的三个开关管只能允许一个导通。当某一相电
压小于另外两相电压时,此桥臂的下开关管的占空比为 1,比如说当 A 相电压比
B、C 相电压小时, Q4 的占空比为 1。

(b) 交流侧的平均开关电流为:
ia (t ) Ts
 iL Ts
 d1  d 4 
ib (t ) Ts
 iL Ts
 d 2  d5  (18-118)
ic (t ) Ts
 iL Ts
 d3  d 6 
(c) 根据功率平衡有:
vd Ts
iL Ts

(18-119)
 van (t ) Ts
ia (t ) Ts
 vbn (t ) Ts
ib (t ) Ts
 vcn (t ) Ts
ic (t ) Ts

将式(18-118)代入到式(18-119)中,可得:
vd Ts

(18-120)
 van (t ) Ts
 d1  d4   vbn (t ) Ts
 d 2  d5   vcn (t ) Ts
 d3  d 6 
(d) 当输入电压的表达式如书中式(18.150)所示时,这里重新给出:
van (t )  VM sin t 
vbn (t )  Vm sin t  120  (18-121)
vcn (t )  Vm sin t  240 
将式(18-121)及各占空比的表达式代入到式(18-121)中,可得:

- 567 -
3
DmVM cos  vd Ts
 (18-122)
2
(e) 根据功率因数的定义,即书中式(16.15),这里重新给出:
average power
PF  (18-123)
 rmsvoltage  rmscurrent 
当三相平衡时,三相总的功率因数等于各相的功率因数,故接下来求 A 相
的功率因数。
根据式(18-123)和式(18-118)有:
2
1
V sin t  iL Dm sin t    d t 
2
M Ts
PF  0
(18-124)
VM iL Ts Dm
2 2
由式(18-124)可得:
PF  cos  (18-125)

Problem 18.9 (待完成)

如书中 Fig18.45 所示的三相 DCM 反激整流器,其输入滤波对交流输入电流


波形中的低频分量几乎毫无影响。晶体管的开关频率为 f s ,占空比为 d 。反激变

压器 T1 、 T2 和 T3 相对于原边的激磁电感为 L ,匝数比为 n :1 。此外反激变压器的

漏感可忽略不计。
(a) 推导交流输入电流和直流输出电流的低频分量表达式。
(b) 推导变换器的平均等效模型,并给出模型中各元素的表达式。
(c) 推导变换器运行于 DCM 模式的前提条件。
Solution:

Problem 18.10

通用输入 boost 整流器功率级设计。此题的目标是利用 boost 变换器对通用


输入整流器进行设计。希望此变换器应用在世界任何地方,故输入电压有效值的
变化范围为 90V~270V ,其频率的变化范围为 50Hz~60Hz 。此整流器可提供
1000W 的 385V 直流输出。开关频率为 100kHz 。你可以认为控制器运行良好,
可产生无畸变的交流输入电流波形,此外直流输出电压也调整得很好。
(a) 整个交流电周期内,推导占空比 d (t ) 的表达式。你可以忽略变换器的动态

特性及损耗。画出交流输入电压最大和最小这两种情况下的占空比 d (t ) 的变化波

形。

- 568 -
(b) 对电感进行设计:
i. 使得当整流器运行在最恶劣情况,且输入电压达到峰值时,此时电感电
流纹波 ig 为瞬时低频输入电流 ig 的 20% 。

ii. 假设效率为 100% ,求解最恶劣情况下的电感电流的峰值和有效值。


(c) 假设效率为 100% ,推导最恶劣情况下 MOSFET 和二极管电流的有效值。
(d) 对电容值进行设计,使得最恶劣情况下,直流输出电压的低频纹波的峰峰
值为 5V 。
(e) 当考虑以下损耗时:
电感绕组的电阻 0 .
1
MOSFET 的导通电阻 0.1
二极管的正向压降 1.5V
开关损耗 ig2  t  0 . 2
5

对于恒定的 1000W 负载,假设控制器运行良好,求解以下两种情况下的效


率:
i. 输入电压有效值为 90V 时。
ii. 输入电压有效值为 270V 时。
Solution:
(a) 对于 boost 变换器,当其运行在 CCM 模式时,有:
V 1
 (18-126)
vg 1  d

对于输入电压有:
vg (t )  Vm sin t  (18-127)

将式(18-127)代入到式(18-126)中,可得:

Vm sin t 
d  1 (18-128)
V
当交流输入电压的有效值达到最大时,有:

270 2 sin t 


d max  1  (18-129)
385
当交流输入电压的有效值达到最小值,有:

90 2 sin t 
d min  1  (18-130)
385
这两种情况下的占空比波形,如图 18-20 所示:

- 569 -
1

0.875

0.75

270 2 sin(  ) 0.625


1
385
0.5
90 2 sin(  )
1
385 0.375

0.25

0.125

0
0 0.393 0.785 1.178 1.571 1.963 2.356 2.749 3.142

图 18-20 两种情况下的占空比波形
(b) 由于负载恒定,故最恶劣情况为输入电压有效值最小且此时输入电压正处
于正弦波的峰值处时,此时输入电流最大。
对于电感电流的纹波有:
2ig
vg  L (18-131)
dTs

将式(18-128)代入到式(18-131)中,可得:

ig 

TsVm sin t  V  Vm sin t   (18-132)
2 LV
故最恶劣情况下,电感电流纹波最大值发生在正弦波的峰值处,故有:

TsVm _ min V  Vm _ min 


ig _ max  (18-133)
2 LV
最恶劣情况下,输入电流的低频分量的表达式可由下式求出:
v
ig  g _ min
Re
Vm2_ min
P (18-134)
2 Re
vg _ min  Vm _ min sin t 
根据式(18-134)可求出:
2P
ig  sin t  (18-135)
Vm _ min

输入电流低频分量的最大值根据式(18-135)为:
2P
ig _ max  (18-136)
Vm _ min

根据题意,有:

- 570 -
ig _ max  0.2ig _ max (18-137)

由式(18-133)、式(18-136)和式(18-137)可得:
L  0.1356mH (18-138)
最恶劣情况下,电感电流的峰值和有效值为:
ig _ max
ig _ rms   11.11A
2 (18-139)
ig _ peak  ig _ max  ig _ max  18.852A

(c) 一个开关周期内,MOSFET 电流和二极管电流为:


 i 0  t  dTs
iQ   g
0dTs  t  Ts
(18-140)
 00  t  dTs
iD  
ig dTs  t  Ts
故:
2 V 
Vm2_ min sin t  1  m _ min sin t  
iQ2  dig2   V 
2
Ts Re (18-141)
sin t 
3 3
V
 1  d  ig2 
m _ min
iD2
Ts Re2V
根据式(18-141),晶体管电流和二极管电流的有效值为:

1 8Vm _ min
d t   I ac _ rms 1 

I Q _ rms  iQ2
0
Ts 3 V
(18-142)

1 8Vm _ min
d t   I ac _ rms

I D _ rms  iD2
0
Ts 3 V

将已知参数值代入到式(18-142)中,可得:
I Q _ rms  9.424A
(18-143)
I D_ rms  5.886A

(d) 在不考虑损耗的情况下,直流侧的输入功率 pin 等于交流侧的输出功率,

故有:

V sin t  
2
vg2 m
pin  pac   (18-144)
Re Re

当直流侧的纹波很小时,负载功率 pload 为:

- 571 -
Vm2
pload  (18-145)
2 Re

故流进电容的功率 pc 为:

Vm2 cos  2t 


pc  pin  pload  (18-146)
2 Re

对于流进电容的功率 pc ,又有:

1 
d  CvC2 (t ) 
dE (t )
pc  c  2  (18-147)
dt dt
根据式(18-146)和式(18-147)可得:

1 Vm2 sin  2t 


C vc (t )  Ec 0 
2
(18-148)
2 4 Re

接下来求 Ec 0 的表达式,因为:

T
1
 vc (t ) dt  V 2
2
(18-149)
T0

将式(18-148)代入到式(18-149)中,可得:
1
Ec 0  CV 2 (18-150)
2
根据式(18-150)和式(18-148),可得:

P
vc (t )  V 2  sin  2t  (18-151)
C
故电容电压纹波的峰峰值即可求出:

P P
2vc (t )  V 2   V2  (18-152)
C C
根据上式可知,电容电压纹波最大值发生在频率最低处,即发生在   2 50
处。为了使电容电压纹波峰峰值小于 5V ,即:
2vc (t )  5V (18-153)

根据式(18-152)和式(18-153),可求出:
C  1654μF (18-154)
(e) 当交流输入电压的有效值为 90V 时,各电流有效值及交流输入电流的表达
式为:

- 572 -
I ac _ rms  11.111A
I Q _ rms  9.424A
(18-155)
I D _ rms  5.886A
ig (t )  15.71sin t  A
根据式(18-155)题中的各个损耗可计算出:
PL  I ac2 _ rms  0.1  12.346W
PQ  I Q2 _ rms  0.4  35.525W
P (18-156)
PD  1.5  3.896W
V
PS  I ac2 _ rms  0.25  30.864W
故输入电压有效值为 90V 的效率为:
P
  92.4% (18-157)
P  PL  PQ  PD  PS

当输入电压有效值为 270V 时,各电流有效值及交流输入电流的表达式为:


I ac _ rms  3.704A
I Q _ rms  1.473A
(18-158)
I D _ rms  3.398A
ig (t )  5.238sin t  A
各个损耗为:
PL  I ac2 _ rms  0.1  1.372W
PQ  I Q2 _ rms  0.4  0.868W
P (18-159)
PD   1.5  3.896W
V
PS  I ac2 _ rms  0.25  3.429W
故输入电压有效值为 270V 的效率为:
P
  99.1% (18-160)
P  PL  PQ  PD  PS

Problem 18.11

如图 18-21 所示的反激变换器工作在 CCM 模式,MOSFET 的导通电阻为 Ron ,

二极管 D1 的导通压降为 VD 。所有其他损耗可以忽略。反激变压器的匝数比为 1:1 。

控制器很好得控制着占空比的变化,使得 ig (t ) 等于 vg (t ) / Re ,Re 为模拟电阻。


Ts

- 573 -
输入电压的表达式为 vg (t )  Vm sin t  。输入滤波器可以将输入电流 ig (t ) 中的开

关频率的纹波移除,且对变换器波形低频分量的影响可忽略不计。
(a) 推导整流器效率的表达式,用 Vm 、 V 、 VD 、 Ron 和 Re 表示。

(b) 当给定下列值时,找出使得变换器的效率为 96% 时的 MOSEFT 导通电阻


的值。
输入电压有效值 120V
直流输出电压 120V
二极管 D1 导通压降 1.5V

负载功率 200W

iin (t ) ig (t ) id (t ) i (t )

L D1
vin (t ) vg (t ) C R v(t )

控制器

图 18-21 CCM 模式下的反激整流器


Solution:
(a) 首先推导变换器低频稳态模型(不考虑动态,仅考虑低频损耗的模型)。
当开关管导通时,稳态等效电路如图 18-22 所示,对于激磁电感 L 上的电压
和输出电容的电流有:

ig (t ) i (t )
ic (t )
vL L
vg (t ) C R v(t )
Ron

图 18-22 开关管导通时的稳态等效电路

vL  vg (t )  ig (t ) Ron
v(t )
ic   (18-161)
R
ig (t )  iL (t )

当开关管关断时,稳态等效电路如图 18-23 所示,故有:

- 574 -
ig (t ) VD id (t ) i (t )

vL L
vg (t ) C R v(t )

图 18-23 开关管关断时的稳态等效电路
vL  v(t )  VD
v(t )
ic (t )  id (t ) 
R (18-162)
id (t )  iL (t )
ig (t )  0

根据激磁电感的伏秒平衡和电容的电荷平衡,联立式(18-161)和式(18-162)可
得:
vL Ts
 d  t  vg  t   iL  t  Ron   1  d  t    v  t   VD   0
 v t    v t  
iC Ts
 d  t     1  d  t   iL  t   0 (18-163)
 R   R 
ig  d  t  iL  t    1  d  t    0
Ts

将式(18-163)化简,可得:
dvg  t   1  d  v  t   1  d VD  diL  t  Ron
v t 
1  d  iL  t   (18-164)
R
ig  t   diL  t 
根据式(18-164)可画出低频稳态等效电路如图 18-24 所示:

ig (t ) dRon 1  d VD 1  d  iL (t )
iL (t )
vg (t ) R v(t )

1: d 1  d :1
图 18-24 低频稳态等效电路
模拟电阻和输入电流的关系为:
vg  t 
Re  (18-165)
ig  t 

根据式(18-164)和式(18-165)可求得:

- 575 -
ReVD  Re v  t   Ron vg  t  ReVD  ReV  RonVm sin t 
d t    (18-166)
Re vg  t   v  t   VD  Re Vm sin t   V  VD 
接下来求负载电流直流分量的表达式。
根据图 18-24 可求得:

I  1  d  iL  t 
1  d  i 1  d  Vm sin t 
Tac
 g t   (18-167)
d Tac
d Re
Tac

将式(18-166)代入到式(18-167)中,故有:
2 Vm  Re  Ron  2 sin 2 
2

 Re2 V  VD  0 1  a sin 
I d
(18-168)
RonVm
a
Re V  VD 
利用书中式(18.137)和书中式(18.138),这里重新给出:

4 sin 2 
d  F  a  
  1  a sin 
2
0

2  4sin 1  a   2 cos 1  a  
 2  2a     (18-169)
a 1  a2 
F  a   1  0.862a  0.78a 2

将式(18-169)代入到式(18-168)中,可得:
2 Vm  Re  Ron  
2

I
 Re V  VD  4
2 1  0.862a  0.78a 2 

1 Vm  Re  Ron 
2


2 Re V  VD 
2 1  0.862a  0.78a 2  (18-170)

RonVm
a
Re V  VD 
对于效率有:
Pout VI
  2 (18-171)
Pin Vm
2 Re

将式(18-170)代入到式(18-171)中,可得:
V  Re  Ron 

Re V  VD 
1  0.862a  0.78a 2 
(18-172)
RonVm
a
Re V  VD 
(b) 当使用题中的给定参数时,有:

- 576 -
Vm  120 2V
V  120V
VD  1.5V (18-173)
Pout  200W
  0.96
根据式(18-171)及式(18-173)可求出:
Vm2
Re   69.12 (18-174)
2 Pout

将式(18-173)和式(18-174)代入到式(18-172)中,可求出:
Ron  0.89225 (18-175)

Problem 18.12

对于使用交流输入电压前馈的平均电流控制的 boost 整流器,如书中图 18.14


所示。推导模拟电阻的表达式 Re Vg ,rms , Rs , kv , vcontrol  。

Solution:
根据书中 Fig18.14 有:
vg (t )
Re 
ig (t )
va  ig (t ) Rs
(18-176)
kv vg vcontrol
vref 1 
Vm2
Vm  2Vg ,rms

当补偿器运行良好时,有:
vref 1  va (18-177)

根据式(18-176)和式(18-177)可求出:
2 RsVg2,rms
Re  (18-178)
kv vcontrol

Problem 18.13

推导 CPM 模式 boost 整流器的静态输入特性,即书中式(18.57)。


Solution:
首先推导 CCM 模式下的 CPM 静态输入特性,考虑人工载波和电流纹波的

- 577 -
影响,有:
m1dTs m d T
ig  t   ic  t   ma dTs  d  d 2 s
Ts 2 2
vg  t 
m1 
L
V  vg  t 
m2  (18-179)
L
vg  t 
d  1
V
vg  t 
d  1 d 
V
根据式(18-179)可求出:
 v t    v t  
ig  t   ic  t   1  g  ma  g  Ts (18-180)
 V  2L 
Ts

接下来推导 DCM 模式下的 CPM 静态输入特性。电路如图 18-25 所示:

ig (t ) v2
vL i2 iC
vg (t ) v1 C R V
i1

图 18-25 DCM 模式下的 CPM 控制方式的 boost 整流器


故有:
ic  t    m1  ma  d1Ts
ic  t  (18-181)
d1 
 m1  ma  Ts
晶体管电流 i1 的周期平均值为:

1 1 1
i1 Ts
 d1Ts m1d1Ts  d1Ts m1d1 (18-182)
Ts 2 2

将式(18-181)代入到式(18-182)中,可得:
ic2 f s m1
i1 
2  m1  ma 
Ts 2

(18-183)
vg v1
m1  
Ts

L L
根据式(18-183)可求出:

- 578 -
1 2
Lic  t  f s
i1  t   2
2
(18-184)
 ma 
Ts

v1  t  T 1  
s
 m1 
根据式(18-184)可求出:
1 2
Lic  t  f s
v1  t  i1  t   2
2
P (18-185)
 ma 
Ts Ts

1  
 m1 

二极管电流 i2 的周期平均值为:

1 1 1
i1 Ts
 d 2Ts m1d1Ts  d 2Ts m1d1 (18-186)
Ts 2 2

对于电感 L ,由伏秒平衡可得:
vg  t  d1  vg  t   V  d 2  0
v1  t  Ts
 vg (18-187)
Ts

v2  t  Ts
 V Ts
 vg
Ts

根据式(18-187)、式(18-186)和式(18-181),可求得:
1 2
Lic  t  f s
v2  t  i2  t  2 2
P (18-188)
 ma 
Ts Ts

1  
 m1 
注意 P 并不是负载功率,其和负载功率的关系为:
vg  t  ig  t   V iD  t  Ts
Ts Ts
(18-189)
V  vg  t   i t  P
 Ts Ts 
 D Ts

根据式(18-188)可求得:
1 2
Lic  t  f s vg  t V
ig  t   2 (18-190)
V  vg  t   vg  t   Lma 
Ts 2

Problem 18.14

如书中 Fig18.47 所示的 boost 整流器,使用带有输入电压前馈的平均电流控

- 579 -
制策略。交流电压的频率为 50Hz 。整流器驱动 500W 的恒定负载。PWM 调制器
为一锯齿波,其峰峰值为 3V 。内部的宽带宽平均电流反馈环路无补偿器(在此,
就认为 Gc  s   1 )。平均电流检测电路的增益为:

va  s  Rs

ig  s   s 
1  
 0 
其余的变换器参数为:
f s  100kHzL  2.5mH
0
f0   10kHzRs  1Ω
2
V  385VVg ,rms  230V

(a) 构建环路增益 Ti  s  的幅值和相位 Bode 图。

(b) 给出 Ti  s  的截止频率和相位裕度。

外部的低带宽反馈环的环路增益为 Tv  s  。此环路的补偿器具有恒定增益

Gcv  s   330 ,放大器的增益 kv  2 。输出电容的值为 C  680μF 。参考电压

vr e f2  3.85V。

(c) 推导输出电压中 100Hz 纹波的峰值。


(d) 推导控制电压 vcontrol 的静态值。

(e) 构建外部反馈环 Tv  s  的幅值和相位 Bode 图。

(f) 推导 Tv  s  的截止频率和相位裕度。

Solution:
(a) 内环的框图如图 18-26 所示:
vref 1 1 d ig  s  ig
Gc  s   1
VM d s
va
va  s 
ig  s 

图 18-26 内环框图

故环路增益 Ti  s  为:

- 580 -
1 ig  s  va  s 
Ti  s   (18-191)
Vm d  s  ig  s 

接下来求从占空比到输入电流的传递函数。
当开关管导通时,有:
dig
L  vg
dt (18-192)
dv v
C 
dt R
当开关管断开时,有:
dig
L  vg  v
dt (18-193)
dv v
C  ig 
dt R
将式(18-192)和式(18-193)进行开关周期平均,有:
dig
L  vg  d v
dt (18-194)
dv v
C  d ig 
dt R
对于式(18-194)可以将输出电压表示成静态量与小信号交流扰动的和,即:
v  V  vˆ (18-195)
将式(18-195)代入到式(18-194)中,可得:
dig
L  vg  d V  d vˆ
dt (18-196)
dv V vˆ
C  d ig  
dt R R
注意,式(18-196)不是小信号方程,要考虑此式中的主要部分,故要忽略值
很小的扰动项,故有:
dig
L  vg  d V
dt (18-197)
dv V
C  d ig 
dt R
根据式(18-197)可画出等效电路如图 18-27 所示:

- 581 -
ig L d ig

vg d V C R V

图 18-27 boost 整流器大信号等效电路


图 18-27 可进一步化简为如图 18-28 所示:
ig L d ig

vg C R V

d  :1
图 18-28 简化的 boost 整流器大信号等效电路

注意图 18-28 相当于有两个输入,一个是输入电压 vg ,另一个是占空比 d  。

为了求从占空比 d 到输入电流 ig 的传递函数,故需忽略图 18-28 中的输入电压,

则图 18-28 可简化为如图 18-29 所示:


ig L d ig

C R V

d  :1
图 18-29 忽略输入电压时的简化的 boost 整流器大信号等效电路
故有:
sLig  d V   1  d V  dV  V (18-198)

根据式(18-198)可求得从占空比 d 到输入电流 ig 的传递函数:

ig  s  V
 (18-199)
d s sL

根据式(18-191)和式(18-199)可求得环路增益 Ti  s  的传递函数为:

1 V Rs
Ti  s   (18-200)
Vm sL  s 
1  
 0 
将题中的参数值代入到式(18-200)中,可得:

- 582 -
128.3
Ti  (18-201)
3.979 108 s 2  0.0025s
环路增益 Ti  s  的幅值和相位 Bode 图如图 18-30 所示:

Bode Diagram
40

20 System: Ti
Frequency (rad/s): 4.24e+04
Magnitude (dB)

Magnitude (dB): 0.0306


0

-20

-40

-90
Phase (deg)

System: Ti
-135 Phase Margin (deg): 55.9
Delay Margin (sec): 2.3e-05
At frequency (rad/s): 4.25e+04
Closed loop stable? Yes

-180
3 4 5 6
10 10 10 10
Frequency (rad/s)

图 18-30 环路增益 Ti  s  的 Bode 图

(b) 环路增益 Ti  s  的截止频率和相位裕度从图 18-30 中可得:

f crossover  6.76kHz
(18-202)
PM  55.9
(c) 根据 Problem 18.10 中的式(18-152),这里重新给出:

P P
2vc (t )  V 2   V2  (18-203)
C C
将给定参数值代入到式(18-203)中,可得:
vc  1.52V (18-204)

(d) 当整流器处于稳态时,输入电流 ig 的峰值为:

2P
I g _ peak   3.074A (18-205)
Vg ,rms

故稳态时,对于参考信号 vref 1 的峰值来说,有:

vref 1_ peak  I g _ peak  3.074 (18-206)

故有:

- 583 -
kv x peak y
vref 1_ peak 
z2
vg _ peak
x peak 
100 (18-207)
y  vcontrol
vg _ peak
z
100
根据式(18-207)可求得:
vcontrol  5 (18-208)

(e) 外环的框图如图 18-31 所示:


vref 2 vˆcontrol vˆ  s  v̂
Gcv  s   330
vˆcontrol  s 

1
100

图 18-31 外环控制框图

为了求出外环的环路增益 Tv  s  ,故需求出从控制信号 vcontrol 到输出电压 v 的

传递函数。
对于输入电压有:
vg  t   2vg _ rms sin t  (18-209)

对于瞬时输入功率,进行开关周期平均后,有:
vg  t 
2

p t  
Ts
(18-210)
Ts
Re vcontrol  t  
将式(18-209)代入到式(18-210)中,可得:
Vg2_ rms
p t   1  cos  2t   (18-211)
Ts
Re vcontrol  t   

根据式(18-211)可画出 boost 整流器大信号等效电路,如图 18-32 所示:

Vg2_ rms Vg2_ rms


 cos  2t  C v
Re vcontrol  t   Re vcontrol  t   Ts

图 18-32 boost 整流器大信号等效电路


为了简化分析,当只考虑低于 2 倍输入频率的波形时,对图 18-32 进行 2
倍交流输入频率的周期平均后,等效电路如图 18-33 所示:

- 584 -
Vg2_ rms
C v
Re vcontrol  t  
T2 L

图 18-33 2 倍交流输入频率的周期平均后的大信号等效电路

接下来,求模拟电阻 Re 的表达式,当内环相比外环足够快时,即对于要考

虑的低于 2 倍输入频率以下的分量时,内环始终处于稳定状态。故有:
vref 1  t   ig  t 
vg  t 
 Re
ig  t 
kv xy
vref 1 
z2 (18-212)
vg  t 
x
100
y  vcontrol  t 
2Vg _ rms
z
100
根据式(18-212)可求得:
Vg2_ rms
Re vcontrol  t    (18-213)
50kv vcontrol  t 

将式(18-213)代入到图 18-33 中,可重新画出大信号低频等效电路,如图


18-34 所示:

i T2 L

50kv vcontrol  t  C v T2 L

图 18-34 大信号低频等效电路
故有:
p 50kv vcontrol  t 
   f vcontrol  t  , v 
T2 L
i (18-214)
T2 L
v vT  T2 L 
T2 L 2L

由于式(18-214)非线性,故需要通过泰勒级数对其进行线性化,忽略二阶以
上的非线性,有:

- 585 -
I  iˆ  f Vcontrol ,V  
f  vcontrol ,V  f Vcontrol , v  (18-215)
vˆcontrol  vˆ
vcontrol v
v control Vcontrol v V

根据图 18-34 可求出:

f  vcontrol , V  50kv

vcontrol V
v control Vcontrol
(18-216)
f Vcontrol , v  50kvVcontrol

v v V
V2

故交流线性分量为:
50kv 50kvVcontrol
iˆ  vˆcontrol  vˆ (18-217)
V V2
根据式(18-217)可画出低频小信号等效电路,如图 18-35 所示:

50kv V2
vˆcontrol C R v̂
V 50kvVcontrol

图 18-35 低频小信号等效电路
根据图 18-35 可得:
50kv vˆcontrol
vˆ  Z
V
1 1 1 (18-218)
 2
  sC
Z V R
50kvVcontrol

根据式(18-218),可求得从控制信号到输出电压的传递函数为:
vˆ  s  50 RVkv
 (18-219)
vˆcontrol  s  CRV s  V 2  50 RVcontrol kv
2

将参数值代入到式(18-219)中,可得:
vˆ  s  1.141107
 (18-220)
vˆcontrol  s  2.988 104 s  296450

将式(18-220)代入到图 18-31 中,可得:

3.766 107
Tv  s   (18-221)
2.988 104 s  296450
其 Bode 图如图 18-36 所示:

- 586 -
Bode Diagram
60

40

Magnitude (dB)
20

-20
0
Phase (deg)

-45
System: Tv
Phase Margin (deg): 90.5
Delay Margin (sec): 0.00125
At frequency (Hz): 201
Closed loop stable? Yes
-90
-1 0 1 2 3
10 10 10 10 10
Frequency (Hz)

图 18-36 外环环路增益 Bode 图


(f) 由图 18-36 可知,其截止频率和相位裕度为:
f crossover  201Hz
(18-222)
PM  90.5

Problem 18.15

某 boost 整流器使用临界导通模式控制策略,其交流输入电流波形如书中
Fig18.19(b)所示。交流输入电压的有效值为 120V ,频率为 60Hz 。整流器以 225V
的输出电压、 120W 的功率向负载提供能量。boost 整流器的电感值为 600μH 。

(a) 推导模拟电阻 Re 的表达式。

(b) 推导变换器开关频率的表达式,用导通时间 ton 和施加的端电压进行表示,

并画出开关频率随着时间变化的图形。
(c) 开关频率的最大值和最小值是多少?
(d) 推导此控制模式下晶体管电流有效值的表达式。并与连续导通模式下的晶
体管电流有效值做对比。
Solution:
(a) 对于模拟电阻,有:
vg  t 
Re 
Ts

ig  t 
Ts

ig _ peak  t 
vg  t  L (18-223)
Ts ton
ig _ peak  t   2 ig  t 
Ts

根据式(18-223)可得:
2L
Re  (18-224)
ton

- 587 -
(b) 对于开关频率有:
1
fs t  
Ts  t 
Ts  t   ton  toff
ig _ peak  t  (18-225)
V  vg  t   L
toff  t 
ig _ peak  t 
vg  t   L
ton

根据式(18-225)可得:
V  vg  t 
fs t  
Vton (18-226)
vg  t   Vm sin t 
(c) 根据式(18-226)可知:
1
f s _ max 
ton
(18-227)
V  Vm
f s _ min 
Vton
(d) 推导晶体管电流有效值的公式为:

I rms  iQ2  t  (18-228)


Ts T2 L

首先求最里面的积分,有:
1 1
iQ2  t   iQ  t   dt  iQ  t   dt
Ts

2 ton 2

Ts Ts 0 T s
0
(18-229)
vg  t 
iQ  t   t
L

根据式(18-229)可得(在此积分中,要认为 vg  t  是常数,即在一个开关周期

内认为输入电压 vg  t  保持不变。另外晶体管电流的表达式也是建立在输入电压

保持不变的前提下):

vg  t   V  vg  t  
2 2
ton
iQ2  t    (18-230)
Ts 3L2V
接下来求式(18-228)中的外层积分,将式(18-230)代入到式(18-228)中,可得:

- 588 -
1 T 2
iQ2  t  iQ  t 
T 0
I rms   dt
Ts T2 L Ts


T
 (18-231)
t vg  t   V  vg  t 
2 2

iQ2  t  
on
Ts 3L2V
vg  t   Vm sin t 
根据式(18-231)可得:

I rms 
Vm2ton
2
 3V  8Vm  (18-232)
18 L2V
对于晶体管的导通时间有:
Vm2
Re 
2P
(18-233)
2L
Re 
ton

根据式(18-233)可求得:
4 LP
ton  (18-234)
Vm2

将式(18-234)代入到式(18-232)中可得:
8P 2  3V  8Vm 
I rms  (18-235)
9VVm2
又因为:
Vm
P I ac _ rms (18-236)
2
将式(18-236)代入到式(18-235)中,可得临界导通模式下 boost 整流器晶体管
电流有效值为:
2 I ac _ rms 8Vm
I rms  1 (18-237)
3 3 V
对于连续导通模式下的 boost 整流器的晶体管电流有效值,其表达式已在式
(18-142)中求出,这里重新给出:

8Vm
I rms  I ac _ rms 1  (18-238)
3V
通过比较式(18-237)和式(18-238),可发现临界导通模式下的晶体管电流有效

- 589 -
2
值是连续导通模式下晶体管电流有效值的  1.155 倍。
3

- 590 -
第19章 谐振变换器

Problem 19.1

对 DCDC 半桥并联谐振变换器进行分析,其运行点处于谐振频率之上。如
图 19-1 所示,元器件 Cb 、 LF 和 CF 的值足够大,其开关频率纹波可忽略不计。

你可以认为所有的元器件均为理想器件。你也可以使用正弦近似方法进行分析。
(a) 画出输入电流 ig  t  的波形。

(b) 构建此变换器的等效电路,类似于书中的 Fig19.22。并给出等效电路中各


元器件值的表达式。
(c) 对等效电路进行求解,推导转换比率 V / Vg  M  F , Qe , n  。

当处于额定负载(最大负载)时,此变换器的电流输出为 I  20A ,电压输出为


V  3.3V 。
(d) 此时变换器的运行频率是多少?
(e) 此时晶体管的峰值电流是多少?
当处于最小负载时,此变换器的电流输出为 I  2A ,电压输出为 V  3.3V 。
(f) 此时变换器的运行频率是多少?
(g) 此时晶体管的峰值电流是多少?和(e)作对比,对效率和损耗进行分析。
ig  t 

Q1 D1
L
Vg Cb 10μH n :1 iR1  t  LF
I
160V
D2
vR1  t 
Q2 C
vs  t  n  20 1.5μF CF R V

图 19-1 半桥并联谐振变换器
Solution:
(a) 如果谐振变换器的运行频率在谐振频率附近,则可认为谐振网络的电流为
正弦波,当运行频率稍高于谐振频率时,并联谐振网络的输入阻抗呈感性,故电
压会稍超前于电流波形,则此时输入电流波形如图 19-2 所示:

- 591 -
vs  t 
Vg

0 0.5Ts Ts t
ig  t 

0 t

图 19-2 输入电流波形

(b) 由于 LF 和 CF 的值足够大,故电感 LF 上的电流近似为直流,则整流桥前

的电流 iR  t  则近似为方波,其波形如图 19-3 所示,注意图 19-3 仅表示波形的

幅值和形状,其时间轴与图 19-2 中的时间轴不一致。


iR  t 

0 t

图 19-3 整流桥前的电流波形

由于网络为并联谐振,故电容 C 上的电压为正弦波,因此 vR1 和 iR 的基波分

量 iR1 可用下式进行表示:

vR1  t   VR1 sin s t 


4 (19-1)
iR1  t   I sin s t 

故从整流桥的前端向负载端看去的模拟电阻为:
vR1  t   VR1
Re   (19-2)
iR1  t  4I

对于负载端,有:
V
R (19-3)
I
为了推导模拟电阻 Re 的表达式,故需推导输出电压 V 和整流桥的输入电压

vR1 之间的关系。显然 vR1  t  的直流分量为输出电压 V ,故有:

- 592 -

1
V sin   d  V (19-4)
 0
R1

根据式(19-4)可得:
2
VR1  V (19-5)

根据式(19-2)、式(19-3)和式(19-5)可得:

2
Re  R (19-6)
8
根据式(19-6),可利用模拟电阻等效整流桥侧的电路,如图 19-4 所示:

ig  t 

Q1 D1
L
Vg ip Cb 10μH n :1 is  t  iR1  t  I
160V
D2
vR1  t 
Q2 C 2
vs  t  vp 1.5μF Re VR1 R V

n  20 2
Re  R
8

图 19-4 等效电路 1
接下来要将变压器右边的电路等效至原边侧,根据图 19-4 有:
vp
n
vR1
is
n (19-7)
ip
vR1
is  sCvR1 
Re
根据式(19-7)可求出:
vp 1

(19-8)
1 1 ip

n 2 Re 1
C
s 2
n
根据式(19-8),可将图 19-4 中的副边侧进一步等效至原边侧,如图 19-5 所
示:

- 593 -
ig  t 

Q1 D1
L
Vg ip Cb 10μH is  t  iR1  t  I
160V
D2 vp
vR1  t 
Q2 C C 2
vs  t  vp n 2 Re 1.5μF Re VR1 R V
n2 n 

n  20 2
Re  R
8

图 19-5 等效电路 2

接下来,对输入侧的半桥进行等效,寻求 vs  t  和输入电压 Vg 之间的关系。

vs  t  的波形如图 19-6 所示,其中横轴时间已标幺化:

vs  t 

Vg

0  2 t

图 19-6 vs  t  的波形

vs  t  的表达式为:

V 0  t  
vs  t    g (19-9)
 0  t  2

vs  t  可用傅里叶级数进行表示,即:

a0  
vs  t     an cos nt   bn sin nt (19-10)
2 n 1 n 1

对于式(19-10)中的系数求解,有:

- 594 -
2 2
a 1
0 vs t dt  20 2  a0    v t dt
0
s

2 2

 vs t  cos  nt  dt  a cos  nt   d t 


2
n
0 0
2
1
an 
  v t  cos  nt  dt
0
s (19-11)

2 2

 vs t  sin  nt  dt  b sin  nt   d t 


2
n
0 0
2
1
bn 
  v t  sin  nt  dt
0
s

根据式(19-11)可求出:
a0  Vg
a1  0 (19-12)
2Vg
b1 

假设 vs  t  波形中只有 1 次基波 vs1  t  起主要作用,直流分量和其余谐波均忽

略不计,因此电流 i p 的波形为正弦,其频率和 vs1  t  一致。根据式(19-12)可求出

vs1  t  和 i p  t  的表达式为:

2Vg
vs1  t   sin t 
 (19-13)
i p  t   I p sin t   

故输入电流的直流分量为:

1 I
ig  t    I p sin t   d t   p cos   (19-14)
Ts 2 0 
根据式(19-13)可画出最终的等效电路如图 19-7 所示:
ip t  
ig  t  I p sin t   
L
Ts Cb 10μH is  t  iR1  t  I

Ip 2Vg vp
Vg
cos   sin t  vs1  t  vp
C
n 2 Re
C vR1  t  Re
2
VR1 R V
160V   n2 n 1.5μF 

n  20 2
Re  R
8

图 19-7 最终的等效电路
(c) 转换比率为:

- 595 -
V  V   I  I R1  VR1   VP   VS 1 
       
Vg  I   I R1  VR1  VP   VS1   Vg 
(19-15)
   8  1       2   H  s  
2 4
 R    2    H  s 
 4   R  n      n 

为了求出转换比率的表达式,接下来还需推导出传递函数 H  s  的表达式(由

于 Cb 的值足够大,故可忽略不计):

n 2 Re 1
H 
CLRe s  Ls  n Re  s 
2 2 2
s
   1
 0  Qe0
n
0   2 f 0 (19-16)
CL
C
Qe  nRe
L

故传递函数 H  s  的模为:

Qe
H s 
F 2  Qe2 1  F 2 
2

(19-17)

F
0
根据式(19-15)和式(19-17)可求出转换比率为:
V 4 Qe
 2 (19-18)
Vg n F  Q 1  F
2 2
e 
2 2

(d) 当运行在最大负载时,此时各参数的值为:
V  3.3V
I  20A
V
R   0.165
I (19-19)
Re  0.204
Qe  1.577
0  5.164 106 rad/s
将式(19-19)代入到式(19-18)中,可求得此时的开关频率应为:
f s  1.032MHz (19-20)

(e) 为了求出晶体管电流的最大值 I p ,接下来要求出输入阻抗 Z i (从 vs1  t  向

- 596 -
输出侧看的阻抗)。
故有:
CLRe s 2  Ls  n 2 Re
Zi  (19-21)
CRe s  1

输入阻抗 Z i 的模为:

 L    n2 Re  CLRe 2 
2 2

Zi  (19-22)
1   CRe 
2

将式(19-19)和式(19-20)中的参数值代入到式(19-22)中,可算出此时的输入阻
抗值为:
Z i  36.075 (19-23)

根据式(19-13)和式(19-23)可求出晶体管电流峰值为:
2Vg
   2.824A
Vs1
Ip  (19-24)
Zi Zi

(f) 当变换器运行于最小负载时,此时各参数的值为:
V  3.3V
I  20A
V
R   1.65
I (19-25)
Re  2.036
Qe  15.768
0  5.164 106 rad/s
将式(19-25)代入到式(19-18)中,可求得此时的开关频率应为:
f s  1.156MHz (19-26)

(g) 此时的输入阻抗的模和晶体管电流峰值为:
Zi  36.034
Vs1 (19-27)
Ip   2.827
Zi
由于轻载时晶体管电流峰值更大,故轻载时的晶体管损耗更高,效率也因
此降低。

Problem 19.2

某 DCDC 谐振变换器包含如书中 Fig19.1(d)所示的 LCC 谐振网络,其输

- 597 -
出滤波器包含一个类似于并联谐振 DCDC 变换器中的滤波电感。
(a) 使用正弦近似法,画出此变换器的等效电路模型,并给出参数值的表达式。
(b) 求 解 模 型 , 推 导 转 换 比 率 M  V / Vg , 用 F  fs / f , Qe  Re / R0 和

n  Cs / C p 进行表示。其中 f  的定义在书中式 19.50 中, R0 的定义为:

L  Cs  C p 
R0 
Cs C p

(c) 当 n  1 , Qe  1, 2,5 时,画出 M 和 F 的关系。

(d) 当 n  0.25 , Qe  1, 2,5 时,画出 M 和 F 的关系。

Solution:
(a) 变换器的电路如图 19-8 所示:
L Cs iR  t  I

Vg vs  t  Cp vR  t  R V

图 19-8 DCDC LCC 谐振变换器


当输出侧的滤波电感值足够大时,则电感电流近似为直流,故整流桥前的输
入电流 iR  t  的基波分量为:

4I
iR1  t   cos t 
 (19-28)
4I
I R1 

对于 vR  t  有:


1
V cos t  d t   V (19-29)
 0
R1

根据式(19-29)可得:
2VR1
V (19-30)

根据式(19-28)和式(19-30),可得:
VR1  2
Re   R (19-31)
I R1 8

- 598 -
对于 vs  t  中的基波分量,有(注意下式中的 t 和式(19-29)中的 t 仅是代表

符号相同而已,其相位可能不一致):
4
vs1  t   Vg sin t  (19-32)

故可根据式(19-31)和式(19-32)画出等效电路如图 19-9 所示:
ip t  
ig  t  I p sin t    L Cs
Ts I

2I p 4Vg Cp
Vg cos   sin t  vs1  t  vR1  t  Re
2
VR1 R V
  
2
Re  R
8
图 19-9 等效电路
(b) 对于转换比率有:
V  V   I  I R1   VR1   Vs1 
M        
Vg  I   I R1  VR1   Vs1   Vg 
(19-33)
   1      2 H  s 
4 8
 R       H  s 
 4   Re    

接下来推导传递函数 H  s  ,故有:

Cs Re
H s  (19-34)
 Cs L 2  1  Re2 2  C p  C pCs L 2  Cs 
2 2

接下来推导传递函数 H  s  的模 H  s  ,故有:

Cs Re
H s  (19-35)
2
 1
 Cs L    Re  C p  C p Cs L  Cs 
2 2 2

 
根据题意有:
L  Cs  C p 
R0  (19-36)
Cs C p
又由于:
Re
Qe  (19-37)
R0

将式(19-36)代入到式(19-37)中可得:

- 599 -
L  Cs  C p 
Re  Qe (19-38)
Cs C p
又由于:
f s s
F 
f  
1 (19-39)
 
Cs C p
L
Cs  C p
根据式(19-39)可推出:
F 2  Cs  C p 
L (19-40)
Cs C p 2
将式(19-38)和式(19-40)代入到式(19-35)中,可得:
Qe F  n  1
H s  (19-41)
F 2Qe2  n  1  F 2  1
4 2

 nF  F 2  1 
2 2

n2
根据式(19-33)和式(19-41),可推导出电压转换比率为:
V 8 Qe F  n  1
 2 (19-42)
Vg  F Q  n  1  F  1
2 2 4 2 2

 nF  F 2  1 
2 2 e

n2

(c) 当 n  1 , Qe  1 时,转换比率 M 和 F 的关系如图 19-10 所示:

2.5

2.188

1.875

1.563

1.25

0.938

0.625

0.313

0
0.01 1.25875 2.5075 3.75625 5.005 6.25375 7.5025 8.75125 10

图 19-10 n  1 , Qe  1 时,转换比率 M 和 F 的关系

当 n  1 , Qe  2 时,转换比率 M 和 F 的关系如图 19-11 所示:

- 600 -
5

4.375

3.75

3.125

2.5

1.875

1.25

0.625

0
0.01 1.25875 2.5075 3.75625 5.005 6.25375 7.5025 8.75125 10

图 19-11 n  1 , Qe  2 时,转换比率 M 和 F 的关系

当 n  1 , Qe  5 时,转换比率 M 和 F 的关系如图 19-12 所示:

12.5

10.938

9.375

7.813

6.25

4.688

3.125

1.563

0
0.01 1.25875 2.5075 3.75625 5.005 6.25375 7.5025 8.75125 10

图 19-12 n  1 , Qe  5 时,转换比率 M 和 F 的关系

(d) 当 n  0.25 , Qe  1 时,转换比率 M 和 F 的关系如图 19-13 所示:

- 601 -
7

6.125

5.25

4.375

3.5

2.625

1.75

0.875

0
0.01 1.25875 2.5075 3.75625 5.005 6.25375 7.5025 8.75125 10

图 19-13 n  0.25 , Qe  1 时,转换比率 M 和 F 的关系

当 n  0.25 , Qe  2 时,转换比率 M 和 F 的关系如图 19-14 所示:

12

10.5

7.5

4.5

1.5

0
0.01 1.25875 2.5075 3.75625 5.005 6.25375 7.5025 8.75125 10

图 19-14 n  0.25 , Qe  2 时,转换比率 M 和 F 的关系

当 n  0.25 , Qe  5 时,转换比率 M 和 F 的关系如图 19-15 所示:

- 602 -
30

26.25

22.5

18.75

15

11.25

7.5

3.75

0
0.01 1.25875 2.5075 3.75625 5.005 6.25375 7.5025 8.75125 10

图 19-15 n  0.25 , Qe  5 时,转换比率 M 和 F 的关系

Problem 19.3

串联谐振变换器的对偶形式。如图 19-16 所示的变换器, LF 1 、 LF 2 和 CF 是

值足够大的滤波元器件,其上的纹波可忽略不计。 L 和 C 是谐振网络的元器件,
其电流 iL  t  和电压 vC  t  的波形十分接近正弦。

(a) 使用正弦近似法,推导开关网络、谐振网络和整流网络的等效电路。
(b) 画出并联谐振网络的输入阻抗 Bode 图。
(c) 求解模型,推导转换比率 M  V / Vg 的解析式,使用 Qe 和 F  f s / f 0 来表

达,并画出 M 和 F 的关系。
(d) 如何评价正弦近似的有效性?(c)中的 M 和 F 的关系图中哪一部分是有
效和准确的?
Ig LF 1

D1 D3
vc  t 
Q1 Q3
C
Vg is  t 
L
D2 D4
iL  t 
Q2 Q4
LF 2 I

CF R V
iR  t 

图 19-16 串联谐振变换器的对偶形式

- 603 -
Solution:
(a) 首先推导开关网络的等效电路,由于 LF 1 的值足够大,故电流 is  t  的波形

如图 19-17 所示:
is  t 

Ig
0
t
I g

图 19-17 电流 is  t  的波形

则电流 is  t  的基波分量为:

4
is1  t   I g sin t  (19-43)

假设 vc  t  的表达式为:

vc  t   Vc sin t    (19-44)

故有:
1 
Vc sin t   d t   Vg

(19-45)
0

根据式(19-45)可求得:
2
Vc cos   Vg (19-46)

故开关侧的等效电路如图 19-18 所示:

4
is1  t   I g sin t 
Ig 

2Vc
Vg cos  vc  t   Vc sin t   

图 19-18 开关侧的等效电路
接下来推导整流电路的等效电路,有:

- 604 -
V
R
I
iR1  t   I R1 sin t 
4 (19-47)
I R1  I


1
  V sin t d t   V
0
c

根据式(19-47)可求得:
Vc  2
 R (19-48)
I R1 8

故可画出最终的等效电路如图 19-19 所示:


4
is1  t   I g sin t 
Ig  iR1  t  I

2Vc
vc  t   Vc sin t   
2
Vg cos  vc L Re Vc R V
 
Zi iL
2
Re  R
8

图 19-19 最终的等效电路
(b) 输入阻抗的表达式为:
1 1 1
 sC  
Zi sL Re
(19-49)
LRe s
Zi 
CLRe s 2  Ls  Re
为了画出输入阻抗的 Bode 图,这里先任意给出各参数值,以便使用 Matlab
画出 Bode 图:
C  47μF
L  20μH (19-50)
Re  10Ω

给定参数值后,输入阻抗的 Bode 图如图 19-20 所示:

- 605 -
Bode Diagram
20
Zi
10

Magnitude (dB)
-10

-20

-30

-40
90
Zi

45
Phase (deg)

-45

-90
3 4 5 6
10 10 10 10
Frequency (rad/s)

图 19-20 给定假设的参数值后的输入阻抗 Bode 图


(c) 因为输入有功功率等于输出有功功率,故有:
Vg I g  VI
V I g  I g  I s1   I R1 
M       (19-51)
Vg I  I s1   I R1   I 
   I  4  I
    s1     s1
 4   I R1     I R1
is  s 
接下来要求出传递函数 的表达式,故有:
iR1  s 

is Z i  iR1 Re
s2 s
 1
is  s  CLRe s  Ls  Re 
2 2
Qe0
  0

iR1  s  Ls s
Qe0
1
0 
CL (19-52)
L
R0 
C
R
Qe  e
R0

F
0
接下来要求出此传递函数的模,故有:

Is F 4Qe2  2 F 2Qe2  F 2  Qe2


 (19-53)
I R1 F
故转换比率为:

- 606 -
F 4Qe2  2 F 2Qe2  F 2  Qe2
M (19-54)
F

当 Qe  15.33 时(与(c)中的参数一致),转换比率 M 和 F 的关系如图 19-21

所示:
160
140
120
100
80
60
40
20
0
0.1 0.463 0.825 1.188 1.55 1.912 2.275 2.637 3

图 19-21 给定 Qe  15.33 时的转换比率 M 和 F 的关系

(d) 对于正弦近似法的有效性,其前提是变换器只对基波分量起主要作用,对
谐波分量产生的响应与基波分量产生的响应相比可以忽略不计。故对此变换器应
用正弦近似法的有效频率区段位于远低于谐振频率的位置,即图 19-21 中左侧的
斜率非常陡的那一区段。

Problem 19.4

当 Problem 19.3 中的变换器运行在谐振频率以下时:


(a) 画出 vc  t  的波形。对于各个子区间,哪些二极管或晶体管导通电流,哪

些阻断电压?
(b) 二极管 D1 至 D4 的反向恢复过程是否导致开关损耗?晶体管 Q1 至 Q4 的输

出电容是否导致开关损耗?
(c) 当运行在谐振频率以上时,再次回答(a)和(b)中的问题。
Solution:
(a) 当运行在谐振频率以下时,且只考虑基波分量时,vc  t  的波形如图 19-22

所示,导通和阻断的半导体器件也在图中给出:

- 607 -
is  t 

0 t

vc  t 

0 t

Q2 D2 Q1 D1
阻断 Q3 D3 Q4 D4
Q1 Q4 Q2 Q3
导通 D1 D4 D2 D3

图 19-22 运行在谐振频率以下时的波形

(b) 二极管 D1 至 D4 的反向恢复过程不会导致开关损耗。晶体管 Q1 至 Q4 的输出

电容会产生开关损耗。
(c) 当变换器运行于谐振频率以上时, vc  t  的波形如图 19-23 所示:

is  t 

0 t

vc  t 

0 t

D Q D Q
阻断 D2 Q2 D1 Q1
3 3 4 4

Q1 Q4 Q2 Q3
导通 D1 D4 D2 D3

图 19-23 运行在谐振频率以上时的波形

二极管 D1 至 D4 的反向恢复过程会导致开关损耗。晶体管 Q1 至 Q4 的输出电容

- 608 -
不会产生开关损耗。

Problem 19.5

某并联谐振变换器的直流输入电压为 Vg  270V 。此变换器向直流负载提供

5V 直流电压。直流负载的功率变化范围为 20W 至 200W 。此变换器中的晶体管


需要运行在零电压模式。原边谐振网络的参数值为 L  57μH 、C p  0.9nF 。此变

换器包含的隔离型变压器的匝数比为 52 :1 。
(a) 当 F 如书中式 19.19 定义时,推导 F 的表达式,用 M 和 Qe 进行表示。

(b) 当变换器运行在最大功率负载点时,推导此时的开关频率、晶体管电流峰
值和电容电压峰值。
(c) 当变换器运行在最小功率负载点时,推导此时的开关频率、晶体管电流峰
值和电容电压峰值。
Solution:
(a) 此变换器的电路如图 19-24 所示:

ig  t 

L 52 :1 iR1  t  LF
I

Vg
vs  t  Cp vR1  t  CF R V
270V

图 19-24 并联谐振变换器
首先将负载侧进行等效,故有:
iR1  I R1 sin t 
4
I R1  I


1 (19-55)
sin t  d t   V

V R1
0

V
R
I
根据式(19-55)可求得:
VR1  2
Re   R (19-56)
I R1 8

再将开关网络进行等效,可画出最终的等效电路,如图 19-25 所示:

- 609 -
L n  52

vs1  t  
4
Vg sin t  Cp n 2 Re v t 

Zi

图 19-25 最终的等效电路
故有:
V V   I  I R1   VR1   V   Vs1 
M       
Vg  I   I R1  VR1   V   Vs1   Vg 
(19-57)
   1  1   4 8
  R         H  s      2 H  s 
 4   Re   n     n

接下来求传递函数 H  s  ,故有:

1
H s  2
s s
 1
 2
0 Qe0
1
0 
CpL (19-58)
Cp
Qe  n 2 Re
L

F
0
接下来求传递函数的模,即 H  s  ,故有:

Qe
H s  (19-59)
F Q  2 F 2Qe2  F 2  Qe2
4 2
e

将式(19-59)代入到式(19-57)中,可得:

2MQe2  M  M 2  4M 2Qe2  4Qe4T 2


F
2MQe2 (19-60)
8
T
n 2
(b) 当变换器运行于最大功率点时,根据式(19-60)可求出:
F  1.197
(19-61)
f s  841kHz

为了求出此时的晶体管电流峰值,还需求出输入阻抗 Z i  s  和它的模 Z i  s  。

- 610 -
故有:

C p LRe n 2 s 2  Ls  Re n 2
Zi  s  
C p Re n 2 s  1
(19-62)
C p2 L2 Re2 4 n 4  2C p LRe2 2 n 4  L2 2  Re2 n 4
Zi  s  
C p2 Re2 2 n 4  1

此时输入阻抗 Z i 的 Bode 图如图 19-26 所示:

Bode Diagram
80

70
Magnitude (dB)

60

50

40
90

45 System: Zi
Phase (deg)

Frequency (Hz): 8.4e+05


Phase (deg): 57.3

-45
4 5 6 7
10 10 10 10
Frequency (Hz)

图 19-26 最大负载功率时的输入阻抗 Bode 图


根据图 19-26 可以验证,此时输入阻抗呈感性,故晶体管运行于零电压模式。
根据式(19-61)和式(19-62)可求出:
Z i  158.046
4 (19-63)
V
Vs1  g
I s1    2.175A
Zi Zi

接下来要求出电容电压峰值,故有:
V  V   I  I R1   VR1 
     
Vc  I   I R1  VR1  Vc 
   1  1  2
  R         (19-64)
 4   Re   n  n
n V
Vc   408.4V
2
(c) 当运行在最小功率点时,根据式(19-60)可求出:
F  1.356
(19-65)
f s  952.6kHz

此时输入阻抗 Z i  s  的 Bode 图如图 19-27 所示:

- 611 -
Bode Diagram
80

70

60

Magnitude (dB)
50

40

30

20
90
System: Zi
Frequency (Hz): 9.54e+05
45 Phase (deg): 87
Phase (deg)

-45

-90
3 4 5 6 7
10 10 10 10 10
Frequency (Hz)

图 19-27 最小负载功率时的输入阻抗 Bode 图


根据图 19-27,可以验证此时输入阻抗呈感性,故可实现零电压软开关。
根据式(19-65)和式(19-62)可求得:
Z i  156.107
4 (19-66)
V
Vs1  g
I s1    2.202A
Zi Zi

对于电容电压峰值,由于输出电压和匝数比均没有变化,故电容电压峰值也
保持不变,即式(19-64)。

Problem 19.6

某谐振逆变器其直流输入电压为 Vg  320V ,逆变器必须产生近似正弦的频

率为 200kHz 的输出电压。当处于空载时,逆变器的输出电压峰峰值应为 1500V 。


额定运行点为 200V 有效值施加在 100 的负载上,此逆变器为非隔离型 LCC 谐
振拓扑。当负载小于 200 时,要求逆变器工作在 ZVS 模式上。
(a) 推导开路输出电压 Voc 和短路输出电流 I sc 的表达式,用 F  f s / f  , Vg ,

R  LCs C p 和 n  Cs / C p 表示。

(b) 为了满足指标,短路电流 I sc 如何选择?

(c) 确定谐振网络的参数值。
(d) 运行在 ZVS 的条件是什么?
(e) 计算空载和短路两种情况下晶体管电流的峰值。
Solution:
(a) 谐振逆变器电路如图 19-28 所示:

- 612 -
L Cs

4
Vg Cp Re v t 

图 19-28 LCC 谐振逆变器


故对于开路电压有:
1
sC p
Voc  Vs1
1 1
sL   (19-67)
sCs sC p
4
Vs1  Vg

根据式(19-67)可得:
Cs 4 Cs 4
Voc   Vg   Vg (19-68)
C pCs Ls  C p  Cs 
2
C pCs L  C p  Cs 
2

又因为:

F

1
  (19-69)
1
L
1 1

Cs C p

根据式(19-69)可得:
F 2  C p  Cs 
L (19-70)
C p Cs 2
又因为:
Cs
n (19-71)
Cp

将式(19-70)和式(19-71)代入到式(19-68)中,可得:
4 n
Voc  Vg  (19-72)
  n  1 1  F 2 
对于短路电流有:

- 613 -
Vs1 CsVs1
I sc   (19-73)
sL 
1 Cs L 2  1
sCs

将式(19-70)和式(19-71)代入到式(19-73)中,可得:
CsVs1
I sc   (19-74)
nF 2  F 2  1

又因为:

F

(19-75)
1
 
R
将式(19-75)代入到式(19-74)中可得:
Cs FVs1 Cs F 4
I sc     Vg (19-76)
R  nF  F  1
2 2
R  nF  F  1 
2 2

(b) 由于空载时,逆变器输出 1500V 的峰峰值电压,故有:


1500
Voc   750V (19-77)
2
对于谐振逆变器,其输出特性必须满足:
v  js  i  js 
2 2

 1 (19-78)
Voc2 I sc2
根据题意有:
s  2  200 103
v  js   200 2 (19-79)
200 2
i  js  
2 2
100
将式(19-77)和式(19-79)代入到式(19-78)中,可得:
I sc  3.054A (19-80)

(c) 对于谐振网络,可用如图 19-29 所示的等效电路进行参数求解:


jX s

4
Vg jX p Re v t 

图 19-29 LCC 等效电路

- 614 -
根据题意有:
Voc 750
H   j     1.841
Vs1 4 V
g
 (19-81)
V
Z o 0  j   oc  245.586
I sc

根据图 19-29 可知:


jX p
H   j  
jX s  jX p
(19-82)
X p Xs j
Z o 0  j  
X p  Xs
根据式(19-81)和式(19-82)可求得(注意,这里能求出两组参数,这里只给出
了一组参数,在接下来的求解过程中,这组参数满足指标要求。没有给出的令一
组参数是否正确并没有验证!):
X s  133.414
(19-83)
X p  292.094

根据式(19-83)可求得:
C p  2.724nF (19-84)

又因为:
1
X s  L  (19-85)
Cs

当将 Cs 的值选择和 C p 的值一样时,根据式(19-85)可求得:

Cs  C p  2.724nF
(19-86)
L  0.3386mH
(d) 对于 ZVS 和 ZCS 两种模式临界处的等效电阻值,有:
Cs L 2  1
Rcrit  Z o 0 Z o   276.832
C p 2  C p Cs L 2  C p  Cs 
Z i 0  jX s  133.414 j (19-87)
Z i  jX s  jX p  158.68 j

因为 Zi 0 呈感性,而 Z i 呈容性,故此变换器处于 ZVS 的条件为:

Re  Rcrit  267.832 (19-88)

(e) 对于空载和短路时的晶体管电流峰值有:

- 615 -
Vs1
I peak _ NullLoad   2.568A
Z i
(19-89)
V
I peak _ Short  s1  3.054A
Zi 0

Problem 19.7

某串联谐振变换器的直流输入电压为 Vg  550V ,变换器向负载提供 30kV 的

电压。直流负载的功率变化范围为 5kW 至 25kW 。要求使晶体管工作在 ZVS 模


式。最大的开关频率为 50kHz ,与谐振网络相串联的隔离变压器的匝数比为 1: n 。
相对于原边侧来说,谐振电容的电压不能超过 2000V 。
(a) 推导谐振网络的电容电压和电感电流的峰值表达式。
(b) 设计谐振网络的电容值、电感值和隔离变压器的匝数比。尽可能得使晶体
管的峰值电流最小。
Solution:
(a) 此串联谐振变换器如图 19-30 所示:
1: n I R1 I

L C
Vs1 ve vR1 R V

4
Vs1  Vg

图 19-30 串联谐振变换器
其等效电路如图 19-31 所示:

L C
Vs1 Re ve  t 

4 8 R
Vs1  Vg Re 
  2 n2
图 19-31 串联谐振变换器的等效电路
故谐振电容电压峰值和电感电流峰值为:

Vs1 1 Vs1
Vc   
  CL 2  1
1 sC
 CRe 
2
sL   Re
2

sC
(19-90)
Vs1 CVs1
IL  
  CL 2  1
1
 CRe 
2
sL   Re
2

sC

- 616 -
(b) 由于负载的功率是有变化范围的,为了在此范围内保持输出电压恒定,因
此需要通过改变运行频率来实现恒定电压输出。对于变压器原边电压 ve  t  的峰

值,有:

Vs1 Vs1
Ve  Re 
1 1  1 
2
sL   Re   
sC 1  L 
Re2  C 
(19-91)
 Ve 1
V  n 4V

R1
 Ve 
V  4 V n
 R1

对于式(19-91),必须要保证所设计的 Ve 的值要比 Vs1 小,否则不会有实数的

根。故有:
4V 4V
Ve   Vs1  n   54.545 (19-92)
n  Vs1

根据图 19-31 有:

1 
sin t d t   I  I R1 

I R1 I (19-93)
0
2

故对于电感电流峰值有:
IL
n (19-94)
I R1

根据式(19-93)和式(19-94)有:

nI IL 
(19-95)
2
为了限制电感电流峰值,根据式(19-95)可知匝数比 n 不能取太大的值,同时
根据式(19-92)可知匝数比 n 又必须大于一个阈值,故这里将匝数比取为 n  60 。
当有了匝数比 n 的值时,就可求出等效电阻 Re 的变化范围,根据图 19-31

可得:
8 1 8 1 V2
Re max  R   40.528
 2 n2 max  2 n2 Pmin
(19-96)
8 1 8 1 V2
Re min  2 2 Rmin  2 2  8.106
 n  n Pmax
根据书中 Fig19.34(a)所示的串联谐振变换器的输入阻抗 Bode 图可知,为
了满足一定负载范围内(也即某一频率范围内)均可实现 ZVS。故需保证在这

- 617 -
一频率范围内,输入阻抗均呈感性,那么这一频率范围一定要在谐振频率点的右
侧。
确定下来频率范围需在谐振点的右侧之后,根据式(19-91)还可知道为了保证
Ve 是定值,当 Re 变小时,运行频率(也即  )也需变小。同理当 Re 变大时,运

行频率也需变大。故当 Re 处于最大值时 Re max ,运行频率也处于最大值 f s max 。

根据题意,运行频率最大为 50kHz ,根据式(19-91)有:


Vs1 1
Ve   Ls max   18.572 (19-97)
2 Cs max
1  1 
1  2  Ls max  
Re max  Cs max 

由于电容电压峰值不能超过 2000V ,根据式(19-90)可知,当 Re 处于最小值

Re min 时,这时运行频率也处于最小 f s min 值,此时电容电压峰值最大,故有:

Vs1
Vc   2000V
2
 1 
Cs min Re2min   Ls min  
 Cs min  (19-98)
2
Vs1  1 
 Cs min Re2min   Ls min  
2000  Cs min 

当频率的变化范围在谐振点的右侧时,为了保证整个负载范围内均实现 ZVS,
故需保证当 Re 处于最小值时 Re min 时(此时的运行频率处于最小值  s min ),输入阻

抗呈感性即可,故有:
1
Ls min  (19-99)
Cs min

根据式(19-98)和式(19-99)可知,式(19-98)成立的必要条件是:
Vs1 Vs1
 Cs min Re min  C  (19-100)
2000 2000s min Re min

当最低运行频率设计在 f s  40kHz 时,根据式(19-100)可得:

Vs1
C  171.9nF
2000s min Re min (19-101)
C  200nF
再根据式(19-97)可确定电感的值:

- 618 -
1
 18.572
Cs max
L  109.8μH (19-102)
s max

Problem 19.8

某谐振变换器包含 LLC 谐振网络,如图 19-32 所示:


(a) 画出短路和空载两种情况下的输入阻抗 Bode 图,即 Z i 0  j  和 Z i  j 

给出谐振频率和渐近线的表达式。
(b) 当开关频率和负载阻值处于什么条件时,变换器处于 ZVS 模式?
(c) 给出 f m 的表达式,当 Zi 0  Zi 时。

(d) 画出传递函数 H   j  的 Bode 图,给出谐振频率和渐近线的表达式。

C Ls I

Vg vs  t  Lp R V

图 19-32 LLC 谐振变换器


Solution:
(a) 短路和空载下的输入阻抗的表达式为:
1 CLs 2  1
Zi 0  s    sLs  Z i 0  j  
sC C
(19-103)
1 C  Ls  Lp   2  1
Z i  s    sLs  sLp  Z i  j  
sC C
其 Bode 图如图 19-33 所示(仅示意图而已,图中有坐标轴是因为画图时必
须要给定参数才行):

- 619 -
Bode Diagram
100
Zi0
Zi00
50

Magnitude (dB)
0

-50

-100

-150
90

45
Phase (deg)

-45

-90
4 5 6 7
10 10 10 10
Frequency (rad/s)

图 19-33 短路和空载下的输入阻抗 Bode 图


根据式(19-103)可求得短路和空载两种情况下输入阻抗的谐振频率为:
1
i 0 
CLs
(19-104)
1
i 
C  Ls  L p 
(b) 正常时的输入阻抗表达式为:
CL p Ls s 3  CR  LP  Ls  s 2  L p s  R
Zi  s  
sC  R  sLp 
(19-105)
 CL R  R  CLs R 2    CL p Ls 3  L p 
2 2 2

Z i  j  
p

C R 2   Lp 
2

根据式(19-105)可求得:
Z i  j  
CL2p R 3   CLs R 2 2  L2p 2  R 2  CLp R 2 2  CL2p Ls 4  j (19-106)
C  L2p 2  R 2 
由于实现 ZVS 的条件是输入阻抗呈感性,故根据式(19-106)可知:
CLs R 2 2  L2p 2  R 2  CLp R 2 2  CL2p Ls 4  0
(19-107)
CL2p Ls 4   CLs R 2  L2p  CLp R 2   2  R 2  0

(c) 对于 f m 的求解,根据图 19-33 可知 Zi 0  jm  呈容性,而 Z i  jm  呈感性,

根据式(19-103)有:

C  Ls  Lp   2  1 1  CLs 2
 (19-108)
C C
根据式(19-108)可求得:

- 620 -
2
m 
C  Lp  2 Ls 
m  2 f m (19-109)
1 2
fm 
2 C  Lp  2 Ls 

(d) 对于传递函数 H   s  ,其表达式为:

CLp s 2
H  s  (19-110)
C  Lp  Ls  s 2  1

根据式(19-110)可求得谐振频率为:
1
 
C  L p  Ls 
(19-111)
1 1
f 
2 C  L p  Ls 

H   j  的 Bode 图如图 19-34 所示:

Bode Diagram
150
H00
100
Magnitude (dB)

50

-50

-100
180

135
Phase (deg)

90

45

0
4 5 6 7
10 10 10 10
Frequency (rad/s)

图 19-34 传递函数 H   j  的 Bode 图

Problem 19.9

如图 19-35 所示的 LLC 谐振逆变器。正常条件下,变换器的运行频率为


f s  100kHz 。所有元器件均为理想器件。

(a) 推导开路输出电压的峰值 Voc 和短路电流的峰值 I sc 的数值。

- 621 -
(b) 画出椭圆形的输出特性曲线。曲线的哪一部分变换器会运行在 ZVS 模式?
在匹配负载点上变换器是否是 ZVS 模式?
(c) 画出 Z i 0  j  和 Z i  j  的 Bode 图,并标出 f 0 、 f  、 f m 和 f s 。

(d) 空载和短路时的晶体管电流峰值是多少?
(e) 当负载阻值在 500 和空载之间变化时,输出电压的变化范围是多少?在
此范围内,变换器是否一直是 ZVS 模式?

C Ls
0.4μF 2.5μH
1: n I

Vg Lp
vs  t  R V
12V 15μH

n  7.5

图 19-35 LLC 谐振逆变器


Solution:
(a) 此变换器的等效电路如图 19-36 所示:
C Ls
0.4μF 2.5μH

Lp
vs  t  Re Ve
15μH

4 R
Vs1  Vg Re 
 n2
图 19-36 LLC 谐振逆变器的等效电路

对于等效电阻 Re 上的开路电压峰值和短路电流峰值,有:

CLpVs1 2
Veoc   20.522V
C  Lp  Ls   2  1
(19-112)
CVs1
I esc   6.345A
CLs 2  1

故输出侧的开路电压峰值和短路电流峰值为:
Voc  nVeoc  153.918V
I esc (19-113)
I sc   0.846A
n

- 622 -
(b) 对于等效电路 Re ,其输出特性表达式为:

v2 i2
2
 2
1
Veoc I esc
Veoc  20.522V (19-114)
I esc  6.345A
等效电阻上的输出特性如图 19-37 所示:
(v2)/(20.5222) + (i2)/(6.3452) = 1
8

0
i

-2

-4

-6

-8
-25 -20 -15 -10 -5 0 5 10 15 20 25
v

图 19-37 等效电阻上的输出特性曲线
匹配的负载等效电阻为:
Veoc
Re _ match  2  3.235 (19-115)
I esc
2
接下来要求出 ZVS 和 ZCS 临界处的等效负载值,即 Re,crit ,故有:

Re.crit  Zo 0 Zo (19-116)

对于这两个输出阻抗,有:
Lp s  CLs s 2  1
Zo0  s  
C  Lp  Ls  s 2  1 (19-117)
Z o  s   sLp
根据式(19-116)和式(19-117)可以求出临界的等效电阻值:
L2p 2  CL2p Ls 4
Re,crit   5.521 (19-118)
C  Lp  Cs   2  1

接下来要求出 Z i 0  j  和 Z i  j  ,其中   2 100kHz 。故有:

- 623 -
1
Zi 0  s    sLs
sC
1
Z i  s    sLs  sLp (19-119)
sC
Zi 0  j   2.408 j
Zi  j   7.017 j

根据式(19-119)可知,当 Re  Re,crit 时,变换器可工作在 ZVS 模式。因此在匹

配的等效负载点上,变换器不能工作在 ZVS 模式。


(c) 根据式(19-119)可画出 Z i 0  j  和 Z i  j  的 Bode 图,如图 19-38 所示:

Bode Diagram
50
Zi0
Zi00
0
Magnitude (dB)

-50
fm fs
-100
f f0
-150
90

45
Phase (deg)

-45

-90
4 5 6
10 10 10
Frequency (Hz)

图 19-38 空载和短路时的输入阻抗 Bode 图


(d) 根据式(19-119)可求出短路和空载时的晶体管电流峰值为:
Vs1
I p_0   6.345A
Zi 0
(19-120)
V
I p_  s1  2.178A
Z i
(e) 当负载电阻的阻值变化范围为 500  时,那么等效电阻的变化范围
500
为: 2  8.89  。
n
为了求出此范围内输出电压的变化范围是多少,首先要求出输出电压的表达
式:
1 1 1
 
Z sLp Re
Vs1 CLp ReVs1s 2 (19-121)
Ve  s   Z
1
 sLs  Z CLp Ls s 3  CRe  Lp  Ls  s 2  Lp s  Re
sC
根据式(19-121)可求出:

- 624 -
CLp ReVs1 2
Ve  j  
 CL R   Re  CLs Re 2    CLp Ls 3  L p 
2 2 2
p e

(19-122)
CLpVs1 2

 CL   1  CLs 2  
1
2 
CLp Ls 3  L p 
2 2 2
p
Re

通过式(19-122)可知,随着 Re 的变大,等效电阻上的输出电压 Ve 是单调递

增的。因此有:
CLpVs1 2
Ve _min 
 CL   1  CLs 2  
1
 CL L   L p 
2 2 3 2
p 2 p s
R
e _ min

 19.286V (19-123)


CLpVs1 2

Ve _max 
 CL   1  CLs 2 
2 2
p

 20.522V
因此输出电压的变化范围为:
Vmin  nVe _ min  144.642V
(19-124)
Vmax  nVe _ max  153.918V

根据式(19-118)可知,在此负载的变化范围内,变换器可始终处在 ZVS 模式。

Problem 19.10 (待完成)

Problem 19.11 (待完成)

- 625 -
第20章 软开关

Problem 20.1

如图 20-1 所示的正激变换器,L 和 C 是值很大的滤波器件,而 Lp 、Ls 和 Cr

的值相对很小。变压器的复位机制没有展示。对于此题,你可以认为变压器是理
想的。
(a) 谐振开关网络属于什么类型?
(b) 哪个半导体器件处于 ZVS 模式,哪个半导体器件处于 ZCS 模式?
(c) 谐振频率是多少?
Lp 1: n Ls Ds L

Dp Cr C R v
vg

图 20-1 正激变换器
Solution:
(a) 属于半波 ZCS 开关网络。
(b) 二极管 Dp 的关断过程处于 ZVS 模式,晶体管 Q 的关断过程处于 ZCS 模

式。
(c) 谐振频率为:
1
0 
n L2
p  Ls  Cr
(20-1)
1
f0 
2 n L2
p  Ls  Cr

- 626 -

You might also like